Você está na página 1de 141

ed.

2 I 2015 I RS18,00 INGLÊS I INGLÊS 11 ESPANHOL I ESPANHOL 11


As diferenças Relembre como As palavras com Entenda a estrutura
711��llllll!II ��IJI!��11 entre o Simple Past
e o Past Perfect
usar o superlativo
e o comparativo
sentido diferente
do que parece
das frases e as
conjugações verbais

Guiado €) ESTUDE SOZINHO A PARTIR DE FATOS DA ATUALIDADE I www.guiadoestudante.com.br

Estudante

IDIO
VEST

H(YI�A� �( IN�l(� ( (��AN��l


TENHA NA PONTA DA lÍNGUA O IDIOMA QUE VOCÊ USARÁ NAS PROVAS

Interpretação: pratique com 46 textos de vestibulares 1 Simulados: 65 questões comentadas


Indispensável}
indispensablel
imprescindible SELO DE QUALIDADE
GUIA DO ESTUDANTE
O selo de qualidade acima, que você também vê
Organização das Nações Unidas (ONU) reconhece 193 nações na capa desta edição, é resultado de uma pes·

A
no mundo. Em um terço delas, fala-se o inglês; em 11%, o quisa realizada com 316 estudantes aprovados
espanhol. Entre essas nações, estão gigantes da economia e
em três dos principais cursos da Universidade
de São Paulo no vestibular 2013. São eles:
grandes fontes de cultura, como Estados Unidos, Reino Unido,
Espanha e México. Isso já dá uma ideia da importância de se dominar �DIREITO, DA FACULDADE DO
esses idiomas, tanto para a vida pessoal quanto para a profissional. É LARGO SÃO FRANCISCO;
com esse espírito que você deve encarar esta edição do GUIA DO ES­ �ENGENHARIA, DA ESCOLA POLITÉCNICA; e
TUDANTE IDIOMAS 2015- como forma de enriquecer seu currículo. �MEDICINA, DA FACULDADE DE
Mas sejamos práticos: o inglês e o espanhol são fundamentais, primeiro, MEDICINA DA USP
para passar no vestibular e avançar no curso universitário. Quem pretende
estudar alguma das ciências humanas terá de ler e interpretar textos ®7 em cada 10 entrevistados na pes·
de história, sociologia, antropologia ou literatura estrangeira. Aqueles quisa usaram material do GUIA DO
que se dedicarem às ciências biológicas ou exatas terão de pesquisar ESTUDANTE durante sua preparação
artigos científicos e compreender não apenas gráficos e esquemas, mas para o vestibular
também as considerações e conclusões dos autores. Seja qual for a área
de conhecimento escolhida, sem conhecer o mínimo do inglês ou do ®Dos entrevistados
espanhol fica muito difícil buscar e consultar documentos na internet. que utilizaram o
Não tem como fugir: você precisa treinar ao menos esses dois idiomas GUIA DO ESTUDANTE:
estrangeiros. Nada de portunhol ou inglês ao pé da letra. 82% disseram que o material ajuda
Esta edição traz textos para interpretação tirados de grandes vestibu­ ou ajudou muito na preparação.
lares. Traz, ainda, dicas de gramática e vocabulário, para você lembrar gB% recomendam o guia
detalhes que podem se transformar em pegadinhas nas questões da prova. para outros estudantes.
Este volume foi elaborado com a consultoria de duas especialistas nesses
dois idiomas - Gisele Rego, em inglês, e Silvia Ferrari, para espanhol.

eEstudante
Esperamos que você aproveite bem esta revisão. E, para o vestibular,
good luck ou isuerte! Guiado
A redação
TESTADO E APROVADO!

A pesquisa quantitativa por meio de entrevista


pessoal foi realizada nos dias 18 e 19 de fevereiro de
2013, nos campi de matrícula dos cursos de Direito,
Medicina e Engenharia da Universidade de São
Paulo {USP).
�Universo total de estudantes aprovados nesses
cursos: 1.455 alunos.
�Amostra utilizada na pesquisa: 316 entrevistados.
�Margem de erro amostrai: 4,9 pontos percentuais.

4 1 GE IDIOMAS 2015
INGLÊS
INTERPRETAÇÃO DE TEXTO
Language classes are popular in America's public schools 10
How to reach long·term happiness 12
Will we ever... 14
lhe magical qualities of guarana 16
Arts and entertainment 18
lhe risks of distraction 20
lhe future of the job market 22
They occupied the streets 24
Climate change: warm words and cool waters 26
Work, e·mail, news, bills... 28
A history of Pi 30
lnterview with Noel Gallagher 32 DICAS DE CONTEÚDO
lhe history of chocolate 34 Simple Present»Falando sobre o cotidiano 13
Past in poetry 36 Present continuous»Quando o gerúndio está liberado 17
Wolverine online leak: howling mad! 38 Vocabulário)) Expressões que confundem 21
About swine fiu 40 Verbos modais)) Completando o sentido 25
lhe Dictionary of American Regional English 42 Adjetivos)) A função dos adjetivos 29
School and longevity 44 Passado)) Diferenças sutis e importantes 33
lhe death of the PC 46 Passado contínuo)) Tudo ao mesmo tempo 37
Volunteer abroad 48 Going to e will )) Como falar sobre o futuro 41
Distant peak car 50 Formação de palavras)) Letrinhas que mudam tudo 45
Another reason to choose a mate wisely 54 Conjunções, preposições...)) Relações entre ide ias 49
Health crisis 58 Aplicações» Passado que continua 52
Big city life 62 Gramática)) O apóstrofo com se phrasal verbs 53
lhe future of rich nations 66 Voz passiva)) Quando o complemento do verbo vira sujeito 56
Comparativo e superlativo)) Superioridade,
igualdade ou inferioridade 57
Regência)) As leis da regência verbal 6o
Discurso direto e indireto)) Duas formas de diálogo 61
Can, may e must)) O uso dos verbos modais 64
Many, much, little»Como usar os quantificadores 65
Orações condicionais)) Como usar o if 68
Pronomes)) Palavras compostas 69
Pronomes relativos»Orações subordinadas adjetivas 70
Pronomes interrogativos)) Na hora de perguntar 71

SIMULADO 72
ESPANHOL EDITORA�Abril
Fundada em 1950
VICTOR CIVITA ROBERTO CIVITA
INTERPRETAÇÃO DE TEXTO (1907·1990) (1936·2013)
82 El espano! invade Brasil Conselho Ed;w;al: Victor
Civita Neto (Presidente),
Thomaz Souto Côrrea(Vice·Presidente), Elda Müller,
84 De igual a igual Fábio Colletti Barbosa, José Roberto Guzzo

86 Venas abiertas de Ame rica Latina Pres;dente: Fábio Colletti Barbosa


Vice-presidente de Operações e Gestão: Fábio Gallo
88 Diversidad cultural en la era de la globalización Diretor-Superintendente de Assinaturas: Fernando Costa
90 llas mascotas forman parte de la familia? Diretora de Recursos Humanos: Cibele Castro

92 Para unas vacaciones tranquilas Diretora-Superintendente: Helena Bagnoli

esuiado
Diretor Adjunto: Di mas Mietto
94 Los celas en el cyber mundo
96 La canción de I bandido
Estudante
98 La muerte según pedra Almodóvar Diretor de Redação: Fabio Vo lpe
Diretor de Arte: Fábio Bosquê Editores: Fábio Akio Sasaki, Lisandra
100 De la ficción a la ciencia Matias, Paulo Montoia. Paulo Zocchi Repórter: Giovana Moraes Suzin
Analista de Informações Gerenciais: Simone Chaves de Toledo Analista de
102 Los peligros de comer bocadillos Informações Gerenciais Jr.: Maria Fernanda Teperdgian Designers: André
Tietzmann, Dânue Falcão Atendimento ao leitor: Carolina Garofalo, Sandra
104 Perdidos en el espacio Hadich, Sonia Santos, Walkiria Giorgino CTI Eduardo Blanco (Supervisor)
INTERNET NÚCLEO JOVEM & INFANTIL Redator Chefe: Rafael Kenski
106 El cerebro es lo más importante Editora-Assistente: Mariana Nadai Repórteres: Ana Prado. Ludmilla
Balduino e Otavio Cohen Designer: Abraaão Corazza, Juliana Moreira
108 Todo depende del punto de vista e Laura Rittrneister Animação: Felipe Thiroux Webmasters: Allyson
Kitamura, Cah Felix e Felipe Oliveria, Leonam Dias Analista de redes
110 Dilema: .:cooperar o aprovecharse? sociais: Lorena Dana e Lucas Baranyi
COLABORARAM NESTA EDIÇÃO Edição: Thereza VenluroH Ilustração: 45
114 El tráfico nos ensefía Jujubas (capa), Teresa Bert1inadi Revisão: Gisele C. Batista Rego (revisora
técnica do inglês), Silvia Aparecida Ferrari (revisora técnica do espanhol)
116 La mujer que escribió un diccionario e Kátia Shimabukuro
www.guiadoestudante.com.br
120 Continuidad sí
PUBUCIDADE SEGMENTADAS- Diretor de publiddade UN SEGMENTADAS:
124 Estudios en Latinoamérica Rogério Gabriel Comprido Diretores: TJagO Afonso. Willian Hagopian Gerentes:
Ana Paula Moreno, Cleide Gomes, Fernanda Xavier, Fernando Sabadin, Regina
130 Como nosotros Maurano Executivos de Negócios: Adriana Martins, Ana Paula Viegas, Cadu
Torres, Camila Roder, Cátia Yalese, Cida Rogiero, Cintia OHvcira, Cristina Loureiro,
134 EI actor activo deI cambio cultural Cristina Marto, Daniela Serafim, Emanuele Coghi, Fábio Santos, Fernanda
Melo, Fernando Lapa, Gabriel Mullcr, Hélio Lima, Juliana Chen Sales, Juliana
Compagnoni, Leandro Thales, Uvy Santos, Lucia Lopes, Luis Augusto Dias Cesar,
Luis Fernando Lopes, Marcelo de Campos, Marcus Vinícius Souza, Maria Helena
Bernardino, Maria Lucia Vieira Strotbek. Marta Ve1oso, Mauricio Ortiz, Mauricio
Amaral Emanuelli, Mayara Brigano, Michele Brito. Paula Perez, Raquel Ienaga,
DICAS DE CONTEÚDO Rebeca da Costa Rix, Renato Mascarenhas, Roberta Maneiro, Sérgio Albino,
Shirlene Pinheiro, Silvano Narcizo, Suzana Veiga Carreira, Vera Reis de Queiroz.
85 Pronomes» Forma ou informal? MARKETING - DIRETOR DE MARKETING: Paulo Camossa DIRETORES:
Louise Faleiros, Wagner Gorab ESTRATÉGIA DIGITAL DIRETOR: Guilhenne
89 Verbos» Estudando conjugação Wemed< PUBLICIDADE REGIONAL· DIRETOR: Jacques Ricardo GERENTES:
Ivan RizentalJoão Paulo Pizarro, Kiko Neto, Mauro Sannazzaro, Sonia Paula,
93 Estrutura>> Tudo em sua ordem Vania Passolongo PUBLICIDADE INTERNACIONAl: Alex Slevens AWLSAS:
Gerente Magali Superbi
97 Vocabulário>> Parece, mas não é APOIO· PL ANEJAMENTO CONTROLE E OPERAÇOES- Gerente: Mar;na
Bonagura PROCESSOS - Gerente: R;cardo Carvalho DEDOC E ABRIL
101 Expressões idiomáticas>> O pouco que diz muito PRESS Grace de Souza PESQUISA E INTELIGENCIA DE MERCADO Andrea
Cosia RECURSOS HUMANOS GERENTE: Daniela Rub;m TREINAMENTO
105 Organização textual>> Masculino ou feminino? EDITORIAL Edward P;mcnla

109 Vocabulário- Imperativo>> Isto é uma ordem! Redação e Correspondência: Av. das Nações Unidas, 7221, 5° andar,
P;nheiros. São Paulo. SP. CEP 05425·902, lei. (11) 3037·2000. Publicidade
112 Condicional»Reforço no argumento São Paulo e informações sobre representantes de publicidade no
Brasil e no Exterior: www.publiabril.com.br
113 Futuro simple »Para falar do que há de vir PUBLICAÇ0ES DA EDITORA ABRIL: Almanaque Abril, Ana Maria,
Arquitetura & Construção. Aventuras na História, Boa Forma, Bons Fluidos,
115 Vocabulário- Verbos irregulares» Fora do padrão Capricho, Casa Claudia, Claudia, Contigo!, Dicas lnfo, Elle, Estilo, Exame,

118 Locuções verbais» Interpretação rica Exame PME,Guia do Estudante, Guias Quatro Rodas, Info. Manequim,
Máxima, Men's Hcalth, Minha Casa, Minha Novela, Mundo Estranho.

119 Demonstrativos»lEsos o aquellos? National Geographic, Nova, Placar, Playboy, Publicações Disney, Quatro
Rodas, Recreio. Runner's World, Saúde, Sou Mais Eu!, Superinteressante,

122 Pretérito perfecto compuesto» Passado que se prolonga Tititi, Veja, Veja BH. Veja Brasnia, Veja Rio, Veja São Paulo, Vejas Regionais.
Viagem e Turismo, Vida Simples, Vip, Viva!Mais, Você S.A., Você RH,

e inclui o presente Women's Health Fundação Victor Civita: Gestão Escolar, Nova Escola

GE IDIOMAS ed.2 2015 (EAN 789·3614·09540 0), é uma pubHcação


123 Possessivos»Adjetivos ou pronomes da Editora Abril. Edições anteriores: Venda exclusiva em bancas, pelo
preço da última edição em banca mais despesa de remessa. Solicite ao
126 Conexões»Coesão e coerência seu jornaleiro. Distribuída em todo o país pela Dinap S.A. Distribuidora
Nacional de P ublicações. São Paulo.
127 Pretérito imperfecto de indicativo»Hábitos antigos A PUBLICAÇÃO não admite publicidade redacional.

128 Pretérito perfecto simple»Ações/situações SERVIÇO AO ASSINANTE: Grande São Paulo: (I!) 5087·2112
Demais localidades: 08007752112 www.abrilsac.com
completas e acabadas PARA ASSINAR: Grande São Pauto: (11) 3 3 47·2121
Demais localidades: 08007752828 www.assineabril.com.br
129 Função advérbio» Um bom conectivo resolve quase tudo IMPRESSA NA GRÁFICA ABRIL
Av. Olaviano Alves de Uma, 4400, CEP 02909·900- Fregues;a do Ó·
132 Pretéritos de I subjuntivo» Condicionais no passado São Paulo - SP

133 Regência» É parecido, mas não é igual I'V'J


136 Acentuação» Regras de uso do acento gráfico
I="IPP $SIP ANER

137
138
Heterotônicos »Grafia igual, mas...
Verbos de câmbio» Estrutura importante
lfAbril MIDtA s.A.
Conselho de Adm;n;stração: Giancarlo Civila (Presidente), &mare
Weideman, Hein Brami, Roberta Anamarta Civita, Victor Civila Neto
Presidente: Fábio Colletti Barbosa
139 SIMULADO www.abril.com.br

GE IDIOMAS lOlS 1 7
I NTERPRETAÇÃO D E TEXTO

Language classes
are popular in
America's public
schools
lt is not true that Americans do not want to
learn a foreign language. They actually do
so that they can be part of a multicultural
plurilinguistic world

but are also exposed to the cultures of French·speaking countries

M
ore than 9 0 0 young students arrive at Kent Gardens students' fluency in English. Richard Gordon, a 6th grade English
Elementary every morning to take part in a language and social studies te acher, disagrees. "One thing I noticed when I
training program known as partia! immersion. Its aim is to teach my students coming from the foreign language program is
help students beco me fluent in a foreign language of their choice. that they are very oral, they are very vocal, and they can express
In this particular school, French is the language being offered. themselves very well in my English classes."
"Half o f the day is spent with a French immersion teacher, where But French teachers like Francoise Brottet admits there are
they learn math and science in French and the other part o f the challenges involved in the immersion program. "Since they do not
day is spent with the English teacher where they are taught social understand everything we say, we have to find creative and different
studies and language arts in English," says Annie Dwyer, a 5th ways to teach them." Young students have their own challenges.
grade French teacher. "It can be a bit confusing at times, because you learn the terms in
Students not only receive instruction in the French language but French and sometimes some tests and homeworkyou get are written
are also exposed to the cultures ofFrench-speaking countries. They in English,'' says 6th grader Kimia Zadegan.
interact with teacher-interns from France. Some students host the Over the past decade, hundreds o f American public schools have
French interns in their homes, establishing personal cultural links. begun offering partia! immersion programs to teach students Latin,
Students at Kent Gardens Elementary come from many different Spanish, German, Japanese and many other languages. Dr. Robyn
cultural backgrounds, and they give many different reasons for Hooker, Kent Gardens' Principal, says that many o f the students at
wanting to be fluent in a foreign language. the school are already multilingual, as well as multicultural, when
"I am actually Muslim," says Rosa Ahmat, "so my parents want me they enroll. "Many o f the children in our school, because it is an
to learn about other cultures and people from other places." Anwar international school, come to us speaking perhaps two o r three and
Mendes says he expects to travei a lot in the future. "So I will have in some instances four languages, so it becomes a part of our res­
a prettygood chance of meeting a person that speaks French," he ponsibility to prepare children for global society - and that always
says. "It will be very hard to get a good job if you do not know the includes the languages."
other languages," says Neha Rana. Hooker believes the training students are receiving at Kent
·
Many educators support that belief. They say immersing young Gardens - and in similar language immersion schools across the
students in foreign language study for at least half of their school country- will help them to communicate more effectively not just in
day gives them the multilingual skills they'll need to succeed in an their own American polyglot, but in the 21st century's increasingly
increasingly global economy. There are other benefits as well. "It global village.
expands the mind of the child and also gives them a greater cultural
awareness of other cultures and makes them more tolerant of others'
differences," says 6th grade French teacher Christine Bedoret. (Universidade Federal de Viçosa (UFV), 2008.
Some critics of immersion programs believe that spending half the (http://www.voanews.com/engl ish/archive/2008·03/2008·03-l9·VOa34.cfm, adaptado)
school day learning in a foreign language could negatively impact Retrieved on ]une 2, 2008. Written by Mohamed Elshinnawi

10 I GE IDIOMAS 2015
EXERCfCIOS - U FV 2008 (adaptado)

L Leia o texto na página anterior e responda às q uestões a s. O trecho em voz passiva "some tests and homework you
seguir: get are written in English" (fi nal do 6° parágrafo) pode ser
De acordo com Richard Gordon, os alunos que participam reescrita na voz ativa como:
dos programas de imersão a) Someone wrote the tests and homework you get in English.
a) dobram seu quociente de inteligência através da prática b) Someone writes the tests and homework you get in English.
do idioma. c) Someone will write the tests and homework you get in
b) melhoram sua fluência em inglês. English.
c) esquecem sua língua m aterna . d) Someone is writing the tests and homework you get in
d) tornam-se, no geral, tímidos e pouco falantes. English.
e) melhoram muito a competência escritora. e) Someone writes the tests and homework you got in English.

z. Em um estudo sobre as relações entre língua e cultura,


Kramsch (1998:81) explica que a comunicação trans, inter
ou multicultural é uma habi!idade mental complexa, que RESPOSTAS
envolve um aspecto específico da aprendizagem de línguas: a 1. Melhoram sua fluência em inglês. No texto, identificamos a fala
capacidade de aprender a língua/cultura de outra pessoa sem do professor na frase entre aspas "One thing I noticed when I teach
perder a sua própria e, assim, transitar entre esses "mundos". my students coming from the foreign language program is that they
Pessoas "multiculturais" conseguem desempenhar papéis are very oral, they are very vocal, and they can express themselves
sociais diferentes, linguística e culturalmente adequados very we/1 in my English classes". Palavras-chave para a compreensão
dentro de uma situação de interação contextualizada. da frase: "theyare veryoral" (eles, os alunos, são muito orais), vocal
(In: KRAMSCH, Claire. Language and Culture. Oxford: OUP, 1998) (vocalizados), "can express themselves very well" (conseguem se
expressar muito bem). Resposta: b
Levand o em conta o pensamento de Kramsch, assinale a
alternativa q u e apresenta a frase do texto da página 10 que z. "lt expands the mind of the child and also gives them a greater
confirma essa explicação: cultural awareness o f other cultures and makes them more tolerant
a) "lt expands the mind of the child and also gives them a o f others' differences." A exp licação de Kramsch trabalha com o as·
greater cultural awareness of other cultures and makes pecto cultural das línguas, e a única alternativa que apresenta uma
them more tolerant of others' differences." frase do texto claramente relacionada ao ass unto é a alternativa (a).
b) "lt can be a bit confusing at times, because you learn the Palavras-chave para a compreensão da frase: "expa nd s the mind"
terms in French and sometimes some tests and homework (expande a mente), "gives cultural a wa reness of other cultures" (dá
you get are written in English." consciência cultural sobre outras culturas). Resposta: a
c) "lt will be very hard to get a good job if you do not know
the other languages." 3. Therefore. A conjunção "so" (então, daí) indica a ideia de con­
d) "So I will have a pretty good chance of meeting a person sequência, assim como "therefore'' (assim, portanto). Resposta: c
that speaks French."
e) "Since they do not u nderstand everything we say, we have 4. "Students" and "educators". O adjetivo possessivo "their" (deles/
to find creative and different ways to teach them." delas) refere-se à palavra "students" anteriormente mencionada na
frase ("its aim isto help students become fluent in a foreign language
J. No terceiro parágrafo, a palavra So na frase "So
I will have a of their choice'; que significa "seu o bj etivo é ajudar os alunos a se
pretty good chance of meeting a person that speaks French" tornarem flu en tes em uma lfngua estrangeira de escolha deles").
pode ser substituída, sem alteração de seu significado, por: }á o pronome do caso reto "they" (eles/elas) se refere a "edu cato rs "
a) however. mencionado na frase anterior("Manyeducators support that belief.
b) despite. They say immersing young students... '; que se traduz por "Muitos
c) th e refo re. educadores sustentam essa crença. Eles dizem que fazer alunos
d) although. jovens participarem de imersão..."). Resposta: d
e) but.
s. Someone writes the tests and homework you get in English. Na
4- (UFV 2008, adaptado) As palavras their (4" linha do 1° parágrafo) voz passiva, o foco da frase pertence ao objeto da ação (neste caso,
e they (1• linha do 4° parágrafo) referem-se, respectivamente "the tes ts and homework", as provas e lições de casa). Na voz ativa,
às palavras: o foco vai para o sujeito ("someone': que aqui significa alguém
a) "schools" e "languages". I ndete rmina do). Resposta: b
b) "students" e "schools".
c) "educators" e "students''.
d) "students" e "educators".
e) "languages" e "educators".

GE IDIOMAS 2015 1 11
I NT ERPRETAÇÃO D E T EXTO

How to reach long·term happiness


Family and friends seem to be the secret for a happy life

H
arvard conducted one of the longest and most comprehensive deeply reflects the human condition. What can be concluded from
studies ofhuman development - the 75 year old Grant Study seven decades of data? It is quite simpie actually; warm relationships
- that's reached some fascinating conclusions regarding the between parents, spouses, children and friends have the greatest
recipe for leading a happy life. The sample group was comprised of impact on your health and happiness in old age. The study found
healthy mal e Harvard college students who, over the course o f their that 93 percent o f the sample group who were thriving at age 65,
lifetime, agreed to meet with an array of scientists and researchers had a dose relationship with a sibling when they were younger. As
who measured their psychological, physical and anthropological George Vaillant, the lead director of the study states, it can ali be
traits. Though ali identities are confidential, it was recently disco­ boiled down into five simpie words: "Happiness is love.Full stop."
vered that John F. Kennedy was a sample participant. Following
these men through times of war, their careers, parenthood and old Business lnsider. Disponível em http://www.goodnet.org/articles/lOSS . Acessado e m
age, the Grant Study has amassed an exorbitant amount of data that 10/6/2013- (Vestib ular ITA 2014)

EXE RCfCIOS - ITA 2014 (adaptado)

LA Grant Study, pesq uisa realizada pela U n iversidade de Harvard, R ESPOSTAS


a) teve por objetivo investigar o comportamento de pessoas L Analisando as alternativas:
idosas e felizes. a) O estudo teve como amostra homens q uando eram estudantes univer­
b) possibilitou o levantamento gigantesco de dados sobre pes· sitários de Harvard- jovens, portanto. Veja o trecho: "The sample group
quisadores de Harvard. was comprised of healthy ma/e Harvard college students... ". Incorreta.
c) comprovou que john F. Kennedy foi um homem extremamente b) Cientistas e pesquisadores são os responsá veis pelo estudo, e não o
feliz. objeto estudado. Incorreta.
d) chama·se the 75 year o/d Grant Study por ser homenagem à c) O nome do ex-presidente ]ohn F. Kennedyé citado apenas como um dos
faixa etária analisada. participantes da pesquisa. Além disso, não h á nenhuma menção aos
e) comprovou que felicidade na vida adulta está atrelada às rela­ resultados relativos a cada indivíduo da amostra. Incorreta.
ções afetivas ao longo da vida. d) A faixa etária é bem mais ampla, pois a pesquisa se prop u n h a a
acompanhar jovens ao longo de sua vida inteira, até os 75 anos de
2. Assinale a opção cuja reescrita não altera o sentido de: "Though idade. Incorreta.
ali identities are confidential, it was recently discovered that john e} A resposta correta está no final do texto: "...warm relationships between
F. Kennedy was a sample partici pant." parents, spouses, c h ildren and friends have the greatest impact
a) john F. Kennedy was a sample participant, although nobody o n your health and happiness in old age" (relações afetivas entre
knew that. pais, cônjuges, filhos e amigos têm maior impacto sobre sua saúde e
b) In spite of being a sample participant,John F. Kennedy's identity felicidade na velhice}. Correta.
was never discovered. Resposta: e
c) The study was confidential, thus the participation of john F.
Kennedy was never discovered. 2. Traduzindo a frase proposta, temos "Apesar de todas as identidades
d) Besides being a confidential study,john F. Kennedy said he used serem confiden ciais, foi descoberto recentemente que john F. Kennedy
to be a participant. foi um dos participantes da amostra". As alternativas podem confundir,
e) In spite of the fact that ali identities are kept confidential, it was mas, sendo o mais fiel possivel à frase original, a melhor alternativa é "In
recently found out thatjohn F. Kennedy was a sample participant. spite o f the fact that ali identities are kept confidential, it was recently
found o u t that]ohn F. Kennedy was a sample participant". "In spite o f"
3. S u bstitu indo os adjetivos long e comprehensive, respectiva­ tem o mesmo sentido de "though" (apesar de, embora), e "found o u t" é
me" n te, por easy e rich na oração "Harvard conducted one of the o mesmo que "discovered" (descoberto).
longest and most comprehensive studies of hu man development", Resposta: e
teremos:
a) the most easy- the richest. 3. A questão exige conhecimentos básicos de superlativos. Os termos
b) the easiest- the most rich. "the longest'' and " the most comprehensive" significam, respectiva­
c) the more easy- the richer. mente "o mais extenso/longo" e "o mais abrangente". Os termos "easy"
d) the easiest- the richest. (fácil) e "rich" (rico), no superlativo, transformam-se em "the easiest" e
e) the most easy- the most rich. "the richest". (Veja mais sobre comparativos e superlativos na pág. 57)
Resposta: d

121 GE IDIOMAS lOlS


DICAS S I M PLE PRESENT » FALA N DO S O B R E O COT I D IANO

Saiba quando usar o Simple Present,


um tempo verbal básico da língua inglesa

A l íngua i nglesa tem, de modo geral, u m a estrutura do tipo Já o verbo have é i rreg ular no presente s i mples. A terce i ra
svc, ou seja, sujeito, verbo e complementos. Nesse aspecto, ela pessoa do s i n g u lar é has.
é bem semelhante à l ín gua portuguesa. Na frase "More than Nas formas negativa e i nterrogativa, u samos um verbo
goo young students arrive at Kent Gardens Elementary every aux il i ar (do not, does not, do, does) que antecede, no caso
morning", podemos perceber essa estrutu ra: o sujeito (More da forma negativa, o verbo pri n c i pal, e, no caso d a forma
than goo young students), o verbo (arrive) e os complementos i n terrogativa, o sujeito d a frase. Além d isso, é i m portante
(at Kent Gardens Elementary every morning) que i n d icam o l e m b rar q u e, nessas formas, o verbo pri n cipal da frase está
lugar e o tempo da ação verbal. Assi m como acontece em sempre na forma do i nf i n itivo sem a partrcu la to.
português, verbos e su bstantivos são n ú c leos de sentido nas Para os pronomes l, you, we, they, usamos o verbo aux il iar
frases, o q u e reforça a i m portânc i a de, n o caso dos verbos, do not para fazer a negação. No caso d e he, she, it, usamos
conhecer suas regras de uso e sua estruturação. does not. Para formar perguntas, usamos o verbo auxil iar do
para os pro n o m es l, you, we e they e does para os pro n omes
THE SIMPLE PRESENT he, she e it. O bserve os exemplos:
Em i nglês, u samos o presente s i mples para: 1 don't speak Mandarin. And you, do you speak a foreign
» I n d icar atividades ou ações rot i n e i ras, cot i d ianas, b e m language? (Eu não falo mandar i m . E você? Você fala alguma
c o m o a freq uência com q u e s ã o desenvolvi d as. língua estrangei ra?)
Ex.: I get up a t 7 o'clock. (Eu levanto às 7 h o ras.) My wife doesn't understând me. Does your wife understand
I do sports twice a week. (Eu pratico esportes d uas vezes you, Henry? (M i n h a esposa não me e n te n d e . S u a esposa
por semana.) entende você, H e n ry?J
» Apresentar i nformações factuais ( i nformações pessoais,
por exemplo). ESTUDANDO CONJUGAÇAO
Ex.: Iam a high schoo/ student. (Eu sou aluno do e n s i n o Observe a tabela e faça o exercfcio proposto.
médio.)
1/ike comic books very much. (Eu gosto m uito de histórias A conjugação de "to speak" (falar)
em quadrin h os.)
» Falar sobre h o rários e locais em que ações n o futuro vão
acontecer (em anú ncios e tabelas com horários de funcio­
namento, por·exemplo).
Ex.: The train leaves the station a t quarter to nine tomor­
row. (O trem parte da estação às q u i nze para as n ove
amanhã.)

A conjugação d e verbos n o presente s i mples d epende d o


sujeito ao qual o verbo s e relaciona. Na forma afirmativa, para
os sujeitos representados pelos pronomes I, you, we, they,
conj uga-se o verbo usando a forma d o i nf i n i tivo, sem a par­
tícu la to. Para os sujeitos da terceira pessoa do s i n gular (he,
she, it), o verbo é acrescido de /s/, /ies/ou /es/, d epe n d e n d o
da termi nação d o verbo. O bserve os exemplos:
1/ove my wife too!
My wife laves me!

Os verbos que terminam em "s", "sh", "ch", "z", "x" e "o"


recebem /es/ (como em to toss, arremessar, he tosses). O s Releia o texto da pági na 10 e s u b l i n h e os verbos q ue estão
verbos q u e term i nam em "y" prece d i d o de consoante têm conjugados no presente si mples. Você consegue compreender
o "y" s u bstituído por /ies/ (com o em to cry, chorar, he cries). o sentido das frases e m que eles se encontram?

GE IDIOMAS 2015 1 13
I N T E R P RETAÇÃO D E TEXTO

Will we ever...
... understand why m usic makes us feel good?

...rlllil EUPHORIA A gush of a


neurotransmitter may be the
explanation for the extreme
happiness people feel in shows

N
o one knows why music has such a potent effect on our no obvious survival value do the same thing? The truth is no
emotions. But thanks to some recent studies we have a one knows.
few intriguing clues. Why do we like music? Like most However, we now have many clues to why music provokes
good questions, this one works on many leveis. We have answers intense emotions. The current favourite theory among scientists
on some leveis, but not ali. who study the cognition of music - how we process it mentally­
We like music because it makes us feel good. Why does it dates back to 1956, when the philosopher and composer Leonard
make us feel good? In 2001, neuroscientists Anne Blood and Meyer suggested that emotion in music is all about what we expect,
Robert Zatorre at McGill University in Montreal provided and whether or notwe get it. Meyer drew on earlier psychological
an answer. Using magnetic resonance imaging they showed theories of emotion, which proposed that it arises when we're
that people listening to pleasurable music had activated brain unable to satisfy some desire. That, as you might imagine, creates
regions called the limbic and paralimbic areas, which are con­ frustration or anger - but i f we then find what we're looking for,
nected to euphoric reward responses, like those we experience be it love or a cigarette, the payoff is all the sweeter.
from sex, good food and addictive drugs. Those rewards come This, Meyer argued, is what music does too. It sets up sonic
from a gush of a neurotransmitter called dopamine. As DJ Lee patterns and regularities that tempt us to make unconscious
Haslam told us, music is the drug. predictions about what's coming next. If we're right, the brain
But why? It's easy enough to understand why sex and food gives itself a little reward- as we'd now see it, a surge of dopa­
are rewarded with a dopamine rush: this makes us want more, mine. The constant dance between expectation and outcome
and so contributes to our survival and propagation. (Some drugs thus enlivens the brain with a pleasurable play o f emotions.
subvert that survival instinct by stimulating dopamine release
on false pretences.) But why would a sequence of sounds with www.bbc.com. Adaptado (Vestibular U n ifesp 2014)

EXERC(CIOS - U N IFESP 2014 (adaptado}

1.Seg u n d o o texto, a pergu nta apresentada no p r i m e i ro z. O texto relaciona a m úsica às d rogas porq u e ambas:
parágrafo: a) liberam os instintos sexuais.
a) é intrigante e merece uma reflexão por parte de músicos b) dependem das preferências pessoais.
e psicólogos. c) incitam a euforia e criam dependência.
b) mostra que a música está relacionada à sobrevivência d) promovem a descarga de dopam i na.
do ser humano. e) ocorrem em contextos semelhantes.
c) introduz uma questão científica ainda não abordada.
d) indica que a música pode auxiliar em tratamentos para 3. Segu ndo Leonard Meyer:
depressão. a) a ansiedade e comportamentos violentos decorrem da
e) pode ser abordada a partir de diversas perspectivas. busca por recompensas.

141 GE IDIOMAS 1015


b) um desejo não atendido gera sensação de perigo e inse­ d} O texto não faz referência a depressão. Incorreta.
gurança. e) Conforme a frase "like most good q ues tions, this o n e works on
c) a música vai de encontro aos padrões do inconsciente. m a ny leveis" (Como a maioria das boas pergun tas, esta pode
d) uma expectativa confirmada gera bem-estar e emoções ser a bordada e m muitos níveis"). Correta.
agradáveis. Resposta: e
e) emoções dúbias como prazer e culpa resultam do consumo
de drogas, como o tabaco. z. O texto relaciona a música às drogas ao a firmar q u e a m bas pro­
movem a descarga d e dopa m ina. Veja o trecho ". . . like those we
4. No trec h o do quarto parágrafo "However, we now have experience from sex, good food and addictive drugs. Those rewards
many clues to why music provokes i ntense emotions", a come from a gush o f a n eurotransmitter called dopamine."(... como
palavra however i n d ica u m a ideia de: o q u e vivenciamos com s exo, boa comida e drogas. Estas recom­
a) avaliação. pensas vêm d e uma descarga d o n eurotransmissor dopamina).
b) expl icação. Resposta: d
c) consequência.
d) finalidade. 3. Segundo o texto, Leonard Meyer adotou teorias já existentes
e) contraste. pa ra explicar o bem-estar provocado pela música. Segundo e las,
quando nossas expectativas não se concre tizam, sen timos frus­
s. O trecho final do q uarto parágrafo - "the payoff is ali tração e raiva. Mas q u a n do são a te n d idas - seja no a m o r, s eja
the sweeter" - pode ser corretamente ente n d i d o como: n u m cigarro -, temos uma doce recompensa. Veja o trecho: ". . . if
a) a retribuição dá muito prazer. we then find what we're looking for; be it lo ve o r a ciga rette, the
b) a moderação vale a pena. payoff is ali the sweeter"
c) a compensação foi menor que a esperada. Resposta: d
d) a sensação de alívio é relaxante.
e) a frustração é substituída pelo amor. 4. Questão simples de voca bulário. "However" (entretan to, contu­
do) é u m marcador discursivo, que transmite a ide ia d e oposição,
6. No trecho do último parágrafo - as we'd now see it -, 'd con traste. Para isso, precisamos analisar a partir da frase anterior.
pode ser reescrito, mante n d o-se a correção e o sentido, (Veja mais sobre marcadores d i scu rsivos na pág i na 49 ).
como: Resposta: e
a) did.
b) had. s. Conforme mencionado na questão 3, "- .mas quando encontramos
c) would. o que procuramos (. . .) a recompensa é doce". "Payoff" significa
d) need to. retribuição. "Swee ter'' é a forma comparativa d e superioridade
e) used to. "mais doce". (Veja mais sobre c o m parativos na página 57)
Resposta: a
1· N o trecho d o ú l t i m o parágrafo - T h e con stant dance
between expectation and outcome thus enlivens the brai n 6.A con tração 'd só se refere a três palavras: "would", "should" ou
with a pleas u rable play of e motions. -, a palavra thus pode "had". Com isso, eliminamos as alternativas a, d e e. Resta analisar
ser corretamente su bstitu ída, mantendo-se o sentido, por: as alterna tivas b e c. Na expressão a n alisada, não podería mos
a) thereby. substituir por "had'; pois neste caso "had" seria u m verbo auxiliar
b) moreover. que exige o uso d e outro verbo n o particípio passado para compor
c) whereas. o passado perfeito. Por exemplo. "as we'd seen" (co m o nós tínha­
d) although. mos visto). já "would" também é u m verbo auxiliar, mas exige na
e) notwithstanding. sequência u m verbo n o infinitivo (s em a partícula "to'']. A frase
"as we would n o w se e it" significa, então "co m o agora vería mos".
(Veja o passado perfeito na pág i n a 33).
RESPOSTAS Resposta: c
L Analisando as alternativas:
a} Não há referências a opinião de músicos e psicólogos. Incorreta 7. fl/ovamente uma questão de vocabulário. "Thus" é um advérbio
b} Segundo o terceiro parágra fo, a música não parece associada à que pode ser substituído por "thereby", pois ambos indicam con­
sobrevivência do ser humano. Incorreta sequência (assim, dessa forma, então). O significado dos demais
c) A resposta não está explícita no texto, mas pode ser deduzida do termos apresentados na questão:
trecho "the current favourite theory among scientists (. . .) dates b} moreover: além disso
back to 1956 (.. .) Meyer drew on earlier psychologial theories o f c) whereas: enquanto, uma vez q u e
emotion. . . " (A teoria favoritd atual entre os cien tistas (. . .) remonta d) a l though. em bora
a 1956 (.. .) Meyer impulsionou as primeiras teorias psicológicas e) n o t withstanding: não obstan te, entretanto
sobre a em oção ... ). Incorreta. Resposta: a

GE IDIOMAS 20!S 1 15
I N T E R P R ETAÇÃO D E TEXTO

The magical qualities of guarana


Drink made with the Brazilian berry i s used as a stimulant ali over the country

uarana is a berry that grows in the Maues region of Northern Brazil that has been found to hold extraordinary properties for

G increased energy and stimulation. Its origins are found deep in the forests of the Amazon where it has been revered for centuries
by the Guarani tribe for its magical and medicinal qualities.
Overtime the popularity of the berry has grown (1) and it is now practically the national drink in Braz i!. As guarana becomes more and
more popular worldwide, as a main ingredient in energy drinks such as "Sobe" and "Rockstar", conflict has risen (2) between the local
producers and distributors of guarana over rights to the marketing of the berry. Multinational corporation that buy the guarana have
formed a "c arte!" that no longer has to compete for the guarana crops, which some claim has resulted in the ruin o f the local economy
and the livelihood o f guarana farmers [...]
The botanical name for guarana is Pau/linfa cupana. It is described as a climbing shrub and is named after C.F. Paullini, a German
medicai botanist who discovered the plant in the 18th century. The substance guaranine in guarana has traces (3) of alkaloids like
theophylline and theobromine that work as a stimulant to increase mental and physical alertness. It also increases the metabolic rate,
which can reduce appetite. The actual medicinal use of Guarana varies from easing headaches, diarrhea, exhaustion and some claims of
an aphrodisiac quality. Other reported benefits include hangover relief and a cure for menstrual-type headaches.

(www.american.edu/ted/guarana.htm, adaptado) (Vestibular Cefet·Al 2oo8)

EXERCÍCIOS - CEFET-AL 2008 (adaptado)

1. Marq ue a ú n ica alternativa falsa de acordo com o texto: J. De acordo com o texto, é correto afirmar que:
a) O nome científico do guaraná vem da tribo guarani, que há séculos a) O guaraná está se tornando famoso no mundo porque é exportado
venera a planta por seus poderes mágicos e medicinais. massivamente.
b) O guaraná é o principal ingrediente de algumas bebidas energéticas b) O guaraná está conquistando fama mundial como ingrediente de
industrializadas. bebidas energéticas.
c) C . F. Paullini, um b otânico alemão, descobriu o guaraná no c) O guaraná sempre foi uma bebida apreciada mundialmente.
século XVI I I. d) Os produtores de guaraná têm lucrado com a internacionalização
d) O guaranina, su bstância presente no guaraná, acelera o da bebida.
metabolismo, o que pode auxiliar em dietas de emagrecimento. e) O cartel formado por corporações multinacionais está contribuin·
e) Além de ajudar na melhora de dores de cabeça, o uso medicinal do do para a melhoria das condi ções dos produtores de guaraná.
guaraná também é indicado para aliviar ressacas.

:z. Marque a ú nica alternativa incorreta de acordo com o texto: RESPOSTAS


a) No trecho "Overtime the popularity o f the berry has grown and it is 1. O nome cientifico do guaraná vem da tribo Guarani, que há séculos
now practically the national drink in Brazil", o pronome it refere-se venera a planta por seus poderes mágicos e medicinais. Cuidado com
ao termo the berry, que, por sua vez, se refere ao termo guarana, a questão, que p ede para assinalar a alternativa falsa. No texto, a infor­
citado anteriormente. mação é q u e o nome cien tífico do guaraná (Pau llinia cupana) deriva do
b) No trecho: "... a German medica/ botanist who discovered the plant nome do cientista C. F. Pauilini. Resposta: a
in the 18th century", segundo a norma gramatical da língua inglesa,
o termo who poderia ser substituído pelo termo that. z. No trecho: "Guarana is a berry that grows in the Maues region o f Nor­
c) No trecho: "Guarana is a berry that grows in the Maues region o f thern Brazil", segundo a norma gramatical da língua inglesa, o termo
Northern Brazil", segundo a norma gramatical da língua inglesa, "that" poderia ser substituído pelo termo "who". O pronome relativo
o termo that poderia ser substituído pelo termo who. "who" é usado para se referir a pessoas anteriormente citadas. Para lugares
d) N o trecho: "The substance guaranine in guarana has traces o f alka­ e objetos usamos o pronome "whích". Na frase em questão, o pronome
/oids like theophylline and theobromine", o termo like poderia ser "that" refere-se à palavra "berry" (a fruta, o guaraná). Assim, "that" poderia
substituído pela expressão such as, sem prejuízo de sentido. ser substituído por "which': mas não por "who ". Resposta: c
e) Acerca do termo has, usado nas três expressões sublinhadas e
enumeradas no texto, é correto afirmar que em (1) e (2) o termo 3. O guaraná está conquistando fama mundial como ingrediente de
tem a função de verbo auxiliar, e em (3) funciona como o verbo bebidas energéticas. Veja o trecho: "As guarana becomes more and more
principal da oração. popular worldwide, as a ma in ingredient in energy drinks". (Conforme o
guaraná s e toma cada vez mais popular no mundo, como o ingrediente
principal em bebidas energ é t icas .) Resposta: b

16 1 GE IOIOMAS 2015
. ,
_- : . ,' --_._ .

·
PRESENT CONTI N U OUS » Q UAN DO O G E RÚ N D IO ESTÁ L I B E RADO

Conheça as situações que exigem


o uso do presente contínuo

Em i nglês, o presente contínuo é usado para: ESTUDANDO CONJ UGAÇÃO


» Indicar ações q u e estão em progresso no momento em q u e Veja como estruturar este tempo verbal.
falamos sobre elas. Conj ugação d o verbo "to speak" (falar).
Ex.: The training that students are receiving is good. (A
formação que os alunos estão recebendo é boa.)
Silvia is having a shower now. She can't talk to you. (Silvia
está tomando ban ho agora. Ela não pode falar com você.)
» Falar de ações q u e são tem porárias.

Ex.: I usual/y start working at 7 but today l'm starting at 10.


(Eu geral m ente começo a trabal har às 7, mas hoje estou
começand o às 10.)
» Falar sobre compromissos agendados para o futuro ("future

arrangements").
Ex.: Sorry, but l'm seeing the doctor tomorrow at 3 p.m. l can
seeyou /ater, after 4. (Descu l pe, mas verei o médico amanhã
às 3 da tarde. Posso vê·lo mais tarde, depois das 4.)

Por ser um tem po com posto, a conjugação d o prese n te


contínuo se dá com a com bi nação de dois verbos: formas do
presente d o verbo to be (am, is, are) e da forma d o particípio
presente (ou gerúndio) do verbo principal (infinitivo acrescido
do sufixo -ing).
Ex.: You are studying now, Roger. Turn off the TV. (Você está COMPLEMENTOS DO VERBO
estudando agora, Roger. Desligue a TV.) Conheça os pronomes do caso reto/oblíquo e os adjetivos
possessivos
Na forma negativa, usamos as formas am not, is not, are Em i nglês, os pronomes d o caso reto são usados na função
not antes do verbo pri ncipal no gerú ndio. Na forma interro­ d e sujeito d e u ma frase e os d o caso o b l fq u o servem como
gativa, posicionamos o verbo to be antes do sujeito d a frase. complementos d o verbo. Já os adjetivos possessivos acom­
Informalmente, usam-se formas contraídas entre sujeitos e panham os su bstantivos e i n dicam, em geral, uma relação de
verbos (/'m, He's, They're) e na forma negativa entre as formas posse. Observe os exemplos:
is/are e not (isn't, aren't). My na me is Luana and Lucy is my sister. She is 24 years old
Ex.: ls Sheila sleeping at this time? Wake her up! (Sheila está and is married with two kids. Their names are Cintia and Luis.
dorm i n d o a essa hora? Acorde-a!) 1 /ove them very much. (Meu nome é Luana, e Lucy é m i nha
The students aren't complaining about the class. They're com­ i rmã. Ela tem 24 anos e é casada, com d o i s fil hos. Os n o m es
plaining about the book. (Os estudantes não estão reclamando deles são Ci ntia e Luis. Eu os amo m u i to.)
da aula. Eles estão reclamando do l ivro.)
Expressões de tem po c o m u mente usadas com o presente
contín uo:
» Now/at this moment/right now

» just/sti/1/this year/summer/month

GE IDIOMAS 2015 1 17
I N T E R P R ETAÇÃO D E TEXTO

Arts and entertainment


New art languages are appreciated by a higher number of fans every year.
Even the not-so-new ones, such as comics

.IIIII IN THE LATE 18oos, a new cultural form was characterized by narrative told in a sequence of pictures or drawings, with continuing casts of characters

W
hether in the Sunday paper o r in a critically acclaimed 20's, Walt Disney (1901-1966) created a very influential world with
graphic novel, co mies have become a distinctive American Míckey Mouse and Donald Duck. Using his childhood insecurities
art form. Comic strips, comic books, and the characters that and failures as material, Charles Schulz (1922-2000) was the writer
people them, are reflections of the country's culture from the end and artist of the incomparable Peanuts, which became the most
of the 19th century to the present. widely read comic strip in the world, inspiring animated cartoons,
In the late 1800s, many conditions seemed ripe for the arrival of a new toys, and reprint books.
form of communication that was neither merely literature nor merely Gary Trudeau's (1948- ) Doonesbury made him the most prominent
graphic art. New and more advanced printing presses were allowing cartoonist-commentator on the política! scene during the 1970s,
newspapers to print more copies, better and faster. Also at that time, bringing him the 1975 Pulitzer Prize, the first ever awarded for
the enormous influx of new immigrants from eastern and southern a comic strip. Calvín and Hobbes is written and drawn by former
Europe, with little or no knowledge of the English language, gave the política! cartoonist Bill Watterson (1958- ), who is known for
medium of visual communication a steady audience. The new cultural having his characters, a manic six-year-old and a levelheaded tiger,
form was characterized by narrative told in a sequence of pictures, with make abrupt mid-strip shifts from fantasy to reality, and from one
continuing casts of characters, and dialogue or text within the picture character's viewpoint to another.
frame. At this point, a new distinction was created that separates most In the approximately 110 year-long-life of comics, they have
comic strips from the pictorial narratives of previous centuries. Comi c encompassed every aspect of American life, from the down-to­
strips were designed to compel the eye to travei forward from pane! earth to the esoteric. Today, due to cutbacks on space for continuity
to pane!, whereas earlier drawings were static and mainly served as strips in newspapers, artists no longer have as much size available
illustrations for text. This new, kinetic, dimension of American comic to include extensive detail in their work. The impact that the arrival
art was a major departure from the cartoons created at that time in other of the compute r age will have on comic artistry remains to be seen.
parts of the world. Many experts designate the "birthdate" ofAmerican Clearly, animation of cartoon characters is making a comeback in
comics as 1895, when the Yellow Kíd first appeared. movies and on digital entertainment websites. Fans o f "the funnies"
Among the many comics artists, one of the earliest was Richard will be waiting to see.
Outcault (1863-1928), who created two o f the field's important
characters, the Yellow Kíd (1895) and Buster Brown (1902). In the late (Vestibular Uerj 2009, www.bpib.com, adaptado)

18 1 GE IDIOMAS 201S
EXERCfCIOS - UERJ 2009 (adaptado)

L Leia o texto na pági na ante rior e assinale a alternativa d) Os q uad r i n h os, com Yellow Kid, nos Estad os U n i d os.
correta. e) O trabal ho d e Charles Schu lz.
Duas situações fundamentais para o recon hecime nto dos
quadrinhos como manifestação artística foram:
a) O s u rg i m e n to dos desen hos ani mados e o a u m en to d o RESPOSTAS
públ ico leitor d e jornais. 1. Os avanços na tecnologia da impressão gráfica e a entrada de
b) A entrada d e i m i g rantes e u ro p e u s e o s u rg i m e nto d os imigrantes europeus. A resposta correta está n o trecho "New a n d
desenhos ani mados. m ore advanced printing presses were allowing n e wspapers t o
c) O aumento d o p ú b l ico leitor d e jornais e os avanços na print more copies, better and fas ter, making it possible to easily
tecnologia d a i m pressão gráfica. reach an ever-increasing public. Ais o at t h a t time, the enormous
d) Os avanços na tecnologia da i m p ressão gráfica e a entrada in flux of n ew immigra n ts from eastern and southern Europe (. ..)
de i m igrantes e u ropeus. gave the medi um of visual comm u n ication a steady a udience".
e) O s u rg i m e n to dos dese n h os ani m ados e os avanços na {Impressoras n o vas e mais avançadas permitiam q u e os jornais
tecnologia d a i m p ressão gráfica. imprimissem mais exemplares, de forma melhor e mais rápida,
tornando possível alcançar facilmente um público em expansão.
1. A partir da leitura d o texto podemos afirmar que Charles Também nessa época, um grande fluxo de n o vos imigra n tes do
Schulz: leste e do sul da Europa ( . .) deu a o meio de comunicação visual
a) É famoso por ter criado Yellow Kid. um público fixo.) Resposta: d
b ) E s c r e v e u h i s t ó r i a s e m q u ad r i n h o s d e s u c e s s o
internac i o nal. z. Escreveu histórias em quadrinhos de sucesso internacional.
c) Tem por i ns p i ração o cenário pol ítico de seu país. A resposta está no trecho "... which became the most widely read
d) Mistu ra realidade e fantasia em suas h istórias. comic strip in the world, inspiring animated cartoons, toys, a n d
e) Foi o p i o n e i ro na p u b l icação d e q uad r i n h o s em j ornais. reprint books". ( . . Qu e se tornou a tirinha m a i s lida n o m u ndo,
inspirando desenhos animados, brinquedos e a reedição de livros.)
3. Na frase "Gary Trudeau's (1948- ) Doonesbury made h i m Uma dica é observar que Sclwlz m orreu em 2000 e q u e somente as
the most prominent cartoonist-commentator o n the political a lternativas (b) e (e) apresentam verbos n o passado. Resposta: b
scene d u ri n g the 1970s", a palavra destacada é exemplo de
um verbo: 3. /rregu/ar no past simple. A forma "ma de" é passado do verbo
a) I rregular n o past s i m ple. irregular "make". Resposta: a
b) Reg u lar n o past perfect.
c) I rregular no past perfect. 4- Comics - art - cartoonist - narra tive. A única a l ternativa q u e
d) Regular n o past s i m p l e. apresenta explicita m e n te pala vras relacionadas a o m u n do das
e) I rregu lar no present perfect. histórias em quadrinhos é a letra (C): "comics" (quadrinhos), "art"
(arte), "cartoonist" (cartunista) e "narra tive" (história). Resposta: c
4- As palavras q u e estão associadas ao assu nto do texto
são: s. Exclusão. A expressão "neither ... no r" (nem ... nem) indica exclusão,
a) comics - eye - cartoonist - s u n day. o u seja, "nem Lima coisa nem ou tra". A tradução da frase é: urna
b) cartoo n ist - c h i l d h ood - s u nday - c o m i cs. nova forma de comunicação q u e não eram n em apenas litera tu ra
c) comics - art - cartoonist - narrative. nem somente arte gráfica . Resposta: b
d) narrative - eye - knowledge - s u n day.
e) art - s u nday - c h i d h ood - c o m i cs. 6. Os quadrinhos, com Yellow Kid, nos Estados Unidos. A resposta
surge em dois trechos. "In the late 18oos, rnany conditions seemed
5- No trecho " ... a new form of communication that was neither ripe for the arrival o f a new form o f cornrnunication" (no fim dos
merely l iterature n o r merely graphic art...", a expressão em anos 18oo, m uitas condições pareciam propícias para a chegada
destaque ind ica a ideia de: de uma nova forma de comun icação) e "Many experts designate
a) ad ição. the 'birthdate' of American co mies as 1 895, when the Yellow K i d
b) exc l u s ão. first appeared" (muitos especialistas designam o nascim e n t o
c)' consequência. dos quadrinhos americanos em 1895, quando Ye l l o w K i d su rgiu).
d) concessão. Resposta: d
e) reforço.

6. De acordo com o texto, no fim do século 19, surgiu (iram):


a) O Prêmio P u l i tzer, nos Estados U n i dos.
b) Os q uad r i n h os, com Yel/ow Kid, n o Leste E u ropeu.
c) Os q uad r i n h os, com Doonesbury, nos Estados U n id os.

GE IDIOMAS 201S 1 19
I NT E R P RETAÇÃO D E TEXTO

The risk of
EXERCICIOS

1. 1t is true to assert that:


a) Both the teen s and t h e e l derly have t h e lowest accident

distraction
rates.
b) Both the teen s and the e l d e rly h ave t h e h i ghest acc i d e n t
rates.
c) Midd le-aged d rivers cause more accidents than teenagers.
Cell phones use while driving a n d alcohol d) Middle-aged d rivers cause more accidents than the elderly.
e) The elderly cause many m o re accidents than teen agers.
causes most of the car accidents
L One can argue that middle-aged d rivers d rive more:
a) recklessly.

D
river distractions are the major causes of car accidents.
They include cell phones and other kinds o f gadgets b) aggressively.
(laptops or electric razors, for example) used by drivers c) dangerou sly.
as they drive. In response to this problem some states in d) cautious ly.
the US have banned the use of hand-held cell phones while e) i m patie ntly.
people are driving. Alcoholic drinks may be also considered
a distraction. That's why anyone with .01 ofblood alcohol is ).Assinale a alternativa que preenche corretamente a lacuna
considered unfit to drive. Other potentially dangerous habits da frase a seguir:
are rubbernecking and tailgating. The first one occurs when H e stopped ... only after the doctor said h e was going to die.
drivers slow down to look at another accident on the road or a) d rank.
anything that looks unusual. That's dangerous beca use the b) dri n k.
driver in the car behind often doesn't have enough time to c) d r i n ks.
slow down. The same applies for tailgating in that a driver d) d ri n ki n g.
who is following a car too closely may not have enough e) to d r i n k.
distance to stop suddenly.

(http://ezinearticles.com/?Top·Causes·of·Car·Accidents&id=302163, adaptado) RESPOSTAS


L 8oth the teens and the elderly have the highest accident rates.
A alternativa indica que "tanto adolescen tes quanto idosos têm
os índices de acidentes mais altos".
Resposta: b

z. Cautiousty. Ou seja, cautelosamente, com precaução.


Resposta: d

3- Drinking. Na frase, temos a indicação de um hábito que foi in·


terrompido, e, nesse caso, o verbo "stop" deve ser acompanhado
de u m verbo no gerúndio. Em português, temos "parar de" fazer
alguma coisa.
Resposta: d

20 I GE IDIOMAS 2015
,·:Ji:� VOCA B U LÁRIO » EXPRESSÕES Q U E CON FU N D EM

Saiba o que são as palavras


cognatas e os falsos cognatos

Uma das estratégias q u e mais usamos d u rante a leitura de PARECE, MAS NÃO t
um texto e m l ín gua estrangeira é a i dentificação de palavras Conheça os falsos cognatos que mais
cognatas, ou seja, aq uelas q u e são pareci d as com palavras confundem na relação inglês/português.
em português e q ue têm o mesmo sign ificado. Na leitura d o
texto da pági na 14, você deve t e r reco n hecido expressões
como "favorite theory", "intense emotions" e "magnetic
ressonance", entre o utras. É possível estabelecer a relação
entre as i d eias dessas expressões, pela s e m e l hança com o
português: "teoria favorita", "emoções i n te nsas" e resso­
nância magnética".

Em inglês, há muitas palavras de origem latina que permitem


fazer esse recon hecimento. Elas são consideradas, em geral,
mais formais, e por isso podem aparecer com frequência e m
textos como o q u e você acabou d e ler (retirado d e u m a revista
médica, em q u e a l i n guagem é normalme nte formal).

C o n t u d o, n e m s e m p r e p o d e m o s r e c o r r e r a e s s a
estratégia s e m pensar nas relações d e sentido q u e as pa­
lavras estab e l e c e m u m as c o m as o utras, p r i n c i pal m ente
porq u e podemos ser e nganados pelos falsos cognatos - pa­
lavras q u e parecem com as da língua portuguesa, mas têm
significados d iferentes. Por exem plo, reconhecer a palavra
physician não como "físico", m as como "médico", faz toda
a d i fe re n ça!

Outros exe m plos de falsos cognatos são as expressões


"college students" e "journal o f medicine". A palavra college
refere-se a facu l dade (e não colégio) e journal, n este contex­
to, não s i g n i fi ca j ornal, mas revista, d iário, periódico. Vale a
pena lembrar também q ue, às vezes, as palavras são cognatas
em um contexto de uso, m as em outro não. U m exem p l o é
a palavra affection, q u e s i g n i fica "afeição" (cognata), mas
tam bém "doença" o u "morbidez".

GE IDIOMAS 2015 1 21
I NTERPRETAÇAO DE TEXTO

The future of
the job market
Speaking at least one foreign language is
definitely an ability that may provide more
opportunities - here or in lndia

.11111 TRANSPORT authorities in lndia want cab


drivers to achieve a 2,ooo-word vocabulary in
English. A driver is the first person a tourist
interacts with on arrival in a city

N
ew Delhi taxi and rickshaw drivers are to be given a English proficiency exam each year. In the first year, a driver is expected
compulsory crash course in the English language to prepare to have learned 20 per cent of the vocabulary and be able to reply to
them for the influx of tourists that will arrive in the city for questions such as, "Where can I get a tourist map of the city?" In the
the next Commonwealth Games. According to the Times ofIndia, second year, the driver should know 35 percent o f the words. By the
transport authorities want the cab drivers to achieve a 2,000-word third year the drivers are expected to have learned 50 per cent of the
vocabulary by the time the games start in 2010. "The programme words. The transport department says that by the Commonwealth
is aimed at the cabbie or auto [rickshaw] driver because he is the Games in four years, drivers will be able to converse confidently.
first person a passenger interacts with on arrival in the city;• said To help drivers the department is to provide special centres where
transport commissioner VS Madan. "That's whywe want drivers to drivers will be able to use audio visual aids to help them become
converse properly and not in sign language:' he added. proficient. Audio tapes will also be provided so that the drivers will
Under the programme, drivers will be taught English relevant to be able to listen in between fares.
the needs o f tourists. Bythe end o f their course drivers should be able
to give directions and suggest good restaurants, among other useful (EL Gazette. English Language]ourna� Opening DoorsAcross the World. lssue NR 319, ]uly, 2006)
tourist advice. The transport department will give the taxi drivers an (Vestibular UFMT 2008)

EXERC(CIOS - U FMT 2008 (ADAPTADO)

L Sobre a i ntencional idade do texto acima, assi nale a afir­ 2. Em relação às capacidades d e uso do i nglês exigidas dos
mativa correta. taxistas ao final d o c urso, considere:
a) Esclarecer aos taxistas de N ova Dél h i q uanto à m e l h o r L S u gerir bons restau rantes aos turistas.
f o r m a d e se tornar proficiente e m i nglês. 1 1 . Oferecer conse l h os úteis aos turistas.
b) I n d i car p rogramas d e c u rsos de i n gl ês e m Nova Dél h i q u e li L I nformar d i reções aos turistas.
sejam rel evantes para o s taxistas. IV. Responder a pergu ntas como: "Onde posso conseg u i r u m
c) I nformar q u e os taxi stas de N ova D é l h i fariío c u rso d e mapa d a c idade?".
i nglês para m e l ho r receber o s t u ristas q u e virão aos j ogos
C o m m o nwealth. São capacidades exigidas
d) Criti car a necessidade d e os taxistas d e N ova Délhi apren­ a) li e 111, apenas.
derem i n glês. b) I, 11 e 111, apenas.
e) I n ce n t ivar os taxistas d e N ova Dél h i para que estud e m i n­ c) 1 1, 111 e IV, apenas.
glês para recepcio nar turistas que vi rão aos jogos Common­ d) I, 1 1; 111 e IV.
wealth daq u i a q uatro anos. e) I, 1 1 1 e IV, apenas.

22 1 GE IDIOMAS 201S
3-A fala "That's why we want drivers to converse properlyand RESPOSTAS
not in sign language" (final do 1° parágrafo) consiste em uma: l.lnformar que os taxistas de Nova Délhi farão curso de inglês para
a) iron ia. melhor receber os turistas que virão aos jogos Commonwealth.
b) ped ido. O texto é bastante descritivo e dá informações sobre o tipo de
c) exp l i cação. curso ("compulsory", ou seja, obrigató rio) que os taxistas vão
d) questionamento. fazer em Nova Délhi, in dia. Não trata explicitamen te sobre a me­
e) advertê ncia. lhor forma de se tomar proficiente em inglês (alternativa a) nem
dos programas relevan tes (alternativa b). Também não faz crítica
4. No texto, as expressões by the time e By the end (1° e 2° (alternativa d) nem incen tiva diretamente os taxistas a estudar
parágrafos) estabelecem relações de: inglês (a lterna tiva e).
a) tem poral i dade. Resposta: c
b) causal i dade.
c) seq uên cia. z. I, 11, 111 e IV. Todas as capacidades são mencionadas no texto: "By
d) con cessão. the end o f their course drivers should be able to give directions and
e) perm issão. suggest good restauran ts, among other usefui tourist advice". (Ao
final do curso, os motoristas deverão ser capazes de dar direção
s. Em relação aos recursos lingu ísticos utilizados no texto, e sugerir bons res tauran tes, den tre outras sugestões/ conselhos
assi nale a afirmativa correta. úteis para turistas.) Responder a perguntas específicas (item I V)
a) Cab drivers e taxi drivers são expressões s i n ô n i mas. está no trecho "be able to reply to questions such as, 'Where can
b) O vocá b u l o compulsory é um falso cognato. f get a tourist map o f the city?"'. (Ser capaz de responder a pergun­
c) O verbo to converse s i g n ifica converter. tas como "Onde posso conseguir um mapa turístico da cidade?".)
d) Them (3• l i n ha do 1° parágrafo) refere-se a tourists. Resposta: d
e) 2,ooo-word é um su bstantivo.
3. Explicação. A expressão "Tha t's why" (Esse é o motivo/a razão)
6. Sobre o programa referido n o texto, assi nale V para as inicia uma explicação. "That's why we want drivers to con verse
afirmativas verdadeiras e F para as falsas. p roperly and not in sign language." (Esse é o motivo pelo qual nós
( ) Até 2010, a cada ano os taxistas terão de fazer um teste queremos que os motoristas conversem apropriadamen te, e não
de proficiência em i n glês. em linguagem de sinais.)
) Ao final do terceiro ano d o cu rso, os taxistas deverão ter Resposta: c
apre n d i d o 2000 palavras e m i n g l ês
.

) Os tax i stas c o n tarão c o m recu rsos a u d i o v i s u a i s para ft. Temporalidade. "By the time/by the end" são expressões q ue
apre n d e r i nglês e m centros especiais. indicam o perlodo em que uma atividade deve ter sido concluída
( ) Os taxistas poderão aprender i nglês no próprio veícu l o no fu t u ro . Por isso, indicam relações de tempo ra lida de: "by the
enq uanto aguardam passagei ros. time" com o significado de "antes" e "by the end" como "ao final".
Resposta: a
Assinale a seq uência correta.
a) V, V, F, F. s. Cab drivers e taxi drivers são expressões sinônimas. A ú n ica
b) V, F, V, V. alternativa correta é a que estabelece "cab" e "taxi" como sinôni­
c) F, V, F, V. mos. Na opção (b), a palavra "compulsory" significa compulsório (é
d) F, V, V, F. cogna ta); na alternativa (c), o verbo "converse'·' significa conversar;
e) V, F, F, V. e em (d) o pronome "them" se relaciona a "drivers" (motoristas),
e não a turistas; na alternativa (e), a expressão "2,ooo-word" tem
função de a djetivo.
Resposta: a

6. Verdadeiro, Falso, Verdadeiro, Verdadeiro. A segunda frase é a


única falsa, pois a expecta tiva das a u toridades é que os taxistas
alcancem um vocabulário de 2 mil pala vras ao final dos qua tro
anos de curso.
Resposta: b

GE IDIOMAS 2015 1 23
INTERPRETAÇAO D E TEXTO

They occupied the streets


A wave of anger is sweeping the cities of the world

T
he protests have many different origins. In Brazil people rose up against
bus fares, in Turkey against a building project. Indonesians have rejected
higher fuel prices. In the euro zone they march against austerity, and
the Arab spring has become a perma-protest against pretty much everything.
Yet just as in 1848, 1968 and 1989, when people also found a collective
voice, the demonstrators have much in common. In one country after
another, protesters have risen up with bewildering speed. They tend to be
ordinary, middle-class people, not lobbies with lists of demands. Their mix
of revelry and rage condemns the corruption, inefficiency and arrogance
of the folk in charge.
Nobody can know how 2013 will change the world - i f at ali. In 1989 the
Soviet empire teetered and fel!. But Marx's belief that 1848 was the first
wave o f a proletarian revolution was confounded by decades of flourishing
capitalism and 1968 did more to change sex than politics. Even now, though,
the incho ate significance of 2013 is discernible. And for politicians who want
to peddle the same old stuff, news is not good.

The Economlst, ]une 29, 2013. Adaptado. (Fuvest 2014, adaptado)

EXERCfCIOS - FUVEST 2014 (adaptado)

LSegundo o texto, os protestos de 2013, em diversos lugares do m undo, d) Ao con trário do proposto na alternativa, o texto afirma que os protestos
a) vêm perd e n d o força por fal has de o rgan ização. são motivados por variados tipos de reivindicação. Incorreta.
b) q u estionam a atuação dos /obbies nas rei v i n d icações das d iver· e) Novamen te, o texto in forma o con trário do q u e propõe a alternativa.
sas classes sociais. Veja o trecho. ". . . the demonstra tors h ave much in common" (os mani·
c) conde nam a corru pção e ou tros com po rtamentos i nadequados festantes têm muito em comum). Incorreta.
da c l asse p o l íti ca. Resposta: c
d) resu ltam de motivações eco n ô m i cas precisas.
e) têm p o u cos aspectos em com u m . 2. Analisando as alternativas:
a) Ao con trário do q u e a alternativa propõe, os protestos são por anseios
2. A o com parar o s protestos d e 2013 c o m movi mentos p o l íticos coletivos. O trech o "they tend to b e ordinary, middle·cfass people, not
_
passados, afirma-se, no texto, que: lobbies with lists o f demands" pode ser traduzido por ''são, em sua maior
a) n e m sem pre esses movi m e ntos expressam anseios col etivos. parte, pessoas comuns, de classe média, e não l o bbies com demandas
b) as crenças d e Marx se confirmaram, mesmo após 1848. específicas). Outro trecho que confirma essa ideia está em "Yetjust as
c) as revoltas de 1968 causaram grandes m u d an ças pol íticas. in 1848, 1968 and 1989, when people ais o found a collective voice. .. " (do
d) não se sabe se os p rotestos d e 2013 m udarão o m u n do. mesmo modo, tanto quanto em 1848, 1968 e 1989, quando as pessoas
e) m u danças d e costu mes foram as p r i n c i pais con seq u ê nc ias de também encon traram uma voz coletiva ... ". Incorreta.
movimentos passad os. b) A menção a Marx está n o trecho "b u t Marx's belief that 1848 was t h e
first w a ve of a proletarian revolution w a s confounded b y decades o f
RESPOSTAS fiourishing capitalism ... ". E significa "mas a crença d e Marx d e q u e
l. A g uestão exige, fundamentalmente, repertório de vocabulá rio. Ana· 1848 era a primeira onda de uma revolução proletária f o i frustrada
lisando cada alternativa: por décadas de capita lismo floresce n t e. . . " Incorreta.
a) O texto não cita nada sobre o r·ga nização. Fala apenas dos m o tivos e c) O trecho é claro: "... and 1968 did more to change sex than politics" (1968 fez
das possíveis consequências dos protestos. Incorreta. mais, principalmen te, por mudanças sexuais do que políticas}. Incorreta.
b) O texto fala em "lobbles". Mas apenas para citar que os protestos não d) A frase "nobody can know h o w 2013 wi/1 change the world" (ninguém
são organizados por esse tipo de organização. Incorreta. pode saber agora como 2013 mudará o m u ndo), confirma a proposição
c) A frase "their mix o f revelry a n d rage condemns the corrup tion, lneffi· da a lterna tiva. Correta.
clency and a rrogan c e o f the folk In charge" pode ser traduzida como e) Não há nenhum trecho no texto que confirme a ideia de q u e os movi·
"sua mistura de a lgazarra e raiva condena a corrupção, a in e ficiência m e n tos pássados alteraram algum costume. Incorreta.
e a a rrogância das pessoas que estão no comando". Correta. Resposta: d

24 1 GE IDIOMAS 2015
DICAS VE RBOS MODAIS » CO M P LETANDO O S E N T I DO

Os verbos modais'I como ''must" e "can"J


indicam o modo de interpretar uma frase

E m i ng l ês, h á três gru pos d e verbos: os reg u l ares (q u e » Must para e xp ressar reco m e n d ação/c o n s e l h o i n c i sivo,
representam ações ou estados), os auxil iares (q u e, além d e o b rigação o u p ro b a b i l idade de grau alto.
poder ser regu lares, desempen ham u m a função estrutu ral Ex.: You real/y must see a doctor. You are losing to o much
nas frases, i n dicando os tempos verbais e as formas negativa weight! (Vo c ê rea l m en t e p re c i s a v e r u m m é d i c o . Está
e i nterrogativa) e os verbos modais ( q u e alteram ou com­ perdendo m u ito peso!)
pletam o sentido d o verbo p r i n c i pal). Os verbos regu lares Ali passengers must wear a seat belt during the flight.
são a maioria e m i n glês (por exe m pl o, know, swim, speak (Todos os passag e i ros devem u sar o c i nto d e segu rança
etc.), e os verbos auxi l iares são be, have e do, por exem p l o. d u rante o voo.)
Os verbos m odais são os segu i ntes: shou/d, ought to, must,
may, might, can, could. No p resen te/futu ro, o s v e r b o s m o d a i s têm a segu i n t e
Os verbos modais i n d i cam id eias q u e dete r m i nam como estrut u ração:
os verbos a e l e relacionados devem ser compree n d idos, o u 1. Os verbos modais são acom panhados do verbo p r i n c i pal
seja, i n d icam o " m o d o " d e i nterpretação q u e o falante q u e r na forma do i n f i n itivo sem a partíc u la to (exceção para o
dar para o verbo princi pal, d a í s e u nome (verbo modal). Veja verbo ought to).
os exem pl os: Ex.: Such items may become public and cou/d subject you
He shou/d study m ore ( e l e deveria e s t u d a r m ais): é u m to ... (Tais itens pode m se tornar p ú b l i cos e p o d e m fazer
consel h o q u e d o u . com q u e você...)
He can study more (ele consegue estudar m ai s): eu sei q u e
e l e t e m capacidade m e n tal para i sso. 2. A forma verbal é sempre a mesma para todos os p ronomes
He may study more (ele pode estudar mais): e u dou permissão (1, you, h e, she, it, we, you, they).
para q u e e l e faça isso. Ex.: Suzie must know the answer. She always studies hard
for exams. (Suzie deve saber a resposta. Ela sem pre estuda
Usamos: m u ito para as p rovas.)
»Should/ought to para expressar recomendação ou conselho. They might go to the beach or s tay home this summer.
They don't kno w yet. ( E l e s p o d e m i r à p raia o u ficar e m
·
» Can para expressar capacidade física/mental, poss i b i l i dade casa n este verão. E l es não sabem ainda.)
ou ainda permissão (em registro i nformal).
Ex.: H e can carry these bags. He's strong e n o ugh, 3. A forma n egativa estrutu ra-se c o m verbo m o d al + not,

don't worry. ( E l e pode carregar essas malas. E l e é forte o havendo tam bém formas contraídas.
bastante, não se p reocu pe.) Ex.: I shou/d not talk to you, but I cannot he/p it. (Eu não ·
d evia falar com você, mas não consigo evitar.)
» Can/could para exp ressar pedidos.
Ex.: Can I come in, Mrs Sanderson? (Posso entrar, sen h o ra 4. A forma i n terrogativa é estrutu rada posicionando o verbo
Sanderson?) modal antes d o sujeito ao qual ele se relac i o n a:
Could I have a coke, p/ease? (Você pode me ver u m a coca, Ex.: May 1 help you? (Posso l h e aj u dar?)
por favor?) Can you say the a/phabet backwards? (Você consegue d izer
o alfabeto d e trás para frente?)
» May para expressar poss i b i l idade/pro bab i l idade (de grau
médio) sobre o futu ro e might para i n d icar possi b i l id ad e/ CONTRAÇÃO DE PALAVRAS
probab i l idade de grau baixo. As formas n egativas contraídas dos verbos modais são:
Ex.: He maycome with us on this trip, but Iam not certain. (Ele
talvez venha conosco na viagem, mas não tenho certeza.) can + not can't (cannot)
=

(Garçom em um restaurante): May I help you, sir? (Posso should + not shouldn't
=

aj udá-lo, s e n h o r?)
might + not mightn't
=

I invited him to see the show, but he's very busy tonight.
must + not mustn't
=
He said he might go with us, though. ( E u o convidei para
ver o show, mas e l e está m u ito ocupado hoje à noite. Mas ought + not oughtn't
=

d isse q u e talvez vá conosco.) A expressão must not (mustn't) indica uma proibição.

GE IDIOMAS 2015 1 25
I NTERPRETAÇAO D E TEXTO

Climate change: warm


words and cool waters
There is a serious debate about why observed temperatures have
not kept pace with computer-modelled predictions

..11111 WARM AND WET Global warming is pointed as a factor that will increase floods, as this one that occurred in 2009 in Manila, Philippines

L
ast week's report that the current "pause" in global warming could There is, however, a serious debate about why the observed
be linked to cyclic cooling in the Pacific will be interpreted by temperatures have not kept pace with computer-modelled predic­
climate sceptics as evidence that global warming isn't happening, tions and where the heat that should have registered on the global
and by politicians as a reason to forget about climate change and carry thermometer has hidden itself. One guess - supported by some
on with business as usual. Both responses would be dangerouslywrong. sustained but still incomplete research - is that the deep oceans
There is no serious argument within climate science about the are warming: that is, the extra heat that should be measurable in
link.between carbon dioxide leveis and temperature. Between 1970 the atmosphere has been absorbed by the se a. This is hardly good
and 1998 the planet warmed at an average of 0,17C per decade, and news: atmosphere and ocean play on each other, and any stored
from 1998 to 2012 at 0,04C per decade. C arbon dioxide leveis in the heat is _ _
I to be returned to the atmosphere sooner or !ater, in
atmosphere, however, continued to rise and are now higher than at unpredictable ways. The real lesson from the latest finding is that
any time in the last 800,000 years. Twelve of the 14 warmest years there is a lot yet to be understood about how the planet works, and
on record have occurred since 2000; the last two years have been precisely how ocean and atmosphere distribute 1 1_ from the
_ _

marked by catastrophic floods in Australia and record-breaking tem­ equator to the poles.
peratures in the US; and the loss of north polar ice has accelerated
at such a rate that climate modellers expect the Arctic Ocean to be The Guardian, September 1, 2013 (www.theguardian.com. Adaptado.)
routinely ice-free in September after 2040. (Vestibular Unifesp 2014, adaptado)

26 1 GE IDIOMAS 2015
' ' ' ' ' ' ' ' ' ' ' ' ' ' ' ' ' ' ' ' ' ' ' ' ' ' ' ' ' ' ' ' ' ' ' ' ' ' ' ' ' ' ' ' ' ' ' ' ' ' ' ' ' ' ' ' ' ' ' ' ' ' ' ' ' ' ' ' ' ' ' ' ' ' ·' ' ' ' ' ' ' ' ' ' ' ' ' ' ' ' ' ' ' ' ' ' ' ' ' ' ' ' ' ' ' ' ' ' ' ' ' ' ' ' ' ' ' 0 • • • • • • • • • • • � • · • • • • • • • • o • • • • • • • • • • • • • • • • • • • • • • • • • • • • • • • • • •. • • • • • • • • O • • • • •• • • • O • • • • • � • • • • • • • � · • • O u -' ..o • • • ..o • � • • • • • � • • •

EXERCfCIOS - U N I FESP 2014 (adaptado)

L As informações apresentadas no segundo parágrafo apoiam c) Os argumentos do segundo parágrafo confirmam o aquecimento
a ideia, presente no texto, de que: global. Correta.
a) os políticos já podem relaxar as medidas q u e visam evitar d) As enchen tes seriam causadas pelo a q uecimento e não pelo
o aq u e c i mento g lobal. resfriamento. Incorreta.
b) a pausa no aq uecimento global tam bém pode desencadear e) Da mesma maneira, os argumen tos reforçam a ideia de acele­
mudanças c l i máticas. ração do aquecimento global indicado pelo ritmo acelerado de
c) o aq u e c i m ento global não está e m desaceleração, apesar derretimento das calotas polares, e não a interrupção do fenô­
do esfriamento do oceano Pac ífico. meno. Veja: "... and the loss o f north polar ice has accelerated a t
d) o ciclo de resfriamento do c l i m a já começou, exemplificado such a ra t e that clima t e modellers expect t h e Arctic Ocean t o be
pelas enchentes na Austrália. routinely ice-free in September after 2040" (... e a perda do gelo
e) o derret i m e nto das calotas polares esfriou os oceanos, no Paio Norte acelerou a uma taxa tão alta que os modeladores
q u e, p o r sua vez, i n terro m p e ram o aq u e c i m ento global. esperam que o Oceano Artico esteja constantemente degelado
depois de setembro de 2040). Incorreta.
z. No trecho d o tercei ro parágrafo the deep oceans are - Resposta: c
warming: that is, the extra heat that should be measurab/e in
the atmosphere has been absorbed by the sea , a expressão - z.A expressão "that is" (isto é, quer dizer, ou seja) sempre introduz
that is i ntroduz u m a: uma explicação - neste caso, sobre a informação dada an teriormen­
a) d iscordância. te: os oceanos profundos estão aquecendo. Isto é, o excesso de calor
b) exe m p l i fi cação. que deveria ser medido na atmosfera tem sido absorvido pelo mar.
c) causa. Resposta: d
d) exp l i cação.
e) general ização. J.A questão exige tanto vocabulário quanto a in terpretação correta
do trecho, no contexto geral do texto. Referindo-se à ide ia de q ue
3· Ass i nale a alternativa q u e c o m p leta c orreta m ente a o excesso de calor seja absorvido pelos oceanos em suas profun­
lacuna I n o texto. dezas, o texto a firma. "that is hardly a good n e ws: a tmosph e re
a) u n l i ka b l e and ocean play on each other; and a ny stored heat is __ _ L ___ to be
b) l i ki n g retumed to the a tmosphere ... " (essa é dificilmente uma boa notícia.·
c ) l i ke l i h ood a atmosfera e o oceano influem um sobre o outro, e qualquer calor
d) u n l i ke l iest estocado _ _ ___ L_ _ é provável que seja devolvido à a tmosfera . . .}. Ou
e) l i kely seja, neste trecho, o autor trata de probabilidade. Se você conhecer
o significado das palavras apresentadas, saberá que a resposta
4. Ass i n al e a altern ativa que c o m p l eta correta me nte a correta é "/ike ly". "To be likely" significa ser prová vel, ser propen­
lacuna 11 n o texto. so. O significado dos termos das demais alterna tivas: un likable:
a) warm ed desagradável; liking: simpa tia, a tração ou p redileção; likelihood:
b) warmth probabilidade; un likeliest: o mais improvável
c) warm Resposta: e
d) warm f u l
e ) warme ft. A questão exige conhecimento sobre sufixos (sobre isso, veja a
página 45). Traduzindo o trecho citado na alternativa: "... and precisely
how ocean and atmosphere distribute _ _ __ _ }[____ from the equator to
RESPOSTAS the poles:: temos "... e como, precisamente, o oceano e a atmosfera
1. O segundo parágrafo apresenta uma série de evidências cien­ distribuem _____ 11... .. do equador aos palas" O verbo distribuir exige
tíficas que apoiam a ide ia de que o mundo está s e aq uecendo: um substantivo como objeto direto (quem distribui distribui alguma
"there is n o serious argument within climate science about the coisa). E o único substantivo apresentado nas alternativas é "warmth"
link between carbon dioxide leveis and temperature" (não há (calor). O significado das demais palavras apresentadas.· warmed:
nenhuma discordância séria na ciência do clima sobre a ligação aquecido (verbo); warm: quente (adjetivo); warmfuf.· caloroso, cheio
entre os níveis de dióxido de carbono e a temperatura). Agora, de calor (adjetivo); warme: não existe essa palavra em inglês.
analisando as alternativas: Resposta: b
a) Associando o segundo parágrafo ao primeiro, concluímos q u e
apesar d e alguns políticos n ã o acreditarem no aquecimento glo­
bal, há evidências de que o fenômeno é uma realidade. Incorreta.
b) No primeiro parágrafo, o texto afirma que a pausa verificada no
aquecimento global pode .::s tar relacionada com o resfria mento
do Oceano Pacifico. No entan to, esse resfriamento não é apon­
tado no restante do texto como causa do fenômeno. Incorreta.

GE IDIOMAS 2015 1 27
I NT E R P R ETAÇÃO D E TEXTO

Work, e·mail,
EXERCICIOS - MACKENZIE 2010 (adaptado)

1.The tips given in the text apply to people:


a) Who are busy showering babies and giving instructions.

news, bills... b) Who never really plan ahead of time or are inaccessi­
ble.
c) Who tends to exaggerate the amount of information
they supply their V I P clients with.
lt is easy to get overloaded by data at this d) Who will only admit getting 35 daily FYis on the
Henderson case.
information-loaded time
e) Who want to learn how to keep from being overwhelmed.

2. Choose the alternative that contain s o n ly adjectives


from the text formed by prefixes or suffixes:
a) i naccessible, reachable, recreational, unplanned.
b) interr u pted, likely, daily, faster.
c) proba b l y, quicker, really, likely.
d) exaggerating, dominating, signature, self-interruption.
e) overl oad, dominating, dip ping, i nstant.

3. Na frase "Peop l e w h o say, 'My boss wants m e to be


reachable after 8 p.m.' are likely exaggerating the control
others h ave over them", a expressão sublin hada pode ser
su bstitu ída, sem alteração de sign ificado, por:
� IS TECHNOLOGY dominating your l ife? a) sho u l d exaggerate.
b) may be exaggerating.
c) must exaggerate.
8 TIPS TO FIGHT INFO OVERLOAD d) ought to exaggerate.
1. Spot the signs. Feel alone even as you communicate with e) may exaggerate.
people ali day? That's a signal technology is dominating
your life.
2. Take baby steps. Try being inaccessible for short spurts to RESPOSTAS
see what happens. The world probably won't implode. 1. Who wan t to Jearn h o w to keep from being overwhe/­
3. Repeat these four words: "I have a choice." People who say, med (que qu erem aprender c o m o evitar ficar s obrecar­
"My boss wants me to be reachable after 8 p.m." are likely regado). O texto apresen ta dicas p a ra q u e m está so­
exaggerating the control others have over them. brecarrega do com i n formações o r i u n das da internet,
4. Set limits. Rein in office e-mail and instant message traffic. exa ta m e n te a ideia expressa no a djetivo "overwhelm ed".
Who truly needs 35 daily FYis on the Henderson case? Resposta: e
5. Give clear instructions. Try an e-mail signature that reads
"I answer e-mail at 10 a.m., 1 p.m. and 4 p.m. If you need a 2. lnaccessible, reachable, recreational, unplanned (inaces­
quicker response, please cal!." sível, alcançável, recreacional, não p la n ejado). No texto,
6. Make a task list. If you're interrupted, you'll get back to essas palavras são a dje tivos (no enunciado, cuidado com a
work faster i f you have one. pala vra "only'� que significa "somente"), formados por prefixos
7. Stick to a schedule. Handle recreational Web surfing and e su fixos.
e-mail at set times. Dipping in and out is classic self­ Resposta: a
interruption.
8. Do a reality check. After five minutes of unplanned surfing, 3· May be exaggerating (pode estar exagerando). A expressão
ask yourself, "Should I really be doing this now?" "a re likely" indica uma probabilidade que deve ser indicada
pelo verbo moda/ "may be", para se adaptar ao uso do gerúndio
(Mackenzie 2010. Por Ron Geraci, www.readersdigest.com) "exaggerating".
Resposta: b

28 [ GE IDIOMAS 2015
:��z�� ADJ ETIVOS » A F U N ÇÃO DOS ADJ ETIVOS

Como identificar informações pela


posição das palavras na frase

Em inglês, os adjetivos antecede m os su bstantivos. É por Em alguns casos, há diferença entre o significado de adjetivos
isso q u e, para a com preen são d e s i n tagma n o m i nal, é pre­ que têm sufixos d iferentes, por exemplo, "economic" (relativo a
ciso "andar de trás para frente". Ass i m, é i mportante fazer economia) e "economical" (econômico, q u e não gasta m uito).
a leitura de toda a frase para q u e você possa i d entificar o
sujeito e com preen d e r as i nformações a e l e relac i o n adas ARVORE G E N EALOGICA
por adjetivos. Observe os exe m plos retirados dos textos. Os Como surgem alguns adjetivos
substantivos estão s u b l i n hados:
» singular voice (voz singu l ar)
» instant message (mensagem i nstantânea)

ADJETIVOS TERMI NADOS EM "-ING" E "- ED "


Em i n g l ês, p o d e m o s formar adjetivos a part i r d e verbos,
com acrésci m o do s ufixo "ing" ou d o s ufixo "ed". Adjetivos
term inados em "ing" descrevem as situações e m si. Adjetivos
term i nados em "ed" d escrevem como as pessoas se sentem
em relação a uma determ i nada situação. O bserve:
>> That show in terests a Jot of people. (O show i nteressa a
muita gente.)
» The show is very interesting. (O show é i nteressante.)

» I am interested in the show. (Estou interessado n o s how.)

ADJETIVOS TERM I NADOS EM "AL"


Em inglês, u samos o sufixo "a/" para formar adjetivos a partir
de su bstantivos. Veja os exem p l os:
» Politica/ (politics)
» lnstitutional (institution)

GE IDIOMAS 2015 1 29
I N T E R P R ETAÇÃO D E TEXTO

A history of pi
A book tells about the mathematical
constant rr as a route of the human history

.<111111 MAIN ROLE Pi, which is related to circumferences measuring, may tell us something about human history

T
he history ofPi, says the author, though a small part ofthe history Dr. Beckmann has authored 11 books and more than 50 scien­
o f mathematics, is nevertheless a mirror of the history of man. tific papers, mostly on probability theory and electromagnetic
Petr Beckmann holds up this mirror, giving the background of wave propagation. History is one of his si de interests; another is
the times when Pi made progress - and also when it did not, because linguistics (he is fluent in five languages and he has worked out a
science was being stifled by militarism or religious fanaticism. The new generative grammar which enables a computer to construct
mathematical levei o f this book is flexible, and there is plenty for trillions o f grammatical sentences from a dictionary o f less than
readers of ali ages and interests. 100 unprocessed words).
He also publishes a monthly pro-science, pro-technology, pro-free
About the author enterprise newsletter Access to Energy, in which he prometes the
Petr Beckmann was born in Prague, Czechoslovakia, in 1924. viewpoint that clean energy can be made plentiful, but that access to
Until 1963, he worked as a research scientist for the Czechoslovak it is blocked by government interference and environmental paranoia.
Academy ofSciences, when he was invited as a Visiting Professor to
the University of Colorado, where he decided to stay permanently BECKMANN, Petr. A History of Pi. New York: Barnes & Noble Books, 1983.
as professor of electrical engineering. (Vestibular ITA 2014, adaptado)

30 I GE IDIOMAS 2015
·EiERdci.as·::·irÃ..2oi4.taCiailiaeio>·························································································································································
:
RES POSTAS
1. O l ivro A History o f Pi: L Analisando cada alternativa:
a) descreve g ran d e parte d a h i stória da matemática e d a a) O trecho "though a sma/1 part of the history of m a th em a tics"
humanidade. sign ifica que o Pi representa apenas pequena parte da história
b) é di reci onado apenas para i n iciantes e m m atemática. da matemática. Incorreta.
c) conta a h i stória de Petr Beckmann em tem pos de repressão b) "The ma thema tical level of this b o o k is fiexible, a n d there is
ao con he c i m ento. plen ty for readers o f ali ages and interests" (o nível da matemá tica
d) associa conceitos m atemáticos a fatos d a vida cotidiana. do livro é flexível e h á m u ito para leitores de todas as idades e
e) é acessível a um pú blico d iversificado. interesses). Incorreta.
c) Petr Beckmann é o a u tor, não personagem do livro. Incorreta.
2. No contexto deste texto, o item lexical "stifled", n o tre· d) Segundo o texto, o conceito da constante matemática rr (pi) é as­
cho "science was being stifled by m i l itaris m or religious sociado a épocas da história, não a fatos do cotidiano. Incorreta.
fanaticism" pode ser trad uzido p o r: e) Como já analisamos na alternativa b, o livro é acessivel a u m
a) sufocada. pú blico diversificado. Correta.
b) desmascarada. Resposta: e
c) organizada.
d) prom u l gada. 2. O verbo "to stifle" significa su focar, interromper, frear. De todas
e) institucional izada. as expressões apresen tadas, a única que se encaixa n o con texto
é "sufocada". Traduzido, o trecho e m questão fica: "a ciência foi
3. Indique o item I ex i cal que pode s ubstitui r o sublinhado no sendo su focada pelo militarismo o u fanatismo religioso".
trecho " ... mostly on probability theory and electromagnetic Resposta: a
wave propagation", sem prej udicar o seu sentido.
a) absol utely. J. O termo "mostly" pode significar "em sua maior parte'� " maioria"
b) chiefly. ou "principalmen te". De todas as alternativas, apenas a b apresenta
c) in herently. um sentido correto. As demais pala vras significam:
d) rand o m ly. • absolutely = absolutamen te;
e) utterly. • inheren tly = inerentemen te, intrinseca m e n t e;
• randomly = aleatoriamen te;
4. A opção q u e contém a reescrita correta de "science was • utterly = completamen te, totalmen te.
being stifled by militarism or religious fanaticism" e m Resposta: b
"Militarism o r religious fanaticism .. " é: .

a) were stifl i n g scien ce. 4-A oração "... science was being stifled by militarism or religious
b) had been stif l i n g science. fanaticism." (... a ciência foi sendo s u focada pelo m i litarismo o u
c) were being sti f l i n g science. fanatismo religioso) está n a voz passiva. A questão pede q u e você
d) has stifl i n g science. a passe para a voz a tiva, com "militarism o r religious fan a ticism"
e) have been sti f l i n g science. como sujeito. O fundamental é prestar atenção no tempo do verbo.
A voz ativa correta da oração é "Militarism or religious fa na ticism
s. I n d i q u e a alternativa q u e contém a referência correta were stifling science" (Militarismo ou fanatismo religioso estavam
para o termo em negrito. sufocando a ciência).
a) "giving the background of the times when Pi made progress Resposta: a
..." � backgro u n d .
b ) "Petr Beckmann holds u p t h i s m i rro r, giving t h e backgro u n d S. Analisando cada uma das alternativas:
of the times w h e n P i m ad e progress - and a l s o w h e n it a) O termo "when" (quando) é um advérbio de tempo. Portanto,
d i d not ..." � m i rrar. refere-se a "times". Incorreta.
c) " U nt i l 1963, h e worked as a research scientist for the Cze· b) O p ro n o m e "it" refere-se a "pi", e não a "mirrar". Incorreta.
choslovak Acade my of Sciences, when h e was i nvited ..." c) Novamente, o advérbio "when" diz respeito ao tempo. No texto,
� research scientist. o a n o de 1963. Incorreta.
d) "h e is f l u e n t in five languages and he has worked out a d) O p ro n o m e relativo "which" (o qual, q u e) não se refere a "five
new generative gram mar which e n ables a com puter to languages" (cinco línguas, ou linguagens), mas a "n ew generative
construct ... " � five languages. gra m mar" (nova gramática generativa). Incorreta.
e) " He ais o p u blishes a monthly pro-science, pro-te c h n o logy, e) Sim, a expressão "in which" (no q u a l, n a q u a l, n os quais, nas
pro-free enterprise newsletter Access to Energy, in which h e quais) refere-se a n e wsletter p u blicada por Beckmann. Correta.
pro motes the viewpoint ..." � newsletter Access to Energy. Resposta: e

GE IDIOMAS 2015 1 31
I NT E R P R ETAÇÃO D E T EXTO

lnterview with
z. De acordo com N oel Gallagher:
a) Algu mas m úsicas prod uzidas pelo Oasis têm uma l i gação
com d rogas.
b) O estú d i o Ab bey Road n u nca teve u m a banda famosa

Noel Gallagher
gravando lá an tes d o Oasis.
c) Oasis n ão teria sido pago se eles n ão tivessem gravado
"Dig Out Your Sou/".
d) E l e mesmo é bastante preoc u pado com o f u t u ro eco­
The Oasis' founder and guitarist explains n ô m i co d a banda.
e) Ele não é tão jovem q u anto M i c k )agger, que recente­
how he creates his songs
mente b rigou com Liam Gal lagher.
asis lead guitarist Noel Gallagher has been described

O as "the elder statesman" o f British rock. Yet this is a


description that he rejects, pointing out that, at the age
of 41, he is still a young man, when compared with the likes of
3. N o excerto "Abbey Road had previously banned Oasis
on account of their unru ly behavior", a expressão verbal
destacada é exemplo do:
Mick Jagger, Keith Richards and even Paul Weller. a) Past s i m p l e, usado para i n d icar uma ação ocorrida em
Gallagher, who has often been in conflict with the Oasis lead u m período d ete r m i n ado no passado.
singer, his younger brother Liam, recently met with the press b) Past s i m p l e, u sado para i n d i car u m a ação anterior a
in arder to pro mote the band's latest album, Dig Out Your Sou/ o utra ação no passado.
(attention: this interview was published before the band was c) Past cont i n u ous, usado para i n d icar u m a ação em an­
closed). As he explained, some of its songs were inspired by damen to no passado.
his memories of LSD trips as a teenager in Manchester. d) Past perfect, usado para ind icar uma ação em andamento
Noel Gallagher: What do you write about when you're 41? I no passado.
don't know. You know, beingfamous? Nobody wants to hear about e) Past pe rfect, usado para i n d icar u m a ação anterior a
that. Being in a band? That was kind of what Definitely Maybe outra ação no passado.
was about. Women? Too boring, you know. Money? Nobody wants
to hear about that. Politics? Boring, you know. Save the planet?
Boring. You know, I don't know. So, the most interesting thing is RESPOSTAS
like, well, I remember when I was 16 and, you know, taking acid, 1. Entrevista. Trata-se de uma en t revista ("interview"). Há indi­
it was like, "Yeah, there's stories to be told there, I think." cação d o a u tor, a explicação das perguntas que foram feitas e
Dig Out Your Sou/ was recorded at London's famous Abbey as respostas do en trevistado (Noel Gallagher) m a rcadas com o
Road studios, whose previous occupants have included The n o m e dele e o sinal gráfico: (dois-pon tos).
Beatles and Pink Floyd. Rumour has it that Abbey Road had Resposta: b
previously banned Oasis on account of their unruly behaviour:
Noel Gallagher: We had to pay the money upfront andpromise z. Algumas músicas produzidas pelo Oasis têm uma ligação com
to be on our best behaviour. We had to pay the money upfront, drogas. A resposta está no trecho "As h e explain ed, so m e o f its
and so they said, "If.. ifyou have to /eave, then you're gonna lose songs were inspired by his m e m o ries o f LSD trips as a teenager
the deposit." Well, it's different now, you know, when we were in in Manchester". (Conforme ele explica, algumas das canções da
there in '97, everybody was in their 20s and whacked out on drugs banda foram inspiradas nas suas memórias das 'viagens' s o b
ali the time, so it's different now. Everybody's got kids, you know. a influência d o LSD durante a adolescência em M a n c h e ster.)
Everybody's a little bit more ... a little bit /ess mental, you know, Resposta: a
but we still have a good time, though. We made a great record,
that's the ma in thing about it, you know, and it's goodfor Abbey 3. Past perfect, usado para indicar uma ação anterior a o utra
Road, that somebody's finally made a great record there! ação no passado. O passado p erfeito é usado para indicar um
acon tecim e n t o a n terior a o u tro n o passado simp les. A ide ia
(Mareei Anders, revista Speak Up, adaptado) é d e que a n tes da gravação d o álb u m D i g O u t Vo u r S o u ! ha via
(Vestibular Universidade Mackenzie 2010) r u m o res de q u e o estúdio t in h a ban1do o Oasis p o r causa do
comportamento desordeiro d a banda. A resposta está n o trecho
"Dig O u t Vou r S o u ! was recorded at London's fa m o us A bbey
EXERCÍCIOS - MACKENZIE 2010 (adaptado) Road s tu dios, whose previous occupa n ts have i n c luded The
Bea tles a n d Pin k Floyd. R u m o u r has it t h a t A b b ey Road h a d
1.0 texto que você acabou de ler é um(a): previousiy b a n n e d Oasis o n acco u n t o f t h e i r u n ruly b e havior".
a) biografia. (O álbum Dig O u t Vou r S o u ! foi gra vado nos famosos estúdios
b) entrevi sta. de Lo ndres Abbey Road, que já foram ocupados por The Beatles
c) verbete e n c i c l opédico. e Pink Fioyd. Havia rumores que o A b b ey Road havia banido o
d) rotei ro d e f i l me. Oasis p o r causa de s e u c o m p o r ta m e n to incon trolável.)
e) l i sta d e i n stru ções. Resposta: e

32 1 GE IDIOMAS 201S
, ,·:��-..- PASSADO » D I F E R E N ÇAS SUTIS E I M PO RTANTES

Aprenda a diferenciar o passado simples


e o passado perfeito

Em inglês, podemos usar três tem pos verbais para tratar . PASSADO S I MPLES
de acontecimentos passados: o passado simples e o passado A conjugação do verbo "to study" (estudar)
perfeito.

THE PAST SIMPLE: Para estrutu rar o passado s i m p l es dos


verbos, é preciso reconhecer se eles são regulares ou i rregu­
lares. Como regra geral, os verbos regulares têm sua forma
afirmativa com o acrésci m o do sufixo " d"o u "ed" à forma do
infinitivo (to remember - remembered). Já os verbos irregu­
lares não seguem a regra geral e suas formas precisam ser
memorizadas. Observe os exem p l os:
He directed }urassic Park. (Ele dirigiu jurassic Park.)
Steven Spielberg grew up in California. (Steven Spielberg
cresceu na Califórnia; grew up = passado de grow up, crescer.)
Nas formas negativa e interrogativa, usamos o verbo auxiliar
did not (didn't) e did, respectivamente, para todos os pronomes
do caso reto, da mesma maneira que organizamos a conjugação
verbal no presente simples.
VERBOS I RREGU LARES
ADVERBS OF TIME: Dois advérbios d e tempo tipicamente Os mais importantes e suas formas
usados com o passado s i m ples são ago e /ast (+noun). Veja
os exem p los:
Five minutes ago, 1 h ou r ago, half an h ou r ago (ago = atrás)
Last week, last Monday, lastyear (last passado/a)
=

No caso de períodos do d ia, pode-se usar yesterday morning


(ontem de man hã), afternoon e evening, mas não com night.
O correto é /ast night (na n oite passada).

THE PAST PERFECT: O passado perfeito é u m tempo verbal


composto, tendo como auxiliar o verbo had seguido do verbo
principal na forma do particípio passado (formas verbais que
em português termi nam em "ado", "edo", "ido", por exemplo
"comido", "estudado", "andado"). Para os verbos regulares,
a forma do particípio é a mesma do passado. Para os i rregu­
lares as formas m udam e precisam ser memorizadas (por
exem plo, to see/saw/seen).
Em frases afirmativas utiliza-se a mesma forma para to­
das as pessoas do d iscurso {1, you, h e, she, it, we, you, they).
Exem plos:
When the children arrived from school, Betsy had already
finished lunch. (Quando as crianças chegaram da escola, Betsy
já havia terminado o almoço.)
Na forma negativa, temos o verbo had not (hadn't) e na forma
interrogativa posicionamos o verbo auxiliar antes do sujeito.
Veja exemplos nas tabelas ao lado.

GE IDIOMAS 201S 1 33
I NT E RP R ETAÇAO D E T EXTO

The history of chocolate


Centuries ago, the product was a bitter mixture drank by the Mayans and Aztecs

..111111 THE AZTECS of central Mexico


attributed the creation of the cacao
beans to their god Quetzalcoatl who
descended from heaven on a beam of
a morning star carrying a cacao tree
stolen from paradise

They prized Xocolatl well above gold and silver so much so that,

D
elicious, delectable, soothing and American. Chocolatewas a New
World discovery, one of the most sought-after treasures brought when Montezuma was defeated by Cortez in 1519 and the victorious
back to Europe from the brave new land across the Atlantic. 'conquistadors' searched his palace for the Aztec treasury expecting
Cacao, from which chocolate is created, is said to have originated to find gold and silver, all they found were huge quantities o f cocoa
in the Amazon at least 4,000 years ago. The cacao tree was wor­ beans. The Aztecs, like the Mayans, also enjoyed cacao as a beverage
shipped by the Mayan civilization. who believed it to be of divine fermented from the raw beans, which again featured prominently
origin. Cacao is actually a Mayan word meaning 'God Food', hence in rituais and as a luxury available only to the very wealthy. They
the tree's modern generic Latin name Theobroma cacao, meaning regarded chocolate as an aphrodisiac and their Emperor, Montezuma
'Food of the Gods'. The word Cacao was corrupted into the more reputedly drank it fifty times a day from a golden goblet and is quoted
familiar 'Cocoa' by the early European explorers. The ancient Maya as saying o f Xocolatl: "The divine drink, which builds up resistance
br�wed a spicy, bittersweet drink by roasting and grinding the seeds and fights fatigue. A cup of this precious drink permits a man to walk
o f cocoa beans with maize and chili and letting the mixture ferment. for a whole day without food".
This drink was reserved for use in ceremonies as well as for drinking Xocolatl or Chocolat or Chocolate. as it became known.was brought
by the wealthy and religious elite. to Europe by Cortez. By this time, the conquistadors had learned to
The Aztecs of central Mexi co attributed the creation of the cacao make the drink more palatable to European tastes by mixingthe ground
beans to their god Quetzalcoatl who, as the legend goes, descended roasted beans with sugar and vanilla (a practice still continued today),
from heaven on a beam of a morning star carrying a cacao tree thus balancing the spicy bitterness of the brew the Aztecs drank.
stolen from paradise. In fact, the Aztecs valued the beans so much
that they used them as currency: a hundred beans bought a turkey (The History ófChocolate. www.aphrodite·chocolates.eo.uk/history_chocolate.htm. Adaptado)
or a slave, and taxes were paid in cocoa beans to Aztec emperors. (Vestibular UFBA 2010, adaptado)

34 1 GE IDIOMAS 201S
EXERCÍCIOS - U FBA 2010 (adaptado)

1. De acordo c o m o texto da pág i n a anterior, os astecas &. As formas infinitivas d o s verbos "brought" (primeiro pará­
explicam a origem d o cacaueiro como: grafo) e "stolen" (terceiro parágrafo) são, respectivam ente:
a) Criação do deus Qu etzalcoatl, que enviou a semen te do a) brought I stole.
cacau por u m raio d e sol. b) b r i n g 1 stole.
b) Invenção do i m perador Montezuma, q u e gostava de beber c) b r i g h t l steal .
chocolate todos os d ias. d) b r i n g I steal.
c) Inve n ção d o s m a i as, q u e c o n s i d e ravam a s e m e nte u m e) b r i n g I sto l e n .
deus.
d) I nvenção do d e u s Quetzalcoatl, q u e trouxe a semente da RESPOSTAS
Amazôn ia. 1. Criação do deus Quetzalcoatl, que desceu do céu em um raio
e) Criação do deus Qu etzalcoatl, que d esceu do céu em u m de sol carregando um cacaueiro ro u ba d o do paraíso. Veja o tre·
raio de u m a estre la d a manhã carregan d o u m cacaue i ro cho "(.. .) a t trib u t e d the creation o f the caca o beans to their god
roubado do paraíso. Qu etzalcoatl who, as the legend goes, descended from heaven
on a beam o f a morning star carrying a cacao tree stolen from
2. De acordo com o texto, é correto afirmar que: paradise". (. . a tribuíam a criação do cacau a seu deus Quetzalcoatl
a) Astecas e m a i as g ostavam d a b e b i d a feita d o cacau q u e, como diz a len da, desceu do céu em u m raio de uma estrela
fermen tado, prese n te p r i n ci pa l m e n te e m rituais. da manhã levando u m cacaueiro roubado do para íso.)
b) Somente os astecas co n s i d e ram o cacau u m a l i m ento d e Resposta: e
origem d ivi na.
c) Some n te os maias usavam o cacau como moeda. z. Astecas e maias gostavam da bebida feita do cacau fermentado,
d) Maias e astecas preparavam o c hocol ate com açúcar. presente principalmente em rituais. A resposta está no trech o
e) Os e u ropeus aprenderam a apreciar o gosto api m entado "The Aztecs, like t h e Mayans, also enjoyed cacao as a b everage
do chocolate. (.. .) featured prominently in rituais and as a /uxury available only
to the very wealthy". (Os astecas, assim como os maias, também
3. Segu ndo o texto, q uando Cortez derrotou Montezu ma, gostavam da b ebida, que aparecia com desta q u e nos rituais e
em 1519, ele esperava: como um luxo disponível apenas aos m u ito ricos.)
a) Con h ecer os segredos da preparação d o chocolate. Resposta: a
b) Conh ecer o paláci o real e as plantações d e cacau.
c) Levar para a E u ropa m u d as do caca u e i ro. J. Encontrar ouro e p ra ta no palácio real. Basta ver o trecho "When
d) Encon trar o u ro e p rata n o palácio real. Montezuma was defeated by Cortez in 1519 and the victorious 'con­
e) Encontrar sementes d e cacau afro d i s íacas. quistadors' searched his palace for the Aztec treasury expecting to
find gold and si/ver, ali they found were huge quantities o f co coa
4. A estrutura da sentença "Chocolate [ ... ] was brought to beans". (Quando Mon tezuma foi derrotado por Cortez em 1519
Europe by Cortez" (último parágrafo) é um exemplo de: e os conq uistadores vitoriosos procuraram o tesouro asteca em
a) past s i m p l e active voice. seu palácio, esperando achar ouro e pra ta, tudo que encontraram
b) past perfect active voice. foram grandes q u a n tidades de cacau.)
c) past s i m p l e passive voice. Resposta: d
d) past conti n u o u s passive voice.
e) past perfect passive voice. ft, Past Simple passive voice. Na fras e, temos uma forma do verbo
"to be" n o passado simples ("was") seguida do verbo "to bring" n o
s. Na frase "The cacao tree was worshi p ped by the Mayan particfpio passado ("brough t"), daí a frase s e r um exemplo de voz
civilization, who bel ieved it to be o f d ivine origin" (segundo passiva no passado simples.
parágrafo), o pronome 11. refere-se a: Resposta: c
a) Mayan.
b) cacao tree. s. Cacao tree. No trecho men cionado, o pronome "it" refere-se à
c) wors h i pped. árvore, ao cacaueiro. Resposta: b
d) civi l izati o n .
e) origi n . 6. Bring I steal. O verbo "to bring" (trazer) tem "brough t " no past
tens e e "brought" no past participle. "To steal" (roubar) tem "stole"
{past tense) e "stolen" (past particip!e).
Resposta: d

GE IDIOMAS 2015 1 35
INTERPRETAÇÃO D E T EXTO

Past in poetry
EXERCÍCIOS - U FG 2007 (adaptado)

L (UFG 2007, adaptado) About the poem "Forgotten Language",


the narrator:
Observe different uses for past tenses in a) Th i n ks he should have appreciated better the language
of nature.
these two poems b) Used to have a relationship with nature which he does
not anymore.
Forgotten Language c) M i sses the countryside where people had time even to
Once I spoke the /anguage ofthe fiowers, talk to plants.
Once I understood each word the caterpillar said, d) Regrets the time he used to spend speaki n g to ani mais
Once I smiled in secret at the gossip of the starlings, and plants.
And shared a conversation with the housefiy e) Could speak a special language which is now spread ali
in my bed. over the world.
Once I heard and answered ali the questions
ofthe crickets, 2. Qual frase descreve a comunicação d o narrador como
And joined the crying ofeach falling dying linguagem não verbal em "Forgotten Language"?
fiake ofsnow, a) Once I spoke the language o f the flowers...
Once I spoke the /anguage of the fiowers... b) Once I u nderstood each word the caterpillar said .. .
How did it go? c) Once I s m i led in secret at the gossi p o f the starlings ...
How did it go? d) And shared a conversation with the housefly i n my bed.
e) O n ce I h eard and a nswere d ali the q uesti o n s o f the
(Shel Silverstein, www.poemhunter, acesso em: 29 ago. 2006, usado em UFG 2007) crickets...

Vocabulário 3. As frases i ntroduzidas pelos termos And, We/1, Okay e In


caterpillar: l agarta fact (linhas de 3 a 7 d o poema The Frog Prince, Continued)
starling: pássaro e u ropeu indicam:
cricket: grilo a) Gradação que estabelece uma i nteração bem-h u m o rada
com o leitor.
b) Apelo emotivo, resultante da sonoridade, q ue aproxima
The Frog Prince, Continued o leitor.
The Princess kissed the frog. c) Passagem temporal, típica da linguagem dos contos de fada.
He turned into a prince. d) Oralidade que caracteriza socialmente as personagens.
And they lived happily ever after... e) Coloq u ialidade q u e transporta o l eitor para a ordem do
Well, /et'sjust say they lived sort of maravil hoso.
happily for a long time.
Okay, so they weren't so happy. RESPOSTAS
Infact, they were miserable. 1. Used to have a relationship with nature which he does not
"Stop sticking your tongue out like that," anymore (tinha uma relação com a natureza que não tem mais).
nagged the Princess. A resposta está disseminada no poema, nas relações que o poeta
"How come you never want to go down to estabelece en tre os verbos mais concretos de comunicação com a
the pond anymore?" whined the Prince. natureza ("spoke "- "understood" - "smiled"). Resposta: b
The Prince and Princess were so unhappy.
They didn't know what to do. 1. Once l smiled in secret at the gossip of the starlings . {Uma vez
. .

[. ..] eu sorri em segredo com as fofocas dos passarinhos...). Na frase,


The Prince kissed the Princess. é preciso compreender as palavras-chave "smiled" (passado do
They both turned in to frogs. verbo sorrir), "secret" (segredo) e "gossip" (fofoca). Resposta: c
And they hopped offhappily ever after.
The End. 3- Gradação que estabelece uma interação bem-humorada com
o leitor. O leitor compreende que a intensidade da relação entre
(SCIESZKA, j. The Frog Prince, Continued. New York: Penguin Books, 1994) o príncipe e a princesa vai diminuindo, o que não é natural nos
(Vestibular UFG 2007) contos de fada, daí o h umor: "We/1, let's just say they /ived sort o f
happily for a long time" (Bem, vamos dizer que eles viveram mais ou
Vocabulário menos felizes por muito tempo)/'Okay, so they weren't so happy"
hop: p u l ar (Tudo bem, eles não eram tão felizes assim)/"ln fact, they were
miserable". (Na verdade, eles estavam muito infelizes). Resposta: a

36 1 GE IDIOMAS 2015
DICAS PASSADO CONTÍN UO » TUDO AO M ESMO TEMPO

O passado contínuo usa o gerúndio


para expressar ação simultânea

O passado contín u o estrutu ra-se da m esma maneira q u e o » Descrição d e atividades em andamento no passado.
presente contínuo, usando as formas do verbo to be no passado Ex.: Wha t were you doing at this time yesterday, Mrs
(passado de am/is = was; passado de are = were). Observe: Richards? (O q u e a s e n hora estava fazen d o o ntem n e ste
I was taking a shower when the phone rang. (Estava to­ horário, s e n ho ra Richards?)
mando banho q uando o telefone tocou.) I was talking to my son's teacher at school. (Eu estava con­
Whi/e Oorothy and Phil were studying, David was listening versando com a professora do meu fi l h o na escola.)
to music. As soon as Phil came in to the roam, his father told
him the whole story. ( E n q u anto D o rothy e P h i l estavam
estudando, Dav i d estava o u v i n d o/ouvia m ú s i ca. Ass i m q u e
Phil chegou n a sala, s e u p a i contou a h istória toda.) Falar é diferente de escrever
Na forma negativa, usamos was not/were not (wasn't/weren't) É im portante conhecer os registros informais
e n a forma interrogativa posicionamos o verbo was/were antes da l inguagem
do sujeito. Observe a tabela de sistematização a seg u i r.
No texto da entrevista com Noel Gallagher (página 32), o cantor
EXEMPLO DE PASSADO CONTÍNUO faz uso frequente de expressões bem informais, típicas do discurso
A conjugação do verbo "to speak" (falar) oral. Observe os exemplos:
Women? Toa boring, you know. Money? Nobody wants to hear
about that. (Mulheres? Mu ito chato, sabe? Dinheiro? N inguém
quer saber disso.)
lf . ifyou have to leave, thenyou're gonna Jose the deposit. (Se ...
.

Se você tem de sair, então vai perder o depósito.)


Expressões como "you know'' (sabe?) ajudam a manter a interação
garantindo que ele compreenda e esteja prestando atenção no que
se diz. Da mesma maneira, perguntas como "Women? Money?" fazem
parte do princípio da economia linguística, uma vez que na interação
face a face há informações subentendidas e qualquer dúvida pode
ser resolvida - em vez de falar "Escrever músicas sobre mulheres?",
ele diz apenas "Mulheres?".
já a expressão informal gonna é uma forma contraída das palavras
going to. Ela dá agilidade ao discurso e é usada para indicar uma
ação no futuro. Outro exemplo desse tipo de expressão no discurso
informal é wanna (forma contraída de want to).
No registro informal, é comum usarmos também expressões que
são consideradas desvios da norma gramatical padrão. Observe o
USAMOS O PASSADO CONTINUO PARA: exemplo retirado da entrevista:
»Atividades que estavam em progresso no passado e que foram Yeah, there's stories to be told there, I think. (Sim, há histórias a
interrompidas por u m a ação pontual (expressa no passado ser contadas ali, eu acho.)
sim ples). Segundo a regra gramatical padrão, o verbo there to be (haver,
Ex.: I was taking a shower when the phone rang. (Eu estava existir) deve ser conjugado, no presente simples, como there is
tomando banho q uando o telefone tocou.) caso se refira a um substantivo no singular, e there are caso se
» Atividades tem porárias no passado. refira ao plural. No exemplo, stories está no plural e para seguir
Ex.: We were living in New York because o f my husband's job. a norma-padrão, portanto, o correto seria dizer "There are stories
(Estávamos vivendo em Nova York por causa do trabalh o do to be told".
meu marido.)

GE IDIOMAS 2015 1 37
I NTERPRETAÇAO DE TEXTO

Wolverine online leak: howling mad!


The entertainment industry is under attack. The crime?
The unauthorized reproduction and distribution of its products

I
t's bold to steal a movie. Any movie. But a surefire blockbuster says director Tony Scott (The Taking ofPelham 1 2 3). In fact, when
like X-Men Origins: Wolverine, Fox's make-or-break $130 million he first heard o f the theft, "I said, 'shoot him!"'. That harsh brand of
summer movie tent pole? That takes nerves of adamantium. justice has swept up those even tangentially involved in the scandal:
And the fact that someone somehow did manage to swipe such Fox News columnist Roger Friedman was fired for downloading the
an incredibly high-profile project is sending shock waves of panic movi e and posting a positive review.
throughout the industry. "This is certainly a lesson for us ali," says When the smoke clears, what impact will any of this have on
Fox co-chairman Tom Rothman. "We, like everybody else, thought the movie's box office haul? Hard to say. When Ang Lee's Hulk
.
our system was secure." Yet, on April 1, there it was: an unfinished was pirated in 2003, Universal Pictures claimed the theft - of a
version o f the movie, available online a full month before its release close-to-finished print - cost it $100 million. (That could have been
date. (Since then, the file has been accessed 100,000 times, and the posturing: The movie was also leveled by brutal reviews. Univer­
blogosphere buzz h as been mixed at best.) Fox is working with the sal had no comment for this story.) At least one industry insider
FBI to find the culprit - "Somebody's going to end up in jail for a thinks the exposure may boost ticket sales. "Everyone was talking
long time," insists Rothman - but no arrests had been made as of about Star Trek before this happened,'' he notes. "Now everyone's
press time. Hollywood bigwigs, who have taken their case for tougher focused on Wolverine."
antipiracy laws ali the way to Congress, are certainly out for blood.
"You have to find the [thief] and you've got to give him serious time," (extraído do vestibular da Universidade Mackenzie 2010, http://www.ew.com, adaptado)

38 1 GE IDIOMAS 201S
EXERCÍCIOS - MACKENZIE 2010 (adaptado)

L The article in the previous page explains that: RESPOSTAS


a) The cri m i nal respo n s i b l e for stea l i n g a copy o f X-Men O ri­ 1. The criminal responsible for stealing a copy of X-Men Origins:
gins: Wolverine before its rel ease date i s sti l l at l arge. Wolverine before its release date is stíll at large. A resposta está no
b) Wolverine has been play i n g i n American m ovie theaters trecho "Fox is working with the FBI to fínd the wlprit - So meb o dy s
' '

since A p r i l l, yet u nf i n ished. going to e n d up in jail for a long time', insists Rothman - but n o
c) Anti piracy laws i n the U S o n ly p u n ish people w hose f i l es arrests h a d been m ad e a s o f press time". (A Fox trabalha com o FBI
have been accessed 1oo,ooo times o r m o re. para encontrar o criminoso - "Alguém vai terminar na cadeia por
d) Directo r Tony Scott has bee n accused of th reate n i n g co l u m­ um longo tempo'� insiste Rothman - mas nenhuma prisão tinha
nist Roger Friedman and bad mo u t h i n g h i m . sido fe ita até o fechamento da e d ição.)
e) Acco rd i n g t o c hairman Tom Roth man, t h e movi e h a s been Resposta: a
revi ewed by a Fax N ews c o l u m n ist, who made the f i l m
available o n l i ne a m o n t h ah ead o f time. 2. A person who has committed a crime or dane something wrong.
A palavra culprit" significa pessoa que cometeu um crime ou fez
"

z. According to the text, cu lprit probably means: alguma coisa errada.


a) A person in charge o f reviewing recently made movies. Resposta: d
b) A person that pri n ts official reports related to movies
and TV series. 3. Universal Pictures. Na frase destacada, "it" refere-se a "ela", a
c) A person that d eals with movi e tickets sold to the p u b l i c empresa, ou seja, ao estúdio Universal Pictures.
online. Resposta: b
d) A person w h o h a s c o m m itted a crime o r d a n e somet h i n g
wrong. 4. O fato ocorrido pode ter um lado positivo. A res p o sta está n o
e) A movie producer w h o is also i nvolved with o n l i n e ad­ trecho "At least o n e industry insider thinks t h e exposure may
vertisement. boost ticket sales". (Pelo menos uma fonte do setor acha que a
exposição pode elevar a venda de ingressos.)
3. Na frase "When Ang Lee's Hulk was pi rated in 2003, U niver­ Resposta: d
sal P ictures clai med the theft - of a close-to-finished print ­
cost it S1oo m i l l ion", a palavra .li refere-se a: s. Fax co-chairman said that was certainly a lesson for them ali.
a) H u l k. No discurso indireto, quando a frase introdutória apresenta um
b) U n iversal Pictures. verbo no passado (no caso das alternativas, temos "said"), o tempo
c) Pri nt. verbal deve retroceder, ou seja, se tem o s uma expressão verbal
d) Ang Lee. no presente simp les, no discurso indireto devemos colocá-la no
e) The th eft. passado simples. Outras m udanças também são necessárias:
advérbios de tempo e pronomes precisam ser alterados em razão
4. No texto há a sugestão de que: dos significados. Na alternativa correta, temos: "O vice-presidente
a) O F B I n ão está fazen d o seu trabalho d i reito. da Fox disse que aquilo foi u m a li ção para todos eles".
b) A U n iversal Pictu res está i nvestindo mais do q ue pode em Resposta: a
sistemas d e segu rança.
c) Tom Roth man é c u l pado p e l o q u e acontece u .
d) O fato ocorrido p o d e t e r u m l a d o pos itivo.
e) To ny Scott vai enco ntrar o c u l pad o pelo ocorrido.

s. A alternativa que apresenta a frase "This is certai nly a


lesson for us ali," says Fox co-chai rman Tom Rothman e m
discurso i n d i reto é:
a) Fax co-chairman said that was certai nly a lesson for them ali.
b) Fax co-chairman said this was certa i n ly a lesson for us ali.
c) Fax co-c hairman said that has been a lesson for them ali.
d) Fax co-chairman said t h i s has been a l esson for u s a l i .
e ) F a x co-c h a i r m a n said t h at w e re c e rta i n ly a lesson fo r
them a l i .

GE IDIOMAS 2015 1 39
INTERPRETAÇÃO D E TEXTO

About swine flu


EXERC(CIOS - U P E 2010 (adaptado)

1. The flu is cal led "swi n e flu" because:


a) lt h as c o m m o n sym ptoms.
Learn about the new strain of influenza that b) People gen eral ly feel u nw e l l .
has spread worldwide since 2009 c ) Peo p l e b e l i eve it c a m e f r o m p igs.
d) l t is not certain the f l u is origi nated in p igs.
e) M ost peo p l e recover w i t h i n a week.

S
wine flu is the common name given to a new strain of
influenza (flu). It is called swine flu because it is thought " Pregnant women" means:
:t.

to have originated in pigs, but this is not certain. a) m u l h eres p ragmáticas.


The most common symptoms are fever, sore throat, b) m u l heres preparadas.
diarrhea, headache, feeling generally unwell and a dry cough. c) m u l h e res com crian ças de colo.
In other words, the symptoms are very similar to seasonal d) m u l h eres grávidas.
(regular) flu. Most people recover within a week, even without e) m u l h eres i m u nes ao vírus.
special treatment.
The virus was first identified in Mexi co in April 2009. It has 3- The sentence "millions of people d ied across the world"
since become a pandemic, which means it has spread around in the i nterrogative form is:
the globe. It has spread quickly beca use it is a new type o f flu a) Did m i l l io n s of people d ied across the world?
vírus that few, if any, people have full resistance to. b) D i d m i l l i ons of people dye across the world'
Flu pandemics are a natural event that occur from time to c) Did m i l l io n s of peo ple d i e across the world?
time. Last century, there were flu pandemics in 1918, 1957 and d) Did m i l l io n s of peo ple dyi n g across the world'
1968, when millions of people died across the world. In most e) Did m i l l i o n s of people are dyi n g across the world?
cases the vírus has proved relatively mild. However, around
the world more than 1,700 people have died and it is not yet 4- De acordo com o texto, pacientes de risco são
ele ar how big a risk the vírus is. For this reason, and beca use a) C rianças maiores de s anos.
ali viruses can mutate to become more potent (stronger), b) Id osos hospital izados.
scientists are saying we need to be careful. c) Pac i e n tes em tratame nto para asma.
Some people are more at risk o f complications i f they cate h d) Profissionais da saúde.
swine flu, and need to start taking antivirals as soon as it is e) Paci e ntes que fazem uso d e as p i r i n a.
confirmed that they have the illness. Doctors may advise
some high-risk patients to take antivirals before they have
symptoms, i f someone close to them has swine flu. RESPOSTAS
People at risk are: L Peopfe believe it carne from pigs. A resposta está no trecho "lt
patients who have had drug treatment for asthma in the
• is called swine flu because it is thought to h ave originated in pigs,
past three years, but this is not certain". (Chama·se gripe sulna porque se acredita
pregnant women,
• ter sido o riginada em porcos, mas isso não está comprovado.}
people aged 65 and over, and
• Resposta: c
children under five.

The most important way is to have good respiratory and z. Mulheres grávidas. Essa é a tradução co rreta do termo.
hand hygiene. In other words, always sneeze into a tissue, and Resposta: d
quickly put it in a bin. Wash your hands and home and work
surfaces regularly and thoroughly to kill the vírus. 3. Did m il/íons of peop/e die across the world? Na forma in·
terrogativa do passado simples, usamos o verbo a uxiliar "díd"
(www.nhs.uk/conditions/pandemic·flu/Pages/lntroduction.aspx, adaptado) an tes do sujeito da frase e o verbo principal no inhnitivo sem
(Vestibular da UPE 2010) a partícula "to"
Resposta: c

4. Pacientes em tratamento para asma. A resposta está na frase


"patients who h ave had drug trea tment for asthma in the past
three years". (Pacientes que têm feito trata mento para asma
nos últimos três anos.)
Resposta: c

40 I GE IDIOMAS 201S
DICAS G O I N G TO E W I L L » COMO FALAR S O B R E O F U T U RO

Reveja as formas verbais e as


aplicações mais comuns em inglês

Em inglês, há várias formas de expressar ideias no futuro. COM "WILL"


Observe os exemplos: Para falar sobre o futuro em geral (previsões, promessas,
» Sandy hopes to become an excellent doctor after she finishes intenções, suposições e pedidos) usamos a forma verbal will +
her course. (Sandy espera se tornar uma excelente médica forma base do verbo principal (sem o "to"). A forma will é usada
depois de terminar seu curso.) para todos os pronomes pessoais (/, you, he, she, it, we, you,
>> l'm going to study more Eng/ish tomorrow. (Vou estudar mais they). A forma negativa é expressa com a combinação de will
inglês amanM.) + not, cuja forma abreviada é won't. Na forma i nterrogativa,

» }ohn might trave/ to the USA next month. Oohn talvez viaje posicionamos o verbo will antes do sujeito da frase.
aos EUA no mês que vem.) Exemplos:
No primeiro exemplo, observamos o verbo to hope (esperar), » They wi/1 come to the party, don't worry. (Eles virão à festa,
que expressa uma ide ia de futuro (apesar de estar conjugado no não se preocupe.)
Simple Present), ou seja "Sandy espera se tornar uma excelente » Wi/1 you marry me? (Você quer se casar comigo? I Casa
médica q uando terminar seu c urso". No segundo exemplo, comigo?)
o falante planeja uma ação futura (Vou estudar mais inglês » .. . by the Commonwealth Games in four years, drivers will be
amanhã) e, no terceiro exemplo, john indica uma probabilidade able to converse confidently (Até os jogos do Commonwealth
para o futuro (talvez john viaje). daqui a quatro anos, os motoristas serão capazes de conversar
Assim, em inglês, há várias formas de expressar o futu ro, com segurança.)
dependendo da i ntenção de quem fala (ou escreve) ao pla­ » Wi/1you elose the windo� please? (Você quer fechar a janela,
nejar ações, fazer previsões ou promessas, indicar probabi­ por favor?)
lidades futuras etc. » If peop/e don't recycle their domestic garbage, we won't sur­
vive much more in this world. (Se as pessoas não reciclarem
COM "GOING TO" o lixo doméstico, nós não sobreviveremos por muito mais
O "going to" (future) é usado em inglês para expressar pla­ tempo neste m undo.)
nejamento de atividades ou ações no futuro (em geral, futuro É possível usar o verbo sha/1 no lugar de wi/1 para as pri­
próximo), ou ainda para expressar previsões para o futuro meiras pessoas do discurso (/, we), em pedidos ou sugestões.
baseadas em evidências no presente. Observe: Exemplos:
» l'm going to p/ay soccer this afternoon. Would you /ike to » lt's getting late. Sha/1 we go? (Está ficando tarde. Vamos

come with me? (Vou jogar futebol esta tarde. Você quer vir embora?)
comigo?) » I got this e-ma i/ and now Iam worried. What shal/ 1 do? (Recebi
» Look at the sky. lt's going to rain, so don't forget your um­ este e-mail e agora estou preocupada. O que devo fazer?)
brella! (Olhe para o céu. Vai chover, por isso não esqueça
seu guarda-chuva!) DEMAIS EXPRESSOES
Para estruturar frases afirmativas, usamos as formas do Além das formas descritas anteriormente, podemos usar
verbo to be no presente (am/is/are) + going to + forma base do outras expressões e verbos para indicar intenções e ambições
verbo principal. As formas negativa e interrogativa seguem as no futuro. Exemplos:
mesmas regras do Presente Contfnuo. Observe: » would /ike + verbo em sua forma-base.

» Keila and Mat are going to the cinema tonight. (Keila e Mat Ex.: I would like to see you tonight. Are you free? (Eu gosta­
irão ao cinema esta noite.) ria de ver você hoje à noite. Você está livre?)
» Sheila isn't going to work on Friday. She h as the day off. » look forward to + verbo no gerúndio
(Sheila não vai trabalhar na sexta-fei ra. Ela tem o dia de Ex.: 1 /ook forward to hearing from you. Write soon. (Espero
folga.) ansioso para ter notícias suas. Escreva logo.)
» What are you going to do tomorrow? Any plans? (O q u e » would rather + verbo na forma-base

você vai fazer amanhã? A l g u m plano?) Ex.: l'd rather go to the mal/. (Eu preferiria ir ao shopping.)

GE IDIOMAS 2015 1 41
I N TE RPRETAÇAO DE TEXTO

The Dictionary of
American Regional English
This ambitious lexicographical project was enhanced by
the unexpected richness of the American vocabulary

.oOIIIIII THE MAIN STREAM ANO THE BRANCHES The language English is one. But it may sound very different depending on the region of the United States

oan Houston Hall, a lexicographer, is concerned about the the laments about the homogenizing forces o f urbanization,

J
use o f regionalisms throughout the country. She works in one mass media and the Internet.
of America's most ambitious lexicographical projects, which Ms. Hall recognizes that the extremes o fregional speech are
culminates with the publication of a dictionary by Harvard disappearing; however, she is pleased to find out that there are
University Press, 50 years after the project was inaugurated still lots o f regional words. I f it has become harder to find many
by Frederic G. Cassidy. different terms for outdated expressions, that's partly beca use
Mr. Cassidy, who died in 2000, did not make it to the end people today are more likely to be talking about fast food or
o f the alphabet. But to scholars and language lovers the work traffic. In fact, in the age o f Twitter and Facebook, linguists
he set in motion is an invaluable guide to the way Americans say, the interactions that most powerfully shape our speech are
not only speak but also live. The dictionary includes nearly still local and face to face. In sum, American English actually
60,000 terms, many of them reflecting the country's rural and has more words for the same things than ever before.
agricultura! past. But among the pages and pages of names for
wildflowers and farm implements, Dare, as the dictionary is Adapted from: < http://www.nyti mes.com/2012/02/2 s/books/d ictionary-of·american·
commonly known, includes enough newer terms to suggest that ·regional·engl ish·reac hes·last·vol ume.html?pagewanted=all>. Accessed on August
the state o f regional English isn't quite as bad as may suggest 14th, 2012. (Vestibular UFSC 2013, adaptado)

42 1 GE IDIOMAS 2015
EXERCÍCIOS - U FSC 2013 (adaptado)

Nas questões a segu i r, pode haver mais de uma alternativa d) S im, o dicioná rio é conhecido por Dare. No trecho: ".. . Da re, as
correta. the dictionary is commonly known". Correta.
e) A ide ia proposta na alternativa é explícita no trecho "Dare ... in­
1. Regardi ng the dictionary mentioned in the text, it is correct c/udes enough newer terms to suggest that the state o f regional
to state that: English isn't quite as bad. . ". C o rre ta .
a) the Dictio nary o f American Regional E n g l i s h is a res u l t o f Resposta: a, d, e
one o f America's m ost ambitious lexicograph ical projects.
b) joan H o u ston H a l l is the o n ly l exicographer respo n s i b l e z. Simples compreensão de texto. Analisando as alternativas:
fo r the d i ctionary p u b l i s hed b y H a rvard U n ivers ity. a) A expressão "The Dictio nary o f A merican Regional English",
c) the d i ctionary w h i c h is a p roduct of a gro u p of students apresen tada aqui, pode se referir ta n t o ao dicionário em si (o
from Harvard consists o f al most 6o,ooo terms. Dare) quanto ao tftulo do texto. Seja com o for, nenhum deles é
d) the Dictionary o f American Regional English is also known um artigo científico. Incorreta.
as Dare. b) A m bas as informações você encon tra na indicação da fonte de
e) the i nc l u s i o n of n ewer terms in the Dictio nary of Ameri­ onde o texto foi extraido, mais precisa men te, do si te de onde
can Regio nal E n g l i s h s u ggests that the state of regional ele foi retirado: "nytimes.com" indica o jornal New York Times
E n g l i s h i sn't so bad. e a data de pub licação é 2012/2/25 (an o 2 012, m ês fevereiro,
dia 25] Correta.
z. Select the correct proposition(s). c) A ide ia está explicita no trecho: "... 50 years after the p rojec t was
a) The Dictionary o f Ameri can Regional E n g l i s h i s a scientific inaugurated by Frederic G. Cassidy" (so anos depois que o projeto
article about a scientific experi ment. foi inaugurado por Frederic G. Cassidy). Correta.
b) The text adapted from The New Yo rk Times was released d) O texto é do jornal New York Times, e não está assinado. Segundo o
origi nally in Febru ary 25th, 2012. texto, ]oan Houston Hall participa do projeto do dicionário. Incorreta
c) The text i s abo u t a d ictionary w hose p roject was started e) Novamen te, a expressão "Th e Dictionary o f American Regional
by Frederic G . Cassidy. English" pode se referir a o nome do dicionário o u ao tftulo do
d) The text was written by joan Houston H ali, a lexicographer texto. Seja como for, nem u m nem outro trata de pessoas que
who works with d i ctionaries. apreciam dicionários. Incorreta.
e) The Dictionary o f Ame rican Regional E n g l i s h is a text prin­ Resposta: b, c
ted i n the mass media abou t people who l i ke d i ctionaries.
3. Analisando cada alternativa:
3. Select the proposition(s) which presents (present) correct a) "not only. .. but ais o" (não apenas. . mas também) é uma expressão
explanations for the expressions highlighted in the text. empregada para adicionar informações. Correta.
a) not o n ly... b u t also: add i n g i n fo rmation b) "Near" (perto), sim, refere-se a distância. Mas o advérbio salien­
b) nearly: i n d icat i n g d istance tado no texto, "nea rly" (cerca de) passa a ideia de arredonda­
c) more l i kely to: expressing probab i l ity mento de um valor. Veja: "nearly 6o,ooo terms" (cerca de 60 mil
d) in fact: em phasi z i n g i nformation term os) Incorreta.
e) i n sum: p rovi d i n g ad d itional i deas c) "More likely to" (mais propenso a) expressa probab ilidade.
f) actual ly: reinforc i n g id eas Correta.
d) "In fact'' (de fato) é uma expressão de ênfase. Correta.
e) "In sum" (em resumo, enfim) não indica a apresentação de novas
RESPOSTAS ideias, mas anuncia uma conclusão que leva e m conta todos os
1. Analisando as alternativas: fa tores apresentados a n teriormente. Incorreta.
a) O primeiro p arágra fo informa que, joan Houston Hall trabalha f) Não se confunda. "actually" é um fa lso cognato q u e significa
em um dos mais ambiciosos projetos de lexicografia da América, "na verdade; e não "a tualmen te". E essas expressões são empre­
que culmina na publicação de u m dicionário publicado pela gadas para reforçar uma ideia. Correta. (Veja mais sobre falsos
Harvard University Press (nome da editora] Correta. cognatos na página 2 1).
b) O trecho " .50 years after the project was inaugurated by Fre· Resposta: a, c, d, e, f
deric G. Cassidy" (. .. 50 anos dep oi s que o proj e t o foi lançado por
Frederic G. Cassidy) demonstra que }oan Houston Hall não é a
única responsável pelo p rojeto. Incorreta.
c) O texto a firma, realmente, q u e o dicionário inclui aproxima­
damente 60 m i l term os, mas não há nenhuma menção ao fato
de estuda n tes terem participado de sua produção. (Não s e
confunda: neste contexto, "scholars" significa especia listas, e
não estudan tes.) Incorreta.

GE IDIOMAS lOJS 1 43
I N T E R P RETAÇÃO D E TEXTO

School and longevity


The more you study1 the healthiest you are

T
o live the longest and healthiest life possible, get smarter. Ins­ school and are half as likely to die between the ages of 15 and 60.
titute for Health Metrics and Evaluation (IHME) data show "Economic growth is also significantly associated with child mortality
that past a certain threshold, health and wealth are just weakly reductions, but the magnitude of the association is much smaller
correlated. However, overall health is closely tied to how many ye­ than that of increased education,'' comments Emmanuela Gakidou,
ars people spend in school. Mexico, for instance, has a fifth the per IHME's director of education and training. "One year of schooling
capita gross domestic product (GDP) of the United States, but, for gives you about 10 percent lower mortality rates, whereas with a 10
women, more than 50 percent of the latter's schooling. percent increase in GDP, your mortality rate would go down only
In line with the trend, Mexico's female adult mortality rate is by 1 to 2 percent."
only narrowly higher. Vietnam and Yemen have roughly equivalent
per capita GDP. Yet Vietnamese women average 6.3 more years in Discover, May 31, 2013. Adaptado. (Vestibular Fuvest 2014)

EXERCÍCIOS - FUVEST 2014 (adaptado)


1. O argu mento central do texto é o de q u e n íveis mais altos de b) O texto não faz menção a crescimento econômico. Incorreta.
escolaridade estão d i retamente relacionados a: c) O texto não faz referência a empregos ("employment" ou 'iobs") lncorreta.
' .

a) índices mais baixos de m o rtal idade. d) Tampouco se refere a taxas de na talidade. Incorreta.
b) cresc i m ento eco n ô m ico acentuado. e) Do mesmo modo, o texto não menciona serviços de saúde. Incorreta.
c) mais e m p regos para as m u l h e res. Resposta: a
d) m e n o res taxas d e nata l i dade.
e) m e l h o rias nos serviços de saúde. 1. A questão exige tanto vocabulário quanto atenção na leitura. Ana/i·
sando as alternativas:
2. N o texto, ao se comparar o México aos Estados U nidos, afirma-se a) Não se confunda com os numerais, nem com a matemá tica. No trecho
q u e, no México, "Mexico, for instance, h as a fifth the per capita gross domestic product
a) o p rod u to i nterno bruto é e q u i valente a so% do prod uto i nterno (GDP} o f the United States ... " (o México, por exemplo, tem um quinto
b ruto dos Estados U n idos. do produto interno bruto per capita {PIB} dos Estados Unidos . . .). Um
b ) os índices de mortalidade ad u lta vêm crescendo, nos ú ltimos anos. quinto ("one fifth") não é o mesmo que 50%. Incorreta.
c) as m u l h eres representam so% da p o p u lação escolarizada. b) O texto se refere à taxa de mortalidade das mulheres adultas, e não à dos
d) as pol íticas ed u cacionais são i n su ficientes e estão defasadas. adultos em geral. Veja a frase: "in !in e with the trend, Mexico's female
e) as taxas de m o rtali dade fem i n i na ad u lta são pouco su periores adult mortality rate is only narro wly higher" (seguindo a tendência, a
às no rte-ameri canas. mortalidade das mulheres adultas no M éxico é só ligeiramente mais
alta.). Incorreta.
3. De acordo com o texto, "about 10 percent lower mortality rates" c) Traduzindo a frase "Mexico, for instance, has a fifth the per capita
é resultado de: gross domestic product (GDP) of the United States, but, for women,
a) " 1 0 percent i n c rease in G DP". more than 50 percent of the latter's schooling", temos "o México, por
b) "ch i l d m o rtality red uctions". exemplo, tem u m quinto do produto interno bruto per capita (PIB) dos
c) "equ ivalent per capita G D P". Estados Un idos, mas, en tre as m ulheres, mais de 50% da escolaridade
d) "eco n o m i c growth". do último [Estados Unidos]). Incorreta.
e) "one year of s c h o o l i ng". d) O texto não faz nenhuma referência a pol!ticas educacionais. Incorreta.
e) Sim, como já vimos na análise da alternativa b. Correta.
Resposta: e
RESPOSTAS
!.Analisando a s alternativas: J.A resposta depende apenas de você compreender o significado da última
a) As frases "h owever, overall health is closely tied to how many years frase do texto: "one year o f schooling gives you about 10 percent lower
people spend in school" (entretan to, a saúde geral está estreitamente mortality rates, whereas with a 10 percent increase in GDP, your mortality
vinculada a quantos anos as pessoas passam na escola" e "one year o f rate would go down only by 1 to 2 percent" (um ano de escolaridade resulta
s-a'rwii11ggives you about 10 percen t lower mortality rates" (um ano de em taxas de mortalidade cerca de 10% menores. já com um crescimento
escolaridade resulta em taxas de mo rtalidade cerca de 10% menores) de 10% do PIB, a taxa de mortalidade cairia apenas 1% ou 2%."
confirmam o que é proposto na alternativa. Correta. Resposta: e

44 1 GE IDIOMAS 2015
D ICAS FORMAÇÃO D E PALAVRAS » LETR I N HAS Q U E M U DAM TU D O

Os acréscimos de prefixos e sufixos transformam


palavras e criam substantivos e adjetivos

E m i ng l ês, u samos prefixos e s ufixos para formar novas 2. Para formar s u bstantivos p o d e m o s u sar, e ntre o u t ros,
palavras. Por exe m p l o, do verbo to drive (dirigir) podemos "ity", "ition", "hood", "ion" e "ship". O b serve:
ter o s u bstantivo driver ( motorista) o u o adjetivo driving
(com o e m driving license, q u e em português se trad uz por
carteira d e h a b i l itação). Pode m os ainda c riar antô n i mos,
como o adjetivo unhappy (infel iz) q u e se forma a partir d o
adjetivo happy (fel iz).

EXEMPLOS DE PREFIXOS
Para criar antô n i m os, podemos usar "un", "dis" e "in":

3. O s ufixo "/ess" pode i nd icar ausência:


• job (emp rego) - jobless (sem e m prego/desem p regado)
• Home (casa/lar) - home/ess (sem-teto/desabrigado)

4. Vários sufixos são usados para criar adjetivos, por- exem p l o


"y", "a i", "ic", "ous" e "fu i":
EXEMPLOS DE SUFIXOS
1. Para criar s u bstantivos i n d i cadores de profissões/profis­
sionais, podemos u sar o s ufixo "e r":
• To teach (ensina r) - teacher (professor)
• Footba/1 (futebol) - footballer (jogador de futebol)
• To swim (nadar) - swimmer (nadador)
· To employ (empregar) - employer (empregador)
C u i d a d o ! To cook (coz i n har) - chef (chefe de cozin ha).
A palavra cooker em i nglês s i gn ifica fogão.

GE IDIOMAS 2015 1 45
INTERPRETAÇAO DE TEXTO

The death of the PC


Millions of com puter users don't want to spend money on softwares that
need to be updated - and a technological revolution is on its way

..ollill COMPUTER technology and


communication has been evolvíng as users'
needs change in a way never seen befo!:e

T
he days of paying for costly software upgrades are numbered. Yet, two little-known companies have a huge head start. Get their
The PC will soon be obsolete. Internet users need faster ways names in a free report from the Motley Fool called, "The Two Words
to communicate and social network with the help of new user­ Bill Gates Doesn't Want You to Hear..."
friendly technology. That's why some people say that the personal
computer is already dead. What is replacing it? BusinessWeek reports Click here for instant access to this FREE report!
70% o f Americans are already using the technology that will replace BROUGHT TO YOU BY THE MOTLEY FOOL
it. Merry Lynch calls it "a $160 billion tsunami". Computing giants
including IBM, Yahoo!, and Amazon are racing to be the first to cash (ENEM 2011, http://www.fool.com. Acesso em 21 jul. 2010, adaptado)
in on this PC-killing revolution.

46 1 GE IDIOMAS 2015
EXERCÍCIOS - ENEM 2011 (adaptado)

1. De acordo com o texto com preendemos q u e o autor: s. Na expressão Computing giants ( l i n ha 7), a palavra com­
a) É u m j o r n a l i sta q u e traba l h a como co rres p o n d e n te i n- puting funciona como:
ternac ional. a) verbo tran sitivo.
b) Não concorda com a situação d escrita n o texto. b) advérbio.
c) Leu revistas de economia e negócios para escrever o texto. c) adjetivo.
d) É dono de uma empresa fabricante de microcom putadores. d) s u bstan tivo.
e) Não tem d i n h e i ro para c o m p rar pacotes de atua l i zação e) verbo i ntransitivo.
de softwares.

1. N o texto, o percentual 70 representa: R E S POSTAS


a) A q uantidade de norte-ame ricanos q u e usam u m a nova 1. Leu revistas de economia e negócios para escrever o texto. A o
tec nologia de com u n i cação. mencionar informações apresentadas n a revista B u s i nessWeek
b) O l u cro obtido pe las e m p resas q u e estão prod u z i n d o u ma c1tar também a empresa Merri/1 Lynch, constatamos que o escritor
nova tec n o l ogia de c o m u n icação. tem acesso a esse tipo de material d e leitura. Para responder essa
c) O n ú me ro d e d ias em q u e a nova tecnologia será l ançada q uestão, c o n h ecimento prévio d o tema é essen cial, o u seja, saber
no mercad o. q u e B u s i n e ssWeek é uma revista de economia e negócios ajuda a
d) A q uantidade d e comp utadores que d everão ser su bsti­ escolher a alternativa correta. Resposta: c
tu ídos pela n ova tec n o logia.
e) O n <J m e ro d e e m p resas q u e estão i nvest i n d o n a n ova 1. A quan tidade de norte-americanos que usam uma nova tecno­

tec no logia. logia de comunicação. A resposta está no trecho "70% o f Ameri­


cans are a lready using t h e technology that wil/ replace it" (70%
3. Leia as afi r m ações abaixo para res p o n d e r à segui n te dos n o rte-americanos já estão usando a n o va tecnologia que irá
questão: substituí-lo) Resposta: a
I. As e m p resas de m i croco m p u tado res form aram um con­
g l o m e rado l i derado por B i l l Gates. 3. // e 111. A afirmação 11 está no trecho "The PC wil/ soon be obso­
1 1 . M i c roco m p u tado res se rão s u b s t i t u íd os por u ma n ova lete (. ) 70% o f Americans are already using the tech n ology t h at
tec n o l ogia d e co m u n icação. will replace i t". ( O PC e m breve estará obsoleto.(.) 70% dos n o rte·
1 1 1 . A nova tecnologia está sendo desenvolvida por e m p resas -americanos já estão u s a n d o a tecnologia que irá substituí· lo.}
grandes e peq u e nas. A afirmação 111 está disseminada no texto, n a m ençao q u e se faz
a gra n des empresas c o m o IB M, Yah o o e Amazon e t a m b é m n o
A(s) ú n i ca(s) afi r m ação(ões) s u stentad a(s) p e l o texto trecho "Yet, two little·known companies h ave a h uge h e a d start ".
é(são): (En tretan to, duas empresas pouco conhecidas já deram um grande
a) I . passo a frente.) Resposta: e
b) 11.
c) I e 1 1 . ft.Às empresas que levam vantagens para serem suas concorrentes.
d) I e 111. A resposta está no trecho " ... two little-kn o w n companies h a ve a
e) li e 111. huge h ead start" (duas companhias p ouco conhecidas deram um
enorme passo a frente}. Como Bi/1 Ga tes é mencionado no título da
4. (ENEM 2011) Ao optar por ler a reportagem completa sobre reportagem, e sabendo que ele é o criador da Microsoft, imaginamos
o assunto anunciado, tem-se acesso a duas palavras que Bill a importância dessas duas empresas n a corrida por desenvolver
Gates não quer q u e o leitor con h eça e que se referem: e comercia lizar essa n ova tecnologia. Resposta: d
a) Aos res pon sáve i s p e l a d ivu l gação d esta i n formação na
i nternet. s. Adjetivo. Em "co m p u ting gia n ts" t e rn os "giga n tes da co m p u­
b) À s marcas mais i m p o rtantes d e m i croco m p u tadores do taçao" É preciso saber q u e, em inglês, os adjetivos a n tecedem
mercad o. os substantivos e q u e m u itos a djetivos tem o s u fixo "-ing" como
c) Aos n o m es dos a m e ricanos q u e i nv e n taram a s u posta terminação. Resposta: c
tec no logia.
d) À s e m p resas q u e l ev a m v a n tag e n s para serem s u a s
conco rren tes.
e) Aos s ites d a i n ternet pelos q uais o p rod u to já pode ser
co n h ecido.

GE IDIOMAS lOIS 1 47
I NT E R P R ETAÇÃO D E TEXTO

Volunteer
EXERC[CIOS - UFMG 2009 (adaptado)

L O texto é d i rigido a
a) B ras i l e i ros i n teressados em d e s e n volver p rojetos d e

abroad
traba l h o no exterior.
b) Estrangeiros i n teressados em d esenvolver projetos de
e n s i n o n o Bras i l .
The announcements below show results c ) Estrange i ros i n teressados em fazer trabalho vol u n tário
n o B ras i l .
of a search about volunteer jobs in Brazil d ) B ras i l e i ros i nteressados em anu n ciar p rojetos de traba­
l h o vo l u ntário em sites da web.
our search for "volunteer in Brazil" has yielded 111 e) Estrangeiros i n teressad os e m dese nvolver projetos de
Y Volunteer Abroad program results conservação do meio a m b i e n te.

Care Work in Brazil with Projects Abroad. Brazil is a beau­ L Dos projetos anunciados, somente três estão relacionados
tiful and exciting destination in South America to volunteer a/ao(s):
with disadvantaged children in orphanages or care homes. a) m e i o a m b i e n te.
Brazil may be the 9th largest economy in the world. But it b) edu cação.
is also a country with one of the largest disparities between c) saúde.
rich and poor. d) crianças e adolescentes.
e) morado res d e fave las.
PROJECT I
Community Work with Young Children in Rio de Janeiro 3. De acordo com o texto, é correto afirmar que:
These projects are both aimed at providing early childhood a) To d o s os p rojetos são d e s e n v o l v i d o s em c i dades l i ­
education and support to young at-risk children from a number torâ neas.
of Rio's favelas (urban slums). b) Os p roj etos são f i n a n c i ad o s p o r o rgan izações n ão
gove rnamentais.
PROJECT 2 c) N e n h u m projeto é d e d i cad o à prevenção de doenças.
Community Health Program Volunteer in Brazil - Recife d) Todos os p rojetos necessitam d e vo l u n tários jovens.
Volunteers will be rewarded as they help in various selfcon­ e) Pelo menos dois p rojetos são v o l tados para ate n d e r
tained community health centers, ali dedicated to the delivery pessoas s e m m o rad i a.
of general health care and disease prevention.

PROJECT 3 RESPOSTAS
Humpback Whale Conservation and Research, Rural Areas L Estrangeiros interessados em fazer trabalho voluntário no Bra­
Based in the beautiful area of Praia do Forte in Brazil, this sil. A resposta está no trecho "Your search for 'volunteer in Brazil'
project gives volunteers an opportunity to get involved in the has yielded 111 Volun teer Abroad program results", detalhada ao
long term conservation of the humpback whales. longo da descrição dos projetos, que têm como ponto em comum
a necessidade de volun tários para desenvolvê-los. Resposta: c
PROJECT 4
Work in an Orphanage - Recife z. Crianças e adolescentes. Somente três projetos mencionam
There are many disadvantaged children in Brazil. Most o f o trabalho com crianças e jovens: projeto 1: "These projects are
these children live at the city's municipal waste site. both aimed at providing early childhood education and support to
young at·risi< children". (Estes projetos são destinados a fornecer
PROJECT S educação infantil e apoiar jovens em situação de risco.) Projeto
Community Work in Rio de Janeiro 4: "There are many disadvantaged children in Brazil". (Existem
These projects are focused on improving the lives o f the m uitas crianças carentes no Brasil.) E o projetos: "These projects
young people who live in the 'favelas', or urban slums, o f Rio are focused on improving the lives o f the young people" (Estes
de Janeiro. projetos têm como foco a m elh o ria da vida de jovens.) Resposta: d

PROJECT 6 3. Todos os projetos são desenvolvidos em cidades litorâneas.


Building Homes Project in Brazil - Recife A respos ta correta não está explícita no texto. É preciso ler os
Our Building Homes Project in Brazil allows volunteers to títulos dos p rojetos, observar os n o m es das cidades (Rio de
construct houses for the homeless, work on the maintenance janeiro, Recife) e saber que são cidades litorâneas. No projeto
of public buildings, and reinforce flood barriers. 3, isso não basta. É preciso ler com atenção o in ício do texto e
identificar "Praia do Forte" como a base do projeto. Resposta: a
(UFMG 2009, adaptado)

48 1 GE IDIOMAS 2015
CONJ U N ÇÕES, P R E POSIÇÕ ES ... » RELAÇÕES ENTRE I DEIAS

Os marcadores discursivos ajudam a


compreensão detalhada durante a leitura

Marcadores discursivos sa.o palavras (conj u n ções e pre­ s u perada por o utros gêneros, principalmente variações d o
posições, principalmente) que estabelecem vários tipos de samba e rock ou h i p-hop em esti lo americano. Mas e l a ainda
re laça.o (de tempo, consequência, oposiça.o, condiça.o, adiça.o pode ser ouvida, u m a nota persistente que caracteriza a
etc.) entre as i dei as em um texto. Eles sa.o i m portantes para iconograficamente bela cidade à bei ra-mar.)
q u e o leitor possa entender a relaça.o entre argu mentos em 2. "If somebody asks me today where they can hear Bos­
u m texto, o u ainda a sequência cronológica d e eventos. sa Nova in Rio, I say 'no where'." (Se alguém me pergu nta
Observe o exem plo: hoje o n d e pode ouvir Bossa Nova n o Rio, eu d i go: e m l ugar
1. "In Rio de janeiro today, Bossa Nova has been sup­ n e n h u m .)
p/anted by other genres, notably o ther samba variations No primeiro exem plo, a palavra but (mas) i n d ica uma re­
and US-style hip-hop or rock. But, it can sti/1 be heard, a lação de oposiça.o, podendo ser su bstituída por s i n ô n imos
persistent note characterizing Brazil's iconically beautiful como however. já n o segundo exe m p l o, a conj u nção /f (se)
seaside city." (No Rio de jan e i ro de hoje, a Bossa Nova foi ind ica uma cond ição.

Soma, oposição1 exemplificação ...

Os mais i mportantes marcadores discursivos

ADIÇÃO: EXEMPLIFICAÇÃO:
what's more, moreover, in addition, and, furthermore, besides for example, for instance, such as
11 He to/d me he wou/d come here with his girlfriend and 11 Daniel plays a lot of sports, for instance volleyban tennis and

stay with us ali night. (Ele me disse que viria aqui com sua soccer. (Daniel pratica vários esportes, por exemplo voleibol,
namorada e ficaria conosco a noite toda.) tênis e futebol.)
11 jane said she had a /ot of work to do and couldn't come to the 11 We can try typical American dishes in this restauran� such as

party. Moreover, she wasn't feeling very we/1. Oane disse que the famous pumpkin pie and onion rings. (Podemos provar
tinha muito trabalho e não podia vir à festa. Além disso, ela pratos típicos americanos neste restaurante, tais como a
não estava se sentindo muito bem.) famosa torta de abóbora e anéis de cebola.)

OPOSIÇÃO/ CONTRASTE: CONDIÇÃO:


however, although, though, but, nevertheless if, unless, provided that
11 julie was not happy with herjob. However, she decided to 11 lfyou have any problem, just cal/ me. (Se você tiver algum

keep it for a while. Oulie não estava feliz com seu emprego. problema, me l igue.)
Entretanto, decidiu ficar nele um pouco mais.) 11 We wi/1 go to the show provided that it doesn't rain. (Vamos ao

11 Although Emily had been born in Chile, she didn't grow up there. show desde que não chova.)
(Embora Emily tenha nascido no Chile, ela não cresceu lá.) 11 Unless janet comes by car, we won't manage to take ali the

children to the c/ub. (A menos que janet venha de carro, não


CONSEQUÉNCIA: conseguiremos levar todas as crianças ao clube.)
because (o�, due to, consequently
11 Due to the heavy rains, the city has faced the biggest flood in OUTROS MARCADORES:
the last 10 years. (Devido às fortes chuvas, a cidade enfrentou a 11 Fortunately/Unfortunately (Felizmente/Infelizmente)
maior inundação dos últimos 10 anos.) 11 On the one hand... on the other hand (por um lado ... por outro lado)

11 Because ofyou I didn't trave/ on vacation. (Por sua causa não 11 On the plus side ... on the down side (no lado positivo ... no lado

viajei em férias.) negativo)

GE IDIOMAS 2015 1 49
I NT E R P R ETAÇAO D E TEXTO

Distant peak car


to car clubs, which offer rental by the hour in their neighbourhood,
and to car-sharing schemes. In particular, the generation who carne
o f age after 2000, the so-called "millennials", express a preference
Carmakers worry that one day demand for for having access to rather than owning cars. But some o f that may
be just talk. In a survey by McKinsey, American millennials said they
cars will stop rising. But that is a long way off expected to use car clubs in the future, but when asked if owning
a car would remain an important status symbol, they were much
more likely to answer "yes" than older consumers.
Economic factors, too, work against car ownership. Sheryl Con­
nelly, Ford's "global trends and futuring" manager, notes that a few
decades ago teenagers in America often got free driving lessons at
school, but now they may have to pay up to $800 for them before they
can sit their test. The cost of adding a young driver to the family's
car-insurance policy too has risen sharply, she says. In Britain the
RAC Foundation study found that fewer young men are driving
because their employers have cut back on providing company cars.
However, studies also show a marked rise in the proportion of
elderly people with driving licences. Baby boomers pretty much
all learned to drive, and now that they are beginning to retire they
expect to continue motoring. The development of assisted driving,
followed one day by fully automated cars, will allow them to stay
� THEY PREFER WALKING Youngs' interest in ouwning a car is decreasing mobile for much longer.
What may be happening in ri c h countries is a one-off shift in the

I
n 1924 Ford ran an advertisement headlined "His First Car", urging timing of people's driving careers, so that they start !ater but then
fathers to buy their teenage sons their first set of wheels. The idea continue well into old age. This may be no bad thing for carmakers. It
caught on. For boys, especially, learning to drive became an essential has long been an open secret in the business that cars are advertised as
part o f growing up. By the late 1970s 86% of American 18-year-olds being for the young but are bought mainly by the middle-aged with the
- ofboth sexes - had a driving licence. But then the trend went into necessary disposable income. In America the average Mercedes buyer
reverse: researchers at the University ofMichigan found that in 2010 is in bis late 50s, and even the supposedly youth-oriented Mini Cooper
only 61% of18-year-old Americans had licences. Other rich countries is typically bought by people in their early 40s. The world's biggest
are going the same way. Teenagers are showing less interest in cars car markets - China, North Ame rica and Europe - are ali greying.
as they turn their attention to smartphones and social networking. So it is not ele ar that declining c ar ownership among young ur­
This is a worry for carmakers, who are wondering where their future banites will have more than a marginal effect on overall car sales.
customers are going to come from. In the two decades to 2008 the Besides, argues Renault-Nissan's Mr Ghosn, for most people "their
number of miJes driven by Americans in their 20s fell by 8%. In Britain car is more than an object." For some it is an extension of their home,
a study for the RAC Foundation, a transport-research body, found a he says, and most people would rather not share their home. For
30% drop among men in the same age group between 1996 and 2006. others it is their pet, and who wants to share their pet?
One reason for concern is that half the world's population now lives Ali in ali, "peak car"- the point at which worldwide demand for
in towns and cities, which have only so much space for cars. Even in cars will stop rising - still seems quite a long way off. I n the rich
rapidly growing car markets such as China, city governments in the world some of the economic factors that have deterred young people
more prosperous parts of the country are beginning to restrict new from taking up driving will fade away: as cars beco me increasingly
car registrations and invest heavily in public transport. self-piloting and accident rates fali, insurance costs should decrease,
Young urban residents may also be meeting up less often in person, and in time there will be little o r no need to take expensive lessons.
thanks to social-networking sites that let them keep in touch digitally.
So they have less need for a car, and when they do need one they turn The Economist, Apri1 2oth, 2013 (Vestibular ITA 2014)

EXERCÍCIOS - ITA 2014 (adaptado)

1.U ma das razões para o menor uso de carros por jovens nos 2. Assi nale a opção em q u e a reti rada do termo em negrito
ú ltimos anos é o(a): com promete o sentido da oração.
a) desinteresse em u sar carro como símbolo de status social. a) "For boys, especially, learn i n g to d rive became an essential
b) realização de festas e m c l u bes particu lares. part of gro w i n g u p".
c) falta d e segu rança nas grand es cidades. b) " ... to restrict new car registrations and i nvest heavily i n
d) u s o d e redes sociais d i gitais. p u b l i c transport".
e) i n suficiência d e estacionamentos e alto custo das vagas c) " ... they were m u c h m ore l i kely to answer 'yes' than older
p rivativas. c o n s u mers".

50 I GE IDIOMAS 2015
d) "The cost o f add i n g a you n g d river to the fami ly's car­ b) Da mesma forma, "hea vily" (pesada me n te), se retirado, não
·insurance pol i cy too has risen sharply ... . " muda o sentido de "investir em transporte p ú b lico". Incorreta.
e) " ... cars are advertised as b e i n g fo r the you n g b u t are c) já "Jikely" na frase em questão não é um advérbio, mas um adjetivo
bought mainly by the m i d d l e-aged ... . " que faz parte da expressão "to be Jikely" (ser provável, ser propenso).
Portanto, se for retirado, a frase perde o sen tido. Correta.
J. Considere as sentenças: d) "Sharply" (acentuadamen te) é um advérbio q u e dá ê n fase a o
I. A geração millennials não se i ncomoda com status social. verbo "ha s risen". S e eliminado, não altera o sentido da frase.· "o
11. A geração millennials já atingiu os 40 anos de idade. custo por acrescentar um m otorista jovem à apólice de seguro
111. A geração ba by boomers faz parte dos apreciadores da de a u tomóvel familiar tem a u m e n tado". Incorreta.
fabricante de carros Mercedes. e) "Mainly" {principalmen te) também é um advérbio que, cortado,
Está(ão) correta(s): não altera ria o sen tido da frase. Veja: "ca rros são apresentados
a) apenas I . como objeto para jovens, mas são comprados por pessoas de
b) apenas l l . meia-idade." Incorreta.
c) apenas 111. Resposta: c
d) apenas I e 11.
e) apenas I e 111. 3. Analisando cada uma das alternativas:
I. A geração do milênio ainda considera o carro próprio um símbolo
4. Assi nale a o pção e m q u e o e m prego s i n tático d o item de status. No trecho ".. .when asked if owning a car wou/d remain
lexical that é d iferente dos demais. an important status symbol, they were much more likely to answer
a) ".. researchers at the University o f M ich igan found that...".
. 'yes'... " (quando perguntados se possuir um carro ainda é um símbolo
b) "One reason for concern is that...". de status, eles são muito mais propensos a responder 'sim'). Incorreta.
c) " ... thanks to social-networking sites that ...". 11. Veja o trecho "the generation who carne o f age after 2000, t h e
d) "She ryl C o n n e l ly, Ford's "global tre n d s and futuring" so-called 'millennials". "Carne o f a g e" significa "atingir a maio­
manager, notes that...". ridade". Então, as pessoas da geração do milênio de q u e trata o
e) "So it is not clear that...". texto teriam, em 2014, perto de 30 anos. Incorreta.
111. "Baby boomers" refere-se às pessoas nascidas na explosão de
s. De acordo com o texto, a expressão "a long way off" pode natalidade entre os anos 1946 e 1964. Então, os baby boomers
ser entendida como: têm, hoje, entre so e 68 anos de idade. Analise, agora, a frase "in
a) emi nente retrocesso. Ame rica the average Mercedes buyer is in his late sos" (nos EUA,
b) acontecimento a longo prazo. o comprador médio de Mercedes tem perto de 5o anos). Como
c) grande possibilidade. essa idade corresponde à idade dos baby boomers, concluímos
d) evento fora de cogitação. que essas pessoas fazem parte da geração que aprecia carros
e) preocu pação factível. M ercedes. Correta.
Resposta: c

RESPOSTAS ft, Analisando as alternativas:


L Analisando as alternativas: a) Na sentença apresentada, (..pesquisadores da Universidade de
a) Para a geração do milênio, o carro ainda representa um símbolo Michigan descobriram que ...), "that" é uma conjunção (usada
de status. A ideia está explícita no trecho "... when asked if owning para ligar ou correlacionar duas orações).
a car would rema in an important status sym bol, they were much b) "Tha t " também é conjunção nesta frase (uma razão para preo·
more Jikely to answer 'yes' than o/der consumers." Incorreta. cupação é que.. . ).
b) A única menção a "club" está no trecho "... they turn to car clubs, c) Mas nesta sen tença ( .. graças aos sites de redes sociais que .. } . ,

which offer rental by the hour in their neighbourhood..." (eles recor­ "that" tem a função de pronome relativo. Refere-se a "sites de
rem a clubes, que alugam por hora em sua vizinhança). Incorreta. redes sociais" e pode pode ser substituldo por "os quais".
c) Não há referência a falta de segurança. Incorreta. d) Na frase proposta nesta alternativa (Sheryl Connelly, gerente de
d) Veja o trecho. "Teenagers are showing Jess interest in cars as they tendências globais futuras da Ford, observa que . . .), n o va m e n te
turn their attention to smartphones and social networking" (Os "that" é uma conjunção.
adolescentes estão mostrando menos interesse por carros, uma vez e) Nesta expressão, "that" é conjunção: "Então, não está claro que .. "
.

que voltam a atenção para smartphones e redes sociais). Correta. Resposta: c


e) O texto não menciona custos com estacionamento. Incorreta.
Resposta: d §. A expressão "a long way o ff" significa "distante no tempo ". Em
tradução livre, a sentença "ali in ali, 'peak car'... sti/1 seems quite a
z.Analisando as alternativas: long way off" poderia ser traduzida como "de modo geral, a estag­
a) O advérbio "especial/y" (principalmen te, especialmente) pode ser nação da demanda global por carros ainda parece muito longe de
retirado da frase sem comprometer o sen tido: "Para os rapazes, acon tecer". De todas as alternativas apresentadas, o termo que mais
aprender a dirigir tornou-se parte essencial do crescimento". corresponde a esse significado é "acontecimento a longo prazo".
Incorreta. Resposta: b

GE IDIOMAS 2015 1 51
DICAS APLICAÇÕES » PASSADO Q U E CONTI N UA

Entenda as aplicações
do presente perfeito

O bserve as frases a seg u i r: Tam b é m existe o Present Perfect Conti n uo u s, obtido com
Gilberto (. . .) has not sung in public in Brazil for five years. o u s o de have/has been + a forma d o gerú n d i o d o verbo
( G i l b e rto n ão canta e m p ú b l ico no Bras i l há 5 anos.) pri nci pal. Pode ser u sado para:
His reputation, though, has never diminished, ever since 1. E nfatizar a conti n u i d ad e de u m a ação q u e começou no
August 1958. (Mas sua reputação n ão d i m i n u i u d esde agosto passado e perd u ra até o presente:
de 1958.) » She's been living here for 10 years. (Ela vive/m o ra aq u i há
As formas verbais em d estaq u e são exem pl o s do tem po 10 anos.)
verbal p resente perfeito. E m i nglês, o p resente p e rfeito é » They've been working in the same company since they
u sado para: moved here, in 1989. (Eles trabalham na mesma e m p resa
» Falar de ações q u e começaram no passado e conti n uam desde q u e se m u d aram para cá, em 1989.)
até o tem po p resente. 2. Para i n d i car u ma ação q u e acabou d e ser finalizada, mas
She has had this car for 2 years. ( E l a t e m esse carro há c ujo res u l tado é v i s ível no presen te:
2 anos.) Look at you! What h ave you been doing? (Ol h e para você!
David and Meg h ave been married since 1999. (David e Meg O q u e você estava faz e n do?)
estão casados desde 1999.) l've been running. Do l look that bad? (Eu estava/estive
» Tratar de ações q u e acon teceram em um tem po i n deter· correndo. Pareço tão mal assi m ?)
m i n ad o no passado e q u e têm resu ltado no p resente. Atenção: para marcar o perfodo d e d u ração d e u m a ação
jessica has had a baby! (Jessica teve um bebê.) até o presente, u samos a preposição for. Para marcar o i n ício
Look at you! You've grow n up! ( O l h e para você! Você d e uma ação, u samos a preposição since. Observe:
c resceu !) He's been working here since 2000. ( E l e trabal h a a qu i
» Falar d e experiência de vida. desde 2000.)
Have you ever flown a hot air bal/on? Yes, I have. Once. He's been working here for 10 years. ( E l e trab a l h a aq u i
(Você já voou em u m balão de ar q u e n te? )á. U m a vez.) há 1 0 anos.)
I h ave never eaten ra w fish. ls it good? ( N u n ca c o m i peixe
cru. É b o m ?) EXEMPLO DE CONJUGAÇÃO NO PRESENT PERFECT:
» Falar d e ações re petidas (e q u e p o d e m c o n t i n u a r a s e
repeti r). Conjugação do verbo "to eat" (comer)
She has written four books. ( E l a escreveu q u atro l ivros.)
How many songs has he recorded? About 20. ( Q u a n tas
m ú sicas e l e grav o u ? Em torno d e 20.) I INiíi'I:'F.IITlíif.'l<I'F.I I .. I
"
• :·
I
I have eaten [raw fish] I haven't eaten [raw fish] Have I eaten [raw fish]?
O P r e s e n t P e rfect é o b ti d o c o m o u s o d o v e r b o a u x i ­
l i ar havejhas + parti c í p i o passado d o verbo p r i n c i pal. Os
advérbios mais usados com o Prese nt Perfect são:
l You have eaten You haven't eaten Haveyou eaten?
J
He has eaten He hasn't eaten Has he eaten?
Always/never

I I
l've always wanted to become a doctor. (Sem pre q u i s m e
She has eaten She hasn't eaten Has she eaten?
tornar u m m é d ico.)
She's never seen a film fike that. ( E l a n u n ca v i u u m f i l m e
lt has eaten lt hasn't eaten Has it eaten?
como e s s e antes.)

Already/ever
Lucy has already had /unch. (Lucy já a l m oçou).
I We have eaten We haven't eaten Have we eaten?
J
You have eaten You haven't eaten Have you eaten?
Haveyou ever seen Gone with the W i nd? (Você j á assisti u

I I
a ... E o Vento Levou?)
They have eaten They haven't eaten Have they eaten?

j usttyet
Mark h as just arrived. (Mark acabo u de c hegar.)
David hasn't done his homeworkyet. (David ainda não fez
s u a l i ção de casa.)

52 1 GE IDIOMAS lOIS
G RAMÁTI CA » O APÓSTROFO CO M "5" E P H RASAL V E R BS

Recorrente na língua inglesa, o 's é usado


frequentemente de forma incorreta no Brasil

Em i ng lês, u samos o. apóstrofo + s ('s) para expressar »Phrasal verbs com sentido não literal: He looked this word
relações de: up in the dictionary to check its meaning. ( E l e procu ro u a
» Posse palavra no d ic i o nário para encontrar o s i g n i fi cado.)
My sister's na me is }anet.
(O nome da m i n h a i rmã é janet.)
>> Paren tesco
Livia's mother is from Rio.
(A mãe de Uvia é do Rio.)
» Associação em geral
This week's biography focuses on Michael}ackson.
(A biografia d esta semana foca-se em M ichael )ackson.)
O apóstrofo + s é sem pre colocado entre o poss u idor e a
coisa possu ída. Caso o poss u i dor seja u m n o m e no p l u ral,
basta acrescentar o apóstrofo. Mas atenção: se o p l u ral for
irregular, conti n u a val e n d o a regra geral. O bserve:
My relatives' farm fies in Tocan tins. (A fazenda dos m e u s
parentes fica e m Tocanti ns.)
The children's toys wi/1 be in this room. (Os brinq uedos das
crianças ficarão n este q u arto.)
O apóstrofo + s pode ser u sado também para marcar as
formas contraídas em tercei ra pessoa dos verbos to be (is) ·

ou to have (has).
She's 19 years old. (She's She is) (Ela tem 19 anos.)
=

She's never studied piano before. (She's She has) (Ela


=

nu nca estudou piano.)

PH RASAL VERBS
Phrasal verbs são combi nações entre verbos e preposições
ou advérbios q u e, em algu n s casos, ass u m e m um sentido
novo, d i ferente d o s i g n ificado d as palavras isolad as. Por
exemplo, na frase "The three concerts tickets sold out within
an hour o f going on safe Thursday", observamos a forma
do passado d o phrasal verb sold out (passado d e sei/ out).
Isoladamente, sei/ s i g nifica "ven d e r" e out s i g n ifica, n o seu
sentido mais geral, "para fora". O sentido d e sei/ out não é a
com b i n ação dos sentidos d e sei/ + out ("ven d e r para fora"),
mas "estar esgotado/esgotar-se". Assi m, temos, e m u m a
tradu ção l ivre, a segui nte i d e ia: "Os i ngressos para o s três
concertos se esgotaram e m menos de u m a hora na q u i nta".
» Phrasal verbs com s e n t i d o l i te ral: He /ooked around to
check i fyou were there. ( E l e o l h ou em volta para verificar
se você estava l á.)

GE IDIOMAS 2015 1 53
I NTERP RETAÇAO DE TEXTO

Another reason to
choose a mate wisely
Should one be happy just for being married? A research tries to prove that this is not that simple

H
appily married people tend to have lower blood pressure than Lunstad, the lead author and an assistant professor of psychology at
their single peers, but being single may be healthier than being Brigham Young University.
unhappily married, a new study suggests. The study, published on On average, the unhappily married had higher daytime and 24-
March 20 in The Anna/s ofBehaviora/ Medicine, sampled 303 generally hour blood pressure readings than single people. Having a wide social
healthy men and women, 204 married and 99 single. Each responded network had no effect on the trends for either married or single people.
to questions about marital quality, social support and mental health. But marital satisfaction was significantly associated with satisfaction
Scales were used to rate stress and life satisfaction. Then each subject with life, lower stress, less depression and lower waking blood pres­
wore a portable blood pressure monitor for 24 hours while performing sure. ':Just being married per se isn't helpful;' Dr. Holt-Lunstad said,
their normal activities. "because you can potentially be worse off in an unhappy marriage.
The groups had little difference in waking blood pressure, but So, choose wisely".
married people had significantly larger dips in blood pressure during
sleep. People whose blood pressure does not dip during sleep are at (vestibular da Unifesp 2009. Por Nicholas Bakalar, em
higher risk for cardiovascular disease, according to Julianne Holt- http://query.nytimes.com/gst/fullpage.html, Apri1 1, 2008. Adaptado)

54 1 GE IDIOMAS 2015
EXERCfCIOS - U N I FESP 2009 (adaptado)

L Accord i n g to the text: 6. No trecho " ... but being single may be healthier than being
a) lt is better to be married rather than si ngle. unhappily married ... ", a palavra s u b l i n hada i nd i ca:
b) Happ i ly married cou p l es h ave a normal blood press u re a) u m a consequê ncia.
and th erefore have a better h ealth. b) um exe m p lo.
c) Blood press u re s h o u l d i n c rease d u ri n g s l eep; otherw ise, c) u m a c o n d i ção.
there would be a h ig h e r risk for cardiovascular d i sease. d) uma h i pótese.
d) The wo rst resu l t of the study was s h o w n by u n hap p i ly e) u ma com paração.
married people.
e) Social network and close friends make s i ngle people m u c h
happ ier t h a n married people. RESPOSTAS
1. The worst result o f the study was shown by unhappily married
z . O estudo apresentado pelo texto: people. A resposta está no seguinte trecho: "But marital sa tisfac­

a) Consist i u em u m a pesq u i sa com 303 casais. tion was signifjcan tly associated with satisfaction with life, lower
b) Comparou a p ressão san g u ínea d e pessoas soltei ras com stress, less depression and lower waking blood pressure. )ust being
a das casadas. married per se isn't h elpful� O r Holt·Lunstad said, 'beca use you can
c) Concl u i u q u e a f l u tuação da pressão sangu ínea é a mesma poten tially be worse off in an unhappy marriage"'. {Mas a satisfação
entre as pessoas casadas e solte i ras. no casamento mostrou estar signifjca tivamente associada à satisfa­
d) I nd icou que as pessoas que são i nfelizes no casam ento têm ção com a vida, menor estresse, menos depress.�o e pressão arterial
prob lemas semel h antes aos d as pessoas solteiras. m a is baixa ao acordar "Apenas ser casado em si não ajuda'; explica
e) Mostrou que as pessoas solteiras sofrem mais de dep ressão a doutora Holt-Lunstad, ''porque você pode, potencialmen te, estar
e estresse do q u e as mal casadas. pior e m u m casamento in feliz".) Resposta: d

J. In the excerpt o f the last paragraph "5o, choose wise/y", the z. Comparou a pressão sanguínea de pessoas solteiras com a das
word so can be replaced, without changi ng its mean i ng, for: casadas. A resposta es tá no trecho "Happily m a rried people tend
a) therefore. to h a ve l o we r blood pressure t h an their single peers, b u t being
b) otherwise. single may b e healthier t h a n b ein g u n h a p p i ly m a rried, a n e w
c) neve rtheless. study suggests. The s t u dy, published o n March 2 0 i n T h e Annals
d) moreover. of Behavioral M e d i c i n e, sampled 303 genera lly h ealthy m e n a n d
e) fu rthermore. women, 204 married and 99 single. . " (Pessoas em casamentos felizes
ten dem a ter pressão arterial mais baixa do q u e os s o l t eiros, mas
4- No trecho d o segu ndo parágrafo " The groups had little ser solteiro pode ser mais saudável do q u e estar casado e i n feliz, é
difference in waking b/ood pressure, but married peop/e had o que sugere um novo estudo, publicado em 20 de março na revista
significantly larger dips in blood pressure during sleep", a Anais de M e d i c i n a Com portamental. No est u d o, u m a a m o stra
palavra groups refere·se a: de 303 homens e m u l h eres com bom estado de saúde geral, 204
a) married people. casados e 99 solteiros.. .) Resposta: b
b) single people.
c) h ealthy m e n and women. 3. Therefore. O s e n tido é de "por isso", "portanto" Resposta: a
d) u n happily married p e o p l e.
e) married peo p l e and s i ng l e people. t,. Married people a n d single people. No texto, localiza m os o
n ú m ero 303 (o grupo total de participantes da pesquisa, h o m ens
s. De acordo com o texto, é incorreto afi rmar que: e mulheres) e a in formação "204 married and 9 9 single" que in dica
a) A rede d e relac i o namentos teve efeito nas te ndê ncias d e dois grupos, ou s eja, os casados e os solteiros. Resposta: e
ambos casados e soltei ros n o estudo.
b) O estudo foi p u b l icado rece nteme nte. s. A rede de relacionamentos teve efeito nas tendências de ambos
c) A pesq u i sa foi realizada na área d e Psicol ogia. casadas e solteiras n o estudo. No texto temos "Having a wide social
d) O estudo u t i l i zou-se de u m bom n ú mero de participantes. network had no effect o n the trends for either m arried a r single
e) Os m e d i d o re s de p ressão s a n g u í n ea foram u t i l izados people'� o u seja, "ter uma ampla rede de relacionamentos não teve
d u rante 24 h o ras. efeito nas tendências de ambos casados o u solteiros". Resposta: a

6. Uma comparação. No trecho indicado o adjetivo "healthy" (sau­


dável) está na for·ma do comparativo, ou seja, "he a l t h ier" (mais
saud,ivel). Resposta: e

GE IDIOMAS 201S I 55
Aprenda a identificar e a utilizar a voz passiva
em inglês em várias situações

Observe os segui ntes pares de frases: Nos casos de verbos transitivos diretos e indiretos, é comum
1) The Mayan civilization worshipped the cacao tree. obter-se a voz passiva com o objeto indireto na posiçiio de
2) The cacao tree was worshipped by the Mayan civilization. sujeito da frase. Observe:
3) The Guarani have revered the guarana berries for centuries. Tom was given the car key. (Deram a chave do carro para
4) Guarana berries have been revered by the Guarani for Tom.)
centuries. Em algumas situações, a voz passiva corresponde à.Partícula
Nas frases 1 e 3, notamos a organização estrutural com a apassivadora "se" em português:
sequência sujeito + verbos + complementos, na qual o sujeito lt is believed that consuming big quantities of omega 3
da frase é aquele que dese m penha a ação verbal. Na frase 1, he/ps protecting the heart against attacks. (Acredita-se que
era a civilização maia que venerava o cacaueiro; na frase 3, cons u m i r grandes q uantidades de ômega 3 ajuda a proteger
são os guaran is q u e reverenciam as sementes de guaraná. o coração contra ataques cardíacos.)
Temos, então, frases na voz ativa.
Nas frases 2 e 4, as situações expressas são as mesmas, mas EXEMPLO DE USO DA VOZ PASSIVA
há uma m udança de foco: a i nformação oferecida em primeiro
l u gar não é quem faz a ação, mas quem ou o que sofre a ação
do verbo (o cacaue i ro era venerado, as sementes do guaraná
são reverenciadas). Temos, então, frases na voz passiva.
A voz passiva é constru ída uti l izando-se u m a forma d o
verbo t o be + particfpio passado do verbo principal. O verbo
to be varia em função do tem po verbal da frase. No caso do
exemplo 2, temos a frase no Simp/e Past, daí o uso da forma
was. Na frase 4, temos o Present Perfect, daí o uso da forma
have been. O quadro ao lado sistematiza a estrutura da voz
passiva em diferentes tempos verbais.
A voz passiva é bastante com u m em notícias e manchetes
de jornais, quando fatos são o foco da i nformação. Exemplo:
2o,ooo injured by Katrina Hurricane. (2o ooo feridos pelo fu­
racão Katrina.)
N e m s e m pre é i m po rtante m e n c i o nar o agente da voz
passiva, mas, quando necessário, o agente é introduzido pela
preposição by: The painting The Su nflowers was painted by
Van Gogh. (A tela Os Girassóis foi pi ntada por Van Gogh.)
f: possível constr u i r a voz passi va com verbos m odais
também. Pode-se, por exemplo, usar o verbo modal + have been
+ particípio passado do verbo principal da frase. Observe:
The Picture must have been stolen by someone who knows
the museum we/1. (O quadro deve ter sido rou bado por alguém
que conhece bem o m useu.)
am/is/are + The song is going A música será
Melanie could have been informed about the h o tel
Going to goir.g to be + to be performed in executada em
cancellation. (Melanie poderia ter sido avisada do cancela­
pastparticiple 5 minutes. s minutos.
mento do hotel.)

56 ( GE IDIOMAS 2015
.: .·:�-� COMPARATIVO E SU PERLATIVO » SU PERIORI DADE, I G UALDADE OU I N FE RIORI DADE

Saiba como são feitas as


comparações entre adjetivos em inglês

Ao faze rmos c o m parações usando adjetivos p o d e m o s SUPERLATIVO


estabelecer relações d e s u perioridad e, i n ferioridade o u Usamos adjetivos na forma do s u perlativo para i n d i car
igualdade. O bserve: que u m elemento dentro de u m grupo de pessoas ou objetos
Happily married people tend to have /ower blood pressure tem destaq ue, tanto do ponto de vista de sua s u perioridade
than their single peers. (Pessoas casadas e felizes tendem a como de sua inferioridade. N essa estrutu ra, os adjetivos são
ter menor pressão sangu ínea do que as solteiras.) preced idos do artigo the (o, a, os, as). Observe:
Your notebook is smaller than m in e. (Seu notebook é Lisa is the most intelligent student in class. (Lisa é a aluna
menor que o meu.) mais i nteligente da sala).
Professor Clark's classes are as interesting as Professor Suzie's shop is the largest in town (A loja da Suzie é a maior
Higgins's. (As aulas do professor Clark são tão interessantes da cidade).
quanto as do professor H iggins.) Como no caso do comparativo de su perioridade, é preciso
His new shop is /ess attractive than his o/d one. (A nova observar a d iferença entre adjetivos curtos e longos q uanto
loja dele é menos atrativa do que a anti ga.) à forma do superlativo. Adjetivos c u rtos têm acréscimo d o
sufixo -est; adjetivos l ongos são preced idos da palavra most.
SUPERIO R I DADE Exemplos:
Para fazermos com parações u sando adjetivos é preciso He is the oldest boy in the soccer team, but he is the most
lembrar q u e eles são organizados em dois grupos: os short talented one (Ele é o mais vel h o no time de futebol, mas é o
adjectives (adjetivos cu rtos, aqueles que possuem uma sílaba, mais talentoso).
ou duas em alguns casos) e os /ong adjectives (longos, duas
sílabas o u mais). Exe m p l os: COMPARATIVO E SUPERLATIVO
» Short adjectives: big, sma/1, fa ir, o/d, new, young, high, low, As variações dos adjetivos irregulares
happy, sad.
» Long adjectives: modem, cosmopolitan, beautiful, extra­

ordinary, handsome.
Os adjetivos c u rtos têm sua forma do com parativo de su­
perioridade com o acréscimo d o sufixo -er. J á os l ongos são
precedidos pela palavra more. Todas as formas de comparação
são acompan hadas da palavra than (do q ue). Observe:
john is o/der than Mary. (Jo h n é mais vel h o que Maria.)
São Paulo is more cosmopolitan than Botucatu.
(São Paulo é mais cosmopolita do que Botucatu.)
Linda /ooks happier than Brad. (Linda parece mais feliz
do que Brad.)

IGUALDADE
Para estabelecer u m a re l ação de i g u a l d a d e u sa m o s a
expressão asjas. Para i n d i car u m a relação de desigu al d ad e,
usamos not as/so ( ... ) as. Exemplos:
Tom is as tal/ asjane. (Tom é tão alto q uanto jane).
Ed is n o t as brig h t as }eff. ( E d n ão é tão b r i l h a n te
q uanto jeff.) CASOS ESPECIAIS
Adjetivos de duas sílabas terminados em "/e", "ow'� e "er"
INFERIORIDADE aceitam as duas formas.
Usamos a expressão less/than (menos/do q u e) para esta­
belecer relações de i nferioridade. Exe m plo:
Organic products are less harmful to ou r health than the
traditional ones. (Produtos orgânicos são menos prej ud iciais
à nossa saúde que os trad icionais.)

GE IDIOMAS 201S 1 57
I NTERPRETAÇAO DE TEXTO

Health crisis
The World Health Organization (WHO) estimates that 50 per cent
of ali medicines sold online are worthless counterfeits

.olllll WORSE than receiving bad news


from a doctor telling you have your
son has a serious disease is knowing
that the medicine he was taking was
fake and that's why he died

T
h e p e rils of c ounterfeit drugs go way beyond being i f not market-aware: in the face of an outbreak of HSNl bird flu in
ripped off by dubious online pill-pushers. The World Health 2005, they began offering fake Tamiflu.
Organization (WHO) estimates that 50 per cent of ali medic ines What can be clone? The WHO coordinates an umbrella body
'
sold online are worthless counterfeits. In developing nations fake pills called the International Medicai Products Anti-Counterfeiting
may account for as much as 30 per cent of ali drugs on the market. Taskforce (IMPACT), an industry initiative that issues alerts when
Even in the developed world, 1 per cent of medicines bought over it finds anomalies in the medicine supply chain. Such events include
the counter are fakes. sudden drops in wholesale prices, hinting at fakes coming onto the
Some key events illustrate the risk these pose. In Nigeria, 2,500 market, or the mimicking o f anti-counterfeiting features on packaging,
children died in 1995 after receiving fake meningitis vaccines. In such as holograms or barcodes, says Nimo Ahmed, head of intelligence
Haiti, Bangladesh and Nigeria, around 400 people died in 1998 after at the UK's Medicine and Healthcare Products Regulatory Agency.
being given paracetamol that had been prepared with diethylene
glycol - a solvent used in wallpaper stripper. The fakers are nothing (Fuvest 2011 - 1' fase New Scientist, )uly 10, 2010, p. 18. Adaptado)

58 1 GE IDIOMAS 2015
.... .. . . . .. .. . . . . . . . . . . . . . . . . . . . . . . . . . .. . . . . . . . . . . . .. . . . . . . . . . . . . . . . . . . .. . . . . . . . . . . . . . . . . . . . . . . . . . . . .. . . . , . . . . . . . . . . . . . . . . . .. . . . . . . . . . .. . . . . . . . . . . . . . . . . . . . . . . . . . . . . . . . .. . . . . . . . . . . . . . . . . . . . . . . . . . . . . . . . . . . . . . . . . . . . . . . . . . . . . . . . . . . . . .
EXERC[CIOS - FUVEST 2011 (adaptado)

L De acordo com o texto, med icamentos falsificados, em RESPOSTAS


geral: 1. São ineficazes e contêm elementos danosos à saúde em sua
a) São c o n s u m i d o s a p e n as e m países p o b res e de p o u co composição. A resposta encon tra-se em "In Nigeria, 2,500 children
acesso à i n t e r n et. died in 1995 after receiving fake men ingitis vaccines. In Haiti,
b) Encon tram d i f i c u l dade d e c o m e rcial ização c o m o apare­ Bangladesh a nd Nigeria, a ro u n d 400 people died in 1998 a fter
c i m e n to d e novas doe n ças. being given paracetamol that had been p repared with diethylene
c) São i n eficazes e contê m e l e m e ntos d a n osos à saúde em glycol - a solvent used in wa!lpaper s tripper". {Na Nigéria, 2. 500
sua c o m p o s i ção. crianças m o rreram em 1995 depois de receberem vacinas falsas
d) Possuem e m balagens atraentes que l u d ibriam o consu m id or. de m e n ingite. No Haiti, Bangladesh e Nigéria, aproximadamente
e) Vêm sendo criteri osamente apree n d i d os pela O rganização 400 pessoas morreram em 1998 depois de receberem paracetamol
M u n d i al da Saúde. que tinha sido preparado com glicol dieti!en o - um solvente usado
em papel de parede.) Resposta: c
z. O texto i nforma q u e os falsificadores:
a) Atuam n a venda d e re m é d i o s n o m e rcado atacad ista. z. Estão sempre alerta à demanda do mercado. A respos ta está
b) Ro u bam o selo de q u a l i dade da O rgan ização M u n d ial da em: "The fakers are nothing i f not market-aware: in the face o f an
Saú de. outbreak o f H5N1 bird nu in 2005, they began offering fa ke Tamifiu".
c) U t i l izam p l acebo nos m e d i ca m e n tos. (Os falsificadores estão conscien tes das demandas do mercado: com
d) Apresentam-se c o m o representantes oficiais da i n d ú stria o surto do H5N1· gripe a viária, eles começaram a oferecer Ta mifiu
fa rmacêuti ca. falsificado.) Resposta: e
e) Estão s e m pre alerta à d e m a n d a do m e rcado.
J. Coordena o trabalho de uma organização que acompanha o
3.Segundo o texto, para conter a venda de medicamentos fornecimento de remédios no mercado farmacêutico, alertando
falsificados, a O rganização M u nd ial da Saúde: para possíveis irregularidades. A resposta está em "The WHO
a) Esti m u l a a v e n d a p ro m oc i o n al de m e d i ca m e ntos i m po r­ coordinates an u m b rella body called the Internacional Medicai
tantes s e m p re q u e n ecessário. Products An ti-Counterfeiting Taskforce (IMPACT), an industry in i·
b) Coo rdena o traba l h o d e uma o rgan ização q u e aco m p a n h a tia tive that issues alerts when it finds anomalies in the medicine
o fornec i m e n to d e re m é d i o s n o m e rcad o farmacêuti co, supply chain". (A OMS coordena uma organização chamada Força­
alertan d o para possíve i s i r reg u l aridad es. -Ta refa Internacional contra a Falsificação de Produtos Médicos
c) Exige que todos os m e d i camentos exi bam o h o l o g rama da (IMPACT), uma in iciativa da indústria que e n via a lertas quando
orga n i zação e o c ó d i g o de barras. encon tra anomalias na cadeia de suprimento médico.) Resposta: b
d ) Controla o l a n ça m e nto d e n ovos m e d i ca m e ntos n o m e r­
cado, a exe m p l o do Tam i f l u . 4. "ripped off" - exploited. "Ripped off" significa explorado. As
e) Autoriza apenas a c o m ercial ização d e med icamentos q u e o u tras alternativas estão erradas.· "drops" é cair; "even" significa
passaram pelo crivo das agências sanitárias i n ternacionais. até mesmo "on the other hand" é p o r o u tro lado; "key" é chave/
importante; "features" significa características. Resposta: e
4. Os termos transcritos à esquerda q u e, sem alteração de
significado, podem ser s u bstituídos pelos apresentados à s. A indústria farmacêutica tem agido para conter a venda de
direita são os segui ntes: medicamentos falsificados. A resposta está no mesmo trecho que
a) "featu res" - n u m be rs. responde a questão 3. A Força- Tarefa Internacional con tra a Falsi­
b) "drops" - i n creases. ficação de Produtos Médicos é uma iniciativa da indústria, apesar
c) "Even" - O n the oth e r h a n d . de coordenada pela OMS. Resposta: d
d) "key" - u n i m portant.
e) " r i p p e d off" - ex p l o i ted.

s.De acordo com o texto, é correto afirmar que:


a) N e n h u m a ação d a i n d ú stria farmac ê u t i ca tem obtido su­
cesso para c o m bater a fal s i f i cação d e m e d i ca m e n tos.
b) To d o s o s m e d i ca m e ntos v e n d i d o s nas far m á c i as usam
agora um selo d e garantia d e q u a l i d ade.
c) Os países da Af r i ca são os ú n icos que sofrem c o m a v e n d a
de m ed i ca m e n tos falsi ficados.
d) A i n d ú stria far m acê u ti ca tem agido para c o n ter a v e n d a
de med icame ntos fal s i f i cados.
e) Os contrabandistas i n ternacionais têm tido mais d ificu ldade
e m c o m e rcial izar m e d i camentos fal sificados nos EUA.

GE IDIOMAS lOlS 1 59
DICAS R EG Ê N C I A » AS LE I S DA R EG Ê N C I A V E RBAL

Como dois verbos podem ser combinados


de forma diferente nas frases

As d iversas maneiras de combi nar verbos são con hecidas verbos podem ser acompanhados tanto do infin itivo q u anto
como verb patterns. As d uas princi pais formas de combi na· do ge rúndio, sem m udança de significado, e, em outros, a
ção são pelo u so do i nfin itivo ou do gerú ndio. Em alguns escolha do i nfi nitivo ou do gerúndio determ ina a mudança
casos, o i n fi n itivo é util izado sem o to. H á casos em que os de significado.

Quando os verbos se encontram


Os d iversos padrões de combinação

!. INFINITIVO COM TO 3. VERBOS/EXPRESSÕES VERBAIS + GERÚNDIO


Need (precisar) My friends need to do some research about this topic. Enjoy (gostar), can't stand (não suportar}, suggest (sugerir), give up
(abandonar, desistir), keep (manter}, mind (importar-se, incomodar-se),
Agree (concordar) I agreed to go to your house.
avoid (evitar).
Refuse (recusar) Michele refused to speak to her husband.
Can't stand (não suportar) I can't stand Jistening to Joud music.
Afford (arcar com) We cannot afford to buy a new car.
Give up (abandonar, desistir) You should give up smoking.
Manage (conseguir) After 4 years at college, he managed to finish his course.
Keep (manter) Don't give up, keep walking.
Decide (decidir) We decided togo to Uruguay for the summer holidays.
Hope (esperar) I hope to beyour teacher in the future.
4. AS DUAS FORMAS (INFIN ITIVO COM TO O U GERÚNDIO)
Promise (prometer) Cintia promised to tell the story.
A) Sem alteração de significado: like, hate, begin, start, love.
Exemplos:
2. OBJETO + INFINITIVO
Peter Jikes going to the cinema.
Alguns verbos pedem um objeto antes do infinitivo, acompanhado Like (gostar)
Peter Jikes to go to the cinema.
ou não da partícula to.
A) Verbos + objeto + infinitivo sem to: I didn't know Helen hated flying.
Hate (detestar)
I didn't know Helen hated to f/y.
Let (deixar) Let me go please.

Help (ajudar) They helped us see the show. B) Com alteração de significado: stop, try, remember, mean, regret.
Make (fazer) Can you make him feel better? Exemplos:
Stop + ing verb form (parar um hábito, uma prática)
B) Verbos + objeto + infinitivo com to:
Louise stopped smoking (Louise parou de fumar)
Stop
Te// (contar, falar) Ann told me to help her son. Stop + infinitive verb form (parar uma atividade para iniciar)
outra): Louise stopped to smoke (Louise parou para fumar)
Ask (pedir) They asked me to drive them home.
Try + ing verb form (experimentar)
lnvite (convidar) }ane invited jim to go to the club.
He tried eating raw fish but it was impossible.
Refuse (recusar) We refused to buy the tickets. Try
Try + infinitive verb form (tentar)
Teach (ensinar) Mary teaches the children to behave we/1. She tried to close the window, but it was very difficu/t.
Remember + ing verb form (lembrar·se de ter feito algo no
passado): I remember Jocking the car doar.
Remember
Remember + infinitive verb form (lembrar-se de fazer algo)
You have to remembertogo to the bank beforeyou drive home.

60 I GE IDIOMAS 2015
� D I SC U RSO D I RETO E I N D I RETO » D UAS FORMAS D E D I Á LOGO

É possível reproduzir conversas usando


o discurso direto ou o indireto

Observe as duas frases a segu i r: DIRECT ANO I N DIRECT SPEECH


1) ... a friendly man informed me that the printer probably O uso correto dos verbos em cada situação
needed on/y to be c/eaned. ( U m h o m e m a m igável m e
i nformou q u e a i m p ressora provave l m e nte preci sava
apenas ser l i m pa.)
2) "Actual/y it's my boss's idea'� the employee replied shee­
pishly. ("Na verdade, foi ide ia do meu chefe", respondeu
o empregado timidamente.)
No primeiro exe m p l o, temos a narração de um d iálogo
feito por um dos partici pantes da história. Já, na segunda
frase, temos a fala de u m dos participantes da h istória, ou
seja, a reprodução l i teral de suas palavras.
A primeira frase é um caso do discurso i n d i reto, q uando
as falas de u m d iálogo são reproduzidas por u m narrador,
participante ou não da história (indirect o r reported speech).
Na reprodução literal das falas dos partici pantes, temos o
discurso d ireto (direct speech).
Em inglês, para reproduzir as falas de um d iálogo na for­ Atenção: advérbios de tempo também m udam no d iscurso
ma do discurso i n d i reto, é preciso segu i r algumas regras, indireto. Nos exemplos acima, você observa a transformação
principalmente em razão dos tem pos verbais, que sofrem de tomorrow para the next day; tonight para that night,
as princi pais transformações, conforme o pri meiro quadro yesterday para the day before.
acima, à d i re ita.
Com relação aos pronomes reflexivos, observe a expressão REFLEXIVE PRONOUNS
destacada na fase abaixo: A construção dos pronomes reflexivos
We usually make more money on repairs if we let peop/e
try to fix things themselves first. (Geralmente ganhamos mais
dinheiro em consertos se deixamos as pessoas tentarem
consertar coisas elas mesmas pri meiro.)
A palavra themselves expressa ênfase n a ação realizada
pelo sujeito da frase, no caso, people. Esta é uma das situações
de uso dos pronomes reflexivos, que têm em comum a palavra
se/f (no p l u ral, selves), "mesmo, próprio". O utra situação de
uso é para i nd icar rec i p rocidade da ação verbal:
She made herself a promise. (Ela fez u ma promessa a si
mesma.)
My boss h urt himself when he was c/osing the car door.
(Meu c hefe se m a c h u c o u q uando e l e estava fechando a
porta do carro.)
Quando o pronome reflexivo é anteced ido da preposição
by, temos em português o significado de "sozi n h o".
The boys stayed at home ali by themse/ves. (Os meni nos
ficaram em casa soz i n h os.)

GE IDIOMAS 2015 1 61
I N T E R P R ETAÇÃO D E TEXTO

Big city life


Moving people and goods quickly, efficiently and cheaply is a central
tenet of the EU's goal for a dynam ic economy and cohesive society

E
uropeans enjoy unprecedented leveis of personal mobility, competition, particularly in the road and ai r sectors and, to a lesser
while goods are shipped rapidly and efficiently from factory extent, for rail. As a result, trucks can operate in countries other
to customer, often in different countries. The European Union than their own, and no longer return empty on international jour­
(EU) has stimulated this freedom of movement by opening national neys. In 2003, a first liberalisation package opened about 70-80%
markets and by removing physical and technical obstacles. But today's o f longdistance rail freight traffic to competition.
transport patterns and growth rates are unsustainable. Liberalisation alone cannot solve severa! deep seated problems.
Moving people and goods quickly, efficiently and cheaply is a These include the dominance o f road over other forms of transport,
cel)tral tenet of the EU's goal for a dynamic economy and cohesive pollution, and the fragmentation of transport systems, including poor
society. The transport sector generates 10% ofEU wealth in terms o f links to outlying regions and between national networks. Congestion
gross domestic product (GDP) and provides more than ten million charging, where users pay for access to scarce infrastructure, is also
jobs. The constant growth in mobility puts severe strains on transport being introduced. One example is the system, pioneered by London
systems. The result is congestion, particularly for road and air traffic in 2003, which charges motorists for driving into central districts of
which reduces economic efficiency, and adds to fuel consumption town and cities. Rail is the first target. A freight train in the EU travels
and to pollution. Although many aspects o f transport policy come at an average speed ofl8 kilometres per hour. Rail must improve speeds
under national governments, it makes sense for the European sin­ and service leveis i f it is to attract freight traffic from roads.
gle market to have a single transport infrastructure. This is why
the EU has opened national transport markets across the Union to (Site europa.eu, adaptado) (Vestibular UnB 2010)

62 1 GE IDIOMAS 201S
EXERCfCIOS - UNB 2010 (adaptado)

L De acordo com o texto da página anterior, considere as 6. Na frase "Li beral isation alone can not solve several deep
afirmações abaixo. seated problems", o verbo s u b l i n hado i n d i ca:
I. Neve r before have goods and i n d i v i d u ai s u n d e rgone s u c h a) i m possi b i l i dade.
m o b i l ity i n E u ro p e. b) p r o i b i ção.
11. The E U h as b r o u g h t about u n q u e s t i o n a b l e tra n s p o rt pat­ c) s u gestão.
te rns. d) necessidade.
111. The EU c u rre n t eco n o m i c a i m , in terms of t ra n s p o rt, is e) s u p o s ição.
based o n o n e main b e l i ef.

Está(ão) correta(s): RES POSTAS


a) I e 1 1 . 1. 1 e 111. As respostas estão nos trechos: "Europeans enjoy unpre­
b) li e 111. cedented leveis o f personal mobility". (Os europeus desfrutam de
c) I e 1 1 1. nlveis de mobilidade pessoal sem preceden tes.); "Moving people
d) Ape nas 1 1 . and goods quickly, efficiently and cheaply is a cen tral ten et o f the
e) Todas. EU's goal for a dynamic economyand cohesive society". (Transpor­
tar pessoas e produtos rapidamen te, com eficiência e de forma
2. De acordo com o texto, é possível afi rmar que: barata, é um objetivo central da comunidade europeia para uma
a) A m a l h a e u ro p e i a d e t ra n s p o rte c o n s i d e ra aspectos indi­ economia dinâmica e uma sociedade coesa.)
v i d u a i s d e cada n ação. Resposta: c
b) Atual m e n te, os s i stemas de tra n s p o rte e u ro p e u s j á alcan­
çaram c o m p l etam e n te os o bj eti vos l i stados p e l a U n i ão z. Ca m in h ões pesados têm permissão para transportar produtos
E u ro pe i a. além do perímetro nacional. A solução está em: "As a result, trucks
c) E m Lo n d res, o s m o tori stas n ão p re c i s a m pagar q u a n d o can operate in coun tries other than their own, and n o longer return
u sam o carro n o c e n tro d a c i d ade. empty on intemational joumeys". (Como resultado, os caminhões
d) Cam i n hões pesados têm p e r m i ssão para tran s p o rtar p ro­ podem operar em outros países além do seu país de origem, e não
d u tos a l é m d o p e r í m e tro n a c i o n a l . mais retornar vazios em viagens internacionais)
e) I n vesti m e ntos n a r e d e ferroviária devem s e r vi stos c o m o Resposta: d
com p l e m e ntares a o u tras p r i o ri d ades.
3. Foi completada e tem resultados no presente. O uso do presente
3. Na frase "The E u ropean U n io n ( E U) has sti m u lated this perfeito "has stimulated") indica uma ação realizada no passado
freedom of m ovement by o p e n i n g national markets and cujo efeito se faz presente.
by removing physical and tech n i cal obstacles", a expressão Resposta: a
subl i n h ada i nd ica q u e a ação verbal:
a) foi c o m p letada e t e m resu ltados n o p resen te. ft. Necessidade. Na frase, o verbo moda/ "must" se traduz como
b) está p ro g ramada para aco ntecer n o futu ro. "deve". "As ferrovias devem melhorar seus níveis de velocidade e
c) acabou e não acontece m ai s . serviço se q uiserem atrair o tráfego de carga das estradas".
d) aco n teceu n o passado e j a m a i s acontecerá n ov a m e n te. Resp os ta: c
e) aconteceu a n teri o r m e nte a o u tra ação n o passado.
s. Da presença de uma preposição que antecede o verbo. O uso do
4- Na frase "Rai l must i mprove speeds and service leveis i f it is verbo na forma do gerúndio é n ecessário em razão da presença de
to attract freight traffic from roads", o verbo must i n d i ca: uma p reposição que o a n tecede, ou seja, "by driving".
a) p r o i b ição. Resp osta: b
b) su gestão.
c) necessidade. 6. Impossibilidade. O verbo "can" pode indicar possibilidade e no
d) perm i s são. caso da frase, temos sua forma n egativa "can n ot".
e) poss i b i l i dade. Resposta: a

s. N o trecho " ... w h i c h charges m otorists for d ri v i n g i nto


central d istricts of town and cities", j u stifica-se o u so do
verbo n o gerú ndio (su b l i n hado) e m razão:
a) do s u j e ito da frase, " m otorists".
b) da p resença d e u m a p re p o s i ção q u e antecede o verbo.
c) d o uso d o te m p o ve rbal p resente c o n t í n u o .
d) d o uso d o p ro n o m e relativo " w h i ch" .
e) da p resença de u m a p re po s i ção logo após o verbo.

GE IDIOMAS 2015 1 63
Aprenda a utilizar corretamente
os verbos modais no passado

Os verbos m odais (veja na página 25) aj u d am a expressar must have been a very handsome man. Ueremy tem 65 anos,
as ide ias dos verbos q u e acom panham. Veja os exem plos: m as, q uando olho para ele, percebo q u e deve ter sido um ho­
1) Mark can speak four languages. (Mark fala q uatro l ínguas.) mem m u ito bonito.)
'
O verbo moda I can. aqu i tem sentido de ter capacidade. I phoned the hotel last night but óaire wasn't in. She might
2) Can you help me, please? (Você pode m e aj u dar?) have been on a tour around the city. (Liguei para o hotel na
3) You maygo now. (Você pode ir agora.) O verbo may expressa noite passada, mas Claire não estava. Ela d eve ter saído em
permissão. um tou r pela cidade.)
4) Cathy should see the doctor. She isn't feeling very we/1. (Cathy Universal Pictures claimed the theft - o f a c/ose-to-finished
deveria ver um médico. Ela não está se sentindo mu ito bem.) print - cost it S1oo mil/íon. That could have been posturing:
Should aparece como conselho, recomendação. the movie was also leveled by brutal reviews. (A U n iversal
alegou que o roubo - d e uma cópia q uase pronta - custou-l he
Nos casos acima, as ide ias referem-se ao presente (exemplos USS 1oo m i l hões. Isso pode ter sido falseado: o filme também
1, 2 e 3) ou ao futuro (exemplo 4). N o passado, temos diferentes foi destruído por críticas brutais.)
estruturas, dependendo das ideias expressas.
Para expressar uma obrigação ou regra n o passado, usamos
1) Para expressar capacidade física o u m ental, usa-se o verbo o verbo have to n o passado.
could (passado do can). A forma negativa de could é couldn't Presente: You have to/must arrive at 8. (Você tem/deve
(could not), e a forma interrogativa é obtida posicionando o chegar às 8.)
verbo could antes do sujeito da frase. Observe os exemplos: Passado: You had to arrive at 8. (Você tinha de/devia chegar
às 8).
I could read when I was 5. (Eu sabia ler aos 5 anos.)
She couldn't walk until she was 4. (Ela não andava/conseguia Tam bém é possível expressar i mpossi bil idade com o can't.
andar até os 4 anos.) Tom can't have studied in this school. He has never lived
Couldyou swim when you were at school? (Você sabia nadar in town. (Tom não pode ter estudado nesta escola. Ele n u nca
q uando estava na escola?) moro u na cidade.)

2) Para expressar probab i l id ad e, recom e ndação ou (im)pos­ 3) Para dar permissão, usamos a expressão be al/owed to (ter
s i b i l i dade no passado, usamos o verbo modal adeq uad o permissão para):
à i d e ia + have + o particfpio passado d o verbo p r i n c i pal. Lisa was allowed to take part into the competition. (Lisa
O b s e rve: tinha permissão para participar da com petição.)

Recomendação: should o u must 4) Para expressar proibição no passado, podemos usar as ex­
You shou/d have ta/ked to her. She didn't understand what pressões be allowed to, be supposed to ou o verbo could na
had happened. (Você deveria ter falado com ela. Ela não forma negativa. Observe:
entendeu o q u e t i n ha acontecido.) Ten years ago, universitystudents in China weren't supposed
to/couldn't /weren't allowed to use the internet on campus.
Os verbos must, may, might e could expressam possibilidade. (Dez anos atrás, u niversitários na China não eram autorizados
}eremy is about 65 now, but when 1 /ook at him, I see that h e a usar a internet d e ntro do campus.)

64 1 GE IDIOMAS 2015
DICAS MA N Y, MUCH, LITTLE » COMO USAR OS QUANT I FICADORES

Many., much e little


indicam quantidades

Em inglês, usamos várias palavras, chamadas quantificado ras,


para i n d icar q uantidades i nd e f i n i das. Observe o exem p lo:
Many peop/e pirate programs regard/ess of their price
tag. ( M u itas pessoas fal s i ficam p rogramas não i m porta n d o
s e u preço.)
Na frase aci m a, a palavra many i n d i ca u m a q u a n t i d a d e
indef i n i d a ( m u i tas pessoas). U samos many/not many c o m
su bsta n t i vo s c o n táveis, g e ra l m e n te e m textos e s c r i tos. O
pro n o m e much/not much é tam b é m usado c o m esse s e n ti­
do ( m u ito, m u ita), mas c o m s u bstan t i vos i n c o n táveis. São
aq u e les q u e, e m i n gl ês, exp ressam q u a n t i d ad e s e m geral e
não ad m i tem p l u ral. Veja no q u ad ro ao lado alguns exemplos
de su bstant i vo s i n c o n táve i s.
Para i n d i car p o u c a q u a n t i dade, t e m o s d u as ex p ressões
em i n gl ês: (a) few e (a) little. A few é usada com s u b stantivos
contáve i s e a little, com s u bsta n t i vos i n co n táve i s . Estu d e
o q u ad ro a o lado para v e r c o m o esses q u an tificadores são
uti l izad o s e o b s e rve m a i s exe m p l os a seg u i r:

Some/Any
Ou tras palavras q u e i n d icam q uantidades i n dete r m i nadas
são some e any (al g u m, a l g u m a, algu ns, a l g u m as). U samos
some em frases afi rmativas e e m alguns ti pos de pergunta (em
oferec i m e n tos e ped idos). Uti l i zamos any em frases negativas
e nos d e m a i s t i p o s d e pergu nta. Veja a l g u n s e x e m p l o s :
» l'd /ike to listen to some songs now.
» Would you like some more wine?
» Can I have some paper, please?
» Are there any English dictionaries at home?
» We don't see any people in the roam.
» Sarah told me she didn't buy any gifts while she was
News l've got some news foryou
on holiday.
Any pode ser usado e m frases afirmativas com o sentido d e
"qualquer". Observe o exe m p lo:
» lt's bo/d to stea/ a movie. Any movie. (É ousado rou bar Paper I don't need much paper to write the message.
um f i l m e . Q u a l q u e r f i l m e.)
Homework

GE IDIOMAS 2015 1 65
I N T E R P R ETAÇÃO D E T EXTO

The future of rich nations


Growth forecasts for rich nations are usually negativel but fertility rates
have been rising in some countries. What will the future be like for them?

Alll1 FERTILITY in a number


of developed economies have
actually been on the ri se. Among
ri c h economies, the jump is most
pronounced in places like the United
Kingdom, France, Spain, and the U.S.

O
ne reason growth forecasts for rich nations are so grim is the lots of women in the workforce. "In parts o f Europe (this method of
common assumption that birth rates are falling. Fewer people calculation) has probably understated true fertility by about 15 to
will produce less income, and shrinking economies. 20 percent;' notes Goldman Sachs economist Peter Berezin. Those
Only the assumption of aging populations may be wrong, according are big numbers, with potentially very significant ramifications. For
to a recent report by Goldman Sachs that looks at key demographic starters, it could be that some of the problems faced by aging nations
trends for the 21st century. with shrinking tax bases (like, for example, overblown health-care
Since bottoming out in 2001, fertility rates in a number of developed spending, crumbling infrastructure, and budget shortfalls) may not
economies have actually been on the rise. Among rich economies, the be quite as bad as once envisioned. In fact, there is some evidence
jump is most pronounced in places like the United Kingdom, France, to suggest that we may soon start seeing mini baby booms, which,
Spain, and the U.S. Larger immigrant populations in these nations in 20 years or so, could have a significant impact on the size of rich
have something to do with this, as they tend to have more children. nations' workforces, a shift that can't come too soon for countries
Yet the effect is only short term, as migrants adopt the fertility rates now struggling with unprecedented leveis of national debt. Another
of their new homes within a generation or so. bright spot - recent surveys by Eurobarometer show that European
Goldman's new analysis shows that another reason for the unexpected women between the ages o f 25 and 39 want to have more children, if
jump, and one that will play a bigger role in the future, is that women only it were easier to find a better work-life balance. Note to Europe­
in rich countries have been having children !ater and !ater in life, an leaders: stop worrying about the effects of immigration and start
something that traditional economic models don't account for. creating better-paid part-time work.
Standard estimates of fertility are still tabulated assuming that most
women are having children in their early 20s, rather than late 20s or
even 30s and 40s, as has become more common in rich countries with (Newsweek, September 28, 2009) (Funece - Uece, 2010)

66 1 GE IDIOMAS 2015
EXERCfCIOS - FUNECE 2010 (adaptado)

The text refer5 to a report about demographic trend5


1. RESPOSTAS
which 5how5 that: L Birth rates have risen in wealthy coun tries since reaching the
a) Fert i l i ty rates h ave fal l e n d ramatically s i nce the begi n n i n g lowest point in the beginning o f the 21st century. A resposta está
of t h e 21st centu ry. em "Since bottoming out in 2001, fertility rates in a number o f
b) W h i l e b i rth rates have r i s e n i n t h e U.S., they have fal l e n developed economies have actually been on the ris e". (Desde que
in t h e U.K. ficaram bem baixas em 2001, as taxas de fertilidade em algumas
c) B i rth rates have r i s e n in wealthy c o u ntries s i n ce reac h i ng das economias desenvolvidas têm, na verdade, subido.)
the lowest p o i n t i n t h e beg i n n i n g of t h e 21st centu ry. Resposta: c
d) Countries l i ke the U.K., the U.S., France, and Spai n are among
those with the lowest b i rth rates i n recent years workplace. 2. Women are choosing to beco me mothers at a /ater age. A res­
e) B i rth rates have fal l e n i n d e ve l o p i n g c o u n t ri es. posta está em "Goldman's new analysis shows that ... women in
rich coun tries have been having children /ater and /ater in life".
z. One of the factor5 that h ave led to the 5Urpri5ing increa5e (A nova análise da Goldman mos tra que ... as mulheres em países
in birth rate5 in wealthy nation5 i5 the fact that: ricos estão tendo filhos cada vez mais tarde)
a) T h e i m m i g ra n t p o p u l a t i o n has been red u ce d . Resposta: b
b) Wo m e n a r e c h oo s i n g to beco m e mothers at a I ater age.
c) You n g w o m e n are h av i n g more and m o re c h i l d re n . 3. Willing to have more babies if they can reach a work-/ife equi­
d ) H u n d re d s of w o m e n a r e leavi n g t h e workp lace. librium. A resposta está em "European women between the ages
e) Wo m e n a n d men are getti n g married at an earl i e r age. o f 25 and 39 wan t to have m o re ch ildren, if only it were easier
to find a better work-life balance". (As mulheres europeias com
3. In European countrie5 women i n the age group ranging idades entre 25 e 39 anos querem ter mais filhos, caso fosse mais
from late 20'5 to late 30'5 are: fácil encon trar um equilíbrio entre o trabalho e a vida doméstica)
a) W i l l i n g to have m o r e babies if t h ey can reach a work-l ife Resposta: a
equilibrium.
b ) Too b u sy t o t h i n k about hav i n g c h i l d re n . 4- 1m balance between the budget and the huge amount o f money
c) Leav i n g t h e i r j o b s to b e c o m e f u l l t i m e m o t h e rs. spent on health-care. A resposta está em ".. .it cou/d be that some
d) The m a i n workforce of these nations. o f the problems faced by aging nations with shrinking tax bases
e) Using d i ffe r e n t m et h o d s in o r d e r to get pregnant. (like, for example, overblown health·care spen ding, crumbling
infrastructure, and budget shortfa lls)" (. . . poderia ser que alguns
4. Among the problem5 in aging nation5, the text mentions: dos problemas enfren tados pelas nações em envelhecimento com
a) An i n crease in i l l i teracy rates a m o n g w o m e n . base em taxas de encolhimento [por exemplo, gastos demasiados
b) S u d d e n s h ri n k i n g of b o t h b u dget a n d s p e n d i ng. com saúde, deficiência na infraestrutura e cortes de orçamen to].)
c) S pe n d i n g o n health-care for i I l egal i m m i grants. Resposta: d
d) 1m balance betwee n t h e b u d get a n d t h e h u ge a m o u n t of
m o ney s p e n t o n h ealt h-care. s. A significant increase of the workforce in approximately two
e) A d e c rease in the q u a l i fi cati o n s needed for j o bs. decades. A resposta está em "In fa ct, there is some evidence to
suggest that we may soon start seeing mini baby booms, which,
s. A 5udden surge of births within the next few years could in 20 years or so, could have a significan t impact on the size of
bring forth: rich nations' workforces". (De fato, há evidência que sugere que
a) A s i g n ificant i n c rease of the wo rkforce i n a p p r o x i m ately podemos e m breve começar a ver um surto de bebês, q u e, em
two d e cades. 20 anos mais ou menos, poderia ter um impacto significativo no
b) A d e c rease in heal th-care i nvest m e n ts. tamanho da força de trabalho das nações ricas.)
c) A s u d d e n c r u m b l i n g o f t h e i n frastructu re. Resposta: a
d) A s h r i n k i n g o f the b u dget.
e) A n i n crease o n eco n o mie p r o b l e m s f o r fam i l i es. 6. More part-time jobs with better salaries should be created
in Europe. A resposta está em "Note to European leaders: stop
6. A piece of advice 5uggested in the text is that: worrying about the effects o f immigration and s tart creating
a) People i n developed cou ntries s h o u l d ad opt c h i l d re n from better-paid part-time work". (Aviso aos líderes europeus: parem
poor c o u n tries. de se preocupar com os efeitos da imigração e comecem a criar
b) M o re m o n ey s h o u l d be i n vested in n u clear p l ants. trabalho de meio período e melhor remunerado)
c) More part-ti m e jobs with better salaries s h o u l d be created Resposta: c
in E u ro pe.
d) W o m e n s h o u l d n o t give b i rt h in t h e i r 40s.
e) Wo m e n stop th i n ki n g about h av i n g c h i l d re n .

GE IDIOMAS 2015 1 67
DICAS O RAÇ ÕES CON D I C I O N A I S » COMO USAR O "IF"

Entenda as diferentes combinações das frases


que usam essa conjunção

1. l f our oi/ runs out - we wi/1 surely become the leading temos a oração i ntrod uzida pela conj u n ção if no passado sim·
producer o f biofuel. (Se o petróleo acabar, certamente nos pies e a oração principal com a com b inação do verbo wou/d +
tornaremos os prin c i pais produ tores d e b iocombustível.) i nf i nitivo do verbo principal. Observe outros exem plos:
2. /f Barack Obama carne to visit Brazil - and he should - we » /f I were* you, I wou/d do what she to/d me. (Se e u fosse
wou/d impress him with ou r bigness in everything. (Se Obama você, eu faria o q u e ela me d i sse.)
viess e visitar o Bras i l - e e l e d everia - n ó s o i m pressiona· » Would Daisy go with me i f I invited her? (Daisy viria comigo
ríamos com nossa grandeza em tudo.) se eu a convidasse?)
3. Brazil wou/d have been a different country if it had been » They'd rebel against the government if we supported them.
colonized by the Spanish. (O Brasil teria sido um pafs d i fe· (Eles se rebelariam contra o governo se nós os apoiássemos.)
rente se tivesse sido colonizado pelos espan hóis.)
(*) Na norma-pad rão da l íngua i nglesa, em orações c o n d i cio·
Em cada u m dos três casos, temos d uas orações relacionadas nais que expressam situação h i potética, a forma d o verbo to
por m e i o da conju nção if, q u e expressa a ideia de condição be n o passado simples é apenas u m a: were. I nformalmente,
("se"). Mas você observou q u e as c o m b i n ações d e tempos a forma was é aceita.
verbais são d iferentes. Isso ocorre por causa d a c o n d i ção
expressa. Analisaremos os três casos separadamente. No caso da frase 3, temos uma s ituação h ipotética no passa·
No p r i m e i ro caso, tem o s u m a relação de causa e c o n se· do, que não pode ser modificada. Daí o uso do passado perfeito
q uência, em u ma s ituação possível d e acontecer. Nesse tipo de (had + particípio do verbo pri nc i pal) na oração s u bord i nada
situação, a oração q ue se i n icia pela conj u n ção i f traz o verbo (com a conj u nção if) e o uso d e wou/d have + particípio do
no presente s i m ples e a oração principal pode se apresentar verbo principal. Observe mais algu n s exemplos:
com o verbo no futu ro, no presente ou ainda no i mperativo. » Ifyou had told me beforehand, I would have invited another
Veja mais exe m plos: friend to go with me, so I wou/dn't have been alone at the
» lfyou want to be a good drummer,you have to practice a /ot. concert. (Se você tivesse me d ito antes, eu teria convidado
(Se você q u e r ser u m bom baterista, deve praticar m u i to.) outro amigo para i r comigo, da f eu não teria ficado sozinho
» We wi/1 have a darker future ifwe don't save water. (Teremos n o s how.)
um futuro som brio se não economizarmos água.) » lf I hadn't studied Mathematics at Unesp, l'd have taken
» Give me a cal/ i fyou need any he/p. (Ligue-me se precisar de Engineering at Unicamp. (Se eu não tivesse estudado mate·
alguma ajuda.) mática na U nesp, eu teria feito E n ge n haria na U n i camp.)

Nesse tipo d e s ituação, podemos usar também a conju nção PARA SITUAÇÕES REAIS E HIPOTÉTICAS
un/ess (a menos que) ou provided (that) (contanto q u e). Principais combinações de orações condicionais
Observe os exe m plos:
» Unless we rush, we wi/1 miss the concert. (A menos q u e n os

apressemos, perderemos o s how.)


Futuro com wi/1/ going to
» They wi/1 give us the discount provided (that) we take the
course together. (Eles nos darão o desconto contanto q u e
Situações irreais would + infinitivo
façamos o c u rso j u n tos.) Simple Past
- hipotéticas do verbo principal

N o caso d a frase 2, a ide ia expressa pela oração condicional Situações hipotéticas would have + past participle
Past perfect
no passado do verbo principal
remete a u m a situação h i potética, q u e não é real. Nesse caso,

68 1 GE IDIOMAS 201S
:� P R O N O M ES » PALAVRAS COM POSTAS

As regras para usar everybody, nobody,


somebody e anybody

Em i n glês, podemos formar várias palavras com a combi­ ou seja, q ualquer pessoa, l ugar, coisa. Ex.: I think anybody in
nação e ntre every, some, any, no e body!one, where, thing. this room can helpyou. (Ach o q u e q ualquer pessoa nesta sala
Usamos every para i n d i car a tota l i d ad e de a l g u m a coisa, pode ajudá-lo.)
lugar ou de pessoas (everywhere = todo l ugar/ everybody
= todo m u nd o) . U s a m o s somebody, somewhere e some­
thing e m frases afi rm ativas para i n d i car a existê n c i a d e Uma combinação para cada situação
algo, pessoa o u l ugar (somebody = algué m, somewhere = O uso do verbo "to wish" depende do contexto
algum l ugar, something = alguma coisa). Usamos anybody/
one, anything, anywhere e m frases para i nd i car a ausência O verbo to wish pode ser usado com diferentes combinações de
(anywhere = nenhum lugar, anybody!one = ninguém, anything formas verbais, dependendo da situação. Observe os exemplos:
= nada). Q u a n d o a frase é negativa, mas tem u m verbo na 1. Para expressar um desejo futuro: wish + would + infinitivo do
forma afi rm ativa, usamos os com postos d e no (nobody/one verbo principal
= n in guém, nowhere = n e n h u m l ugar, nothing = nada). Nas >> I wish something good would happen to her. (Desejo/quero que

frases interrogativas, também usamos os compostos d e any: algo de bom aconteça a ela.)
anybody!one (alg u é m ), anywhere (alg u m l ugar), anything » }enna wishes the kids would come for the Christmas party at the
(alguma coisa). Observe os exem plos: end of theyear. Oenna deseja que as crianças venham para a festa
de Natal no fim do ano.)
FRASES AFI RMATIVAS
» You can look everywhere, you wi/1 not find it. (Você pode 2. Para expressar desejo de mudança de uma realidade: wish +
procurar e m todo o l ugar, não i rá encontrá-lo.) verbo principal no passado
» Everybody wants to see this film. (Todo m u n d o quer ver » I wish I was ta/ler. (Eu queria ser mais alta.)
esse f i l m e.) » }enna wishes she had more money to buy the presents. Oenny
» Don't look: somebody is watchingyou. (Não o l he: alguém queria ter mais dinheiro para comprar os presentes.)
está o bservando você.)
3. Para expressar desejo sobre uma situação passada: wish +
FRASES N EGATIVAS passado perfeito do verbo principal
» I don't need anybody to do that for me, thanks. (Não preciso » I wish you hadn't lost that pen. (Eu queria que você não tivesse

de n i nguém para fazer isso por m i m, obrigada.) perdido aquela caneta.)


» I don't have anything to do, anywhere to go. How boring! (Não » I wishyou had never said that. Peter was never the same after

tenho nada para fazer, n e n h u m l u gar para ir. Que chato!) that. (Eu queria que você nunca tivesse dito aquilo. Peter nunca
» No one knows what it is like to be the bad man. ( N inguém foi o mesmo depois disso.)
sabe como é ser o cara mau.)
É possível usar a expressão if only para expressar desejos. Observe:
FRASES I NTERROGATIVAS » /f only I wasn't so short, I would be a good basketba/1 player.
» Does anyone know where my passport is? (Alguém sabe (Se eu não fosse tão baixo, seria um bom jogador de basquete.)
o n d e está meu passaporte?) » I feel so lonely here in Paris. /f only I cou/d speak Freneh, I would
O uso d e compostos com a palavra any e m frases afirma­ talk to people. (Sinto-me tão sozinho aqui em Paris. Se soubesse
tivas pode acontecer q uando o sentido de any é "qualquer", falar francês, eu conversaria com as pessoas.)

GE IDIOMAS 2015 1 69
DICAS P R O N O M ES R E L AT I VO S » O RAÇ Õ E S S U B O R D I N ADAS A DJ ETIVAS

Conheça os diferentes pronomes relativos


usados para relacionar frases

E m i n glês, as o rações adjetivas (relative clauses) são in­ As vezes, é possível omitir o p ronome relativo nas o rações
trod uzidas pelos pronomes relativos who, which, whose. Os s u bord i nadas adjetivas. I sso acontece q ua n d o o pro n o me
pronomes relativos substituem u m elemento anteriormente não é o sujeito do verbo da oração s u bordinada. O bserve a
m e n c i onado, estabelecendo a relação e ntre d uas orações d i ferença n estes dois exem plos:
por m e i o d essa s u bstituição/referência. Usamos who para » Cristina is a journalist. She works at home.
pessoas, which para coisas e animais e whose para i n d i car a » Cristina is a journalist who works at home. (Cristi na é u m a
relação de posse. Veja alguns exem plos: jornalista q u e trabalha e m casa.)
» And it's notjust scientists who are taking notice. (E não são N esse caso, who é sujeito da oração s u bord i nada e não
apenas os c i e ntistas q u e estão percebendo.) pode ser o m itido.
» We are going to talk about the subject which worries us » Cristina is a journalist. l met her two months ago.
most: global warming. (Vamos falar sobre o ass unto q u e » Cristina is a journalist (who) I met two months ago. (Cristina
mais n o s preocu pa: o aq uecimento global.) é uma jornalista que e ncontrei dois meses atrás.)
» }ames Lovelock, whose last book talks about the greenhouse Aqu i, who não é sujeito da oração, mas o complemento do
effect, is a we/1-known scientist. O ames Lovelock, cujo último verbo met (passado d e meet) e pode ser om itido.
l ivro fala do efeito estufa, é um cientista bem conhecido.) N a função d e complemento d o verbo é possível usar o pro­
nome relativo whom (mais formal) em vez de who. Observe:
É possível s ubstitui r os pronomes who e which por that, » This is the man whom I talked to last night. (Este é o homem
q ua n d o se trata d e o rações adjetivas restritivas ( o u seja, com quem conversei na noite passada.)
aquelas q ue caracterizam u m elemento da oração princi pal
d e forma a restri ngir e/ou especificar sua i d entidade). Nesses
casos, essa oração não vem entre vírgulas. já no caso de orações
Um caso complicado
adjetivas explicativas (aquelas i n seridas entre vírgulas), não
Estes verbos confudem m uita gente
podemos usar that. O bserve os exemplos:
» H e is the man who!that gave me the ticket for the bus. (Ele LI E/LAV
é o homem que m e deu o b i l h ete para o ô n ib us.) 1. Lie (mentir): verbo regular (passado e particípio = lied)
» Shakespeare, who wrote Hamlet and Romeo and }uliet, is 2. Lie (estar deitado/situado): verbo irregular (passado: lay/
the most important English literature writer of ali times. partidpio: lain/gerúndio: lying)
(Shakespeare, q u e escreveu Ham let e Romeu e j u l ieta, é o 3- Lay (deitar, botar, colocar): verbo irregular (passado e
mais i m portante escritor da l iteratura b ritânica de todos particípio: laid)
os tem pos.)
Usa-se that (e não what) após something, anything, nothing RAISE/RISE:
e everything. Exem plos: 1. Raise (cultivar, criar, aumentar): verbo transitivo e regular
» The Simpsons col/ected everything that they needed to take (passado e particípio: raised)
to the trip before leaving home. (Os S i m psons j u n taram 2. Rise (elevar-se, levantar-se): verbo intransitivo e irregular
tudo q u e precisavam levar para a viagem antes de d eixar (passado: rose; particípio: risen)
a casa.)

70 I GE IDIOMAS 2015
Eles são usados para formular
questões diretas e indiretas

Recorremos a p ronomes ou expressões i nte rrogativas para Em i n gl ê s, a estrutu ra geral de pergu n tas é a segu i n te:
form u lar pergu ntas de i nformação (information questions). P r o n o m e i n te rrogativo + verbo auxi l iar + s u j e i to + verbo
O b s e rve os exe m p l os: + i nformação, c o m o n e stes exe m p l os.
» Wha t's your 10 number? (Qual o n ú m e ro d o seu RG?) » Where do you live? ( O n d e você m o ra?)
» What color is your car? (Qual a c o r d o seu carro?) » I want to know where you live. ( E u q u e r o saber o n d e você
» Which dentist did you visit: Doctor Mi/es of Doctor Ste­ m o ra?)
vens? (Qual d e n t i sta você v i s i to u : D o u to r M i les ou d o utor » What course does she take at university? ( Q u e c u rso você
Steve n s?) faz na u n iversidade?)
» When didyou /ast ta/k to you father? ( Q u a n d o você falou Mas também é possível fazer p e r g u n tas i n d i retas (chama­
c o m s e u pai pela ú l t i m a vez?) das de embedded questions) usan d o frases d e i nt r o d u ção
» Where do you come from? (De o n d e você ve m ?) como I don't know/Can you te// me .. ?/Do you know... ?.
» How /ong does it take you to arrive at school? (Quanto N es s e t i p o de c o n stru ção, a p e rg u n ta q u e c o m p l e m e n ­
te m po você leva para c h egar à escola?) t a a o ração d e i n trod ução d eve s e r v i sta c o m o u m a frase
» How much is that? ( Q u a n to cu sta?) afi r m ativa e não se estrutu ra c o m o u m a p e rg u nta d i reta.
» How do you go to work? (Como você vai para o trabalho?) t o caso de:
A d i ferença entre what e which é p e q u e na: what é m a i s » Can you te// me what course she takes at university? (Você
geral e w h i c h i n d ica q u e o falante está pe nsando e m u m gru­ p o d e me d iz e r q u e c u rs o ela faz na u n i v e r s i d ade?)
po específ i c o d e o pções de resposta. O bse rve os exe m p l os :
» What's your name? ( Q u a l o s e u n o m e?) ORDEM INVERTIDA
» Which co/or do you /ike: blue or red? ( D e q u al cor você Exemplos de perguntas indiretas
gosta: azul o u ve r m e l h o?)
O pronome How pode ser usado e m conj u n to com adjetivos
e m d i ve rsas pergu ntas:
» How tal/ are you? (Qual a sua altu ra?)
» How o/d is your sister? (Quantos a n o s tem a s u a i rmã?)
» How far is it from your house to the airport? (Qual a d i s­
tância da s u a casa até o aeroporto?)
» How long have you been /iving in this town? (Há q u anto
te m p o vocês m o ram n esta c i dad e?)
Existem casos es peciais. Leia as frases:
» Who did you ta/k to at the party? (Você c o n v e rs o u c o m
q u e m na festa?)
» Who talked to you at the party? ( Q u e m c o n v e r s o u c o m
você na festa?)
No pri m e i ro exe m p l o, temos a estrutura geral de perguntas
Leitura estratégica
em i n g l ês, c o m o u s o de verbo auxi l iar antece d e n d o o s u j e i ­
Técnicas para estudar vocabulário e, assim, entender
t o . N o segu n d o e x e m p lo, a est r u t u ra é d i fe r e n te, p o i s não
melhor um texto
conta c o m o uso do verbo aux i l i ar e c o m o verbo p r i n c i pa l
co n j u gado n o passado s i m p l es. E m i n g l ês, q ua n d o o s u j e i to >> Organize palavras em forma de mapas semânticos: .u ma
da p e rg u n ta é o próprio pronome i nterrogativo, conj u ga-se o palavra central que faça você lembrar de várias outras
verbo c o m o se fosse u m a frase afi rmativa, i nd ep e n d e nte d o palavras, em torno de um mesmo tema associativo.
tempo v e r b a l . I sto acontece c o m o s p ro n o m e s who e what. » Organize famílias de palavras: um verbo, por exemplo,
Veja m a i s a l g u n s casos: tem um substantivo, adjetivo, advérbio e sinônimos
» What didyou do? (O q u e você fez?) correspondentes.
» What/ Who did this to you? (O q u e/Quem fez isso com você?) » Contextualize o(s) significado(s) de uma palavra por meio de
» Who was phoning? (Q u e m estava telefonand o?) ilustrações e frases.
» Who were you phoning?(Para quem você estava telefonando?)

GE IDIOMAS 2015 1 71
L (Unisantos 2009) Leia o texto abaixo sobre a tragédia ocor­ d) lt is rat h e r i m portant to c o n v i n c e p e o p l e to f o l l o w yo u r
rida em Santa Catarina no fim do ano passado e escolha a b e l iefs.
alternativa q u e apresenta a seq uência de tempos verbais e) l t is n o t v e ry i m p o rta n t to l i s t e n t o y o u r h ea r t a n d
que melhor preenche as lacunas: i ntuition.

H e avy rai n s in the state over two m o n t h s O texto a seguir, base para responder à s q u estões, é u m frag­
ago. D u r i n g N ovember, the area 3 5 i n c h es mento do prefácio de julian Lennon para o livro autobiográfico
of rai n, w h i c h e i g h t t i m e s h ig h e r t h a n t h e John, escrito por sua mãe Cynthia Powell, primeira esposa do
average rai nfall o f 4-3 i nc h es f o r t h e month. Santa Cata­ ídolo de tantas gerações, o ex-Beatle john Lennon.
rina governor Luiz H e n r i q u e da Silve i ra as
say i n g, " T h i s t h e w o rst c l i m at i c t ragedy o f JOHN
Santa Catarina's h istory." Foreword by julian Lennon
(www.reliefweb.int/rwtrwb.nsf/dbgooSID/VDUX·LXShN?OpenDocument&RSS2o=o2·
P · lgk-) Growing up asjohn Lennon's son has been a rockypath. Ali my
life l've had peop/e coming up to me saying "1 /oved your Dad".
a) started - received - i s - was q u oted - is. I always have very mixed feelings when I hear this. I know that
b) starts - rece i ves - was - i s q u oted - was. Dad was an ido/ to millions who grew up /oving his music and
c) started - rece i ves - i s - is q u oted - is. his ideais. But to me he wasn't a musician or a peace icon, he
d) starts - received - was - i s q u oted - was. was the father 1 /oved and who let me down in so many ways.
e) startied - received - was - was q u oted - was. After the age of five, when my parents separated, I saw him
only a handful of times, and when I did he was often remate
z. (Unicsal 2oo8) Assinale a alternativa q u e completa o texto and intimidating. l grew up /onging for more contact with him
de maneira lógica. but felt rejected and unimportant in his life.
Dad was a great talent, a remarkable man who stood for
What do doctors say about fast food? ________ peace and love in the world. But at the same time he found it
Chips, hamburgers, hot dogs and other greasy food make you very hard to show any peace and lave to his first fami/y - my
fat and provoke bad cholesterol regardless of you r age. mother and me. ln many accounts of Dad's life Mum and I are
either dismissed, o r at best treated as insignificant bit players
a) I n gen e ral t h ey say that t h i s k i n d of food is h a r m l ess. in his life, which sad/y is something that continues to this day.
b) T h ey agree t h i s food i s healthy for c h i l d re n . Yet Mum was his first rea/ /ove and she was with him for half
c) They reco m m e n d it f o r yo u ng. peo p l e o n ly. his adult life, from art college, to the genesis o f the Beatles,
d) T h ey say t h i s k i n d of food is h ealthy for everyo ne. to their overwhe/ming worldwide success. That's why l'm so
e) In g e n e ral t h ey agree we m u st n o t h ave i t everyday. happy that she's decided to write her side of the story (...)
(POWELL, C. john. ln: Speak Up Magazine, 2007, n.239, p.42.)
Read this text to answer q u estion 3.
G lossary: rocky path - cam i n ho difícil, ped regoso; remate
Your time is limited, so don't waste it /iving someone and intimidating - distante e i ntimidatório; /onging - ansian­
else's life. Don't be trapped by dogma - which is living with do; stood for - simbol izava; dismissed - desconsiderados;
the results o f other peop/e's thinking. Don't /et the noise o f overwhelming - estrondoso.
other peop/e's opinions drown out your own inner voice.
And most important, have the courage to follow your heart 4- (EAFA 2008) Segundo o texto:
and intuition. Somehow, they already know whatyou truly a) ) o h n Le n n o n, apesar da v i d a artística atri b u l ada, foi u m
want to become. Everything e/se is secondary. You can't pai am o roso e d e d icado.
change the past, but you can ruin the present by worrying b) j o h n Le n n o n , a p e s a r do g ra n d e tale n to, não e ra o pai
too much about the future. b i o l ó g i co d e ) u l i an.
Internet <thinkexist.com> (adapted). c) ) o h n Le n n o n, apesar d e s e r í d o l o pacifista, c a u s o u m u ito
sofri m e nto a s u a fam fl ia.
3. (UnB 2009) From the text, it i s correct to deduce that d) john Lennon, apesar do sucesso, não desc u idava da família.
a) O n e s h o u l d l ive o n e's own l i fe. e) ) o h n Le n n o n, apesar da v i d a atri b u l ada, n u n ca se separou
b) D o g m as are p rece pts to be fo l l owed by p e o p l e as t h ey de s u a fam í l ia.
r e p re s e n t o t h e r persons' reas o n a b l e set o f bel iefs.
c) Yo u r i n n e r v o i c e m u st not b e let out as it may d i stu r b s. (EAFÁ 2008) " (...) he was the father 1 /oved and who let me down
o t h e r i n d i v i d u ai s . in so many ways". Nessa passagem, o narrador revela q ue:

72 1 GE IDIOMAS 2015
a) E m bora o pai o te n h a aban d o n ad o de m u itas f o r m as, e l e a) I was_ very good stu d e n t.
o amava. b) My b ro t h e r has _ p retty g i r l f r i e n d .
b) O pai s o u b e a m á- l o d e m u itas formas. c) I d o n't n e e d t o b uy _ e l e p h a n t l l !
c) Le n n o n foi um pai p resente e m u ito a m ad o . d) "My assets i n c l u d ed __ t h o u sa n d po u n d s i n t h e b a n k
d) Lamentave l m ente, o p a i n u nca soube como a m a r a fam ília. ( . . .) "
e) Len n o n a m ava o f i l h o, de m u i tas f o r m as d i ferentes. e) "Why d o n't you l o o k for __ u n iversity c o u rse?"

6.(Eafa 2oo8) Na frase, "I always have very mixed feelings when n. (Eafa 2008) Passando-se toda a frase "(...) she was with him
I hear this", a palavra destacada faz referência a: for ha/f his adult life (.. .)" para o masc u l i no correspondente,
a) A l i my l i fe. obteremos:
b) " l loved y o u r Dad." a) H e was with her for half her ad u l t l i fe.
c) m i xed fee l i n gs. b) H e was with s h e for h a l f his ad u l t l i fe.
d) j o h n Le n n o n's s o n . c) H e was w i t h s h e f o r half her ad u l t l i fe.
e) a rocky path. d) H e was w i t h h e r for h a l f s h e ad u l t l i fe.
e) H e was with h e r for half h i s ad u l t l i fe .
7. (Eafa 2008) No segu ndo parágrafo, o marcador d iscu rsivo
But expressa: 1J. (Eafa 2008) A q u e palavra o pronome who se refere em "/
a) com paração. know that Dad was an ido/ to m illions who grew up loving
b) c o n t raste. his music and his ideais"?
c) causa. a) Dad.
d) d úv i d a. b) i d o L
e) ad i ção. c) m i l l i o n s .
d) h i s m u s i c.
l(Eafa 2008) "That's why l'm so happy that she's decided to write e) I .
her side o f the story." Nessa passagem, o narrador:
a) Revela o d e s c o n te n tam en to p e l a d e c isão da m ãe de es­ Read t h e following passage a n d choose t h e option w h i ch
creve r a versão dela s o b re os fatos. best completes each q u estio n, accordi n g to the text:
b) Faz u m a com paração da tri ste h i stória de sua própria vida
c o m a d e o u tras pessoas. G ETTI N G REAL ABOUT THE H I G H PRICE OF C H EAP FOOD
c) R e v e l a o q u a n to se s e n te i n c o m o d a d o por ser fi l h o de
J o h n e Cy n t h i a . Horror s tories about the food industry have been with us
d) Mostra-se s u r p reso p e l a m ãe ter reso l v i d o se tornar es­ since 1906, when Upton Sinclair's novel The j ungle told ug/y
critora. truths about how America produces its meat. Nowadays,
e) M o stra-se fe l i z p e l a m ãe ter d e c i d i d o escrever a v e rsão things have got much better, and in some ways much worse.
dela sobre os fatos. The U.S. agricultura/ industry can now produce unlimited
quantities of meat and grains at remarkab/y cheap prices.
f, (Eafa 2008) Em "Dad was a great talent (...)", o verbo está no But it does so a t a high cost to the environment, animais
simp/e past tense. Se substituirmos adequadamente o tempo and humans. Some o f those hidden prices are the erosion o f
verbal pelo simp/e present, obteremos: fertile farmland and the rise o f antibiotic-resistant bacteria
a) Dad are a great tal e n t ( . . . ) . among farm animais.
b) Dad w e re a great tal e n t (. . . ) . Some Americans are noticing such warnings and working
c) Dad am a great tal e n t ( . . . ) . to transform the way the country eats - farmers who are
d) Dad is a great ta l e n t ( ... ) . raising sustainab/e food in ways that don't ruin the earth.
e) Dad be a great tal e n t (. .. ) . Documentaries and the work of journalists are reprising
Sinc/air's work, awakening a s/eeping public to the realities
10. (Eafa 2008) Na expressão "great talent", a função do termo o f how we eat. Change is also coming from the very top. First
em destaque é: Lady Michel/e Obama's White H ouse garden has so far raised
a) caracte rizar o seg u n d o e l e m e n to. a /ot o f organic produce - and tons o f powerfu/ symbolism.
b) e x p ressar u m a co n s e q u ê n c i a. Nevertheless, despite increasing public awareness, sustainab/e
c) i n d i car u m a ação. agricu/ture, remains a tiny enterprise: according to recent
d) exp ressar u m a d úv i d a. data from the U. S. Department o f Agriculture, less than 1% of
e) d e s c rever u m a s i t u ação . American cropland is farmed organically. Sustainable food is
ais o pricier than conventional food and harder to find.
11. (Eafa 2008) Em i n glês, d uas palavras podem ser util izadas Unless Americans radically rethink the way they grow and
como artigos i ndefinidos: a e an. Assinale a opção cuja frase se consume food, they face a future of eroded farmland and
completa adequadamente com a mesma palavra destacada high health costs. Sustainable food has an elitist reputation,
em "Dad was an ido/ to millions ( ... )": but each o f us depends on the soi/, animais and p/ants. And

GE IDIOMAS 2015 1 73
S I M U LADO

a s every farmer knows, if you don't take care o f your land, from the hospital, and researchers fol/owed them for the
it can't take care o f you. next eight years.
(BRYAN,W. Friday, Aug. 21, 2009. http://www.time.com/time/health/article/o,Bs99,1917458,oo. After control/ing for age, sex, obesity, physical inactivity,
html, adaptado.) smoking, education and other factors, they found that the
more chocolate people consumed, the more likely they were
J4. (PUC·Minas 2010) U pton S i nclair's novel The }ungle was the to survive. While the chocolate eaters in the study had a
fi rst to: statistically insignificant reduction in the risk of death from
a) cal I p e o p l e's atte n t i o n to t h e q u a l i ty of food produced in any cause over the eight·year span, the reduced risk for dying
Ame rica. o f heart disease was highly significant.
b) i n struct Americans o n how to prod u ce and sei I better meat. And it was dose·dependent - that is, the more chocolate
c) deal with the p ro b l e m s concern i n g America's food i n d u stry consumed, the lower the risk for death. Compared with people
p rofits. who ate none, those who had chocolate less than once a
d) te l l h o rror sto ries t h at led to the c h a nge of agr i c u l tu ra! month had a 27 percent reduction in their risk for cardiac
i n d u stry. death, those who ate it up to once a week had a 44 percent
e) advise people to eat m o re q ual ity vegetables. reduction and those who indulged twice or more a week had
a 66 percent reduced risk o f dying from a subsequent heart
15- (PUC·Minas 2010) Today, the production o f meat and grains event. The beneficiai effect remained after controlling for
in the U.S. is: intake o f other kinds of s weets. The co·author o f the pape r,
a) i ns u ff i c i e n t. Dr. Mukamal, said that data from other studies suggests that
b) l i m i ted. chocolate lowers blood pressure and this might be a cause
c) d ecreas i n g. o f the lower cardiac mortality found in the study.
d) e n o rm o u s. (http://www.nytimes.com, September, 2009, adaptado)
e) wo rse.
According to the text, i t i s true to say that:
11- (PUC·PR 2010)
1&. (PUC·Minas 2010) First Lady Michelle Obama's Wh ite House a) T h e study was i n p rogress for th ree m o n t h s after patients'
garden has been an effort to: fi rst heart attack.
a) transform A m e r i ca's eco n o my. b) Peo p l e w h o ate less c h o c o l ate had better resu lts.
b) p ro mote A m e r i ca's food i n d u stry. c) The c o n s u m pt i o n of c h oc o l ate also red u ced the risk of
c) teach p e o p l e to g ro w t h e i r o w n food. death from other causes.
d) e n c o u rage A m e ri ca n s to cook at h o m e . d) T h e c o n s u m p t i o n of c h o c o l ate i s b e n ef i c i a i for d i abetic
e) c ha n ge t h e way A m e r i c a n s eat. pati e n ts.
e) The c o n s u m pt i o n o f choco late after a h eart attack e n han
17. (PUC·Minas 2010) The problem with organi c food is that it i s ces t h e c h a n ce of s u rvival.
______ than conventional food.
a) m o re ex p e n s ive and more d i ff i c u l t to f i n d . :ae. (PUC·PR 2010) I n paragraph three, the sentence "The bene·
b) u n h e al t h i e r a n d extre m e ly m o r e c a l o ri c . ficial effect remained after contro l l i ng for i ntake of other
c) m o r e fatte n i n g a n d h a r d e r t o d i gest. kinds of sweets" means:
d) more h a r m f u l and more d a n g e r o u s. a) People conti n u e h av i n g beneficiai effects i n d e p e n d e n t of
e) m o r e d i ffi c u l t to grow a n d c h eaper. eati n g o t h e r sweets.
b) Peo p l e w h o a d d e d o t h e r k i n d s of sweets to t h e i r d i ets
11. (PUC· Minas 2010) U n l ess Americans radically reth i n k the way felt t e r r i b l e .
they grow and consume food, c) P e o p l e w h o eat c h o c o l ate feel l i ke eat i n g o t h e r k i n d s of
a) t h ey w i l l rad ical ly i m p rove t h e i r way of l i v i n g. sweets m o r e ofte n .
b) t h ey w i l l h ave p r o b l e m s w i t h t h e i r l a n d a n d h ealth. d) Peo p l e m u st control t h e a m o u n t of other sweets t h ey eat
c) t h e i r l i fe w i l l c o n t i n u e t h e s a m e for a long t i m e . to conti n u e havi n g beneficiai effects.
d) t h e i r h a b its w i l l m ake t h e m f a m o u s w o r l d w i d e . e) Peo p l e who added other k i n d s of sweets to t h e i r d i ets felt
e) t h ey w i l l start to deve l o p n e w tec h n o l ogy for t h e agri c u l · even m o re b e n efits.
t u re i n d u stry.
A BETTER DEAL ON MALARIA
Answer q uestions 19 and 20 based on the following text:
Novartis is breaking even selling Coartem. But it chose to
CHOCOLATE INCREASES S URVIVAL RATES lose money to save more lives. For the past nine years, the
AFTER H EART ATTACK drug company Novartis has been selling Coartem, one of the
most effective antimalarials on the market, to public·health
Scientists followed l,l69 nondiabetic men and women who officials in the developing world ata loss totaling more than
had been hospitalized for a first heart attack. The patients S253 million - not counting the mil/íons spent on Research &
had a health examination three months after their discharge Development. That's added up, the firm reports, to more than

74 1 GE IDIOMAS 2015
550,000 lives saved. In late )anuary, the company unveiled z s. o problema sobre o q ual o homem d esabafa diz respeito
the first pediatric dose o f Coartem - tess bitter and easier to ao fato de:
swaltow than the adult version - which is expected to help a) S u a m u l he r não gostar m a i s d e l e .
in the battle against a disease that kills more than 7oo,ooo b) E l e não f a l a r i n g l ês.
children under 5 each year. (...) "Novartis could be making c) S u a m u l h e r não c o m preendê-lo.
a lot more money making hypertension or diabetes medi­ d) E l e não amar m a i s a m u l h er.
cations that the people in the U.S. and Europe woutd buy," e) A m u l h e r não c o n ve rsar m ai s com ele.
says A waCoii-Seck, executive director o f Rolt Back Mataria,
a global partnership founded with the goal o f halving the 26. O texto provoca h u mo r porq u e brinca com:
world's mataria cases by 2010. "lnstead, it's investing real a) As relações afetivas e n tre homens e m u l h e res e a n eces-
funds in finding medicines that wifl neve r be profitable." s i d a d e de c o m u n i cação.
Though mataria is both preventable and curable, many b) A i m portância q u e os homens dão a conve rsas de boteq u i m .
o f those in the developing world struggle to get affordable c) A falta d e c o m u n icação e n tre m u l h e res.
treatment, particularty in sub-Saharan Africa, where the d) o desejo que todas as pessoas têm d e falar outros i d i o mas.
mosquito-borne disease is most prevalent. e) A necessidade de conve rsar com a m igos para d e sabafar.
(Time, March 9, 2009)
27. (UFPA 2" fase - 2009) SOMETH I N G ABOUT ARGENTINA
u. (ESPM 2009 ) Vou can i nfer from the text that Novartis: Eight largest in the world,
a) h as been m a k i n g a lot of m o ney w it h Coarte m . country is fult o f natural wonders
b ) has d e m o n strated social awareness b y s e l l i n g Coartem to
d eve l o p i n g c o u ntries. Argentina, meaning "land of si/ver': is a rich and vast land­
c) intends to profit m o re with p e d i atric Coartem than with second largest (after Brazi/) in South America and eighth targest
t h e m e d i c i ne for ad u lts in the world. lts heartland is a broad grassy plain known as the
d) has i n vested in researc h i n g and deve l o p i n g Coartem be­ Pampas. The Spanish first arrived around 1516, and Argentina
cause i t i s a m e d i c i n e with h i g h c o m m e rcial val u e . gained independence in 1816. The small native population died
e) has been i nvesti n g l ots of m o n ey to make hyperten s i o n from European diseases, and today's population is over 95
and d i abetes m ed icat i o n s . percent European. Today the fiteracy and urbanization rates
are high, the birthrate and the infant mortality rate are tow,
u. (ESPM 2009 ) Choose the correct translation for: less bitter and most Argentines consider themselves middle class.
and easier to swallow: The reçent past is tumultuous. Some 30,000 people disappe­
a) m e n o s d i s pe n d i o s o e mais fác i l d e tomar. ared in the "dirty war" during the Military's junta 1976-1983
b) m e n o s ácido e mais fác i l d e p rod u z i r. rufe. In Apri/ 1982, Argentine Forces invaded the British-held
c) m e n o s a m argo e m a i s fác i l de e n go l i r. Falkland lslands, which Argentina calts lsfas Malvinas. Defe­
d) m e n o s azedo e m a i s fác i l de fabri car. ated by Britain during 1982, Falkland lslands war toosened
e) m e n o s l u c rativo, m as c o m resu ltados m a i s fac i l m e n te the military dictatorship's stranglehold on democracy.
c o m p rovados. Since then much has been won: greater freedom o f the
press, tolerance o f opposition, and increased foreign invest­
23. (ESPM 2009 ) According to the text, "affordable" refers to: ment. However, a deep recession caused economic coltapse
a) low cost treat m e nt. at the end of 2001 foltowed by fragile economic growth.
b) effective treat m e n t . Unemployment plagues the economy even as the nation
c) h i g h p r i ce treat m e n t . enjoys the continent's highest per capita income.
d) efficac i o u s treat m e n t . (UFPA 2' fase 2009, http://travel.nationalgeographic.com/places/countriestcountry_argen·
.
e) adeq u ate treat m e n t . tina.html, adaptado)

De acordo com as i nformações do texto, o tempo de d u ração


e o momento de ocorrência da G uerra das Malvi nas estão
corretamente ind icados na alternativa:
a) u m ano - em 1983.
b) u m ano e m e i o - em 1816.
c) um ano e um m ê s - em 1976.
d) d o i s anos - antes do reg i m e m i l i ta r.
24. O texto acima é exemplo de u m/uma: e) m e nos d e u m a n o - d u rante o reg i m e m i l i tar.
a) pro paganda.
b) t i r i n ha. :ai. o texto afirma q u e u m a das consequências da G uerra
c) fo l h eto. das Malvinas foi o:
d) cartaz. a) a u m ento d o e m p rego na Arge n t i na.
e) m a n u a l . b) fortalec i m ento d a eco n o m i a arge nti na.

GE IDIOMAS 2015 ) 75
c) d e sapareci m e nto de 30.000 arge n t i n os. One cliché is that punk was less a musical genre than a
d) esfac e l a m e n to d a Força Aérea arge n t i na. state of mind- but that was true in the days before it became
e) e n f raq u ec i m e nto d o regi m e m i l i tar arge n t i n o. fashionable to become a punk fashion victim.
Punk remained an underground scene unti/ 1976, when two
:ag. O texto menciona algu m as con q u istas consegui d as após bands - The Ramones and The Sex Pisto/s - made the outside
a q ueda do reg i m e m i l itar na Argenti na. E ntre essas con­ world take notice.
q u i stas está o(a): Not only did they beco me hugely successful in their own
a) rec u p e ração do p o d e r aq u i s i tivo da c l asse m é d ia. right, but they aiso provided an inspiration to peop/e who
b) a u m e n to d e i nvesti m e nto estra n g e i ro . realised you did not need to be able to p/ay an instrument
c) s u rg i m e n to d e parti d o s p o l íti cos. to be in a band - you just had to have something to say.
d) p o p u l arização da i m p re n sa. YOUNG5, I. BBC News Online, 23 dec. 2002. Trecho com cortes. Disponível em: <http://news.
e) c resci m e n to eco n ô m i co. bbc.co.uk/2/hi/entertainmenV2601493.Stm>. Acessado em 11 de março de 2011

JO. Marca o movimento c u l t u ral descrito n o texto:


a) A cena m u s i cal, q u e, com c l i c hês, se manteve n o estrelato.
b) o p r i m o r m u s ical, q u e se d i f u n d i u para o o u tro l ad o do
m u ndo.
c) A m ú s i ca p o p u l ar, por i n corporar e l e m e n tos d o m u n d o
fas h i o n .
·
d) A o r i g e m c o m s h o w h i st ó r i c o d e S e x P i s t o l s j u n to c o m
R a m o n e s.
e) A expressão i d eológi ca, que d e u um novo sentido à i d e i a
de banda.

Fonte: http://www.joyoftech.com/joyoftech/joyimages/1514.jpg
Acessado em 11 de março de 2011

A expressão "/'m so excited to announce ou r newest product"


explícita para o leitor a f u nção d o texto, q u e é:
a) P ro m ov e r a v e n d a de n ova tec n o l o g i a para p r o d u z i r m ú ­
si ca, p o r m e i o d e d i s c u rso p u b l i citário.
b) A n u n c i a r p rod uto s i m i l a r e c o n c o r re n te d e o u tro l íd e r d e
m e rcado, p o r m e i o d o a p e l o à e m oção. Fonte: Disponível em < http://insoniainsana.zip.neVimages/DownloadedKid.jpg>.
c) Cri ti car o i m pacto social d o p r i m e i ro p rod u to, p o r m e i o Acessado em 22 de março de 2011
d e d i s c u rso i rô n i c o e m rel ação a o se g u n d o .
d) C a u s a r h u m o r, p o r m e i o d e d i s c u rso c r ítico a o s p rod u tos O texto verbal reforça o efeito de h u m o r d o cartu m ao pôr
c o m baixa q u a l i d a d e tec n o l ó g i ca. em q u estão:
e) o r i e ntar o c o n s u m i d o r, por m e i o d e d i s c u rso escl arece­ a) as referênc ias da i n ternet na v i são de m u n do e exp ressões
dor das vantag e n s e c u stos dos p r o d u tos. l i n g u ísticas das n ovas gerações.
b) a falta de c o n h eci m e n to que as pessoas d e m e i a- i d a d e
J1. A BRIEF H I STORY O F P U N K poss u e m da i n ternet e s e u s recu rsos.
c) a c u r i o s i dade i n fan t i l s o b re q u estões d i f íc e i s, como essa
The original punk music scene ran like the careers o f many a res peito d o i n íc i o e d o fim da v i d a .
o f its stars - burning brightly for a short time before crashing d) o d e s p reparo dos p a i s para i m por l i m i tes a o s f i l hos n o u so
to the ground in f/ames. de j o gos e c o m p u tado res.
But its importance can be judged by the echoes heard in e) a falta d e te m p o para d i a l o gar com f i l h o s, d e r ivada d o
music e ver since, as we/1 as the /egends and c/iches that have excesso de f u n ções da m ã e m o d e r n a .
grown up around it.

76 1 GE IDIOMAS 2015
c) provocar o riso e a refl exão, ao i ron izar o ráp i d o enve l h eci­
m ento d e rec u rsos tecnológi cos.
d) criticar as relações sociais, ao representar os processos de
desu man ização causados pelas tecn o l og ias.
e) regu lamentar as n o rmas d e eti q u eta, ao i nd icar possíveis
usos das tecn ologias.

3S.TEXT 1
Leonard Hofstadter and Sheldon Cooperare both brilliant
physicists. They are colleagues, best friends, and roommates,
although in ali capacities their relationship is always tested
primarily by Sheldon's regimented, deeply eccentric, and
non-conventional ways. They are also friends with their Cal
Tech colleagues mechanical engineer Howard Wolowitz and
astrophysicist Rajesh Koothrappali. The foursome spend
their time working on their individual work projects, playing
vídeo games, watching science-fiction movies, or reading.
As they are self-professed nerds, ali have little or no luck
lt was a stupid gift, but they were smart with popular women.
enough to h ide the axe before giving it.
Fonte: "Huggo". The Big Bang Theory story/ine (fragmento).
Fonte: disponível em http://voices.allthingsd.com/files/2o08/12/1188.gif. Disponível em: http://www.imdb.com/title/tto8g8266. Acesso em 22/1/2012.
Acessado em 22 de janeiro de 2012.
TEXT 2
No cartum, a expressão q u e ajuda o leitor a estabelecer rela­ A woman who moves in to an apartment next door to two
ções com o discurso de sustentabilidade é: brilliant but socially awkward physicists shows them how
a) "real g i ft". little they know about fite outside o f the laboratory.
b) "stu p i d gift". Fonte: "Huggo". The Big Bang Theory storyline (fragmento).
c) " h a n d m ad e". Disponível em: http://www.imdb.com/titlettto8g8266. Acessado em 22 de janeiro de 2012.
d) "green g i ft".
e) "h i d e the axe". Os textos são resenhas da série The Big Bang Theory. A pa­
lavra que no texto 1 serve para tipificar comportamentos
como o que foi destacado no texto 2 é:
a) c o l leagu es.
b) room mates.
c) fou rsome.
d) s e l f· p rofessed.
e) n e rds.

RESPOSTAS COM ENTADAS

1. Started - received - is - was quoted - is. No texto, temos


(tradução livre): "Fortes chuvas no estado começaram há dois
meses. Em novembro, a área recebeu 35 polegadas (574 cm3)
de chuva, oito vezes mais do que a média de 4.3 polegadas
(70 cmJ) para o mês. O governador de Santa Catarina, Luiz
Fonte: Disponível em: http://www.joyoltech.com/joyoftech/joyimages/881.gil. Henrique da Silveira, afirmou: "Esta é a pior tragédia climática
Acessado em 22 de janeiro de 2012. da história de Santa Catarina". Resposta: A

As expressões l inguísticas e as representações de costumes 2. In general they agree we must not have it everyday. Na
típicos em festas de aniversário concorrem para que o texto resposta p a ra a pergun ta, temos: "Bata ta, h a m búrguer,
tenha a função de cachorro quente e outras comidas gordurosas engordam e
a) causar o h u m o r e o deboche, a o satirizar o u so d e apare­ aumentam o colesterol, independentemente de sua idade".
l h o s tecnol ógicos em si tuações i nad e q u adas. Assim, a respos ta correta deve ser a alternativa (E), que pode
b) promov e r o c o n s u m o, ao i l u strar a d i versidade d e m e i o s ser traduzida como: "Em geral, eles concordam que não de·
tecnológicos q u e p o d e m s e r ofereci d o s c o m o presentes. vemos ingeri-la todos os dias". Resposta: E

GE IDIOMAS 201� J 77
3. One should live one's o wn life. A resposta está no trecho the food industry h ave been with us since 1906, when Upton
"Your time is limited, so don't waste it living som eon e else's Sin clair's nove l The ) u ngle told ugly truths about how America
fite". (S e u t e m p o é l i m i ta do, portan to, não o desperdice produces its meat". (His tórias terríveis so bre a indús tria ali­
vivendo a vida de o u tra pessoa.) Resposta: A men tícia têm estado conosco desde 1906, quando o romance
de Upton Sinclair, A Sel va, mostrou verdades horrorosas sobre
ft,John Lennon, apesar de ser ídolo pacifista, causou muito como a A m érica produz sua carne). Resposta: A
sofrimento a sua família. A resposta está no trecho "In many
accounts of Dad's life Mum and I a re either dismissed, o r a t 15. Enormous. A resposta está no trecho "The U.S. agricultura/
best treated a s insignificant bit players in his fite, which sadly industry can n o w produce unlimíted q uan tities o f meat and
is something that continues to this day". (Em vários episó­ grains at rema rka b/y cheap prices". (A indús tria agrícola dos
dios da vida de meu pai, minha mãe e eu fomos rejeitados EUA consegue agora produzir quantidades ilimitadas de carne
o u, na melhor das hipóteses, tratados como insignifican tes e grãos a preços n o tavelmente baixos.) Resposta: D
figuran tes em sua vida, o que tristemente é algo que continua
a té h oj e.) Resposta: C 16. Change the way Americans eat. Encontramos a resposta
na fras e "Fírst Lady Michel/e Obama's White House garden
s. Embora o pai o tenha abandonado de muitas formas, ele o has so far raised a lot o f organ ic produce - and tons o f po­
amava. A resposta está no trecho "h e was the fa t h er t loved werful sym b olis m". (O ja rdim da prime ira -dama Michel/e
and who let m e down in so many ways". (ele foi o pai q u e eu Obama na Casa Branca produziu a té agora a limen tos o rgâ­
a m a va e q u e m e desapon tou de tantas formas diferentes.) nicos - e toneladas de s i m b o lismo poderoso). Resposta: E
Resposta: A
17. More expensive and more difficult to find. A resposta se
6. "1 /oved your Dad". Na frase, temos "Eu sempre tenho senti­ encon tra no trecho "Sustainable foo d is also prícier than
men tos muito con troversos quando ouço isso'� referindo-se conventional food and h a rder to fin d ". (Comida sustentável
à ideia an teriormente pos ta, o u s eja, "Eu adorava seu pa i ". é também mais cara do que a convencional e mais difícil de
Resposta: 8 se encon trar.) Resposta: A

7. Con traste. "But" significa "mas" e "neverthe/ess" sign ifica 18. They wi/1 have problems with their larid and hea lth. A
"con tudo", ou seja, indicam con traste. Resposta: 8 resposta está no trecho "Unless A m ericans radica/ly re t h í n k
t h e way t h ey gro w a n d c o n s u m e food, t h ey fa c e a fu ture
B. Mostra-se feliz pela mãe ter decidido escrever a versão dela o f e ro d ed fa rmland a n d h ig h health cos ts". (A m en o s q u e
sobre os fatos. A resposta e n con tra-se em "That's why l'm so o s a m ericanos repensem radica l m e n te o modo como cul­
happy that s h e 's decided to write her si de o f the story". (Por tivam e c o n s o m e m s e u s a li m e n t o s, eles e n frentarão um
isso estou feliz q u e ela tenha decidido escrever o lado dela futuro de lavouras desgastadas e altos custos com a saúde.)
da história.) Resposta: E Resposta: B

g. Dad is a great talent ( ... ) O presente s imples de "was" é 19. The consumption o f chocolate after a heart attack enhan­
"is". Resposta: D ces the chance o f survival. A resposta está no trecho " ... they
found that the more ch ocolate peop/e consumed, the m o re
10. Caracterizar o segundo elemento. A pala vra "grea t " (gran­ likely they were to survive". (. .. eles descobriram q u e, qua n to
de) é u m a dje tivo que cara cteriza o s u bs ta n tivo "ta l e n t " mais chocola te as pessoas consumia m, maior a probabilida­
(talen to). Resposta: A de de elas sob revi verem). Resposta: E

u. l don't need to buy _ elephant!!! O artigo indefinido "a n" zo. People must con tra/ the amount o f other sweets they eat
a n tecede pala vras iniciadas por vogais. Resposta: C to continue having beneficia/ effects. A resposta está na
frase "The beneficiai effect remained after controlling for
l:Z. He was with her for half her adult fite. O pro n o m e "sh e intake o f other kinds o f sweets". (O efeito ben éfico perma­
"(ela) deve s e r s u bstituído p o r s e u e q u ivalente masculino neceu após con trolar a ingestão de o u tros tipos de doces).
"h e". O mesmo ocorre com "him", substituído por "her", e o Resposta: D
"his'� q u e deve ser substituído por "h e r". Resposta: A
u. Ha s demonstrated social a wareness by selling Coartem
13. Millions. Na frase "I kno w that Dad was an ido/ to mil/íons to developing countries. A resposta está no trecho "Novartis
w h o gre w up loving h is m us i c a n d h is ideais", a pala vra h as been selling Coartem, one o f the most effective a n tima­
"who" (quem) se refere à palavra an terior, ou seja, "m il/íons" larials on the market, t o public-health o fficia/s in the deve­
(milh ões). Resposta: C loping world a t a !oss t o ta!ing more than S253 mil/ion {. . .).
That's added up, the firm reports, to m o re than sso,ooo Jives
lft. Cal/ people's attention to the quality of food produced sa ved". (A Novartis tem vendido Coartem, um dos mais efi­
in America. A resposta está n o trecho "Horror stories about cien tes remédios a n timalá ria do mercado, a s e cretarias de

78 1 GE IDIOMAS 2015
sa úde pública em países em desenvolvimen to, totaliza·ndo 30. Criticar o impacto social do primeiro produto, por meio
um prejuízo de mais de 253 milhões de dólares ... Isso corres­ de discurso irônico em relação ao segundo. O anúncio do
ponde a mais de 550 m il vidas salvas.) Resposta: B novo produto é irônico, pois o plugue de ouvido serve para
q u e as pessoas não sejam o brigadas a o u vir as músicas
zz.Menos amargo e mais fácil de engolir. Essa é a correta criadas n o Garageband, u m aplicativo produzido pela em­
tradução da frase. Resposta: C presa para outro de seus produtos, o iPad. Resposta: C

23. Low cost treatment. "Affordable" significa "que se pode 31. A expressão ideológica, que deu um novo sentido à ide ia
pagar". Resposta: A de banda. O texto demonstra q u e a expressão ideológica
sempre foi importante no movimento; além disso, cita duas
24. Tirinha. O texto é uma tirinha. Pela organização textual bandas punks de grande expressão - Ramones e Sex Pistols - que
(uso de linguagem não verbal com ilustrações, texto verbal possuíam membros com pouca familiaridade com os ins­
dentro de balões, sequência de quadros) é possível identificar trumen tos, já que era mais importante ter algo a dizer do
a tipologia textual. Resposta: 8 que, de fa to, apresentar apuro m usical. Resposta: E

zs.Sua mulher não compreendê-/o. Ao dizer q ue "My wife JZ.As referências da internet na visão de mundo e expressões
doesn't understand me': a personagem reclama da falta de linguísticas das novas gerações. A resposta da mãe permite
compreensão de sua parceira (Minha esposa não me enten­ inferir que a criança acreditava ter surgido por download,
de/compreende), a lgo que faz parte do estereótipo dos re­ revela n d o q u e sua visão acerca do m u ndo, de q u estões
lacionamentos afetivos entre homens e mulheres, especial­ importan tes com o a de sua origem e sua linguagem, está
mente no casamento. O leitor espera que ele esteja falando tomada por referências da internet. Resposta: A
sobre essa crença. Resposta: C
33. "Green gift". Na língua inglesa, "green" é u tilizado como
zG. Asrelações afetivas entre homens e mulheres e a necessi­ sinônimo de susten tável. A tradução do termo tam b é m é
dade de comunicação. Durante a leitura, cria·se a expectativa usada em português: "ideias verdes'� "tecnologia verde" etc.
de que a falta de compreensão relatada pelo marido se deva "Real gift" significa literalmente "presente real, de verdade"
a razões profundas, dentro do relacionamento afetivo en tre e indica que o garoto não considera o artesanato feito com
ele e sua esposa. No entanto, no último quadro ficamos sa­ material reeie/a do um presente válido. "Stupid gift" significa
bendo que a esposa não fala inglês, razão esta para a falta de presente estúpido, péssimo. "Handmade" significa artesanal,
comunicação em qualquer tipo de inter-relação e não apenas feito a mão. "H ide the axe" significa esconder o machado,
na situação do casamento. Daí o humor. Resposta: A mas a expressão não tem relação alguma com sustentabili­
dade nem com proteção ao meio ambiente. Ela concorre para
27. Menos de um ano - dura n te o regime militar. Releia os efeitos de humor ao sugerir que os pais o teriam escondido
estas frases: "Some 30,000 people disappeared in the 'dirty antes de dar "o presente verde". Resposta: D
war' during the Military's junta 1976-1983 rui e. In Apri/ 1982,
Argentine Forces invaded the British-held Falklan d /slands, 34. Provocar o riso e a reflexão, ao ironizar o rápido envelhe­
which Argen tina calls lslas Ma/vi nas. Defeated by Britain cimento de recursos tecnológicos. Pois o conjunto de falas
during the 1982". (Cerca de 30 mil pessoas desapareceram e representações de costumes típicos em festas de aniver­
na 'guerra suja' durante a ditadura militar de 1976-1983. Em sário garante uma visão bem-humorada e crítica, conteúdo
a bril d e 1 982, as forças a rgen tinas i n va diram as ilhas tfpico do gênero cartum, sobre a rapidez com que recursos
Falklan d, britânicas. Derrotados pelos ingleses em 1982 .. .) . tecnológicos vão sendo superados e descartados. Vale res­
Resposta: E saltar que a inovação, muitas vezes, obedece mais à intenção
de lucrar com os impulsos consumistas do que, de fa to, à
zB. Enfraquecimento do regime militar argentino. Aresposta obso/escência das tecnologias. Resposta: c
está contida em: "Falkland Jslands war loosened the m ilita­
ry dictatorship's stranglehold on democracy". (A Guerra das 35. Nerds. o termo inglês "nerd" é u tilizado para indicar ou
Malvinas a frouxou o gargalo da ditadura militar sobre a estereotipar pessoas que se dedicam a intensas a tividades
democracia). Resposta: E intelectuais, muitas vezes em detrimento do convívio social.
"Colleagues" significa colegas. "Roommates" significa cole­
zg. Aumento do investimento estrangeiro. Basta ler: "Since gas de quarto - no caso, é utilizado para indicar que Leonard
then much has been won: greater freedom of the press, to­ e Sheldon dividem u m aparta mento. "Foursome " significa
lerance of opposition, and increased foreign investmen t". quarteto, retomando os q uatro colegas: Leona rd, Sheldon,
(Desde então muito se ganhou: maior liberdade de imprensa, Howard e Rajesh. "Self-professed" significa autoproclamados,
tolerância da oposição e aumento do investimento estran­ e no contexto faz referência aos personagens que se chamam
geiro.) Resposta: 8 a si mesmo de "nerds". Resposta: E

GE IDIOMAS 2015 1 79
I NT E R P R ETAÇÃO D E TEXTO

El espaiiol invade Brasil


Los gobiernos espaíiol y brasileíio se preparan para formar a nuevos profesores
y alumnos de espaíiol con proyectos de difusión cultural

..111111 CLASE DE ESPANOL: según las estimaciones deI Ministerio de Educación brasileíio, a lo largo de los próximos anos se necesitarán 210 ooo profesores de espaíiol

E
! impulso a la ensefíanza de! espano! en Brasil, donde se calcula "brasilefíistas" en las áreas de Economía, Derecho, Ciencias Sociales,
que once millones de estudiantes aprenderán e! castellano como Historia, Lengua y Literatura, Geografía.
segunda lengua, centrará la conversación que e! presidentente Por su parte, e! rector de la Universidad de Salamanca, José
de la Junta de Castilla y León, Juan Vicente Herrera, y e! de! país Ramón Alonso, que también estará presente en esta reunión, aseguró
sudamericano, Luis Inacio Lula da Silva, mantendrán hoy en un que intentará "explorar nuevas áreas de colaboración" entre ambos
desayuno de trabajo en Madrid. El hotel Westin Palace acogerá a países. Así, explicó que quieren plantear una investigación conjunta
ambos dirigentes en un encuentro que comenzará a las 8.30 horas en materia de energías renovables ya que Brasil "es un país líder
y se prolongará una hora. sobre todo en biodiesel", un aspecto en e! que la regíón está tenien­
Según las estimaciones dei Ministerio de Educación brasilefío, a do "un desarrollo en los últimos tiempos". En concreto, matizó, la
lo largo de los próximos afíos se necesitarán 210.000 profesores de Universidad de Salamanca quiere aportar investigación y también
espano!. Las expectativas de crecimiento convertirán ese asunto en e! conocer las líneas de trabajo de este país sobre este tema.
eje dei encuentro entre ambos mandatarios, que también abordarán En cuanto a la importancia de la colaboración, Alonso aseguró que
las posibilidades de impulsar la cultura brasilefía en la Comunidad, y Brasil "ha planteado la necesidad de tener 200.000 profesores de
la rehabilitación de! Palacio de Maldonado (en Salamanca), prevista espano!" y la USAL tiene un prestigío en la formación de educadores,
para el cuarto trimestre de 2008. La recuperación de! emblemático y en la edición de material didáctico para la ensefíanza de espano!
edificio salmantino fue una de las cuestiones que Herrera y Lula da como lengua extranjera. En la actualidad, hay un amplio grupo de
Sirva abordaron en una reunión celebrada en octubre de 2005 en la estudiantes de doctorado y profesores brasilefíos que se forman en
Cumbre Iberoamericana de jefes de Estado y de Gobierno que tuvo Salamanca y lo que quieren es "mejorarlo". Además anunció que
lugar en Salamanca. Allí acordaron que el centro se convertirá en la se planteó ai embajador de Brasil en Espana que se desarrolle en la
sede oficial dei Centro de Estudios Brasilefíos de la Universidad de ciudad una semana de Brasil en Salamanca con actividades culturales,
Salamanca y de la Fundación Cultural Hispano Brasilefía. gastronómicas y tradicionales brasileíi.as. Otro de los aspectos es el
El Centro se puso en marcha en e! afio 2001, ante la constatación de colaborar con empresas espafíolas que tienen intereses en Brasil
de que en Espana hay muy pocos expertos en Brasil, a pesar de ser y con actividades brasilefías.
un país con gran cantidad de investigadores americanistas, y nació
con el objetivo de crear la primera generación de investigadores (Extraído de vestibular da UFPA 2008, adaptado de www.eladelantado.com)

82 1 GE IDIOMAS lOlS
EXERCÍCIOS - UFPA 2008 (adaptado)

1. Com base no texto da página anterior, é correto afirmar que: 4- A i m portância do desenvo lvi mento do ensino da língua
a) O p r e s i d e n te L u la e o p re s i d e n te d o gove r n o e s p a n h o l, espanhola no Brasil tem sua j ustificativa em:
j u a n V i c e n te H e rr e ra, e n c o n tram-se em M a d r i para fi rmar a) " E I C e n t ro se p u s o e n m archa e n e l afio 2001, ante l a c o n s­
convê n i os relac i o n ad o s ao e n s i n o d o espan h o l n o Bras i l . tatac i ó n de q u e e n E s p a n a hay m uy p o c o s e x p e rtos e n
b) O p r e s i d e n te L u l a e j u a n V i c e n te H e r r e ra, p re f e i to d e B ras i l . . ."
Salaman ca, se re u n i rão e m Mad ri para f i r m a r c o n vê n i o s b) " S e g ú n l a s e st i m ac i o n e s d e i M i n i s te r i o d e E d u ca c i ó n
q u e d e s e n vo lverão o e n s i n o d o e s p a n h o l n o B ras i l . bras i l e fi o, a l o largo d e l o s p r óx i m o s anos s e n ec e s i tarán
c ) O preside nte L u l a e j u a n V i c e n te H e r r e ra, reitor da U n i ver­ 210.000 p rofesores d e espano i."
sidad e de Salamanca, se re u n i rão e m Mad ri para tratar do c) "AI I í a c o r d a r o n q u e e l c e n t r o s e c o n v e rt i rá en l a s e d e
d e s e n vo lv i m e n to do e n s i n o do espan h o l n o B ras i l . o f i c i a l d e i C e n tro d e Est u d i o s B ras i l e fi o s d e l a U n i v e r s i ­
d) O pres i d e n te L u l a e o preside nte d a j u nta de Castela e Leão, dad d e Salamanca y d e l a F u n d a c i ó n C u l t u ral H i s p a n o ­
j u a n V i c e n te Herrera, se reu n i rão em M a d r i para tratar d o B ras i l e fia."
d e se nvolvi m e nto d o e n s i n o d o espan h o l n o Bras i l . d) "En l a actu a l i d ad, hay un a m p l i o g r u p o de e s t u d i an tes
e) O presidente L u la e o m i n i stro da Edu cação da Espanha, j u an d e d octo rad o y profesores b ras i l e fi o s q u e se f o r m a n e n
Vicente Herre ra, se re u n i rão em Madri para fi rmar convênios Salamanca y l o q u e q u i e re n es ' m ej o rarlo'."
que desenvolverão o e n s i n o d o espan h o l n o Bras i l . e) "Por s u parte, el recto r de l a U n iversidad de Salaman ca,
J o sé Ram ó n A l o n so, q u e tam b i é n estará presente en esta
1. A informação de que Lula e Herrera se encontrarão durante re u n i ó n, aseg u ró q u e i n te ntará 'ex p l o rar n u evas áreas de
o café da manhã está expl ícita em: c o l ab o rac i ó n ' e n tre ambos países."
a) " L a rec u pe rac i ó n d e i e m b l e m át i co e d i f í c i o sal m a n t i n o f u e
u n a de l a s cuestiones q u e H erreray Lula da Si lva abordaron
en u n a re u n i ó n c e l e b rada e n octu b re d e 2005 en la C u m bre RESPOSTAS
l be roamericana d e jefes d e Estado y d e G o b i e rn o q u e tuvo 1. O presidente Lula e o presidente dajunta de Castela e Leão,}uan
l u gar e n Saiam an ca." Vicente Herrera, se reunirão em Madri para tratar do desenvolvi­
b) " ... cen trará la conversación q u e el preside nte de la j u nta mento do ensino do espanhol no Brasil. A informação correta está
de Cas t i l l a y León, j u an Vicente H e rrera, y el dei país s u d a­ no primeiro parágrafo do texto. Resposta: d
meri cano, L u i s l n acio L u l a da S i l va, mantend rán hoy en u n
desay u n o d e trabajo e n Mad r i d ." z. ... centrará la conversación que e/ presidente de la }unta de
"

c) "E I hotel West i n Palace acogerá a ambos d i rigentes en u n Castil/a y León, }uan Vicente Herrera, y e/ de/ pafs sudamericano,
e n c u e n tro q u e come nzará a las 8.30 h o ras y s e p r o l o n gará Luis lnacio Lula da Silva, man tendrán h oy en un desayuno de
u na h o ra." trabajo en Madrid." Nesse trecho, a palavra "desayuno" significa
d) "Los dos mandatarios tam b i é n abord arán la d i f u s i ó n d e "café da man hã". Na frase seguinte a essa, na qual está a pala vra
la c u l t u ra d e este país e n l a reg i ó n y la restau rac i ó n d e i "desayuno", se faz menção ao h o rário do encontro, que também
Palacio de Mal do nado, futura sede de l a F u n dación H i s pano nos leva à conclusão de que os dois manda tários tomarã o o café
Bras i l e fi a." da manhã jun tos. Resposta: b
e) " H e rrera y L u l a da Si lva cen trarán hoy su e n t revista en e l
i m p u lso de la e n se fi anza d e i e s p a n o i e n Bras i l ." 3. É interesse da Universidade de Salamanca con tribuir com as
investigações sobre o programa brasileiro de energias renováveis,
3.Segu ndo o texto: bem como conhecer as áreas de trabalho do Brasil sobre esse tema.
a) Lula e H e rre ra, e m seu e ncontro em Madri, tratarão apenas A resposta está no quarto parágrafo do texto, no qual o reitor da
do d e se n volvi m e n to d o e n s i n o do espan h o l n o Bras i l e da Universidade de Salamanca a firma q u e vai propor a discussão
d i f u são da c u l t u ra bras i l e i ra na Espa n h a. desse tema n o encon tro com os dois manda tários. Resposta: c
b) é grande, na Espan ha, o n ú m ero de i nvestigadores de temas
re lac i o n ados ao Bras i l . ft. "Según las estimaciones de/ Ministerio de Educación brasílefio,
c ) é i n teresse da U n iversidade d e Salamanca contri b u i r c o m a lo largo de los próximos afias se necesitarán no.ooo profesores
a s i nvestigações s o b re o programa b ras i l e i ro de e n ergias de espafiol." A resposta está no segundo e no último parágrafos do
ren ováveis, bem co m o c o n h ec e r as áreas d e traba l h o do texto. Resposta: b
Bras i l sobre esse tema.
d) no Palácio de Maldonado, atual m ente, f u n c i o nam o Cen tro
de Estudos Brasi l e i ros da U n ivers i d ad e de Salamanca e a
F u n d ação C u l t u ral H i s pano-Bras i l e i ra.
e) se realizará em Madri e em Salamanca uma semana de ativi­
dades c u l t u rais, gastro n ô m icas e trad i c i o n a i s b rasi l e i ras.

GE IDIOMAS 2015 1 83
I NT E R P R ETAÇÃO D E TEXTO

De igual
EXERCÍCIOS - UFBA 2012 (adaptado) desilude quando é barrada sua entrada num
país estrangeiro, a alterna tiva está incorreta.
L Com base na leitura do texto, são verdadeiras c) A canção não propõe q u e os imigrantes eu­
as proposições: ropeus sejam desrespeitados. A o contrário,

a igual
a) A identidade de cada povo é menos respei­ o próprio título já indica q u e o q u e se pede
tada na América Latina do que na Europa. é o respeito mútuo. Incorreta.
b) Os presidentes têm l ivre acesso para liberar d) Os adjetivos gen tflicos usados na canção para
os i m igrantes ilegais que são barrados. referir-se aos imigrantes latino-americanos
Un poema que nos habla del c) Os i m igrantes europeus também devem ser revelam uma visão preconceituosa. Os gen·
prejuicio entre los pueblos de desrespeitados, como os latino-americanos tílicos como "sudaca" e "bolita" equivalem

diferentes culturas o são nos países europeus. a formas de trata m e n to como "baia no" ou
d) Os adjetivos gentílicos usados para os i m i­ "paraiba': empregadas n o Brasil para falar,
Soy bolita en Italia, grantes lati no-americanos revelam uma vi­ de forma preconceituosa e generalizada, dos
soy co lombo en Nueva York, são preconceituosa da qual eles são vítimas nordestinos. Correta.
soy sudaca por Espaiia pelo m u ndo afora. e) Na canção não há menção a imigrantes ilegais
y paragua de Asunción. e) Os im igrantes i legais, como forma de protes­ colarem cartazes nas embaixadas como for­
Espaiiol en Argentina, (5) to, colam cartazes nas em baixadas para que ma de protesto. A palavra "cola", e mpregada
alemán en Salvador, seus d i reitos e son hos sejam respeitados. na canção, significa "fila", em português. Ou
un francés se fue pa' Chile, seja, os imigra n tes não colam cartazes, mas
japonés en Ecuador. 2. Em referência à linguagem do texto, é cor­ sim fazem fila diante das em baixadas para
El mundo está amueblado reto afirmar: conseguir os documen tos e se legalizarem.
con maderas dei Brasil (10) a) A oração "Si me pedís que vuelva otra vez Incorreta.
y hay grandes agujeros donde nací!yo pido que tu empresa se vaya Resposta: d
en la selva misionera. de m i país" (17 e 18) pode ser considerada
Europa no recuerda como u m a "oración condicional" na q ual z.AnaUsando as alternativas:
de los barcos que mandó. "Si" fu nciona como nexo. a) A alternativa é correta porque a função do
Gente herida por la guerra (15) b) "En tanto" (25) é uma expressão que i n d ica n exo/conectar "si", em espano/, é a de intro­
esta tierra la salvó. i ntensidade. duzir orações condicionais. Correta.
Si me pedís que vuelva otra vez donde nací c) "lo" (27) exerce, no texto, a função de objeto b) A expressão "en tanto" é equivalente a "mien·
yo pido que tu empresa se vaya de mi país. di reto e tem como referente a expressão "e I tras tanto" (enquanto isso}. Não indica inten­
Y así será de igual a igual gran ladrón" (25). sidade, mas simultaneidade, o u s eja, tem um
Y así será de igual a igual. (20) d) "si n aliento" (32) pode ser trad uzido como s e n tido temporal. Incorreta.
Tico, nica, e! boricua, sem forças. c) A função gramatical de "lo" é de objeto direto
arjo, mejo, e! panamefío e) "se" em "se jactaba" (35) i n d ica uma ação e substitui a expressão "gran ladrón". (Veja
hacen cola en la Embajada passiva. mais sobre objeto d i reto na pág. 93}. Correta.
para conseguir un sue fio. f) "su" e m "su honor" (36) e e m "su ley" (36) d) Trata-se de um caso de e q uivalência de vo­
En tanto e! gran ladrón, (25) é um determi nante que poss u i o mesmo cabulário. A palavra "a liento" tem o sentido
lleno de antecedentes, referente. d e n o ta tivo d e "hálito", em portugês, mas
si lo para Inmigración pode ser usada com o sentido cono tativo de
pide por e! presidente. RESPOSTAS: "sopro'� "fôlego". Correta.
Los llamados ilegales L Analisando as alternativas: e) O pro n o m e "se" é reflexivo. Associado a u m
que no tienen documentos (30) a) Segundo o início da canção, a iden tidade verbo, expressa uma ação reflexiva - aquela
son desesperanzados dos povos é desrespeitada tanto na A mérica na qual o sujeito executa e, ao mesmo tempo,
sin trabajo y sin aliento. Latina quan to na Europa e na A m é rica d o sofre a ação do verbo. Neste caso, ''iactarse"
Ilegales son los que Norte, onde os estrangeiros são tratados com significa "gabar-se'� e logiar-se de forma exa­
dejaron ir a Pinochet. termos depreciativos: "bolita en /ta/ia': "co· gerada e presunçosa. Incorreta.
Inglaterra se jactaba (35) lombo en Nueva York", "sudaca por Espana", f) Nos dois casos, o possesivo "su" tem a função
de su horror y de su ley. "paragua de Asunción", "espan o / en Argenti· gramatical de determinante de u m substan­
na", "alemán en Salvador'� "un fra ncés se fue tivo, respectiva m e n te de "honor" e "ley". E
GIECO, León; GUREVICH, Luís. De igual a igual. Dispo· pa' Chile", "japonés en Ecuador". lncorreta. a m b os refere m-se à Inglaterra. Lembre: de­
nível em: <http://www.cancioneros.com/nc/n738/o/ b) A o con trário do que afirma a alternativa, a termina n tes são p a la v ra s q u e a n te c e d e m
de·igual·a·igual-leon·gieco>. Acesso em: 30 ago. 2011. canção afirma q u e "... si lo para lnmigración s u bstantivos, dando precisão a seu s ignifica·
(Vestibular U FBA 2012, adaptado) pide por e/ presiden te, los l/a mados ilegales do - neste caso, estabelecendo uma relação
q u e no tienen documentos son desesperan­ en tre o substan tivo e quem o possui. Correta.
Obs.: Entre parênteses, o número do ver­ zados", ou seja, o presidente não têm voz Resposta: a, c, d e f
so, para facilitar a localização dos pontos para libera r os imigra n t es ilegais que são
pedidos nos exercícios. desiludidos. Entendendo que um imigrante se

84 1 GE IOIOMAS 2015
DICAS PRONOMES » FORMAL OU I N FO RMAL?

Saiba como definir o tipo de tratamento


usado com relação a outra pessoa

Para começar, vamos con hecer (ou relem brar) os pronomes fam i l iar. Ustedes se contrapõe a vosotros/as, usado e m um
em espan h o l . registro i nformal. N o espan h o l falado nas I l has Canárias e
e m toda a América Lati na, as formas vosotros/as não são
- usadas, sen d o s u bstitu ídas por ustedes tanto nos registros
formais q uanto nos i n formais e fam i l iares.
yo me

.:Quieres (tú) tomar algo? (i nformal - você) � .:Quiere usted


tú te
tomar algo? (formal - s e n h o r/a)

usted se
Vosotros sois muy elegantes. (informal - vocês) -+ Ustedes
son muy elegantes. (formal - sen h o res/as)
él!ella se

O uso de pronomes pessoais não é a ú n ica marca l i ng u ís­


nosotros;nosotras nos
tica da opção pelo registro formal ou i nformal no texto. Os
possessivos (mi, tu, su etc.) e as term inações verbais tam bém
ustedes os
nos mostram que forma d e tratamento é usada pelo emissor
d e um e n u nciado.
vosotros/vosotras se
Em algu mas regiões d e alguns países d a América, como na
Argentina, a forma tú é su bstituída por vos e, muitas vezes, vem
e/loS/e/las se
acompanhada de uma forma verbal especffica. Esse fenômeno
é chamado d e voseo. Ex.: .:Qué querés vos? (O que você q uer?)
Em m u itos casos, não é n ecessário expl icitar o p ronome
pessoal porque o verbo conjugado i n d ica quem é o sujeito Atenção!
da oração. Usamos os pronomes pessoais q uando queremos Cu i dado ao usar e/lo
evidenciar o emprego de usted e ustedes em relação às formas
tú e vosotros, ou em relação a é/, e/la, e/los e e/las. No caso dos O pronome neutro e/lo é empregado para designar coisas
demais p ro n o mes, usamos os pessoais q ua n d o q ueremos indeterminadas, um conjunto de coisas ou ideias, e não sofre flexão
enfatizar o sujeito com quem estamos falando. de número nem de gênero. Equivale a isto/isso do português.
As formas de trata m e nto ustedjustedes i n d i ca m u m a Não pode ser usado no lugar do pronome masculino singular É/:
forma d e tratamento ceri m o n iosa e respeitosa para com o É/ está en paro. Por e/lo, no puede gastar sus ahorros. (Ele está
i nterlocutor e são usadas em u m registro formal. Usted se desempregado. Por isso, não pode gastar suas economias.)
contrapõe ao p ro n o m e tú, u t i l izado e m u m registro mais

GE IDIOMAS 2015 I SS
I NT E R P R ETAÇÃO D E TEXTO

Venas abiertas de
América Latina
Habla e l periodista y escritor uruguayo Eduardo Galeano acerca de la
relación de los países lati noamericanos con los países desarrollados

..11111 A LA UNIDAO La escultura Mão, deI brasileiío Oscar Niemeyer, en São Paulo, representa el sentimiento de lucha y unión que debe predominar entre los pueblos latinoamericanos

L
a división internacional dei trabajo consiste en que unos países necesario construir para quienes padecen los negocias. Nuestros
se especializan en ganar y otros en perder. Nuestra comarca sistemas de inquisidores y verdugos no sólo funcionan para e! mercado
dei mundo, que hoy llamamos América Latina, fue precoz: se externo dominante; proporcionan también caudalosos manantiales de
especializá en perder desde los remotos tiempos en que los europeos ganancias que fluyen de los empréstitos y las inversiones extranjeras
dei Renacimiento se abalanzaron a través dei mar y !e hundieron los en los mercados internos dominados. "Se ha oído hablar de concesiones
di entes en la garganta. Pasaron los siglos y América Latina perfeccionó hechas por América Latina a! capital extranjero, pero no de las
sus funciones. Este ya no es e! reino de las maravillas donde la realidad concesiones hechas por los Estados Unidos ai capital de otros países.
derrotaba a la fábula y la imaginación era humillada por los trofeos de Es que nosotros no damos concesiones", advertía, aliá por 1913, e!
la conquista, los yacimientos de oro y las montaíi.as de plata. Pero la presidente norteamericano Woodrow Wilson. É ! estaba seguro: "Un
región sigue trabajando de sirvienta. Continúa existiendo ai servido país - decía - es poseído y dominado por e! capital que en é! se haya
de las necesidades ajenas, como fuente de reservas dei petróleo y e! invertido". Y tenía razón. Por e! camino hasta perdimos e! derecho de
hierro, e! cobre y la carne, las frutas y e! café, las materias primas y los llamarnos americanos, aunque los haitianos y los cubanos ya habían
alimentos con destino a los países ricos que ganan consumiéndolos, asomado a la histeria, como pueblos nuevos, un siglo antes de que los
mucho más de lo que América Latina gana produciéndolos. Son peregrinos de! Mayflower se establecieran en las costas de Plymouth.
mucho más altos los impuestos que cobran los compradores que los Ahora América es, para e! mundo, nada más que los Estados Unidos:
precios que reciben los vendedores; y a! fin y a! cabo, como declaró nosotros habitamos, a lo sumo, una sub América, una América de
en julio de 1968 Covey T. O liver, coordinador de la Alianza para e! segunda clase, de nebulosa identificación.
Progreso, "hablar de precios justos en la actualidad es un concepto
medieval. Estamos en plena época de la libre comercialización.". Galeano, Eduardo. Las Venas Abiertas de América Latina. Sigla Veintiuno Editores, 53' ed.,
Cuanta más libertad se otorga a los negocias, más cárceles se hace México, 1988. Adaptado. (Vestibular UFBA 2012, adaptado)

86 1 GE IDIOMAS 201S
EXERCÍCIOS - U FBA 2012 (adaptado)

1. De acordo com as i n formações contidas no texto, é cor­ en él se haya inve1tido ... " (um pais é dominado pelo capital
reto afirmar: que é investido ne l e). Mais adian te, "... ah ora América es, para
a) A América Lat i n a co n t i n u a a serviço d e i n teresses alheios. e/ mundo, nada más que los Estados Unidos ... " (agora a América
b) A América Latina tem melhorado suas funções, o que marca Latina nada mais é que os Estados Unidos". Correta.
um avanço no c resc i m ento. e) Sim, o texto diz que há um problema de iden tidade no uso dos
c) As tri b u tações apl icadas aos países ven d e d o res sem pre termos América e americano. Isso fica claro no trecho "... por e/
acabam sendo i nferio res às dos cons u m i d ores. camino hasta perdi mos e/ derecho de l/amamos americanos... "
d) As características d e um país que rece be i nvest i m e ntos (ao longo do caminho, a té perdemos o direito de nos chamar
estrangeiros ad equam-se, segundo o autor, ao pensamento america nos. . .}. Correta.
man i festad o pel o presidente n o rte-ame ri cano W. W i lson. f} Segundo o texto, o capital estrangeiro, na forma de emprésti­
e) A própria n o m eação do co nti n ente se configura como u m mos e investimen tos, tem como objetivo o enriquecimento dos
cl aro problema d e i d entidade. investidores estrangeiros e investidores internos que trabalham
f) O capital estra n g e i ro beneficia-se por f u n c i o nar e m favor a seu próprio favor. Isso consta do trecho "nuestros sistemas de
dos m e rcados locais, oferecendo e m p résti mos e também inquisidores y verdugos no só l o funcionan para e/ mercado exter­
i nvest i m en tos. no dominante; proporcionan también caudalosos manan tiales
de ganancias que nuyen de los em préstitos y ias inversiones
z. N o texto, possuem equ ivalência de sign ificado os termos extranjeras en los mercados internos dominados." Incorreta.
e/ou expressões transcritos nas seguintes proposições: Respostas: a, c, d
a) "comarca del m u n do" e "América Lati na".
b) "abalanzaron" e "habían asomado". z. A questão exige conhecimento de vocabulário e de figuras de
c) "más" e "pe ra". linguagem. Exige, além disso, certa familiaridade com o linguajar
d) " m u c h o más" e "a lo s u m o". das ciências econômicas. Analisando as alternativas:
e) "i nversio nes" e "concesio nes". a) Sim, comarca do m u n do é uma figura de l inguagem q u e, n o
f) "i nvers i o n e s extranje ras" e "cap i tal extranjero". con texto, se refere à América Latina. Correta.
b) "abalanzaron" (lançar-se, arrojar-se) não é sinônimo de "habían
3. Quanto ao uso da linguagem, no texto, é correto afi rmar: asomado" (apareceram, em português}. Incorreta.
a) No trecho "a través de I mar y le h u n d ieron los d i entes e n c) "más" (mais) é um advérbio de intensidade; já "pera" (mas, porém)
la garganta.", o termo d estacado tem a f u n ção d e c o m ­ é uma conjunção adversativa. Incorreta.
p l e m e nto d i reto. d) "mucho más" significa "muito mais" e "a lo sum o '' significa "no
b) No trecho " Este ya no es e l rei n o d e las marav i l las donde máximo". Incorreta.
la rea l i d ad d e rrotaba a l a fáb u l a ...", a palavra "donde", do e} "inversiones" são investimentos e "concesiones", concessões.
ponto d e vista semântico, expressa i d e i a d e l u gar. Incorreta.
c) No trech o "con desti no a los países ricos que ganan con­ f} "inversiones extranjeras" (investimentos estrangeiros) e "capital
sumiéndolos, m u c h o más d e lo que América Lat i n a gana extranjero" (capital estrangeiro) são sinônimos no contex t o
produciéndolos. ..", o termo "los" em "consu miéndolos" e em aqui apresentado.
"prod uciéndolos" exerce, no texto, a mesma fu nção si ntática. Respostas: a e f
d) No trecho "y ai f i n y ai cabo, como dec laró en j u l i o de 1968
Covey T. O l i v e r, ...", a l o c u ção adve rbial "ai f i n y ai cabo" 3. Analisando as alternativas:
poderia ser s u bstitu ída, sem p rej u ízo d o senti do, por "ai a} O p ronome "/e" (lhe, e m p ortuguês) tem a função de objeto
fi n y a l a postre". indireto. Incorreta.
b) "Donde" é um pronome relativo que indica l ugar. Correta.
RESPOSTAS c) Nos dois casos, o pronome "los" tem a função de objeto direto.
I. Analisando as alternativas: Correta.
a} Segundo o texto, a A mérica Latina "... continúa existiendo ai d] As duas expressões são locuções adverbiais e ambas significam
servicio de las necesidades ajenas . . . " Correta. "finalmente", "por fim". Correta.

b} O trecho "... pasaron los sigl as y América Latina perfeccionó sus Respostas: b, c, d
funciones" diz que a América Latina está aperfeiçoando suas
funções. Mas o texto não fala em crescimento derivado desse
aperfeiçoamento. Incorreta.
c) O texto nos diz o con trário do que é proposto na alternativa. A tri­
butação é maior para os vendedores do que para os consumidores:
"son mucho más altos /os impuestos que cobran los compradores
que los precios que reciben los vendedores" Incorreta.
d) O pensamento do presidente norte-americano é citado no trecho
"... un país - decia - es poseido y dominado p or e/ capita l que

GE IDIOMAS 2015 1 87
I NT E R P R ETAÇÃO DE TEXTO

Diversidad cultural en
la era de la globalización
La globalización tecnológica necesita estar acompanada por una evolución

L
os cambios generados por la globalización invitan a una redefinición constante de las políticas culturales. La globalización
tecnológica necesita estar acompaiíada por una evolución política y cultural, capaz de facilitar una mejor comprensión
entre los ciudadanos dei mundo. En un mundo donde la influencia de los medi os masivos de comunicación sobre la vida
cotidiana es cada vez más intensa, los países no pueden sustraerse a la oleada de cambios globales.
El diálogo intercultural abre nuestros horizontes a lo nuevo y nos predispone a escuchar a los otros, modificando
nuestros puntos de vista, como una condición imprescindible para la convivencia pacífica. Solamente a través de un intercambio
fluido tendremos la posibilidad de encontrar nuevas soluciones a nuestras diferencias culturales. A esto debemos
agregar una tarea importante de nuestra época: hacer más humano el proceso de globalización. Debemos trabajar juntos para
construir un proyecto futuro, de manera que ningún grupo sea excluido de la dinâmica contemporânea. La actividad
cultural ha de jugar un papel fundamental a la hora de concebi r y aplicar políticas que contrarresten el impacto negativo de la
globalización y moderen las ambiciones lucrativas dei mercado. Todos debemos trabajar juntos. Cada l2ªÍâ debería crear no
só lo normas sino también espacios de encuentro, que permitan participar de este diálogo intercultural. La humanidad es una,
y sus culturas, numerosas. Para que la sociedad dei siglo XXI as uma nuevas formas de solidaridad en un mundo cada yez más
interdependiente, hay que establecer políticas culturales capaces de "humanizar" la globalización.

(Extraído de PUC·RS 2007, http://www.scielo.cl, Literaturay Linguística, 2003, adaptado)

EXERCfCIOS - PUC 2007 (adaptado)

1. EI texto: c) globalizaci o nes, cu ltu rales, d iál ogos, pos i b i l idads, paises,
I. aborda un tema de carácter sociocu l t u ral. vezes.
11. d i sc u te p royectos para i ntrod u c i r cam b i as. d) global izaciónes, c u l t u rales, d iálogos, pos i b i l idades, países,
1 1 1. eva l ú a propuestas d e m ej oras futuras. vezes.
IV. anal i za i deas s o b re armonía y d iversidad. e) globalizaciones, culturales, d iálogos, posibilidades, país, vezes.

El análisis de las afi rmativas permite conclu i r que están cor·


rectas solamente RESPOSTAS
a) l y l l . 1. /ey I V. "E/ diál ogo intercu lt u ra l a b re n u es tros horizontes a
b ) I y IV. lo nuevo y nos predispone a escuchar a los otros, m odificando
c) 1 1 y 1 1 1. nuestros puntos de vista, como una condici6n imprescindible para
d) l i y I V. la con vivencia pacifica. Solamente a través de un in tercambio
e) l l l y IV. fluido tendremos la posibilidad de encon trar n uevas soluciones
a nuestras diferencias culturales." Resposta: b
1. La alternativa en l a q u e solo h ay pai abras m asc u l i nas
reti radas dei texto es: 2. E/ diálogo, e/ encuentro, e/ proyecto, los ciudadanos. Para
a) los cam b i a s, los c i ud adanos, l o n u evo, e l i nterca m b i o. responder a essa questão, podemos eliminar as alternativas que
b) lo n u evo, el i n te rca m b i o, el p royecto, el papel. contêm o ar t igo neutro "lo" acompanhado do adjetivo "nuevo".
c) e l d i ál ogo, e l e n c u e n tro, lo n u evo, e l p royecto. Em espanhol, o artigo neu tro, em nenhuma hipótese, acompanha
d) los países, lo n u evo, los cam b i as, e l papel . substantivos masculinos. Resposta: e
e) e l d iálogo, e l e n c u e ntro, e l proyecto, l o s c i u dadanos.
3. Globalizaciones, cu/tu rales, diálogos, posibilidades, países,
3- EI plural de las paiabras subrayadas en el texto es, respecti· veces. As opções b, c, d e e apresentam erros gramaticais e de
vamente: acen tuação gráfica: uso indevido/fa lta do a ce n t o gráfico e m
a) global izaciones, c u l t u rales, d iálogos, pos i b i l idades, países, "globalizaci6nes" e "países"; ausência d a letra e na formação
veces. do pl u ral de palavras terminadas e m consoan tes, como em "po­
b) g l o bal i zaciónes, c u l t u rales, d iálogos, posi b i l idad s, países, sibilidads"; troca de z (do sin g u la r) por c (no plural) em palavras
veces. terminadas em z, como em "veces". Resposta: a

881 GE IDIOMAS 2015


DICAS VERBOS » ESTU DANDO CONJ U GAÇÃO

Veja como conjugar e usar o presente do indicativo

Usamos o presente d o i nd i cativo para expressar: Com a i rregularidade vocál ica o > ue:

Ações atuais Trabajo en un bufete de abogados.


yo sueiio vue/vo duermo
Ações habituais Los domingos, ceno con mis abuelos.
tú sueflas vuelves duermes
Fatos históricos En 1642, nace lsaac Newton.
usted! é/! e/la sue/la vuelve duerme
Ordens L/amas a Marta y /e das e/ recado.
nosotros/as soflamos volvemos dormimos
Ações futuras Esta tarde, voy ai cine con Nati.
vosotros/as sofláis volvéis dormís
Verdades universais La luzes más rápida que e/ sonido.
ustedes/ e/los/ e/las sueflan vuelven duermen

Os verbos regulares são c o n j u gados da segu i nte forma: Com a i rregularidade vocá l i ca e > i:
.
Pronombres Habiar . - Beber . , Vi�i�. ·'
' ' - .

yo hablo bebo vivo yo pido me visto

tú hab/as bebes vives tú pides te vistes

usted/ é/! e/la habla bebe vive usted! é/! e/la pide se viste

nosotros/as hablamos bebemos vivimos nosotros/as pedimos nos vestimos

vosotros/as habláis bebéis vivís vosotros/as pedls os vestís

ustedes/ e/los/ e/las hablan beben viven ustedes/ e/los/ e/las piden se visten

Os i rregu lares sofrem alterações vocá l i cas e/ou conso­ Verbos com variação consonantal n a la d o s i n g u lar:
nantais, exceto no caso da la e da 2a pessoas d o p l u ral q u e
sem pre se conjuga m c o m o reg u l ares.
Com a i rregu laridade vocá l i ca e > ie: yo salgo traigo cojo conozco
" ' • f •' ,.... ..'
�mpezar ::
'�+ "
\'
Pronombres • ,· · Querer . Pre�rlr ·,:� tú safes traes coges conoces
yo empiezo quiero prefiero
usted! é/! e/la sa/e trae coge conoce
tú empiezas quieres prefieres
nosotros/as salimos traemos cogemos conocemos

usted! é/! e/la empieza quiere prefiere


vosotros/as sa/ís traéis cogéis conocéis

nosotros/as empezamos queremos preferimos


ustedes/ e/los/ e/las salen traen cogen conocen

vosotros/as empezáis queréis preferís


Verbos q u e c o m b i nam alterações:

ustedes/ e/los/ e/las empiezan quieren prefieren

tú oyes

usted/ é/! e/la oye

nosotros/as oímos

vosotros/as oís

ustedes/ e/los/ e/las tienen oyen

GE IDIOMAS 2015 1 89
I NT E R P R ETAÇÃO D E TEXTO

(las mascotas forman parte


de la familia?
Los animales generan una cercania emocional mayor que un pariente lejano

� LOS ANIMALES juegan un papel fundamental en la estructura de la familia

H
ace ya tiempo que las mascotas se han sumado a la vida familiar, mascotas tienen su lugar en esa construcción, ya que, sea como
pero ahora muchas personas las consideran prácticamente compafíía o como auxiliar, los animales suelen generar una cercania
miembros de la familia. Este tema se disparó casi ai azar, emocional mayor que un pariente lejano ai que sólo llamamos o
cuando un grupo de sociólogos de la Universidad de Warwick hizo una vemos un par de veces ai afio".
encuesta sobre cómo las personas consideraban que estaba formada El impacto psicológico que las mascotas pueden tener sobre
su família. Lo curioso es que los sociólogos empezaron a recibir, sus duefíos, es algo investigado en profundidad en muchos países
co � mucha frecuencia, la siguiente pregunta de sus encuestados: occidentales como Estados Unidos, Inglaterra, Australia, Francia
ápuedo incluir a mis mascotas en la estructura familiar? Y nada y Espafía, aunque hay datos también de estudios serios en países
menos que 44 de los consultados, o sea, casi un 20%, espontáneamente latinoamericanos como Colombia, México, Venezuela, Argentina,
sumaron a sus mascotas en la encuesta de descripción de su familia. Guatemala, Costa Rica y Chile.
Nickie Charles, profesor de la Universidad, sefíaló que los limites Lo cierto es que los animales domésticos aportan compafíía,
usualmente construidos entre la sociedad y la naturaleza se van devuelven sobradamente e! afecto que reciben e introducen en nifíos
desdibujando con las mascotas. "De hecho", continuá el experto, y jóvenes los necesarios conceptos de rutina y responsabilidad.
"tiene sentido que cuando se describe a la família y a los amigos se
incluya a las personas más cercanas y útiles a la estructura, y las (Extraído de PUC·RS 2009. http www.saludyciencias.com.ar, 28/03/2008, adaptado)

90 I GE IDIOMAS lOIS
EXERCÍCIOS - PUC·RS 2009 (adaptado)

Com base nas informações apresentadas pelo artigo da pági­ RESPOSTAS


na anterior, responda às quatro pergu ntas desta página. L O impacto psicológico de mascotes sobre os seus donos tem sido
1. De acordo com o texto, é correto afi rmar apenas que: investigado não só na Europa e nos Estados Unidos. Mas também
a) as mascotes s e m p re foram c o n s i d e radas m e m b ro s das em países latino-americanos, como Colômbia, México, Venezuela,
famfl ias. Argentina, Guatemala, Costa Rica e Chile, segundo o texto.
b) os soci ó l ogos da U n iversidade d e Warw i c k pergu ntaram Resposta: d
aos s e u s e n t revi stad o s s e c o n s i d e ravam as m as c o t e s
m e m b ro s d a s u a fam íl ia. z. "Os limites usualmente construídos entre a sociedade e a natureza
c) os sociól ogos d a U n iversidade de Warw i c k pe rgu n taram a estão perdendo a precisão com as mascotes" e "de fato". A chave
20% d o s s e u s entrevistados se co n s i d e ravam as mascotes para responder a esta questão está no significado da locução verbal
c o m o parte da sua fam í l i a. "se van dedibujando" (estão se tornando menos claros, precisos).
d) o impacto psicológico de mascotes sobre os seus donos tem "De hecho" (de fa to) é uma expressão linguística que introduz uma
s i d o i nvestigado não s ó na E u ro p a e nos Estad os U n i d os. constatação.
e) entre as pessoas entrevistadas pelos sociól ogos da U n ive r­ Resposta: a
s i dade de Warwick, 44 p e d i ram para i n c l u i r seus a n i mais
d e esti m ação como m e m b ro s d e s u a fam í l ia. 3. Familia. O uso da expressão "esa construcción" corresponde
a uma estra tégia d e referência sequencial de substituição ("esa
z. Os dois e n u nciados "los lím ites usualmente construidos construcción" por "família").
entre la sociedad y la naturaleza se van desdibujando con las Resposta: c
mascotas" e "de hecho" podem ser corretamente traduzidos,
respectivamente, por: 4- Acostumbran. o verbo "soler'' tem o significado de "costumar'�
a) "Os l i m i tes usual m e n te construídos entre a sociedade e no português, e confere ao verbo ao qual acompanha uma ideia de
a natu reza estão p e r d e n d o a precisão com as mascotes" habitualidade.
e " d e fato". Resposta: b
b) "Os l i m i tes u s ualm e n te construídos entre a sociedade e
a natu reza estão s e n d o redesen hados com as mascotes"
e "de fato".
c) "Os l i m ites u s u a l m ente con stru ídos entre a sociedade e
a natu reza foram f i c a n d o m a i s tolos com as mascotes" e
"além d i sso".
d) "Os l i m ites construídos entre a sociedade e a natu reza se
p e rd e ram com as mascotes " e "al é m d i sso".
e) "Os l i m i tes u s u a l m e n t e c o n s t r u í d o s e n tre a sociedade e
a natu reza estão s e n d o repe n sados com as mascotes" e
"de fato".

3. No mesmo texto, a expressão salientada "esa construcción"


(fi nal do primeiro parágrafo) faz referência à palavra:
a) sociedad.
b) natu raleza.
c) fam i l ia.
d) macotas.
e) personas.

4. A forma verbal "suelen", salientada no texto (fi m do pri­


m e i ro parágrafo), poderia ser trocada, sem prej u ízo de
sentido, pela forma:
a) parec e n .
b) acostu m b ra n .
c) p u e d e n .
d) p er m it en .
e) logra n .

GE IDIOMAS 2015 1 91
I NT E R P R ETAÇÃO D E TEXTO

Para unas
• �

vacac1ones Agencia de
Viajes Virtual 2


tranquilas
Puede oonsultat· y reservaron-llne un setvlcio
desde su dotn lcll lo os u lugar de t m ba jo, :11

:��
sln desplazamlentos, a horr<lndose las
molestlas det t rátioo y los a parcamlentos





Una agencia que promete �

provindenciarlo todo con


rapidez y eficiencia

(Vestibular UFSC 2012, adaptado)

EXERCfCIOS - UFSC 20012 (adaptado)


RESPOSTAS
1.Clasificamos el género textual presentado en el texto como: 1. O texto é uma propaganda. Ta nto a linguagem q u a n to os argumentos
a) p u b l i ci d ad apresentados nesse gênero pretendem atender às supostas necessidades
b) reportaje do leitor, que é o potencial consumidor. No caso do texto desta questão,
c) receta alguns dos argumentos usados referem-se às dificuldades da vida moder­
d) b i l l ete na como m o vimentar-se pelas cidades, perder tempo em filas ou faltar
e) poesia ao trabalho. As imagens também co n tribuem para vender o produto.
Mostram um destino de viagem e a tra n quilidade dos consumidores que
z. Acerca de las i nformaciones conte n idas en el texto, es correcto podem planejar e comprar as viagens sem sair de casa.
afi rmar que: Resposta: a
a) U rci To u r es u n a age n c i a d e viajes q u e e nvia u n agen te hasta e l
d o m i c i l i o d e sus c l i en te s para faci l itar l a com pra d e paquetes z. A n alisando cada uma das alternativas:
tu rísticos. a) A agência de viagens não oferece o serviço em dom icílio, mas pela
b) los cl ie ntes d e U rci To u r t i e n e n l a opción d e planear sus vaca­ internet. Ta nto é que o cartaz define a Urci como uma "agencia de viajes
c i o n es a través de la i ntern et, s i n la n ecesidad de d i r i g i rse a l a virtual". Incorreta.
agencia. b) Sim, o cliente tem a opção de contratar os serviços da agência pela
c) la agencia de Viajes U rci Tou r ofrece la programación más am plia internet. Correta.
d e i m e rcado. c) O texto do a n úncio é explícito: a agência o ferece a mais ampla progra·
d) para c o n s u l tar los servicios d e U rci Tou r e s necesario estar e n mação do mercado. Correta.
s u d o m ici l i o. d) O a n ú ncio não limita o atendimento virtual a q u em está em casa.
e) con U rci Tou r se puede h acer viajes v i rtuales. O consumidor p o d e estar ta m b é m em seu local d e tra balho: "Pu ede
consultar y reservar on fine un servicio desde su domicilio o su lugar de
3. En el apartado de I texto "Puede consultar y reservar on-l i n e un trabajo". Incorreta.
servi cio desde su domicilio o su lugar de trabajo, sin desplazamien­ e) A agência é virtual, mas não são virtuais as viagens q u e ela o fe rece.
� ahorrándose las moi estias de I tráfico y los aparcamientos", las Incorreta.
expresiones subrayadas pueden ser sustituidas consecutivamente, Resposta: b e c
sin alteración de signifi cado, por las expresiones:
a) extrafia m i e n to, las gravedad es, los estacionamien tos. 3. A questão pede que o estudante identifique sinónimos dos termos
b) alejamiento, las e nfermedades, las plazas de estacionamie ntos. sublinhados. "Desplaza m ie n tos" é o mesmo que deslocamen tos, tras­
c) trasladarse, las i n co m od i dades, los estac i o n a m i entos. lados. "Aparcamientos" são estacionamentos. "Aiejamien tos", da opção
d) d es pega m i e n tos, los i n fortu n ios, los s i t i as. b, significa afastamento. Cuidado com a pala vra "m o/estias", que pode
e) tras l ados, los i n conve n i e ntes, los estacionamientos. ser con fundida com "moléstias" (doen ças, em português). Em espanhol,
"mo/estia" significa "inconveniência". Tra ta-se de um falso cognato (veja
mais sobre falsos cognatos na pág. 97).
Resposta: e

92 J GE IDIOMAS 2015
DICAS ESTRUTURA » TUDO E M S UA O R D E M

Entenda a construção da oração em espanhol

SUJ EITO + V E R B O + O B J ETO D I R ETO + O UTROS Grafia h ispânica


C O M P LE M E NTOS Os principais países de l íngua espanhola
e seus gentílicos
Los pasajeros esperan e/ avión de la una para ir a Barcelona.
{Os passageiros esperam o avião da uma para ir a Barcelona.) Argentina - argentino, argentina
Bolivia - boliviano, boliviana
É possível alterar a ordem dos elementos que constituem a oração Chile - chileno, chilena
que vimos no modelo. A alteração depende da ênfase que o emissor Colombia - colombiano, colombiana
quer dar a um ou a outro elemento constitutivo da oração. Costa Rica - costarricense, costarriqueno, costarriquena
Cuba - cubano, cubana
Para ir a Barcelona, los pasajeros esperan e/ avión de la una. Ecuador - ecuatoriano, ecuatoriana
{Para ir a Barcelona, os passageiros esperam o avião da uma.) E/ Salvador - salvadoreno, salvadorena
Espana - espano� espano/a
Los pasajeros, para ir a Barcelona, esperan e/ avión de la una. Guatemala - guatemalteco, guatemalteca
(Os passageiros, para ir a Barcelona, esperam o avião da uma.) Honduras - hondureno, hondurena
México - mexicano, mexicana, mejicano, mejicana
O espanhol tende a su bstitu i r o verbo por um si ntagma. Nicaragua - nicaragüense
Isso expl ica a abu ndância de sintagmas verbais na l íngua Panamá - panameno, panamena
espanhola. Por exemplo: saltar ..., dar saltos (dar pu los, saltar)/ Paraguay - paraguayo, paraguaya
envejecer -+ hacerse viejo (enve l hecer). Perú - peruano, peruana
Puerto Rico - puertorriqueno, puertorriquena
Nas orações interrogativas, é comum a inversão da ordem entre República Dominicana - dominicano, dominicana
o sujeito e o verbo: Uruguay - uruguayo, uruguaya
iEs usted espano/ o argentino? Venezuela - veneza/ano, venezolana

Nas orações negativas, o advérbio "no" precede o verbo:


No quiero salir contigo. Atenção!
No caso de respostas negativas, o advérbio "no" se repete no Há adjetivos que possuem uma única forma,
início da oração: tanto para o masculino quanto para o feminino:
i Vamos ai cine conmigo? No, no puedo ir contigo, tengo que trabajar. el!la nicaragüense, el!la costarricense.

GE IDIOMAS 2015 1 93
I NT E R P R ETAÇÃO D E TEXTO

Los celos en el cyber mundo


Un a relación amorosa puede arruinarse gracias a la tecnología y ai acortamiento d e todas las d istancias

.olllll DEFINITIVAMENTE, la tecnologia ha cambiado la forma de relacionamos, es como si en internet los conflictos estuvieran a un clic de distancia

C
on la masificación de internet, no só lo aumenta e! número de !e caen mal a mi pololo. Con uno no me deja ni juntarme, así que para no
parejas que nacen aquí, sino que también están las que mueren perderlo de vista nos escribimos por e-mail. A veces siento que igual es
por culpa de una conexión defectuosa o por una ciber pelea infidelidad. Pero es la única forma en que no sea una pelea más".
amorosa, con ciber celos. Definitivamente, l a tecnologia ha cambiado la forma de
"Es raro, pero muchas veces peleo con mi pololo por lo celoso relacionamos, es como si en internet los conflictos estuvieran a un clic
que se pone en e! messenger. Si me demoro en responder, cree que de distancia. Como si lo que pasara en la pantalla no fuera más de lo
estoy hablando con otro. É ! es súper tranquilo, relajado, casi tímido, mismo que se vive en carne y hueso, sino que un terreno pantanoso y
por mensajes es más atrevido, más jugado. Pero también es celoso... difícil, donde es fácil hundirse si no se sabe desde antes donde estás
Si veo que se conecta, me pongo invisible para no pelear", cuenta pisando. Las extrafias e indefinidas regias de! romance 2.0.
Byota, sicóloga que trabaja en selección de personal.
Para Loi, chef que se dedica a! catering, internet es un infierno para (Extraído de vestibular da UFCG, 2006. ALDANA, Claudia. Amor en Tiempos de Internet:
los celosos. "Es fácil conocer más gente o comunicarme con los ex que Romance 2.0. Revista Ya E/ Mercurio, n• 1056, 2003, p. 31)

94 1 GE IDIOMAS 2015
EXERCÍCIOS - UFCG 2006 (adaptado)

1. O artigo destina-se às (aos): s. De acordo com o texto, a palavra pololo sign ifica:
a) agências de acon se l hamento matri m o n ial, pois trata d e a) namorado.
relações amorosas. b) computador.
b) p s i c ó l ogos, u m a vez q u e anal i sa o c o m portamento d e c) m e n sagem.
pessoas a n s iosas. d) marido.
c) sexólogos, te n d o e m v ista q u e m ostra o despertar das e) confl i to.
paixões dos ado l escentes.
d) usuários de salas de bate-papo, porq ue trata dos possíve is
ri scos desse m e i o. RESPOSTAS
e) provedores d e i nternet, pelo n ú m ero crescente de i nte­ 1. Usuários de salas de bate-papo, porque trata dos possíveis
ressados no assu nto em q uestão. riscos deste meio. Uma pista de que o destinatá rio do texto são os
usuários está no último parágra fo, no qual o próprio autor se coloca
z. As assertivas a segui r apresentam ideias relacionadas ao como usuário ao empregar o plural "nosotros": "Definitivamen te,
texto. Assinale se são verdadeiras (V) ou falsas (F). la tecnologia h a cambiado la forma de relacionamos, es como si en
( ) A i nternet promove a realização de mu itos casamentos. internet los conflictos estuvieran a un clic de distancia". Resposta: d
( ) Os serviços oferecidos pela internet facil itam as relações
amorosas. z. F F · V - F - V. Segundo o texto, a intern e t não promove casa­
) A i nternet é u m instrumento que prod uz fantasia. mentos, mas ajuda a formar novos pares ou a destruí-los. Não são
) O ciúme de alguns parceiros estimula a busca de novos os serviços o ferecidos que facilitam as relações amorosas. Além
relacionamentos pela i nternet. disso, segundo o texto, as relações pela internet causam ciúme
) A i nternet tem mudado as formas de convivência social. e ntre os casais. Resposta: c

A sequência correta é: 3. /nsegurança pode gerar problemas de relacionamentos por meio


a) v- F - v - F - v. da rede. De acordo com o texto, "interne t es un in fiemo para los
b) V - V - F - V - V. celosos". (internet é um inferno para os ciumen tos.) Ou seja, quem
c) F - F - V - F - V. é ciumento pode ver esse sentimento aumentar pelas relações que
d) V - F - V - F - F. seu pa r man tém com o utras pessoas pela internet. Resposta: e
e) F - F - F - V - V.
4- Como algo relacionado ou que perdura no presente. O verbo
3. A ide ia principal sugerida no terceiro parágrafo é que a(os): conjugado n o pretérito perfecto compuesto de indicativo revela
a) i n f i d e l i dade nas relações via i nternet não existe. que se trata de uma ação do passado cujos efeitos perduram no
b) i nternet é um excel ente meio para manter contato com presente. Resposta: e
ex-am igos.
c) parcei ros c i u me ntos têm m u itas d i ficu ldades para u sar 5 Namorado. No Ch ile, onde foi publicado origina lmente o artigo,
a i n tern et. a palavra "pololo" corresponde à palavra espanhola "navio", que
d) i nternet permite que os nam orados se com u n iq u em com significa nam orado. Resposta: a
mais freq u ê nc ia.
e) i nsegu rança pod e gerar pro b l e m as d e relacionamentos
por m e i o d a rede.

4. Com o uso da forma verbal n o trecho "la tecnología ha


cambiado la forma de relacionamos", o autor mostra q u e
considera essa m udança:
a) habitual n o passado e n o presente.
b) sem n e n h u m a relação com o presente.
c) passageira e sem cont i n u idade no presente.
d) anterior a o utras m u danças q u e oco rrerão n o presente.
e) como algo relaci onado ou que perd u ra n o presente.

GE IDIOMAS 2015 1 95
I NT E R P R ETAÇÃO D E TEXTO

La canción
EXERCÍCIOS - U FMG 2006 (adaptado)

L Senale la opción en que la correspondencia entre la paiabra


y su sinónimo entre corchetes está incorrecta:

del bandido
a) zares (ref. 4) = [USUARIOS].
b) afu e ras (ref. 2) = [ALR E D E DORES].
c) botín (ref. 3) = [DESPOJO]
d) capas (ref. 1) = [J EFES].
El llamado "narcocorrido" ha venido haciendo
carrera en la industria musical 2.Los corridos, según el texto, son:
a) canciones sobre la vida d e narcotraficantes.
b) "periód icos con m ú s ica" que cantan y m itifican h istorias.

L
a subversión es elemento frecuente en la cultura mexicana. c) un tipo d e su bve rsión frecuente e n la c u l t u ra m ex i cana.
Ejemplo de ello es e! caso de! bandido quien, admirado d) un géne ro m us i cal d e las afue ras d e C u l iacán.
por e! pueblo, representa - para bien o para mal - la
contrapartida de las instituciones omnipotentes. Actualmente 3. E n "Desde h ace más de un siglo los corridos son " u n
son ante todo los capos (1) de! narcotráfico los que atizan periódico c o n música" para el pueblo li ano, la voz de aquellos
la fantasia anarquista. Ejemplo de ello es e! llamado que no tienen voz", la expresión subrayada significa:
"narcocorrido", un género que desde hace algunos afíos ha a) narcotraficantes.
venido haciendo carrera en la industria musical. b) todos os m ex i canos.
En las afueras (2) de Culiacán, capital de! Estado Federal de c) autores dos corridos.
Sinaloa, en e! noroeste de México, se encuentra una capilla. d) m exicanos m ai s s i m ples.
Antes era más pequena y se encontraba supuestamente sobre
la tumba de "San" Jesús Malverde. Pero este lugar de 4. De acordo com o texto, são corretas as afirmações:
peregrinación constituía u n dolor d e muelas para las I . O mais provável é que os corridos tenham se originado na
autoridades sinaloenses; debia de desaparecer y con ella e! tradição do romance trazida pelos espanhóis ao México.
recuerdo de un personaje ai que la Iglesia nunca hizo proceso 11. jesús Malverde era um dos capos do narcotráfico mexicano.
de santificación. La medida levantó un huracán de protestas, 1 1 1 . j esús Malverde é o patrono dos corrid istas.
así que se tomó una decisión "a la mexicana", surreal y IV. O narcocorrido é um tipo de corrido que surgiu faz poucos
paradójica. Se demolió la capilla y se construyó una nueva, anos.
más hermosa. a) Todas.
"Ahora queda una tumba I de aquel hombre tan querido, I que b) I, 11 e IV.
todo lo que robaba I !o repartia entre los pobres, I por eso es que c) I, 1 1 1 e IV.
hoy en día I se !e hacen grandes honores" dice e! texto de "La d) 1 1, 1 1 1 e IV.
muerte de Malverde" de Seferino Valladares, uno de los muchos e) N e n h u ma.
corridos acerca de este Robin Hood en versión mexicana, que
compartia su dinero con los pobres, como afirman algunos,
mientras que las malas lenguas murmuran que con e! botin RESPOSTAS
(3) mantenia e! buen negocio de las cantinas. La temeridad y L Zares (ref.
4) [USUARIOS]. Essa palavra não faz referência aos
=

descaro de Malverde no se detenian ante nadie, tampoco ante usuários d e drogas, mas aos capos, chefes do narcotráfico. Ela
las fuerzas dei gobierno. Asi que no es de asombrar que a causa significa, em português, "czar·: an tigo soberano russo. Resposta: a
de sus correrias se tornara e! santo de los zares (4) de la droga
y e! patrón de sus cronistas, los corri distas. 2. "Periódicos con música" que cantan y mitifican historias. A res·
La historia, en doble sentido, ha impresionado y apasionado posta a essa questão está no trecho "Se narra, se mitifica, y de
siempre a los mexicanos. Se narra, se mitifica, y de preferencia, preferencia, se canta. Desde hace más de un siglo los corridos son
se canta. Desde hace más de un siglo los corridos son "un 'un periódico con m úsica' para e! pueblo /Iano, la voz de aquellos
periódico con música" para e! pueblo llano, la voz de aquellos que no tienen voz". Não cantam somente histórias do narcotráfico,
que no tienen voz. Los orígenes de! corrido no son claros não são somente um gênero musical dos arredores de Culiacán e
y constituyen un tema de debate entre musicólogos. Los nem sempre são subversivos. Resp osta: b
nacionalistas pretendeu enraizado en la cultura azteca; más
plausible es la suposición de que llegó con los conquistadores J. Mexicanos mais simples. Nesse contexto, a palavra "/!ano" significa
espafíoles y la tradición de! romance. que não goza de privilégios. Resposta: d

ESSL, Andréas. La Canción Del Bandido. Revista Humboldt, n. 141, 1004, adaptado. 4- l, 111 e IV. A única afirmação incorreta é a /I: no texto, não se afirma que
(Vestibular UFMG 1006) }esús Ma/verde era um dos capas do narcotráfico mexicano, apesar
da possibilidade de que participasse de contravenções. Resposta: c

96 1 GE IDIOMAS 2015
DICAS VOCA B U LÁRIO » PARECE, MAS NÃO É

Fique de olho no vocabulário


para não se confundir

Em qualquer leitura e m espan hol, você já deve ter identi· » restar - s u btrair, d i m i n u i r
ficado m u itas palavras cognatas, ou seja, parecidas com o » salada - salgada
português e q u e têm o mesmo significado em função de u m a » sitio - l ugar
origem comum: o latim. Contudo, h á palavras que podem enga­ » sótano - porão
nar porque, apesar da semelhança formal, apresentam signifi­ » taco - palavrão ou tort i l la mexicana
cados d i ferentes. São chamadas heterossemânticos. Vejamos » tacón - salto do sapato
algumas dessas palavras q u e podem causar confusão: » talón - calcanhar
» taza - xícara
» acordar - c h egar a um acordo » tirar - arremessar, d eixar cair
» a/mohada - travesseiro
» balcón - terraço, sacada
» basura - l ixo
» bols o - bolsa Heterogenéricos
» borracha - bêbada Confira palavras que possuem forma igual
» borrar - apagar ou semel hante e m espanhol e e m português,
» brinco - salto mas pertencem a gêneros diferentes
» cajón - gaveta
--

» cano - grisal h o Masculinas em espanhol, Femininas em espanhol,


» casco - capacete, f o n e s d e ouvido femininas em português masculinas em português
» chaqueta - paletó
» cena - jantar e/ árbo/ (a árvore) la "a" (e as outras letras

» cinta - fita do alfabeto) (o "a")


» copo - floco e/ colar (a cor) la baraja (o baralho)
» crianza - criação
elcutis (a cútis) la costumbre (o
» cubiertos - tal here s
costume)
» embarazo - gravidez
» enderezar - e n d i re itar e/ desorden (a desordem) la feche (o l e i te)
» escoba - vasso u ra
e/ dolor (a dor) la mie/ (o mel)
» escritorio - escrivan i n ha
» estafar - e nganar e/ color (a cor) la nariz (o nariz)
» éxito - sucesso e/ equipo (a equipe) la protesta (o protesto)
» exquisito - gostoso
» experto - especial i sta
e/ estreno (a estreia) la sal (o sal)

» extrafíar - estra n har, s u rpree n d e r, ter saudade d e e/ funes (e demais dias da la sangre (o sangue)
» tacha - aspecto, fascistas semana) (a segunda·feira)
» faro - farol
e/ mensaje (e outros la sonrisa (o sorriso)
» fecha - data
substantivos en -je) (a
» flaco - magro
mensagem)
» ganancia - l u cro
» habitación - q u arto e/ puente (a ponte) la risa (o riso)
» inversión - i nvesti mento e/ vais (a valsa) la cumbre (e outros
» ladrillo - tijolo substantivos terminados
» largo - comprido, longo em -umbre) (o cume)
» latir - p u l sar
» /en til/as - l entes d e contato e/ vértigo (a vertigem)
» luego - d e p o i s e/ análisis (a análise)
» pastel - b o l o

GE IDIOMAS 2015 1 97
I NT E R P R ETAÇÃO D E T EXTO

La muerte según Pedro Almodóvar


El director espafíol habla acerca de la del icada cuestión de la aceptación de la muertel
tema de su película Volver

4111111 PENtLOPE CRUZ en una escena de Volver, dei director Pedro Almodóvar: una película sobre la muerte y la soledad

V
olve r es un título que incluye varias vueltas, para mí. He vuelto, La principal vuelta de Volver es la dei fantasma de una madre,
un poco más, a la comedia. He vuelto ai universo femenino, a La que se aparece a sus hijas. En mi pueblo estas cosas pasan (me he
Mancha (sin duda es mi película más estrictamente manchega, criado oyendo historias de aparecidos), sin embargo yo no creo
el lenguaje, las costumbres, los patios, la sobriedad de las fachadas, en las apariciones. Sólo cuando les ocurren a los demás, o cuando
las calles empedradas). He vuelto a trabajar con Carmen Maura (hace ocurren en la ficción. Y esta ficción, la de mi película (y aquí viene
diecisiete anos que no lo hacíamos), con Penélope Cruz, Lo la Duenas mi confesión), ha provocado en mí una serenidad como hace tiempo
y Chus Lampreave. He vuelto a la maternidad, como origen de la no sentía (realmente, serenidad es un término cuyo significado es
vida y de la ficción. Y, naturalmente, he vuelto a mi madre. Volver a un misterio para mí).
La !Vl ancha es siempre volver ai seno materno. En los anos que llevo de vida, nunca he sido una persona serena
Tengo la impresión, y espero que no sea un sentimiento pasajero, (ni me ha importado lo más mínimo). Mi innata inquietud junto
de que he conseguido encajar una pieza (cuyo desajuste, a lo largo a una galopante insatisfacción me han servido generalmente de
de mi vida, me ha provocado mucho dolor y mucha ansiedad, diría estímulo. Ha sido en los últimos anos, en los que mi vida se ha ido
incluso que en los últimos anos había deteriorado mi existencia, deteriorando, consumida por una terrible ansiedad. Y eso no era
dramatizándola más de la cuenta). La pieza a la que me refiero es "la bueno ni para vivir, ni para trabajar. Para dirigir una película es más
muerte", no só lo la mía y la de mis seres queridos sino la desaparición importante tener paciencia que talento.
implacable de todo lo que está vivo. Nunca lo he aceptado, ni lo he
entendido. Y eso te pone en una situación angustiosa ante e! cada (www.cl u bc uI tu ra.co mtcl u bc i ne/c I u bci n e astas/ ai modovar/voi ve ri a pe I i cu la/e n pai a·
vez más rápido paso de] tiempo. braso2.htm) (Vestibular U FMG 2007)

98 1 GE IDIOMAS 201S
EXERCfCIOS - U FMG 2007 (adaptado)

1. Segundo o texto da página anterior, Almodóvar, no seu 5. Nos seguintes e n un ciados retirados do texto, "hace die­
filme Volver, voltou a: cisiete anos q u e no lo HACIAMOS" e "en los ú ltimos anos
a) considerar la mate rni dad tanto como origen de la vida HABIA DETERIORADO m i existencia", os verbos destacados
como de la ficción. e m letras mai úsculas estão, respectivamente, no:
b) hacer reír ai públ ico cinematográfico con el lenguaje de a) pretérito perfecto s i m ple e pretérito perfecto compuesto
La Mancha. de i nd i cativo.
c) trabajar el u n i verso de la mujer, sus costumbres y sobrie­ b) pretérito i m perfecto e pretérito p l u scuam pe rfecto de
dad, como antes. i nd icativo.
d) ver el fantasma de una mad re, l a suya, que se les aparece c) preté rito p l u sc uam perfecto e preté rito i m pe rfecto de
a sus h ijas. i nd i cativo.
d) pretérito perfecto compuesto e pretérito perfecto s i m p l e
z. Releia o seguinte trecho: "La pieza a la que me refiero es de i n d icativo.
'la muerte', no só lo la mia y la de mis seres q u eridos sino la e) pretérito perfecto com puesto e preté rito pluscuam per­
desaparición implacable de todo lo que está vivo". As expres­ fecto de i n d i cativo.
sões sublin hadas acima estabelecem entre os e n u nciados
apresentados uma relação de:
a) justaposição. RESPOSTAS
b) com paração. 1. Considerar la maternidad tanto como origen de la vida como de la
c) causa. ficción. No primeiro parágrafo do texto, Almodóvar dá, literalmen te,
d) soma de argumentos. essa informação a o fazer a seguinte comparação: "H e vuelto a la
e) contraste. matemidad, como origen de la vida y de la ficción". Resposta: a

3. No trech o "Volver es un títu lo que incluye varias vueltas, z.Soma de argumentos. No trecho destacado na questão, temos o
para m í. H e vuelto, un poco más, a la comedia", é estabelecida encadeamento de enunciados por conexão, com o uso de conecto·
entre esses dois e n u n ciados uma relação de: res. Em espanhol, os conectares no sólo/sino correspondem a não
a) correção. só/mas também do português e promovem uma relação de soma
b) especificação. de argumentos!ideias n o texto. Para recordar como funciona o
c) tempo. encadeamento por conexão, vá à página 124. Resposta: d
d) motivos.
e) causa e efeito. 3. Especificação. No primeiro enunciado desse trecho do texto,
Almodóvar afirma que seu filme representa para ele vários tipos
4. Observe o seguinte trecho: "Tengo la im presión, y espero de retorno, enquanto no segundo enunciado especifica um deles,
que n o sea un sentim iento pasajero, de que he conseguido a volta às comédias. O texto da página 124 explica como funciona
encajar una pieza, cuyo desajuste, a lo largo de m i vida, m e o encadeamento por justaposição. Resposta: b
haprovocado mucho dolor y mucha ansiedad, ..." Se as formas
verbais s u b l i nhadas acima estivessem na mesma pessoa e ft. Consegui1 provoqué. A conjugação dos verbos no pretérito per­
no pretérito perfecto simple, teríamos: fecto simple de indicativo é explicada na página 126. Resposta: a
a) conseguí!provoqué.
b) consegu ías/provocaban. s. Pretérito imperfecto e pretérito pluscuamperfecto de indicativo.
c) consegu ía/provocaba. Para responder à ques tão, é útil reler a conjugação desses tempos
d) consegu í!provocó. verbais na página 125. Resposta: b
e) consegu í!provocaba.

GE IDIOMAS 2015 1 99
I NT E R P R ETAÇÃO D E T EXTO

De la ficción
Interpela temas sociales, políticos y culturales contemporáneos.
Provoca asombro, estimula la imaginacián y la creatividad.
Es fuente de inspiracián para e! futuro y colabora para que

a la ciencia
nuestra especie abandone los intereses personales y aspire a
nuevas posibilidades.

(http://axxon.com.ar) (Vestibular da Uerj 2010. )


La inspiración de muchas ideas de
la moderna era espacial surgió de las
EXERC[C I O S - U E RJ 2010 (adaptado)
aventu ras de la ciencia ficción
1. La c i e n c i a ficción p u d o contri b u i r positivamente para
ulio Verne imaginá el primer viaje del hombre a la !una en el progreso de la sociedad. Según el texto, la presencia de

J
su novela De la tierra a la /una, en 1865. Arthur C. Clarke civi l izaciones extraterrestres en esa clase de obra ayudó a
anticipá las estaciones espaciales y las computadoras sen­ los científicos a:
Sibles en su clásico 2001: una odisea de/ espacio. Ray Bradbury a) i n vesti gar plati l los vo ladores.
escribiá sobre civilizaciones extraterrestres en sus Crónicas b) local izar estac i o nes orbitales.
marcianas muchos afíos antes que el Mars Rover de la NASA c) real izar viajes i nterplan etarios.
explorara el vecino planeta. d) co m u n i carse con a l i e n ígenas.
Estas historias, escritas antes de que los viajes espaciales
fueran posibles, fueron fuente de inspiracián para generaciones 2. El autor dei texto busca crear una atmósfera en pro de
enteras de científicos y exploradores espaciales. la ciencia ficción. Para eso, hace uso por varias veces dei
"La ciencia ficcián ayuda a que se encienda la chispa de la sigu i ente rasgo característico de la argumentación:
imaginacián en lo que muchos piensan que es un tema acabado" a) m odal izac i ó n .
- opiná Kurt Lancaster, escritor de ciencia ficcián y profesor b) com paración.
adjunto de estudios de medi os y comunicacián en Fort Lewis c) generalizac i ó n .
College, Colorado. d) eje m p l ificación.
En 2001, la Agencia Espacial Europea (ESA) realizá un
análisis minucioso de las primeras obras !iterarias, plásticas y 3. Los verbos q u e están conjugados correctame n te e n el
cinematográficas de ciencia ficcián para determinar si algunos m i smo tiempo verbal que podría son:
de los conceptos y de los ejemplos de tecnologia imaginados en a) q uerría, i r ía, sali ría.
esas obras podrían servir de inspiracián para naves y misiones b) q u erería, i ría, sald ría.
espaciales actuales y futuras. c) q uerría, i r ía, sald ría.
La agencia recogiá más de 250 conceptos entre científicos, d) q u e re ría, iría, sali ría.
ingenieros, escritores de ciencia ficcián y gente común. Un
folleto ilustrado reúne estas ideas, que en algunos casos podrían
convertirse en realidad gracias al trabajo de investigadores RESPOSTAS
espaciales europeos. 1. Realizar viajes interplanetarios. A resposta p ode ser justificada
La literatura, las obras de arte y las películas de ciencia ficcián com a seguinte passagem do texto: "'Sin la ciencia ficción, jamás
suelen ser producto exclusivamente de la imaginacián de sus hubiéramos ingresado en la era espacia l' - aseguró Lancaster. 'Los
autores y a veces contienen errares. No hay ningún hotel Hilton jóvenes lectores de finales de/ sigla XIX recib ie ro n una inspiraci·
en la !una, contra lo que describe Clarke en 2001. Sin embargo, ón tan fuerte de julio Veme y H. G. Wells q ue muchos de e/los se
algunos pronásticos, sistemas y tecnologias propu estas en las especializaron en cohetes para así pod e r inventar la tecnol ogia
primeras obras de ciencia ficcián se hicieron realidad. q u e Jes permitiria viajar a otros m u n dos'." Resposta: c
"Sin la ciencia ficcián, jamás hubiéramos ingresado en la era
espacial" - asegurá Lancaster. "Los jávenes lectores de finales 1. Ejemplificación. O autor do texto utiliza-se de exemplos do
del siglo XIX recibieron una inspiracián tan fuerte de Julio cinema e da literatura para criar u m a a tm osfera positiva para a
Verne y H. G. Wells que muchos de ellos se especializaron en ficção cien tífica . Resposta: d
cohetes para así poder inventar la tecnologia que les permitiria
viajar a otros mundos." 3. Querrfa, iría, saldría. Todos os verbos estão no condicional
Entre los conceptos de la ciencia ficcián que se hicieron simp les. O tem a é abordado na página 130. Resposta: c
realidad se pueden mencionar los lanzadores de proyectiles
ultraveloces, los cohetes propulsores, las cápsulas de descenso,
los trajes presurizados, las estaciones orbitales, los veleros
solares o velas de fotones y las comunicaciones satelitales.
Para Lancaster, la ciencia ficcián es la mitologia de nuestra
época. Apunta al futuro y nos sefíala nuestro lugar en el cosmos.

100 I GE IDIOMAS 2015


DICAS EXPRESSÕES I D IOMÁT I CAS » O POUCO Q U E D I Z M U ITO

Conheça algumas expressões idiomáticas

O espan h o l possui várias expressões i d i omáticas, ou seja, >> Creerse e / ombligo de/ mundo. - Pen sar q u e é o centro d o
construções q u e não d evem ser trad uzidas ao pé da letra, U n iverso.
somando os sign i ficados i n d ividuais dos vocábulos q ue as >> Cuando e/ río sue na, agua /leva. - A voz do povo é a voz d e
formam, sob o risco de l evar a i nterpretações e q u i vocadas. Deus.
Essas expressões variam de acordo com a c u ltura dos l u ­ >> Cuando las barbas de tu vecino veas cortar, pon las tuyas
gares o n d e são e m pregadas. Representam j u ízos d e valor a remojar. - Pôr as barbas de m o l h o.
sobre variados temas e, embora predom i ne m na l i nguagem >> Cuatro ojos ven más que dos. - D u as cabeças p e n sam
oral, é i m p o rtante conhecê-las, porq u e são recorren tes nos melhor d o q ue u ma só.
textos escritos. >> Dar la lata (bara). - Encher a paciência.
As expressões i d i omáticas se caracterizam por d izer m u i­ >> Dar la nota. - Fazer-se n otar, exi b i r-se de forma negativa.
ta coisa com poucas palavras. Por isso, com fre q u ê ncia, as >> Dar/e calabazas a alguien. - Dar o fora em alguém.
pessoas util izam essas expressões para apresentar u m argu­ >> Darse a todos los demonios. - V i rar o d iabo, f i car m u ito
mento de consenso, ou seja, um ponto de vista praticamente zangado.
i rrefutável, por estar ancorad o e m u m a ideia socialmente >> Dar un dedo de la mano 1 Dar un ajo de la cara. - Dar tudo
recon hecida. Veja algu mas d essas expressões: e m troca d e alguma coisa, estar d isposto a pagar qualquer
preço para o bter algo.
» A buenas horas, mangas verdes - usada q uando alguém >> Dar las uvas. - Criar raízes, esperar sentado por algo.
aparece quando não é mais n ecessário. >> Darse una torta. - Cair, ferir-se.
» Agarrar (coger/tener) e/ gusanillo - Estar com um b u raco >> Decir a a/guien cuántas son tres y dos. - Dizer poucas e
no estômago, estar morto de fome. boas.
» iAgárrate y no te muevas! - Deus me I ivre! >> Decir ni mú. - Não dar um pio.
> > Agua que no has de beber, déjala correr. - V i va e d e ixe >> Dejar paso. - Abrir espaço, d ar passagem.
viver. >> Dejarse de bramas. - Deixar d e brincad e i ra, parar com a
>> A la cama no te irás sin saber una cosa más. - Podemos bri ncad e i ra.
apre n d e r algo n ovo a cada d ia. >> Del dicho a/ hecho hay mucho trecho. - Falar é fác i l, fazer
>> AI decir. - Segu n d o dizem. é q u e é d ifíc i l .
> > iAhí va! - Veja só! >> D e tal paio tal astilla. - Tal pai, tal f i l ho.
> > Ande yo caliente, ríase la gente. - Dar pouca i m portância >> Donde Cristo dia las tres voces. - M u ito l o n ge.
aos comentários ou fofocas das pessoas e não se preocupar >> Donde las dan las toman. - Tudo o que vai volta.
com a o p i n ião a l h e i a. >> Echar mal ojo. - Pôr mau-olhado.
>> A paio seco. - De estômago vazi o . >> Echarse algo a las espaldas. - Deixar para trás; d e ixar para
>> iA q u é viene ? - E o q u e isso t e m a ver? Q u a l a relação? Com lá. Não se preocu par com algo.
que o bj etivo? Expressa desco n h ec i m en to d e algo. >> Echarse algo sobre las espaldas. - Carregar nos o m b ros,
>> A río revue/to, ganancia de pescadores. - Quando as coisas encarregar-se, ass u m i r u ma respo n sab i l i dade.
não são claras, alguém poderá aproveitar-se d a situação. >> Echar (tomar) algo a brama. - Levar n a b r i n cade i ra.
>> Aunque la mona se vista de seda, mona se queda. - O hábito >> Echar tripa. - Criar barriga.
não faz o m o n ge >> Echar una mano. - Aj u dar.
>> Cerrarse en banda. - Fechar-se n u m cas u l o, ser como u m >> E/ plato de la mesa ajena se antoja más que e/ propio. - A
caramujo. grama d o vizi n h o é sem pre mais verde.
> > Comerse los pufíos. - Roer as u n has d e arre p e n d i m ento, >> E/ que avisa no es traidor. - Quem avisa a m i go é.
ter vontade de matar-se. >> E/ que mucho abarca poco aprieta. - Quem tudo quer nada
>> Creer que todo e/ monte es orégano. - N e m t u d o o q u e tem.
reluz é o u ro, acreditar q u e t u d o é fáci l . >> Empinar (alzar) e/ cada. - Encher a cara, e m bebedar-se.

GE IDIOMAS 2015 1 101


I NTERP R ETAÇÃO D E TEXTO

Los peligros de comer bocadillos


Los riesgos de la contam i nació n
en la preparación de a l i m e ntos

� ENFERMEDADES EN LA LONCHERA. Aunque sea apetitosa, la merienda puede llevar microorganismos patógenos, causadores de intoxicaciones

R
ápidos, fáciles y sabrosos. Los bocadillos son una preparación una cocción que elimine gérmenes peligrosos, como la salmonella.
muy recurrente en excursiones de un día, desayunos o me­ En estas casos, es posible una contaminación cruzada porque los
riendas escolares, e incluso, para cenas. Las posibilidades son microbios de los alimentos crudos pueden extenderse a otros ali­
numerosas, como lo son los ingredientes que pueden utilizarse para mentos, por contacto con superficies, utensilios o manos. Si en los
prepararias. En todos los casos, tanto si se elaboran con productos bocadillos se mezclan alimentos crudos con cocinados, e] riesgo es
crudos como cocinados, deben prepararse con las mismas precau­ mayor, mientras que preparaciones ricas en proteínas (cerda, patés,
ciones sanitarias que cualquier otro plato más elaborado, ya que en marisco) pueden causar intoxicaciones alimentarias si se dejan a
e !los también pueden crecer bacterias patógenas. temperatura ambiente.
Embutidos, carne, verduras, huevo o chocolate; fríos o calientes;
con pan de molde o en barra. Las posibilidades para elaborar bo­ Bocad i l los frios
cadillos son infinitas, tanto en lo referente ai relleno como al tipo Son sencillos de elaborar, pera no por e!lo están exentos de ries­
de pan. No obstante, cuando los ingredientes no requieren cocción gos. Cuando se prepara un bocadillo con jamón cocido envasado ai
(tomates, lechugas, atún y embutido, entre otros), deben tratarse vacío o en atmósfera modificada, se manipulan lanchas y a menudo
con unos estrictos princípios de higiene para evitar riesgos, como se consumen ai día siguiente las no utilizadas. Pera estas entran
contaminaciones cruzadas. Cualquier relleno, tanto frío como ca­ en contacto con las manos, la mesa o utensilios y es posible una
liente, puede contaminarse. contaminación microbiana, que crece de forma importante si las
condiciones de tiempo y temperatura son favorables. Uno de los
Bocad i l los cal ie ntes bocadillos fríos que mayor atención merece desde e! punto de vista
Las posibilidades para elaborar bocadillos calientes son tan infini­ sanitario es e] de atún. Dejarlo muchas horas a temperatura ambiente
tas como lo se a la imaginación de cada uno. Desde los tradicionales puede provocar que se infecte con microorganismos.
con tortilla o !orno con queso, a combinaciones más sofisticadas que
ponen a prueba la fantasía de quien los elabora. En todos los casos, Por Marta Chavarrías
durante la fase de preparación y presentación, se requiere calor. (Disponível e m : <www.consumer.es/seguridad-alimentaria/sociedad-y-consu­
Este proceso facilita la eliminación de ciertos patógenos, aunque en mo/2011/05/25/200834-Php>. Acesso em l0/6/2011. Adaptado)
e] caso de la tortilla, es imprescindible cuajarla bien y someterla a (Vestibular UFRN 2012. adaptado)

102 1 GE IDIOMAS 2015


EXERCfCIOS - U FR N 2012 (adaptado) d) Do mesmo modo, a q u es tão da temperatura é apontada como um
dos fatores de risco para a contaminação dos alimentos. Não é o tema
1. A ide ia principal do texto é: central do texto. Incorreta.
a) Os c u i dados h i g i ê n icos q u e se devem ter na preparação d e Resposta: a
d iversos tipos d e lanches.
b) Os problemas que são causados à saúde pela i n gestão de lanches.
c) As fac i l idades d e preparo e os t i pos d e recheio q u e se podem l. A palavra "es tas" é um pronome demonstrativo que substitui a pal� vra
colocar nos lanches. "lanchas" (fatias), da frase an terior. Isso elimina as alternativas c e d, que
d) As d i ferenças de tem peratura que p reci sam ser consideradas na associam "estas" a "manos" (mãos). já a expressão "...y a menudo se consumen
p re paração d e lanches. ai dfa siguiente... " (e frequentemente são consumidas no dia seguin te) indica
que o termo "estas" se refere a fatias que não são consumidas imediatamente.
2. A palavra "estas", grifada e m negrito no ú ltimo parágrafo, se Resposta: b
refere a:
a) fatias d e presunto con s u m i das. 3.Ana/isando as alterna tivas:
b) fatias d e presunto não c o n s u m i das. a) A primeira frase, "Los bocadil/os son una p reparación muy recurrente
c) mãos protegidas q u e man i p u lam as fatias. en excursiones de un día, desayunos o meriendas escolares, e incluso,
d) mãos desprotegidas q u e man i p u lam as fatias. para cenas" (os lanches são uma preparação muito comum em excursões,
cafés da manhã ou merendas escolares e, inclusive, como jantar) indica
3. Além de serem saborosos, rápidos e fácei s de fazer, os lanches que o texto considera os lanches opção para qualquer refeição - também
podem ser: no café da manhã ("desayunos"). Incorreta.
a) c o n s u m i dos raramente n o café da man hã. b) No mesmo trecho analisado acima, a pala vra "recurrente" significa
b) con s u m i d o s d iariame nte em q ua l q u e r refei ção. frequen te, comum. Incorreta.
c) p re parados com um t i po d e pão específico. c) Ao con trário do que propõe a alternativa, o texto diz que os lanches
d) p reparados com variados i n gred i e n tes. podem ser preparados com diferentes tipos de pão. No segundo parágrafo,
encontramos explicita m e n te uma referência a essa possibilidade: "las
4. É correto afirmar que: posibilidades para elaborar bocadillos son infinitas, tanto en lo referente
a) o m e l ete é u m tipo d e recheio de lanche q ue d eve ser bem cozido ai relleno como ai tipo de pan". Incorreta.
para e l i m i nar as bacté ri as. d) O texto é explfcito. Ainda no primeiro parágra fo, i n forma que "las
b) recheios crus e cozidos, q uando m isturados com proteínas, geram posibilidades son numerosas, como lo son los ingredientes que pueden
um risco de i ntoxi cação ali me ntícia maior. utilizarse para preparar/os". Correta.
c) lanches q u en tes são, geral m e n te, c o n s u m idos no verão. Resposta: d
d) l o m bo com q u eijo é um tipo sofisticado d e recheio para lanches.
4. Sim, o texto diz que omeletes (tortillas) devem ser bem cozidas a fim de
s. Na frase "No obstante, cuando los i ngred ientes no req u ie ren eliminar germes. Isso é claro na frase "... aunque en e/ caso de la tortilla,
cocción (tomates, lechugas, atún y em butido, entre otros), deben es imprescindible cuajarla bien y someterla a una cocción que elimine
tratarse con u nos estrictos princi pias de h igiene para evitar ries­ gérmenes peligrosos, como la salmonella". Correta.
gos, como contaminaciones cruzadas", a expressão "no obstante" b) De acordo com o texto, sanduíches q ue miswram alimentos crus e
pode ser su bstitu ída, sem prej u ízo semântico, por: cozidos apresentam maior risco de contaminação. Alimentos ricos em
a) aun q u e. protelnas podem causar intoxicação, se não forem man tidos refrigerados.
b) p u es. Veja o trecho: "si en los bocadiilos se mezclan alimentos cru dos con coei­
c) s i n o . nados, e/ riesgo es mayor, mien tras que preparaciones ricas en proteínas
d ) s i n em bargo. (cerda, pa tés, marisco) pueden causar intoxicaciones alimentarias si se
dejan a temperatura ambiente." Incorreta.
c) Não há. nenhuma referência sobre a estação do a n o na qual são con·
RESPOSTAS sumidor sanduíches quen tes. Incorreta.
1. Analisando as alternativas: d) Segundo o texto, sanduíches de lombo com queijo ("Iomo con queseo")
a) Sim, o texto fala da importância dos cuidados higiênicos que se deve são tradicionais (tradicionales), não so fisticados. Incorreta.
ter na preparação de diversos tipos de lanches. Isso fica claro no primeiro Resposta: a
parágrafo, na expressão ". . . deben prepararse con las mismas precau­
ciones sanita rias" (devem ser preparadas com as mesmas precauções s.A expressão "no obstante" (não obstan te, no e n tan to, porém) é um co­
sanitárias). Correta. nectar que tem valor contrastivo e contra-argumentativo (ou adversativo).
b) O texto fala sobre eventuais problemas de saúde - em particular, de E é sinônimo de "pera" e "sin embargo". Traduzindo a frase: "no entan to,
intoxicações alimentares -, mas como consequência da falta de cuidado quando não requerem cocção (tomates, alfaces, atum e embu tidos, entre
na preparação de lanches. Este não é o tema principal. Incorreta. o u tros}, os alimentos devem ser tratados com princípios rígidos de higie·
c) Apesar de citar que uma das características dos sanduíches é a facilidade ne para evitar riscos, como contaminação cruzada". (Sobre con ectares,
de preparo e de mencionar a possibilidade de diferentes tipos de lanche, veja as págs. 124 e 127)
este também não é o tema principal do texto. Incorreta. Resposta: d

GE IDIOMAS 2015 1 103


I NT E R P R ETAÇÃO D E TEXTO

Perdidos en
que rodar un episodio a la semana y no solo de una serie, sino
de tres, suponía un seria handicap para un desarrollo mínimo
de guiones con un poco más de seriedad.

el espacio
(Perdidos en e/ espacio. Guión: lrwin Allen. Ano de lanzamiento en EEUU: 1965)
(Vestibular Uerj 2010)

Los guion istas d e l a se r i e n o ten ían EXERCÍCIOS - UERJ 2010 (adaptado)


el más m í n i m o pudor en usar cual q u i e r L Tras algunas décadas, es posible detectar los problemas que
te ma para l o s capítu los tenía la se ri e. Sus problemas ten ían como causa principal:
a) los autores q u e creaban exte n sos g u i o n es.
b) la e m presa q u e produ cía de masiadas series.
c) e l e q u i p o que i magi n aba m uchos m o nstru os.
d) los personajes que e ran excesivam ente caricatos.

z. En el texto, el uso reiterado d e verbos conj u gados e n


pretérito i m perfecto de i n d i cativo se debe:
a) a que p red o m i nan los párrafos descriptivos e n e l texto.
b) a q u e p red o m inan los párrafos narrativos e n e l texto.
c) a q u e pred o m i nan los párrafos d isertativos e n e l texto.
d) a la mezcla de párrafos d isertativos y narrativos en el texto.

RESPOSTAS
.Ali LOS PERSONAJES de Perdidos en el espacio: una de las series de ciencia 1. La empresa q u e p rod ucía demasiadas series. A resposta para
ficción más emblemáticas de los anos 6o essa q u estão se encon tra no fim do texto, quando se informa
acerca do trabalho febril que contribuía para que os argumentos

L
a serie narraba las aventuras de una familia, los Robinson, de outras séries passassem a esta, e vice-versa. Também se fa la
claramente inspirada en la novela Los Robinsones Suizos, sobre a dificuldade de filmar um episódio por semana de três
embarcados en un viaje a bordo de la nave Júpiter II rumbo séries simultaneas, o que demandava roteiros com pouca qua·
a Alpha Centaury, con e! fin de fundar una colonia. Debido a /idade. Resposta: b
un sabotaje dei Dr. Smith, agente de una potencia extranjera,
que queda atrapado en la misma nave, su viaje se convierte en z. A que predominan los párrafos descriptivos en e/ texto. No texto,
imposible y, como su propio nombre indica, se pierden en e! predomina a descrição de como eram filmados os capítulos da série
espacio sin posibilidad de poder volver a la Ti erra. televisiva, as circunstâncias que caracterizaram as filmagens e a
Pero: c\qué tiene esta serie para convertirse en un referente apresentação da série na televisão nos anos 1960 e nas décadas
de la ciencia ficción de los sesenta? Pues la verdad es que no posteriores. Resposta: a
lo sé, pero sí recuerdo que no me la perdia cuando se emitia
en aquellos lejanos afias.
Las conversaciones disparatadas dei robot con e! repelente
Dr. Smith, cuya frase favorita era estamos perdidos, moriremos
todos, los campos de fuerza que siempre fallaban, probablemente
adquiridos en tiendas de todo a un euro, los espantosos trajes
de astronauta plateados, los peinados de mamá Robinson que
jamás se I e movían pese a las circunstancias, o los impagables
monstruos, semana sí y semana también (el más escandaloso
de todos fue el hombre zanahoria), configuraban una serie
singular que con diez afias podia verse asombrado, pero que
con e! paso dei tiempo se ha convertido en la más carcajeante
de la historia de la ciencia ficción televisiva.
Como puede observarse en esta breve sinopsis, los guionistas
no tenían e! más mínimo pudor en usar cualquier tema para los
capítulos, sin importarles si eran desquiciados o no. El trabajo
febril de la factoría Allen contribuía a que argumentos de otras
series pasaran a esta y viceversa. Además, e! hecho de tener

104 1 GE IDIOMAS 201S


DICAS ORGAN IZAÇÃO TEXTUAL » MASC U L I N O OU FEM I N I N O?

Saiba como descobrir o gênero dos


substantivos sem fazer confusão

Em espan hol, ass i m como em português, todos os su bstan­ ART ICULOS


1
I
tivos possuem um gênero gramatical: fem i n i n o ou masc u l i no.
Para d escobrir o gênero d e um s u bstantivo d evemos estar Determinados Indeterminados
atentos à q u eles q u e pode m ser fe m i n i nos o u m ascu l i nos
d e acordo com sua termi nação (de s i n ê n cia) e àq ueles nos Masculino e/ los un unos
quais o gên e ro se manifesta m e d i an te a conco rdância com
Feminino la las una unas
o utras palavras, como artigos, possessivos, d e m o n strativos
o u adjetivos.
C o m o em português, a maioria dos s u b stantivos termi­ Quanto à formação do plural: em espanhol, as palavras termi­
nados e m -a é fem i n i na e a maioria dos term i nados e m -o é nadas em vogais formam o plural com o acrésci m o d e -s:
masc u l i na. Porém, há tam b é m : persona � personas
c liente � clien tes
Substantivos q u e mudam d e terminação d e acordo com o gênero:
e/ cliente/ la clienta. E as termi nadas e m consoantes formam p l u ral com o acrés­
Substantivos em -ista e -nte, cujo gênero depende da concordancia c i m o de -es:
com outras palavras, por exemplo, o artigo: facultad � facultades
e/ periodista I la periodista, e/ estudiante 1 la estudiante. capital � cap i ta les
Substantivos que designam alguns animais devem vir acompanhados do
Atenção: Palavras temi nadas em -ción e zón perdem o acento
adjetivo macho ou hembra: e/ ratón macho /e/ ratón hembra.
---
gráfico no plural:
Alguns substantivos terminados em -a que designam pessoas dependem da
nación � naciones
concordAncia com outras palavras: e/ demócrata Ila demócrata.
A maioria dos substantivos terminados em -cia, ·ción, ·dad, ·ez, -eza,
As palavras terminadas em -ez trocam o z por c:
-ie, ·ncia, ·nza, ·sión, ·tad, ·tud, ·umbre é feminina:
vez � veces
la democracia/ la nación/ la edad! la vejeZ/ la certeza/ la carie/
la ganancia/ la esperanza/ la expresión/ la facultad.
A maioria dos substantivos terminados em -aje, -ambre, ·an, -or é masculina:
O uso do /o
e/ viaje/ e/ enjambre/ e/ dolor

O artigo neutro lo é específico do espanhol. Ele é invariável


A l g u n s s u b stantivos m ud a m d e sign i ficado conforme o e o usamos para substantivar adjetivos com valor abstrato
gênero: (no singular ou plural), advérbios e orações introduzidas pelo
pronome relativo que.
e/ capital/ o capital (bens) la capital/ a capital (cidade)

Lo mejor de la vida, todavía no lo has vivido.


e/ cura I o pároco, padre la cura I a cura (tratamento)
No sabes lo rápido que corre este coche.
Nunca escucho lo que me dicen.
e/ corte 1 o corte (ação de cortar) la corte 1 a corte (tribunal)

Atenção!
e/ frente I a frente de batalha la frente I a testa
É incorreto usar /o no lugar do masculino singular e/.

e/ ordem 1 a organização la ordem 1 a ordem (comando)


Lo tráfico estaba mala. (incorreto)
E/ tráfico estaba mala. (correto)
e/ policia I o policial la policia 1 a policia (corporaçao)

GE IDIOMAS 2015 1 105


I NTERP RETAÇÃO D E T EXTO

El cerebro
es lo más
importante
Siempre más com plejo de lo
que se espera, él es objeto de
esperanza de beneficios para la
salud de I ser humano

..111111 LA DtCADA DEL CEREBRO alcanzó pronto


la primera plana en el media académico
internacional. El tema central era una toma
de conciencia general sobre los beneficios de I
estudio científico de I cerebro

A
continuación se presenta parte de una reflexión personal sobre El proyecto "Década del cerebro" fue una iniciativa patrocinada por
una posible evaluación del logro de los objetivos propuestos por la Biblioteca del Congreso (LC) y el Instituto Nacional de Salud Mental
el gobierno de los Estados Unidos de América, algunas agencias (NIMH) de los Estados Unidos. Su objetivo primordial era dar cuerpo a
internacionales de investigación y otras entidades internacionales, los objetivos propuestos en la resolución 174 de! Congreso. Públicamente
para la denominada "década de! cerebro". presentada el 17 de julio de 1990 por e! entonces presidente de los Estados
A solo un afio de! cierre oficial de la década de! cerebro, el balance Unidos de Norteamérica George Bush, la década del cerebro alcanzó
general no parece ser tan bueno como se desearía. Indudablemente pronto la primera plana en el medio académico internacional. El tema
e! avance alcanzado en estas últimos nueve anos en las discipli­ central de la resolución era el fortalecimiento de una toma de conciencia
nas, campos y áreas de trabajo cobijadas bajo el término genérico general sobre los beneficios dei estudio científico del cerebro.
neurociencia, fue inmenso. La proclamación presidencial 6158 en la que se declaraba la década
No obstante, la relación expectativas-logros no parece satisfacer iniciada el 1 de enero de 1990 como década del cerebro, se centró en
los esperados, ni dentro de la comunidad académica particular ni cuatro puntos principales: (l) el incremento en el aparecimiento de
dentro del marco social más general. Si realizáramos una comparación enfermedades cerebrales y mentales de tipo degenerativo, traumático
de las relaciones expectativas-logros entre las décadas de! cerebro y congénitas; (2) los avances tecnológicos en microscópica y
y aquella del espacio, encontraremos razones para no calificar de neuroimagenologia; (3) los avances conceptuales en la comprensión
pesimistas las aseveraciones anteriores. de algunos procesos patológicos así como en el desarrollo de algunas
Uno de los logros tangibles de la década del espacio fue la llegada ciencias básicas, por ejemplo, la genética y la bioquímica, y (4) avances
del hombre al sue lo lunar. El Homo sapiens del siglo XX, ai igual que en disciplinas intermedias como la biologia molecular o la genética
sus coespecíficos de siglos pasados, conquistaba nuevos y vastísimos molecular. Se hacía referenda también, en este discurso, al ahora
territorios allende sus fronteras; "un pequeno paso para un hombre, famoso capítulo de la drogadicción y de la fármaco dependencia. (.. .)
pero un gigantesco salto para la humanidad", diría Neil Amstrong
aquel día. Por el contrario, hoy, a las puertas dei siglo XXI, finalizando (Fernando Cárdenas M. S. Departamento de Psicobiología, Universidad de São Paulo, São
el plazo propuesto para e! conocimiento del cerebro, no tenemos Paulo, Brasil. Marisol Lamprea. Universidad Nacional de Colombia. Artículo publicado el
mucho que decir, salvo que e! cerebro resultó más complejo de lo 10 de abril de 2001. www.psicologiacientifica. com/bv/psicologia) (Vestibular UFCSPA 2007)
que esperábamos. (...)

106 1 GE IDIOMAS 2015


EXERCÍCIOS - UFCSPA 2007 (adaptado)

1.Atendiendo a aspectos semánticos de la lengua es p an o la, s. Según el texto, es correcto afirmar q ue:
senale la alternativa correcta. a) el mayor logro d e la década fue la ! l egada dei hom bre ai
a) La palabra "cob ijada" (párrafo 2), es antó n i m o d e "refu­ suelo l u nar.
giada". b) el proyecto "década dei cerebro" fue una i n iciativa dei medi o
b) La locución "no o bstante" (párrafo 3) es s i n ó n i m o d e "to­ académ ico i n ternacional apoyada por el presidente de los
davia". Estados U n idos, George Bush.
c) La palabra " p ro nto" (párrafo s), es s i n ó n i m o d e " l e n ta­ c) los avances e n la década dei cérebro y e n la dei espacio no
m e nte". han sido idénticos.
d) La palabra "toma" en la expresión "toma de conciencia" d) e l estu d i o d e la d rogad i c c i ó n y de la fármaco d e p e n d e n­
(párrafo s) es u n verbo conj u gado en p resente del modo cia no formaban parte de los objetivos d e la "década d e i
indicativo. cere b ro".
e) E l adjetivo "com plejo" (párrafo 4) es antó n i m o de "si m ple". e) el autor no expresa su o p i n i ó n acerca d e los logros de la
"década dei cerebro".
2. Uno de los logros tangible� de la década de/ espacio fue la
/legada de/ hombre a/ sue/o lunar. E/ Ho mo sapiens de/ siglo
XX, ai igual que sus coespecificos de siglos pasados, conquis­ RESPOSTAS
taba nu e vos y vastisimos territorios allende sus fronteras; 1. E/ adjetivo "complejo" (párrafo 4) es antónimo de "simple".
"un pequeno paso para un hombre, pero un gigantesco salto Tra ta-se de u m caso de a n t on ímia, ou uso de co n trários. Esse tipo
para la humanidad'� diria Neil Amstrong aquel dia. POR EL de q uestão é bastante explorado em pro vas, e você pode estudar
CONTRARIO, hoy, a las puertas de/ sig/o XXI, finalizando o tema com a ajuda de listas de a n tônimos com base nos adjetivos
e/ plazo propuesto para e/ conocimiento de/ cerebro, no mais empregados no espanhol. Resposta: e
tenemos mucho que decir, salvo que e/ cerebro resultá más
complejo de lo que esperábamos. z. Contrajunción (Co n t rajun ção), isto é, a apresentação de dois
argumentos opostos ligados, neste caso, pela expressão "por e/
En el párrafo, la expresión destacada establece entre las con trario". Resposta: c
partes del texto que conecta una relación de:
a) condicionalidad. 3.EI área/ e/ logro/ e/ avance. Nessa q uestão, o substan tivo "á rea"
b) generalización. pode levar à formação equivocada do plural por estar acompanha­
c) contraj u nción. do do artigo masculino "e/". Apesar de estar precedido do a rtigo
d) com paración. masculino, esse substan tivo é feminino. Isso se deve ao fato de q ue,
e) conj u nción. em espanhol, substa n tivos femininos q u e começam c o m a vogal
tônica a ou com o encon tro h+a {tônico); no singular, não podem

3.EI singu lar de las pai abras /as áreas/ los logros/ los avances, vir precedidos do artigo feminino "ia" para evitar cacofonia. No
sacadas del texto, es: e n t a n to, n o p lu ra l, esses substa n tivos devem vir precedidos do
a) el área/ e I logro/ el avance. artigo feminino "ias" porque o encontro do artigo com o substan tivo
b) la área/ e I logro/ el avance. não provoca problemas fonéticos. Resposta: a
c) el área/ lo logro/ lo avance.
d) la área/ lo logro/ lo avance. 4. Quisieran, hubieras, fuera. Todos os verbos estão no pretérito

e) lo área/ el logro; el avance. impertecto de/ s u bj u n tivo. Pa ra inform a r-se s o b re esse t e m p o


verbal, l e r a s in formações d a página 132. Resposta: b
4. Si realizáramos una comparación de las relaciones ex­
pectativas- logros entre las décadas de/ cerebro y aquella s. Los avances en la década de! cérebro y en la dei espacio no han
de/ espacio, ... sido idénticos. (Os avanços na "década do cérebro" e as conquistas
o b tidas n a era espacial não têm sido idên ticos.) A resposta a essa
La alternativa en la que todos los verbos están conjugados ques ttio está n o 2° parágrafo: "Si realizáramos una comparación de
en el mismo tiempo del verbo destacado es: las relaciones expectativas-logros entre las décadas de/ cerebro
a) q u isiéramos, habría, haga. y aq uella dei espacio, encon traremos razones para no calificar de
b) q u isieran, h u bieras, fuera. pesimistas las aseveraciones a nteriores". E também n o 3°parágrafo:
c) q u i s ieron, h u b iera, haga. "Por e/ c o n tra rio, h oy, a las p uertas dei sigla XXI, finaliza n d o e!
d) q u isieron, h u bo, fuera. plazo propuesto p ar a e/ conocimiento d ei cerebro, n o tenemos
e) q u isierais, h u bieron, haga. mucho q u e decir, salvo q u e e/ cérebro resultá más co m piejo de lo
q u e esperábamos". Resposta: c

GE IDIOMAS 201S 1 107


I NT E R P R ETAÇÃO D E TEXTO

Todo depende del punto de vista


Y O N O AOUANTABA MÁS VIVIR HASTA QUE SUPE EL "FENO SHUI" ES UN MILENARIO
EN ESTE TUOURIO MISERABLE . DEL "FENO SHUI" . CHINO DE �RREOLAR EL HOOAR PARA
EN ARMONIA Y EQUILIBRIO CON LA CASA Y
UNO MISMO.

(Vestibu lar U FRGS 2010)

EXERC[CIOS - U FRGS 2010 (adaptado)

1. Considere o texto a seg u i r: La joven d ijo: "La tierra es e/ c) s u b ú r b i o, p i n tado, e n canamento e piso.
sue/o de mi casa. ;Qué ignorante h e sido tantos anos!}amás d) barraco, fu rado, esgoto e p i so.
me detuve a pensar sobre eso." e) cabana, fu rado, p rivada e piso.
Assinale a alternativa que apresenta as formas corretas para
completar a reescrita desse texto em discurso i n d i reto.
Lajoven d ijo q u e la tierra el sue lo de____ RESPOSTAS
casa. Reconoció q u e i gnorante tantos anos. z. Era - su - había sido - se había detenído. Quando passamos
jamás a pensar sobre eso. o e n u n c i ado para o discurso indireto, introduzimos a pa r tícula
a) e ra - su - había s i d o - se había dete n ido. "que" (pa ra unir a frase introdutória à s u bordinada), trocamos o
b) era - m i - ha s i d o - se h a dete n ido. possessivo "mi" por "su" e passamos os verbos a o utros pretéritos do
c) es - m i - h a s i d o - se había dete n ido. indicativo. Mudamos também a pessoa na q ua l os verbos estavam
d) fue - su - había s i d o - se detuvi e ra. conjugados. Resposta: a
e) f u e - su - f u e ra - se detuviera.
z.Enumeração, exemplificação e conclusão de ideias. Até o quadrinho
1. Nos n ove p r i m e i ros q uad r i n h o s da h i stória, a m u l he r 8, a mulher enumera os princípios do Feng Shui e fa la sobre cada
estabelece n a s suas falas uma relação d e : um deles com um exemplo de sua casa . A partir do quadrinho 9,
a ) e n u meração, exe m p l if i cação e c o n c l u são d e i d e ias. estabelece-se uma conclusão para a exposição das ideias. Resposta: a
b) e n u m e ração, exe m p l i f i cação e contraj u n ção d e ide ias.
c) con cessão e c o n c l u são d e ide ias. 3. Ba rra co furado, esgoto e piso. Podemos chega r ao significado
,

d) cau sa, exe m p l i f i cação e con seq u ê ncia d e ide ias. dessas palavras em po rtuguês por meio das ilustrações, q u e mos·
e) exe m p l i f icação e contraj u n ção d e id eias. tram todos os elemen tos citados. "Tugurio" está associado ao lugar
onde vive a mulher e, pelas i magen s, podemos ver que se trata de
3. N o texto l i d o, as palavras tugurio, pinchado, cloaca e um ba rra co. "Pinchado" pode ser entendido pela relação com a
mosaico signifi cam: palavra "ca fi o ". "Cloaca" p ode ria ser associada também ao sentido
a) s u b ú rb i o, p i ntado, p rivada e m osaico. que essa pala vra tem em outro campo semân tico (nas aves, significa
b) s u b ú rbio, fu rado, privada e m osaico. "ânus"), e "mosa ic o " fica claro pela relação com "chão ". Resposta: d

108 1 GE IDIOMAS 2015


DICAS VOCABU LÁRIO - I M P E RATIVO » I STO É U M A O R D E M !

O emprego do modo imperativo


na língua espanhola

Em pregamos o i m perativo para dar ordens, fazer ped i dos, No caso das orações potenciais, o p resente do s u bj u nti­
expressar u m d esejo ou u m a proibição, dar i nstruções etc. vo é p reced ido pelas segu i n tes expressões modalizad oras
Em função de seu papel apelativo (dirigido a um i nterlocutor (mostram a forma como o falante vê o q u e está d izendo): "no
presente n o m o m ento da produ ção do e n u nciadoL só existe creo I no me parece que", "es posible I imposible que", "es
para a 2• pessoa do singu lar e do p l u ral (túlusted - vosotrosl probable que", "qué raro que", "que extrafío que", "es necesa­
aslustedes) e para a 1• do p l u ra l (nosotroslas). rio que", "es innecesario que", etc. Nas o rações desiderativas
(expressam d esejosL esse tem po verbal vem preced i d o d e
I M P E RATIVO NA FORMA AFI RMATIVA expressões como "oja/á", "espero que", "deseo que".
Verbos regu lares: conjugamos os verbos e l i m i na n d o as Confi ra a conj u gação do presente do s u bj u n tivo:
termi nações -ar/ -er/ - i r do i n f i n itivo e acrescentando as
seg u i ntes desinên ci as:

As desinências são as mesmas dos verbos


regulares, mas os verbos sofrem alte·
rações na raiz. Apresentam as mesmas
Desinências
irregularidades da 1 ' pessoa singular (yo)
•1• conjugação: amar = ame,
do presente do indicativo:
usted hab/ - e beb - a/escrib - a ames, ame, amemos, ameis,
Alteração vocálica e > ie I o >eu I e > i:

amen.
beb - amos I escrib - pensar -+ piense 1 dormir -+ duerma 1
nosotros/as habl - emos •2, e 3, conjugação: beber 1
amos pedir -+ pida.
escribir = beba/escriba, bebas/
I nterposição de consoante em todas
vosotrosjas hab/ - ad beb - ed 1 escrib - id escribas, beba /escriba, bebamos/ •

as pessoas do verbo: decir -+ diga, digas,


escribamos, bebáistescribáis,
diga, digamos, digais, digan 1 conocer -+
ustedes habl - en beb - an I escrib - an beban/escriban.
conozca, conozcas, conozca, conozcamos,
conozcáis, conozcan 1 huir -+ huya,
huyas, huya, huyamos, huyáis, huyan.
Não faça isso!
O impe rativo na forma negativa

O imperativo na forma negativa forma-se a parti r do presente


do subjuntivo. Os verbos que apresentam irregularidades Observe
no presente do indicativo mantêm essa irregularidade no Pretérito perfecto del subju ntivo
presente do subjuntivo e, portanto, também no imperativo na
forma negativa. Este tempo verbal se usa no mesmo tipo de oração em
Exemplo: No hagas ruido, e/ nino está durmiendo. que vimos que se pode usar o presente del subju ntivo,
hacer __, hago (presente do indicativo) __, que yo haga potenciais e desiderativas, s u bordi nadas ou não (Deseo
(presente do subjuntivo) que hayas aprobado e/ examen). Confere às orações u m a
perspectiva temporal de passado e se conjuga d a
segui nte forma:
PRESENTE DO SUBJ U NTIVO
Entre suas utilidades, tam bém está a de estruturar orações Verbo auxil iar + particípio do verbo principal
potenciais e desiderativas haber decir

E m espan hol, o presente do s u bj u ntivo confere aos e n u n­ (yo) haya dicho


ciados a noção tem poral de presente ou de futu ro, de acordo (tú) hayas
com o contexto em q u e aparece. Exceto no caso das o rações (usted!él!ella) haya
pote nciais (Quizá esté enfermo) e desiderativas (Ojalá venga (nosotros/as) hayamos
a mi casa), em geral, esse tempo verbal é u sado em o rações (vosotros/as) hayáis
s u b o rd i nad as m ostra n d o a n te r i o r i d ad e, s i m u l ta n e i d ad e (ustedes!ellos!ellas) hayan
o u posteridade e m relação à ação/estad o expressa p e l o
verbo d a o ração pri n c i pal (Aunque esté enfermo, no faltará
a la elas e).

GE IDIOMAS 2015 1 109


I NT E R P R ETAÇÃO D E TEXTO

Dilema: (cooperar o aprovecharse?


Lo q ue motiva a una persona a colaborar con otras
por Débora Freidmann

Allll COOPERAR PARA SER FELIZ Las personas ayudan unas a las otras no por interés personal sino para sentirse bien consigo mismas

A
diario las personas deciden si cooperan con quienes tienen existencia de una red de contactos no influye en la cooperación. "Veinte
una relación afectiva, como vecinos o compaiíeros de trabajo. aiíos de trabajos teóricos habian dado lugar a la creencia de que el
Eligen, por ejemplo, entre tirar la basura en ei lugar indicado en interaccionar con otros a través de una red podría fomentar el que la
el edifício o dejarla en un pasillo o entre dejar la mesa limpia o sucia gente cooperara más, ya que entonces los cooperadores podrían estar
después de almorzar en el comedor común de la empresa en la que juntos y beneficiarse mutuamente. Ahora sabemos que no es así", agrega
trabajan. Incluso, resuelven si en determinadas situaciones vale la el experto Domingo Anxo Sánchez. La explicación que encontraron es
pena aprovecharse dei resto. áQué es lo que pesa para tomar uno u otro que a la gente !e importan las acciones que tomen los demás - cuántos
camino? Esta fue la inquietud central de un grupo de investigadores de han cooperado con ellos - y su estado de ánimo, más aliá de los beneficios
la Universidad Carlos 111 de Madrid y de la Universidad de Zaragoza, que obtengan. Es decir, se coopera cuando buena parte de los vecinos
que decidieron investigar estos mecanismos y desarrollaron e! mayor coopera, y en cuanto no es así, la gente deja de hacerlo. Aunque pueda
estudio efectuado hasta e! momento sobre el tema. Los resultados, li amar la atención que las personas no cooperen según e! beneficio que
afirman, fueron sorprendentes: las personas cooperan en función de crean que vayan a lograr, para Sánchez esa reacción se origina en la
su estado emocional y no actúan pensando en el beneficio propio. (...) Prehistoria con los cazadores recolectores.(...)
En el estudio participaron más de 1200 alumnos de bachillerato Otra de las hipótesis es que en muchos contextos lo único que las
en Aragón, que tenian compaiíeros de juegos asignados por una personas saben de sus compaiíeros es si e !los han ayudado o no, y no
computadora que eran sus vecinos fisicamente. En cada ronda, cada cuánto se beneficiaron por eso. "Estamos acostumbrados a fijarnos
·
uno decidia si queria cooperar o no y en función de eso obtenia un en la información disponible e ignoramos el beneficio", resume el
pago. "Si la decisión era cooperar, obtenia 1 (una) unidad de pago por experto. Los autores de la investigación prevén que los resultados
cada vecino que hubiera cooperado y nada por cada vecino que no ayuden a comprender cómo toman decisiones las personas, en
hubiera cooperado. Si la decisión era no cooperar, obtenia 1,4 unidad especial cuando hay que decidir entre colaborar o aprovecharse de
de pago por cada vecino que hubiera cooperado y nada por cada vecino los otros, y a partir de allí puedan elaborarse estrategias que induzcan
que no hubiera cooperado. De esta manera, claramente es tentador no a la gente a cooperar.
cooperar, porque entonces uno se aprovecha de todos los que cooperan
a su alrededor, pero si todo el mundo piensa lo mismo y nadie coopera, Texto extraldo y adaptado de: E/ País, Uruguay; junio de 2012
nadie gana nada. (...) El hallazgo más importante dei estudio es que la (Vestibular Universidade Mackenzie 2014)

110 I GE IDIOMAS 2015


EXERCÍCIOS - U N IVERSI DADE MACKENZIE {adaptado)

1.EI verbo subrayado "hubiera cooperado" también se puede RESPOSTAS


sustitui r por la forma: L A forma verbal "hubiera cooperado" está no pretérito p/uscuam­
a) habrfa cooperad o perfecto de/ subjuntivo. Nesse tempo verbal, a expressão é formada
b) h u biese cooperado pelo verbo auxiliar "haber" e o particípio do verbo principal. Há
c) tend rfa cooperado dois tipos de desinências corretas para o verbo auxilar "haber",
d) haya cooperado em "-ra" e "-se". Assim, a forma equivalente a "hubiera cooperado"
e) h a cooperado é "hubiese cooperado" (veja mais sobre s u bj u n t ivo n a pág.132)
Resposta: b
z. El sinónimo del sustantivo "hallazgo", destacado en ne­
grita e n el texto, es: L Mesmo que você não conheça a palavra "hallazgo", é possível des­
a) consabido cobrir seu significado por o u tros elementos do texto. Mas podemos,
b) e n cantam iento a n tes, eliminar as a lternativas incorretas analisando a q ue classe
c) descubri mie nto gramatical pertence "haflazgo". Pela frase em que a palavra aparece,
d) desmed ido "e/ hallazgo más importante ... '; percebe-se que se trata de um subs·
e) vol u m i n oso tantivo (an tecedido por um artigo e seguido de adjetivo). En tão, o
sinônimo também deve ser um substantivo. Com isso, eliminamos as
3. El sinónimo de la palabra "lograr", destacada e n negrita alternativas a, d e e. Nelas são apresentados adjetivos: "consabido"
e n e l texto, es: (compartilhado), "desmedido" (desmedido) e "voluminoso" (volumoso)
a) alcanzar Ficamos com as alternativas b e c. Recorrendo ao texto, repare que a
b) arren dar palavra se refere a a lgo que foi encontrado pelo estudo, ou seja, uma
c) c u brir descoberta (descubrimiento).
d) apelmazar Resposta: c
e) e n fatizar
3o "Lograr" é sinônimo de "alcanzar". O verbo tam b é m existe em
4. Seg ú n el texto: português, com diversos significados.· alcan çar; conseguir, enganar.
a) Las personas están d i spuestas a cooperar d esde que les Os demais verbos que aparecem nas demais alternativas significam,
pague n u n a cantidad mayor. respecttvamen te, alugar, co b rir, amassar ou amaciar, e e n fa tizar.
b) Las personas solamente cooperan en situaciones i m­ Alternativa correta: a
puestas.
c) Las perso nas cooperan de acuerdo con su estado e mo­ 4- Analisando cada uma das alternativas:
cio nal y experi e n cias previas. a) O texto não fala so b re remuneração o u pagamento. Incorreta.
d) Las personas solo coo peran cuando hay un benefi cio b) O ato de coopera r é apresentado como uma escolha. Incorreta.
expl ícito. c) Segundo a pesquisa, as pessoas cooperam de acordo com seu
e) La decisión de cooperar está d i rectam ente relacionada estado emocional. Veja o trecho.· "... las personas cooperan en
con una red de contactos entre otras personas. fu nción de su estado emocional y no actúan pensando e n e/
beneficio pro pio". Correta.
s. Según e l texto, los resu l tados de l a i nvestigaci ó n nos d) A pesquisa indica exatamente o oposto: as pessoas não cooperam
l levan a: pensando em levar alguma vantagem. Incorreta.
a) Com prender como las personas deciden entre colaborar e) O texto n ega q u e a rede de contatos seja determinante na de­
y aprovecharse de los de más y a partir de eso, los cientf­ cisão de cooperar ou não. Isso fica claro no trecho "E/ ha llazgo
ficos seguirán elaborando estrategias para q u e la gente más importante de! estudio es que la existencia de una red de
e m p i ece a cooperar. c o n tactos n o in fluye en la cooperación". Incorreta.
b) Anal izar q u e las decisiones no están i nvolucradas con Resposta: c
otros beneficios obte n idos sino con las u n idades de pago
rec i bi das por cada vecino. s. Preste a tenção: a questão pede a ide ia geral dos resultados do
c) O bservar q u e a través d e una red, la gente i n teractú a estudo. A ú l tima frase do texto in forma q u e os pesquisadores
m u c h o más ráp i d o p ue s h ay u n f o m e n to para q u e l a compreenderam como as pessoas decidem cooperar com o utras.
gente coopere más. E, a partir daí, eles esperam pensar e m estratégias que levem as
d) N otar q u e a l a gente no l e i m portan las acciones q u e pessoas a coooperarem mais. A o contrário do que se a firma nas
t o m e n los de más, p o r eso, hay u n a mayor cooperación. demais alterna tivas, a decisão en tre cooperar o u não cooperar não
e) Observar que l as perso nas cada d ía deciden s i van a tem relação com o pagamento recebido durante a pesquisa, com a
cooperar o no, de acuerdo con el tipo de relac i ó n q u e rede de contatos do indivíduo, nem com a rede de relacionamentos
tienen con la otra persona. ou da relação entre m e m b ros do grupo.
Resposta: c

GE IDIOMAS 2015 1 111


DICAS CON DICIONAL » REFO RÇO NO ARG U M E NTO

Verbos que reforçam uma ideia

Vamos agora detal har verbos m uito úteis nos textos argu­ O cond icional é m u ito usado para:
mentativos porq u e reforçam aspectos i m portantes já apre­ » Dar conselhos: Cada persona debería saber dónde se
sentados pelas ide ias. halfa la ansiada 1/avecita de la felicidad, pero a menudo
O cond icional s i m p l e expressa as ações como prováveis ou no está a nuestro alcance.
h i potéticas. Veja como funciona sua conjugação. » Pedir um favor d e manei ra gentil. Le pediría e/ favor de
apagare/ cigarrillo, si no /e molesta. (Se não for i ncômodo,
Verbos regulares: acrescentamos ao i nfinitivo as desinências poderia apagar o cigarro?)
próprias do condicional, que são as mesmas para os verbos » Expressar d esejos: Me gustaría comprarme una casa
das três conjugações. nueva. (Gostaria de comprar u m a casa nova.)
» hablar 4 hab/arfa, hab/arfas, hablaría, hablarfamos, ha­ » Expressar condição para a realização de algo: Si tuviera
b/arfais, hablarían. dinero, iría ai cine esta tarde. (Se eu tivesse d i n he i ro, i ria
» correr -+ correría, correrías, correría, correríamos, ao cinema esta tarde.)
correríais, correrfan. » Fazer convite s: ,! Vendrías conmigo a/ cine esta noche?
» vivir 4 vivirfa, vivirfas, viviría, viviríamos, viviríais, (Gostaria de ir ao cinema com igo hoje à noite?)
vivirfan. » Falar do futuro com respeito a um momento do passado:
Los jóvenes /ectores de fina/es de/ sig/o XIX recibieron
Verbos i rregul ares: as raízes dos verbos i rregu lares apre­ una inspiración tan fuerte de julio Verne y H. G. Wel/s
s entam as mesmas alterações sofridas n o futuro s i m p l e, que muchos de el/os se especializaron en cohetes para
e a e l as são acres c e n tadas as d e s i n ê nc i a s d o s v e r b o s asf poder inventar la tecnología que les permitiria viajar
regu lares. a otros mundos.
Exemplo: poner 4 pondría, pondrfas, pondría, pondríamos,
pondríais, pondrían.
» caber 4 cabr- Apócope
» haber 4 habr- Algumas palavras perdem uma parte quando
» saber 4 sabr- ficam diante de certas palavras
» poder 4 podr-
» poner � pondr- Há palavras que sofrem apócope - perda de uma letra ou de uma
» salir 4 saldr- sílaba - quando as colocamos imediatamente antes de certas palavras.
» venir -+ vendr­ Veja alguns casos:
» tener --+ ten d r­ n perdem o "o" antes de substantivos masculinos no singular: primero

» querer 4 q uerr­ e tercero (primer día e tercer colocado), alguno e ninguno (algún
» hacer 4 h ar- dinero e ningún alumno), bueno e maio (buen día e mal tiempo).
» decir 4 di r­ n perde a sílaba final antes de substantivo no masculino ou feminino

» valer -+ vald r- no singular: grande (gran mujerjhombre).


n perde a sílaba final antes de adjetivos ou advérbios: tanto (tan

Já o cond icional composto expressa a anterioridade de uma inteligente/rápido).


h i pótese e m rel ação a o utra, situada no passado: Si hubieras )) perde a letra "a" antes de palavras no masculino ou feminino no
cenado, no te habrías desmayado. (Se você tivesse jantado, singular: cualquiera (cualquier cosa/dia).
não teria desmaiado.)
Esse tem p o é c o m posto e formado p e l o verbo haber no
condicional s i m ple, mais o particípio d o verbo conj ugado.
Exe m plo: comer 4 habría comido, habrfas comido, habría
comido, habríamos comido, habríais comido, habrían comido.

1121 GE IDIOMAS 2015


DICAS FUTURO S I M P L E » PARA FALAR DO Q U E HÁ D E V I R

Saiba como conjugar os verbos para


expressar condição e concessão

E m espanhol, o pretérito i m perfecto e o pl uscuam perfecto Verbos i rregu l ar e s : são formados a parti r d a 3• pessoa
d e i subju n tivo conferem aos e n u n c iados a n oção te m p o ral do p l u ral d o pe rfecto s i m p l e d e i n d i cativo. Apresentam as
d e passado. São e m p regados em orações s u bordi nadas, con­ mesmas d e s i n ê n cias dos verbos regulares. Por exemplo:
d i cionais e concessivas, e também em orações desiderativas
(q u e exp ressam um d esejo):

» Si dijera la verdad, se libraría de la culpa. (Se d issesse a


fueselfuera tuvieseltuviera
verdade, se livraria da cul pa.) (con d i c i o nal)
» Si hubiera venido a la fiesta, no lo dejarían entrar. (Se tivesse fueseslfueras tuviesesltuvieras
vindo à festa, não o deixariam entrar.) (condicional)
» Aunque dijera la verdad, no se libraría de la culpa. (Mes­ fueselfuera tuvieseltuviera
mo q u e d i ssesse a verdad e, não se l iv raria da c u l pa.)
(concessiva) fuésemoslfuéramos tuviésemosltuviéramos

» Aunque hubiera dicho la verdad, no se habría librado de


fueseislfuerais tuvieseisltuvierais
la culpa. (Mesmo q u e tivesse d ito a verdade, não teria
se l i vrado d a c u l pa.) (co ncessiva) fuesenlfueran tuviesenltuvieran
» Ojalá hubiera venido a mi casa ayer. (Que bom seria se
tivesse vindo a m i n h a casa ontem.) (desiderativa)
Já o p retérito pl uscuamperfecto é formado pelo verbo ha­
Vam o s c o n h ec e r ago ra a c o n j u gação do pretérito i m ­ ber no pretérito i m perfecto dei s u bj u ntivo, mais o particípio
perfecto. d o verbo p r i n cipal. Por exem p lo:
Verbos reg u l are s: ad m i te m d uas desi nências d i ferentes,
q u e servem para conjugar verbos das três ter m i nações.

. . ' 'P".� hubiera/ hubiese visto


Estudiar comertvivir : . . :L :$..-"'

estudiaselestudiara comieselcomiera vivieselviviera


·
hubieras 1 hubieses visto

estudiaseslestudiaras comieseslcomieras vivieseslvivieras


·
hubiera 1 hubiese visto

estudiaselestudiara comieselcomiera vivieselviviera


·
hubiéramosl hubiésemos visto

estudiásemosl estudiáramos comiésemoS/comiéramos viviésemoS/viviéramos


·
hubierais 1 hubieseis visto

estudiaseislestudiarais comieseislcomierais vivieseislvivierais


·
hubieran I hubiesen visto

estudiasenlestudiaran comiesenlcomieran viviesen/Vivieran


·

GE IDIOMAS 2015 1 113


I NT E R P R ETAÇÃO D E TEXTO

El tráfico nos enseiia


Los accidentes son más que la buena educación en el tráfico
VERANO 2 00 7 ; N U E VO R É CO R D D E AC C I D E NT E S E N C A R R ET E R A
El JuevC!Is� stn que sirva de precedente... rettextona: i-Para qué sirven de verdad los radares?
L.Ya le hemos perdido e l rniedo a la nueva Ley
de Tráfico?
SI;NO� A(;EMTE
'AS Í - - -�QJG"vA
LoS

EL JUEVES. Barcelona, set. 2007 , La til a de oroz

EXERCICIOS - U FG 2008 (adaptado)

1. En el tono de la exposición de problemas hecha en cada c) m iedo.


una de las viíietas, se nota un ánimo de s ubrayar: d) todo e l e n u nciado.
a) los valores e nfrentados entre las partes i m p l icadas. e) n i n g u n a d e las palabras del e n u nciado
b) la mala d i s posición de mostrada por la ind ustria de coches.
c) las actitudes d ialogantes fomentadas desde la autoridad.
d) el desaliento por los choques registrados en las carreteras. RESPOSTAS
e) lo perj u d i cial en la colocación de avisos para los motoristas. z. Los valores enfrentados en tre las partes implicadas. Nos três
quadrinhos que compõem a tira, temos o enfrentamento entre as
z.EI conductor de I utilitario, ai dirigirse ai guard ia civil - e l a u toridades de trânsito, ou a n o va Lei de Trânsito, e os motoristas
agente -, demuestra u n a (unas): cujas a titudes transgridem as normas impostas por essas autorida­
a) falta de respeto hacia la madre de I policía. des. Há um claro confronto de valores, direto e até mesmo agressivo,
b) ganas por no q uedarse s i n la sobremesa. principalmente no primeiro quadrinho. Resposta: a
c) i n d iferencia ante la pena por sus chulerías.
d) recepción cordial ante la m u lta i m p u esta. z. lndiferencia ante la pena por sus chulerías. A forma sarcástica
e) desídia debida ai cansancio de manejar. como o m otorista trata o guarda, dizendo-lhe provocações, mostra
que ele não teme a pena que pode sofrer diante de suas grosserias.
3. En relación con el titular general, la vi neta de I radar pone Resposta: c
de relieve que e l uso de este encierra:
a) efecto contraproducente. ).lnterés espurio. O enunciado "y nuestro beneficio" faz referência
b) propaganda h i l ariante. ao fato de beneficiar-se ilicitamente de algo; nesse caso, do benefício
c) jocosidad dafíina. ilícito que as autoridades obtêm das multas de trânsito em razão
d) corru pción autonómica. do excesso de velocidade. Resposta: e
e) i nterés espurio.
4- Ley de Tráfico. O pronome átono "/e" faz referência ao objeto in­
4- En el e n u nciad o c:Va )_g_ hemos perdido e l m i ed o a la n u e­ direto "Leyde tráfico". Temos na oração em questão uma repetição
va Ley de Tráfico?, el pronombre áton o s u b rayado h ace do complemen to, na forma de pronome (/e) e como um sintagma
referenda a: nominal (Ley de tráfico). Essa repetição do complemento indireto
a) Ley d e Tráfico. é muito usual no espanhol, mas só é permitida quando se trata da
b) n osotros. 3' pessoa do singular o u do p lural. Resposta: a

114 1 GE IDIOMAS 1015


DICAS VOCA B U LÁRIO - VE RBOS I RREG U LARES » F O RA DO PAD RÃO

Conheça os verbos irregulares no


imperativo e saiba como conjugá-/os

A conj ugação dos verbos i rregulares no i m perativo na for­


ma afirmativa se parece muito com a dos verbos i rregulares Posicão enclítica
no presente do i n d i cativo. As desinências são as mesmas c
A colo ação pronominal com i m perativo
dos verbos regulares n o imperativo e as irregu laridades se
apresentam na raiz dos verbos: Os pronomes em posição enclrtica com o imperativo na forma
afirmativa: os pronomes átonos ou reflexivos vêm depois do
a) verbos que sofrem a alteração vocálica e > i e, o > ue e e > i nas verbo e u nidos a ele sem o uso do hífen.
pessoas tú, usted e ustedes. Por exemplo: piensa;' duerme I No caso da utilização de dois pronomes, a ordem é objeto
pide (tú), piense/ duerma/pida (usted}, pensemos/ durmamos/ indireto (me, te, se, nos, os, se) + objeto direto (lo, la, los, las).
pidamos (nosotros/as), pensad/ dormid!pedid (vosotros/as}, Exemplo: dfmelo.
piensen/ duerman/ pidan (ustedes).
Os pronomes átonos /e e les, quando usados junto com
b) verbos term i nados e n -ocer e - u c i r, q u e ap resentam os pronomes de objeto direto, transformam-se em se para
a inclusão da consoante z nas pessoas usted, nosotros e evitar a cacofonia.
ustedes, como em conoce (tú), conozca (usted), conozcamos Exemplo: repíteselo.
(nosotros/as), conoced (vosotros/as), conozcan (ustedes).
Nas construções reflexivas, os verbos nas pessoas nosotros
c) verbos termi n ados em -ir nos q uais há encontro de três e vosotros perdem, respectivamente, o -s e o -d finais
vogais, sendo i a vogal do meio. Essa vogal é su bstituída do imperativo na forma afirmativa: acostar + nos/ os =
por y nas pessoas nas quais se d á o encontro vocál ico. acostémonos/ acostaos.
Exe m plo: huye (tú), huya (usted), huyamos (nosotros/as), Os pronomes em posição proclftica com o imperativo na forma
huid (vosotros/as), huyan (ustedes). negativa: os pronomes átonos ou reflexivos vêm antes do verbo.
No caso da utilização de dois pronomes, a ordem é objeto
d) verbos q u e são i rregu lares na la pessoa d o s i n g u l ar e indireto (me, te, se, nos, os, se) + objeto direto (/o, la, los, las).
apresentam também u m a forma monossi lábica para a Exemplo: No se lo repitas.
pessoa tú. Por exemplo:

» hacer: haz/ haga/ hagamos/ haced! hagan.


» decir: di! diga/ digamos/ decidi digan.
» tener: ten/ tenga/ tengamos/ tened! tengan.
» poner: pon/ ponga/ pongamos/ poned/ pongan.
» salir: sal/ salga/ salgamos/ salid/ salgan.
» venir: ven/ venga/ vengamos/ venid! vengan.

Os verbos ir e ser são total mente i rregulares: ve (tú), vaya


(usted), vayamos (nosotros), id (vosotros), vayan (ustedes) /
sé (tú), sea (usted), seamos (nosotros/as), sed (vosotros/as),
sean (ustedes).

GE IDIOMAS 2015 1 115


I NT :E R P R ETAÇÃO D E TEXTO

La mujer que escribió un diccionario


María Moliner fue la primeira mujer
cuya candidatura se presentó en
la Academia de la Lengua

.oillllll MARIA MOLINER hizo una proeza con muy pocos


precedentes: escribió sola, en su casa, con su propia
mano, el diccionario más completo, más útil y más
divertido de la lengua castellana

H
ace tres semanas, de paso por Madrid, quise visitar a Maria biblioteca. En 1967, presionada sobre todo por la Editorial Gredos,
Moliner. Logré un contacto con su hijo menor, que me hizo di o e! diccionario por terminado. Pero siguió haciendo fichas, y en
saber que no era posible visitar a su madre por sus quebrantos e! momento de mo rir tenia varios metros de palabras nuevas.
de salud. Pensé que era una crisis momentânea y que tal vez pudiera .\Cómo trabajaba? Un día se levantó a las cinco de la maiíana,
veria en un viaje futuro a Madrid. Pero la semana pasada, cuando ya dividió una cuartilla en cuatro partes iguales y se puso a escribir
me encontraba en Bogotá, me llamaron por teléfono para darme la fichas de palabras sin más preparativos. Sus únicas herramientas
mala noticia de que María Moliner había muerto. Yo me sentí como de trabajo eran dos atriles y una máquina de escribir portátil, que
si hubiera perdido a alguien que, sin saberlo, había trabajado para sobrevivió a la escritura de! diccionario. Su marido fingía una
mí durante muchos aiíos. María Moliner hizo una proeza: escribió impavidez de sabio, pero a veces medía a escondidas las gavillas
sola, en su casa, con su propia mano, e! diccionario más completo, de fichas con una cinta métrica, y les mandaba noticias a sus
más útil y más divertido de la lengua castellana. Se llamaDíccíonarío hijos. María Moliner tenía un método infinito: pretendia agarrar
de Uso de/ Espano/, tiene dos tomos de casi 3 000 páginas en total, a] vuelo todas las pai abras de la vida. Só lo hizo una excepción: las
que pesan tres kilos. María Moliner lo escribió en las horas que !e mal llamadas malas palabras, que son muchas y tal vez las más
dejaba libre su empleo de bibliotecaria, y e! que ella consideraba su usadas en la Espana de todos los tiempos. Es e! defecto mayor de
verdadero oficio: remendar calcetines. Uno de sus hijos, a quien !e su diccionario, y Maria Moliner vivió bastante para comprenderlo,
preguntaron un día cuántos hermanos tenía, contestó: "Dos varones, pero no lo suficiente para corregirlo.
una hembra y e! diccionario". Pasó sus últimos aiíos en un apartamento dei norte de Madrid. A
Cuando e! menor de sus hijos empezó la carrera de ingeniero, veces le llegaba un periodista desperdigado. A uno que !e preguntó
María Moliner sintió que !e sobraba demasiado tiempo después por qué no contestaba las numerosas cartas que recibia, !e contes­
de sus cinco horas de bibliotecaria, y decidió ocupado escribiendo tó, con más frescura que la de las flores que cultivaba: "Porque soy
un diccionario. La ide a !e vino de! Learner's Díctíonary, con e! cu a! muy perezosa". En 1972 fue la primera mujer cuya candidatura se
ap'rendió inglés. Es un diccionario de uso; es decir, que no solo dice presentó en la Academia de la Lengua, pero los muy seiíores
lo que significan las palabras, sino que indica también cómo se usan, académicos no se atrevieron a romper su venerable tradición
y se incluyen otras con las que pueden reemplazarse. María se sentó machista. Sólo se atrevieron hace unos aiíos, y aceptaron entonces
a escribir su diccionario en 1951. Calculó que lo terminada en dos la primera mujer, pero no fue Maria Moliner. Ella se alegró cu ando
aiíos, y cuando llevaba diez todavia andaba por la mitad. "Siempre lo supo, porque !e aterrorizaba la idea de pronunciar e! discurso de
!e faltaban dos aiíos para terminar", me dijo su hijo menor. AI admisión. ".\Qué podia decir yo" - dijo entonces - si en toda mi vida
principio !e dedicaba dos o tres horas diarias, pero a medida que los no he hecho más que coser calcetines?"
hijos se casaban y se iban de la casa !e quedaba más tiempo disponible,
hasta que llegó a trabajar diez horas ai día, además de las cinco de la Gabriel Garcia Márquez, EI País. (Vestibular UFPE 2008, adaptado)

116 1 GE IDIOMAS 2015


EXERCfCIOS - U FPE 2008 (adaptado)

1. En el segundo párrafo de! texto aparece la siguiente expre­ RESPOSTAS


sión: "Siempre LE faltaban dos aiios para terminar, me dijo su 1. Se refiere a Maria Moliner, autora de/ diccionario. Trata-se de
h ij o menor". Repare en la forma LE, que aparece destacada. um pronome de complemento indireto. Uma vez q u e é um com­
De e l l a p od e m os decir, conside rando excl usivamente e l plemento indireto do verbo, não sofre flexão de gênero, é idên tico
contexto e n e l q u e aqu i aparece, que: para o masculino e o feminino. Resposta: e
a) se trata d e u n pronombre personal.
b) hace referenda ai autor de! texto, Gabriel Garcia Márquez. z. Son, respectiva men te, un p ro n o mbre interrogativo y u n
c) se refiere ai d i ccionario d e uso d e l espano!, que fue men­ pronombre exclamativo. Todos o s p ro n o m es i n t e rrogativos e
c i onado con anterio ridad. exclamativos (inclusive aqueles q u e figuram em orações iilterro­
d) es una forma variable e n cuanto ai gênero. gativas e exclamativas indiretas, nas quais não se usam os sinais
e) se refiere a Maria M o l i n e r, autora de! d i ccionario. de interrogação/excla m a ção) são acentuados graficam e n te em
espanhol. Resposta: e
1. En e l trecho " Hay que saber cómo fue escrita la o bra para
entender cuánta verdad i m p l ica la respuesta", las palabras 3. Fue contra ese criterio de embalsamadores que Maria Mofiner
subrayadas son acentuadas gráficamente porque: se puso a escribir su d/cclonario en 1951. Com o verbo de câm b io
a) son esdrúju las y todas las esdrúju las l levan tilde. "ponerse" se expressa o início de uma atividade em um momento
b) son l lanas term i n adas e n vocal. preciso, no caso o ano quando María Moliner começou a escrever
c) son agudas term i nadas e n vocal. seu dicionário. Resposta: a
d) son h eterotón icos con relac i ó n ai português.
e) son, respectivame n te, u n pronombre i nterrogativo y un �. Costurar meias. "Coser" é costurar e "calcetines" são meias.
pronom bre exclamativo. Resposta: c

J. EI e n unciado que reem p l aza correctamente a " Fu e contra s. Maria Mofiner no ha incorporado a su diccionario las palabrotas
ese criterio d e e m balsamadores que Maria Moliner se sentá dei espano/. A resposta a essa questão está no terceiro parágrafo
a escribir su diccionario e n 1951" es: do texto. A palavra "pala brotas" é s i nôn im o de "malas pala bras",
a) fue c o ntra ese c riterio d e e m bal samad o re s q u e Maria q ue significa, em português, "palavrões". Resposta: e
M o l i n e r se puso a escri b i r su diccionario en 1951.
b) fue c o n tra ese c riterio de e m balsamadore s q u e Maria
M o l i n e r se quedó a esc r i b i r su d iccionario e n 1951.
c) fue c o ntra ese criterio de e m bal samad o re s q u e Maria
M o l i n e r se volvió a escri b i r su d i ccionario e n 1951.
d) fue c o n tra ese c ri terio de e m bal samadores que Maria
M o l i n e r se h izo a escri b i r su d iccionario e n 1951.
e) fue c o nt ra ese criterio de e m bal samadores q u e Maria
Moliner and uvo esc r i b i e n d o su d iccionario en 1951.

" En el texto, Maria Moliner afirma que su oficio era coser cal­
cetines. En português, la expresión subrayada sign ifica:
a) costurar cal c i n has.
b) costurar camisas.
c) costurar meias.
d) costurar calças.
e) costurar rou pas de criança.

s. Según e l texto, podemos afi rmar que:


a) Maria M o l i ne r ten ia c i n co h ijos.
b) Maria Moliner manten ia a la fam i l ia cosiendo calcetines.
c) Marra M o l i n e r tardó m u chos anos para esc r i b i r su d iccio­
nario porque era perezosa.
d) Maria Mol i ne r fue la primera mujer a la que se h a aceptado
en La Real Acad e m i a de La Lengua Espano l a.
e) María M o l i n e r n o ha i nc o rporado a su d i c c i o n a r i o l as
palabrotas d e l espan o!.

GE IDIOMAS 2015 1 117


DICAS LOC U ÇÕ ES V E RBAIS » I NTERPR ETAÇÃO R I CA

Os verbos dizem muito


mais do que aparentam

E m espan h o l, é c o m u m o u s o d e l o c u ções verbais q u e


s e formam c o m a u n i ão d e u m verbo auxi l iar conj u gad o a Atenção!
outro verbo, q u e pode estar no i nf i n itivo, no gerú n d i o ou no Verbos com encontro vocálico sofrem uma adaptação no gerúndio:
particípio. Co n heça algu mas d essas con struções: leer = leiendo -1 leyendo.
Verbos com a alteração e > i no presente do indicativo mantêm
LOC U ÇÕ E S VERBAIS DE I N FI N ITIVO essa alteração no gerúndio, pedir -1 pidiendo, enquanto os
» Ir + a + i n f i n itivo = i n d ica u m a ação q ue começa a realizarse verbos morir/ dormir/poder -1 muriendo I durmiendo 1 pudiendo
ou q u e consideramos d e c i d ida. Ex.: Tronaba mucho. lba a trocam o por u.
1/over. (Trovejava m u i to. la c h over.)
» Echar a/ pon erse a/ romper + a + i n f i n i tivo = i n d ica o i n íc i o

d e u m a atividade e m u m m o m e n to p reciso e q u e pode re­ LOCU ÇÕ ES V E R BAIS D E PARTIC [ P I O


presen tar u m a ação desco ntrolad a q u e começa de forma N essas locuções, o particípio concorda com o co m p l e m e n to
repenti na. Ex.: Me pu se a gritar cuando vi a los ladrones. ou suje ito. Não i n d i cam ação em c u rso ou conti n u idade d e
(Comecei a gritar q uando vi os lad rões.) ação.
» Volver + a + infinitivo. Ind i ca repetição da ação. Ex.: Volvimos » Llevar/ tener + particípio = tem sentido c u m u lativo, i n d ica

a encon trar/o. (Encontramo-lo d e n ovo.) o n ú mero total d e algo. Ex.: Llevo leída la mitad de/ libra.
» Dar/e (a uno) + por + i n f i n itivo. Expressa um ato caprichoso, (Já li a metade do l ivro.) I Tengo escrito todo e/ trabajo. (Já
u m a extravagância. Ex.: Me dio por maquillarme. (Cismei escrevi todo o trabalho.)
e m me m aq u iar.) » Andar + particípio = i n d i ca o estado n o q ual o s ujeito se

» Quedar + en + i nf i n i tivo. I n d i ca conco rdância para fazer encontra. Ex.: Ando cansada de trabajar. (An d o cansada de
a l go. Ex. Quedamos en vemos esta tarde. (Co m b i namos trabalhar.)
d e nos encontrar esta tarde.)
» Esta r + por/ a punto de! para + i n f i n i tivo. I n d ica uma ação

i m i n e n te . Ex.: Estoy por divorciarme. (Estou q u as e m e Expressões idiomáticas


d ivorciando.) Algumas expressões importantes do espanhol

LOC U Ç Õ E S V E R BA I S D E G E R Ú N D I O )) De hecho = De fato.


» Esta r + gerú n d i o = i n d i ca u ma ação e m cu rso. Ex.: Los chicos n En brama = De brincadeira.

están durmiendo. (Os m e n i nos estão d o r m i ndo.) )) En casa de herrero, cuchillo de paio = Em casa de ferreiro, o
» Anda r + gerú n d i o = expressa ação d u rad o u ra. Ex.: Andan espeto é de pau.
diciendo m uc h o s a b s urdos. (An d a m d i z e n d o m u i to s n Encontrárselo todo hecho = Ter/ nascer com sorte na vida.

a b s u rdos.) n Erre que erre = Insistir.

» Seguir + gerú n d i o = i n d ica a conti n u i dade de u ma ação. Ex.: )) Estar a las duras y a las maduras = Ser solidário, tanto nos bons,
Sigo comiendo demasiado en la cena. (Conti n u o comendo quanto nos maus momentos.
d e m ais n o jantar.) )) Estar a medias tintas = Ser leigo no assunto.
n Estar como una(s) pascua(s) = Ficar exultante.

U s a m o s o gerú n d i o para e x p ressar u ma ação anterior n Estar como un fideo I Estar en los huesos = Ser muito magro,

o u s i m u ltânea à d e o utro ve rbo, n u nca poster i o r à ação magro como um palito.


exp ressa por e l e. )) Estar de rodillas = Estar de joelhos, submeter·se.
Perdí mis docu m e n tos, q u e dá n d o m e sin poder hacer la )) Estar empapado hasta los huesos I metido hasta los codos =
ma trícula. ( i n correto) Estar envolvido em algo até o pescoço, completamente.
Perdí mis documento y me quedé sin poder hacer la ma trí­ )) Estar en Babia! Estar entre Pinto y Valdemoro = Estar indeciso.
cula. (correto) )) Estar hasta las narices (la coronilla I e/ mono) = Estar cheio de,
não aguentar mais.
Formação do gerú n d i o: )) Estar patas arriba = Estar uma bagunça, de cabeça para baixo.
» Verbos te r m i nados em -ar = -ando -J bailar = bailando )) Estar (quedarse) sin blanca = Estar sem dinheiro, ficar no
» Ve rbos te r m i nados em -erl - i r = i e n d o -1 comer I vivir = vermelho.
comiendo 1 viviendo

118 1 GE IDIOMAS 2015


DICAS D E M O N S T RA T I V O S » ' E S O S O AQ U E L L O S ?

Com poucas explicações diretas e duas tabelas,


conheça uma classe importante de palavras

Os d e m onstrativos são fundamentais para a com preen­ Q u a n d o u m d e m o n strativo reto ma/s u bstitu i u m a pala­
são d e um texto. E l es i d e n ti f i cam e assi n alam os o bjetos vra c i tada ante r i o r m e n te, tem a fu nção gramatical d e u m
em relação ao espaço, ao tempo e à posição e m relação ao p ronome:
emissor e ao receptor. Veja q uais são e l es: Quiero solo aquel libro. Este es inútil.

' Em espan hol, os demonstrativos não ad m item a contração


. este 1 estos ese I esos aquel l aquellos
Demonstrativos com as preposições en e de:
esta 1 estas esa 1 esas aquella 1 aquellas
Luis ya no vive neste piso. (errad o)
Luis ya no vive en este piso. (certo)
Esos anos Aque/los anos
Estas anos son
me divertí de nina fueron P o r fi m, con heça os d e m o nstrativos n e utros. E l e s são
Identificação los mejores.
mucho. maravi/losos. e m p regad os para i n d i car aconte c i m e n tos, coisas ou frases
no tempo (Refere·se ao
(Refere·se aos (Refere·se a muitos sobre os q uais se fez refe rência anteriormente n o texto:
tempo atual)
últimos anos) anos antes) Los operarias están de huelga.
No estoy de acuerdo con eso.

Esta puerta
Esa puerta Aque/la puerta
(próximo do
(próximo do (longe do emissor
Identificação emissor) =
interlocutor) e do interlocutor) =
no espaço equivale às
= equivale à equivale às formas
formas cerca,
forma ahí /ejos, a/lí, a/lá
aquí, acá

GE IDIOMAS 2015 1 119


Continuidad si

.oOIIIIII LOS GUARANI hablantes se han acercado con mayor fuerza a la adquisición dei contra la cultura y aun contra la lealtad que las paraguayas y paraguayos
castellano, y algunos castellano hablantes perdieron el miedo ai guarani y superaron sienten por su querida lengua. El guarani, lengua de comunicación si
los prejuicios en contra de él y mil veces si; lengua de imposición, no.

MELIAÂ l, B. Disponível em: http:// staff.uni·mainz.de. Acesso em: 27 abr. 2010 (T. A.).

L
a comunidad paraguaya ha encontrado en la lengua guarani una
funcionalidad real que asegura su reproducción y continuidad. (ENEM 2011)
Esta, sin embargo, no basta. La inclusión de la lengua guarani
en el proceso de educación escolar fue sin duda un avance de la
Reforma Educativa.
Gradas precisamente a los programas escolares, aun en con­ r--.....- ­
___.., _.,...,

textos urbanos, el bilingüismo ha sido potenciado. Los guarani


h ablantes se han acercado con mayor fuerza a la adquisición dei
castellano, y algunos castellano hablantes perdieron e! miedo ai " ,..
L
guarani y superaron los prejuicios en contra de él. Dejar de fuera !. !... '
-
--
.
de la Educación Media ai guarani seria echar por la borda tanto - !.- L-
:!...... !.. w
!_
trabajo realizado, tanta esperanza acumulada.
.. ... ...
..
___ ____ __ __ _,
Cualquier intento de marginación dei guarani en la educación ... _______ _____ _

......
. .. ...
-· --------
""__ _. ____ ______

paraguaya merece la más viva y decidida protesta, pera esta postura


ética no puede encubrir el continuismo de una forma de enseiianza .oOIIIIII DEJAR DE FUERA de la Educación Media ai guarani seria echar por la borda tanto
dei guarani que ya ha causado demasiados estragos contra la lengua, trabajo realizado

120 i GE IDIOMAS 2015


EXERC(CIOS - ENEM 2011 (adaptado)

L No ú ltimo parágrafo do fragmento sobre o b i l i nguismo no 4- No trecho do texto "Aun sin escuela e i ncl uso a pesar de
Paraguai, o autor afirma que a l íngua guarani, nas escolas, la escuela, paraguayos y paraguayas se están com u n icando
deve ser tratada como l íngua de com u n i cação e não de im­ en guaran i", a locução verbal s u b l i n hada nos i ndica q u e a
posição. Qual dos argumentos abaixo foi usado pelo autor ação expressa pelo verbo:
para defender essa ide ia? a) Está em andamento.
a) O guarani continua sendo usado pelos paraguaios, mesmo b) Está d escontrolada o u começa d e forma re p e n t i n a.
sem a escola e apesar d ela. c) Mostra u m ato caprichoso, u m a extravagância.
b) O e n s i n o médio no Paraguai, sem o guarani, desmereceria d) Tem sentido c u m u lativo.
todo o trabalho realizado e as esperanças acu m u ladas. e) Começa a realizar-se.
c) A língua guaran i encontrou u m a f u n c i o n a l i dade real q u e
asse g u ra s u a re p ro d u ção e c o n t i n u i d a d e, m a s só i s s o
n ã o basta. RESPOSTAS
d) A i ntro d u ção do guaran i nas escolas pote ncializou a d i­ 1. O guarani continua sendo usado pelos paraguaios, mesmo sem
f usão da l ín g u a, mas é n ecessário q u e haja u m a postura a escola e apesar dela. Essa resposta se justifica porque este é
ética em seu ensi no. o único argumento apresentado que trata o guarani como uma
e) O b i l i ng u i s m o na m an e i ra d e e n s i nar o guaran i tem cau­ língua de comunicação. Resposta: a
sado estragos contra a l íngua, a c u l t u ra e a lealdad e dos
paraguaios ao guaran i . 1. /nc/usão e permanência do ensino do guarani nas escolas. O texto
nos mos tra que a mobilização social no Paraguai foi responsável
2. E m alguns países bilíngues, o u s o de uma l íngua pode se pela inclusão do ensino do guarani nas escolas. Resposta: d
sobrepor à outra, gerando uma mobilização social em prol
da valorização da menos proemi nente. De acordo com o J,Argentina, Chile e Panamá. Apesar de na Espanha a língua oficial
texto, no caso do Paraguai, esse processo se deu pelo (a): ser o espanhol (castelhano), este país não é híspano-americano, se
a) Falta de conti n u idade do e n s i n o guaran i nos programas localiza na Europa. No Suriname e no Haiti, a língua oficial não é o
esco lares. espanhol. Resposta: c
b) Preconceito existente contra o guara n i p r i n c i pal me nte
nas escolas. 4. Está em andamento. Da mesma forma q u e a tradução para o
c) Esperança acu m u lada na reforma ed ucativa da e d ucação português "estão se comunicando em guara n i". Resposta: a
méd ia.
d) I nclusão e permCI.nência do ensino do guarani nas escolas.
e) Contin u ísmo do ensino do castelhano nos centros urbanos.

J. De acordo com o texto, o Paraguai é um país h ispano-falante


da América do Sul. Em que alternativa temos somente países
hispano-americanos nos quais a língua oficial é o espanhol?
a) Argentina, Bolívia e Espanha.
b) Peru, U ruguai e S u riname.
c) Arge n t i n a, Chile e Panamá.
d) Venezuela, Colômbia e H ai ti.
e) México, Arge n t i n a e Espanha.

GE IDIOMAS 2015 1 121


DICAS PRETÉRITO PERFECTO COMPU ESTO » PASSADO QUE SE PROLONGA E I NCLUI O PRESENn

Conheça um dos tempos verbais utilizados


para falar de fatos que já aconteceram

E m espan h o l, para falar d e aconteci m entos passad os,


podemos usar o p retérito perfecto c o m p u esto de i n d i ca­ Listas de referência
tivo. E l e exp r i m e u m aco n tecimento encerrado, mas q ue, Conheça a formação do particípio de
d e a l g u m a forma, está re lacionado ao presente o u cujos verbos regulares e irregulares
efeitos d u ram, tam bém, n o m omento em q u e se fala. Essa
relação com o presente é expressa por meio de u ma série de Infinitivo em -ar: formam o particípio sempre em -ado, ou seja,
referentes temporais: não há verbos com particípio irregular:
» tempo próx i m o ou que i n c l u i o presente: esta manana/ >> estudiar --+ estudi + ado = estudiado

tardejnoche, hace, minutos/horas/segundos, hace un » pagar --+ pag + ado = pagado


momento, esta semana, este mes/ano/siglo, hoy. Ex.: Hace
una hora h e ido a/ supermercado. Infinitivo em -er/ ir: formam o particlpio em -ido:
» advérbios: todavía no/aún no (ai nd a não), ya (já}, a/guna » beber --+ beb + ido = bebido
vez, una vez, dos/tres ... veces, nunca/jamás. Ex.: Aún no ha » venir � ven +ido = venido
empezado la c/ase.
Os verbos se c o n j u gam da seg u i n te forma: o p retérito Já os verbos irregulares apresentam uma forma própria
perfecto compu esto é formado pelo verbo auxi liar haber no de particípio:
p resente d o i n d i cativo mais o verbo pri n c i pal n o particfpio. » hacer/deshacer --+ hecho!deshecho
O particfpio permanece i n variável, o u seja, não faz concor­ » romper --+ roto
dância c o m o s ujeito, q u e é expresso pela pessoa na q ual o » escribir/circunscribir/describir/transuibir --+ escrito/
verbo haber é conj ugado. O verbo haber é u sado para formar circunscrito/ descrito/transcrito
todos os tempos com postos. » volver/desenvolver/devolver/envolver --+ vuelto/desenvuelto/
devuelto/envuelto
-----.-- u�,... ·�--r
� �;.... .";L "t - ' •
» abrir --+ abierto
Pretérito perfecto compuesto -·.. ·:' .: H�ber + Particípio ·.
» absolver!disolver/resolver --+ absuelto/disuelto/resuelto
yo he andado » poner/anteponer/componer!disponer/exponer/ imponer/
posponer/presuponer/proponer/transponer --+ puesto/
tú has andado
antepuesto/compuesto!dispuesto/expuesto/pospuesto/
usted 1 é/! e/la ha andado presupuesto/propuesto/transpuesto
» ver/antever/prever/rever --+ visto/ante visto/previsto/revisto
nosotros;as hemos andado » cubrir!descubrir/encubrir --+ cubierto/descubierto/encubierto
» imprimir ....,} impreso
vosotros/as habéis andado
» decir --+ dicho
ustedes 1 ellos;ellas han andado » morir --+ muerto
>> satisfacer --+ satisfecho

o pretérito perfecto compuesto, os pronomes oblíquos


devem ser colocados sempre a:ntes do verbo:

No he me sentido bien hoy. (errado)


No he sentidome bien hoy (errado)
No me he sentido bien hoy. (certo)

1221 GE IDIOMAS 2015


DICAS ADJ ETIVOS POSSESSIVOS

Duas formas para uma


mesma função sintática

O s possessivos podem ter a f u n ção d e adjetivo (acom­


pan ham um s u bstantivo, mostrando q u e m o possu i ) o u de Amplie seu repertório
p ro n o m e (no l u gar d e um s u b stantivo ou grupo n o m i nal, Continue a conhecer expressões idiomáticas
q ua n d o d es ejamos retomá-lo, evitand o a sua repetição).
Con heça os possessivos com a fu nção d e adjetivo: 11 Fijarse en algo (alguien). � Prestar atenção.
11 Flipar/alucinar. � Estar louco, perder o juizo, ficar doido, não
» Ato nos: antecedem os su bstantivos e não ad mitem artigos. estar muito bom da cabeça.
Exe m plo: 11 Gallina vieja da buen caldo. � Panela velha é que faz comida boa.
Aquel/a fue la mi gran oportunidad. (errado) 11 Guardarse las espaldas. � Precaver-se.
Aquella fue mi gran oportunidad. (certo) 11 Hablando dei rey de Roma, por la puerta asoma. � Falando do
diabo, apareceu o rabo.
11 Hacer algo adrede. � Fazer algo de propósito.
11 Hacer como si nada. � Fazer-se de desentendido.
yo mi casa I mi coche mis casas1 mis coches
11 Hacer filigranas. � Fazer elucubrações, fundir a cuca.
11 Hacer la pascua a alguien. � Estragar a festa, incomodar.
tú tu casa I tu coche tus casasI tus coches
11 Hacerle la pelota a alguien. � Bajular alguém, puxar o saco.
11 Hacerse e/ (la) interesante. � Exibir-se.
usted! 11 Hacerse mala sangre. � Zangar-se, ficar de mau humor.
su casa1 su coche sus casa I sus coches
el!ella
11 Hacerse un lío con algo. � Confundir-se, atrapalhar-se com.
nosotros/as nuestra casa 1 nuestro coche nuestras casasI nuestros coches 11 Hacer sombra a alguien. � Superar alguém por merecimento
ou qualidades.
vosostros/as vuestra casa I vuestro coche vuestras casas1 vuestros coches 11 Hacer pufíetas. � Mandar plantar batatas, pentear macacos.
11 jHala! � Vamos! Coragem!
ustedes/
e/los/e/las
su casa 1 su coche sus casas I sus coches 11 Hecha la Jey, hecha la trampa. � Aprovada a lei, o caminho
para contorná-la está aberto.
11 Hincar los codos. � Matar-se de estudar.
» Tôn icos: vêm sem pre depois do s u bstantivo. Exe m plo: 11 Hinchársele las narices a alguien. � Subir o sangue à cabeça;
E/ viaje fue idea m ía . zangar-se.
11 Ir de punta en blanco. � Estar muito elegante.
11 lrse de picos pardos. � Divertir-se.
11 Llegar y besar e/ santo. � Fazer alguma coisa rapidamente.
yo casa mfa I coche mfo casas míasI coches míos
11 Llevarse por de/ante. � Arrasar, destruir.
11 Lo mío es.. � O que me interessa é... (a música, o futebol etc.).
.

tú casa tuya 1 coche tuyo casas tuyas I coches tuyos


11 Los tuyos nunca te olvidan. � A sua família não se esquece
de você.
usted! 11 Voy a lo mío. � Preocupo-me apenas com as minhas coisas.
casa suya 1 coche suyo casa suyas I coches suyos
el/ella
11 Siempre se safe con la suya. � Consegue sempre aquilo
nosotros/as casa nuestra I coche nuestro casas nuestrasI coches nuestros que q uer.

vosostros/as casa vuestra 1 coche vuestro casas vuestras I coches vuestros

ustedes/
casa suya Icoche suyo casas suyas 1 coches suyos
e/los/ e/las

GE IDIOMAS lOlS 1 123


I NT E R P R ETAÇÃO D E TEXTO

Estudios en Latinoamérica
Los caminos para organizar un período de estudios en un país latinoamericano

� UNA EXPERIENCIA ONICA Pasar una temporada de estudios en un país extranjero exige tiempo para obtener información y organizarse

E
n esta sección hemos intentado dar cabida a los apartados de consultar las redes iberoamericanas; muchas de e lias conceden becas
más interés para quienes se plantean una estancia formativa en e! marco de sus programas de movilidad.
en Latinoamérica. Si ya hemos decidido la universidad y e] programa académico que
Para empezar, si dudamos entre diferentes destinos o aún no nos nos interesa, y hemos sido admitidos, toca empezar a organizar el
hemos decidido por ninguno en concreto, es probable que deseemos viaje. Antes de salir rumbo a Latinoamérica debemos haber solven­
saber más de los países que conforman América Central y América dei tado los problemas dei visado, alojamiento, seguro médico, billetes
Sur, además de conocer México (Norteamérica) y algunas islas caribenas. para viajar ai país de destino, etc.
En esta guia podremos consultar información sobre sus ciudades, sus Para solicitar el visado y conocer los trámites se debe contactar
sistemas universitarios, sus universidades y las titulaciones que ofertan. con la embajada de nuestro país en el país latinoamericano en e! que
Una vez que localicemos el centro de educación superior que queramos estudiar, investigar o ensefiar.
nos interesa, probablemente queramos conocer las becas o ayudas Otra fuente de información con la que debemos contactar es el
económicas de que disponemos para costear nuestra estancia. Lo Ministerio de Asuntos Exteriores, tanto de nuestro país como dei
mejor es probar a acogerse a alguno de los muchos programas de de destino.
movilidad que se han firmado entre países, tanto para intercambio Es recomendable realizar todos los trámites con la máxima ante­
de estudiantes como para movilidad de profesores. lación posible, ya que la obtención dei visado suei e llevar un par de
También pueden interesarnos los proyectos de cooperación ibe­ meses como mínimo, y además cuanto antes se localice una vivienda
roamericanos, en los que Espana desempena un papel fundamental, y los billetes de avión, más barata resultará la estancia.
ya que ai compartir la misma lengua la comunicación y colaboración Por último, hemos incluído una sección para solventar las dudas
interuniversitaria es especialmente fácil y productiva, permitiendo que puedan surgir ai regresar de nuestro país en lo relativo a la
el desarrollo de programas conjuntos de docencia e investigación homologación de títulos.
en ambas orillas de] Océano Atlántico.
Por último, en la búsqueda de ayudas financieras es interesante (Vestibular UFSC 2012, adaptado)

124 1 GE IDIOMAS 2015


EXERCICIOS - U FSC 2012 (adaptado)

Seíiala la(s) proposición(es) correcta(s): RESPOSTAS


1. 0 texto fornece orientações sobre estudos em países latino-americanos,
L lA qué público se destina el texto? que podem interessar a qualquer pessoa que pretenda passar um perío·
a) A extranjeros q u e desean estud iar en Latinoaméri ca. do de estudos neste continente. Fala em ensino superior, mas não há
b) Solamente a extranjeros q u e d esean hacer un c u rso d e post· nenhuma informação que restrinja o acesso a alunos de pós-graduação.
grado en Latinoamérica. Tampouco orienta com relação a tra balho. E não cita brasileiros em
c) A todos los extranjeros q u e van a trabajo a países lati noame­ nenhum trecho. Duas alternativas são válidas.
ricanos. Resposta: a e e
d) A ciudadanos bras i lefios que obtuviero n una beca para estudiar
en países lati n oamericanos. z.Analisando as alternativas:
e) A q u i e n se propone desarrol lar estud ios e n países latinoame­ a) O texto não apresenta nenhum aspecto econômico dos países - não
ricanos. fala de moedas correntes ou balança comercial, por exemplo. Apre­
senta aspectos das cidades, dos sistemas do ensino superior e das
z. EI texto sugiere al lector la búsqueda de algunas i nformaciones titulações oferecidas. Incorreta.
en cuanto a: b) A Espanha é citada como país parceiro importante em projetos ibero­
a) los aspectos económ icos d e l destino e n q u e se p ropone estu­ americanos. No entanto, em nenhum trecho o texto menciona projetos
d iar. de cooperação com o Brasil. Incorreta.
b) los proyectos de cooperación entre Espafia y Brasil, que ofrecen c) Sim, o texto traz informações sobre o auxflio econômico que o estudante
becas a p rofesores y est u d iantes. pode obter para se man te r no país no qual vai estudar Por exemplo, em
c) las ayudas financieras para e l coste de l a estancia; l a oferta de "las becas [bolsas de estudo] o ayudas económicas de que disponemos
becas, e ntre otros. para costear nuestra estancia {estada]", ou em "en la búsqueda de ayudas
d) a los trámites involucrados en los viajes i nternacionales, tales financieras es interesante consultar las redes iberoamericanas; muchas de
como: seg u ro médico, vi sado, b i l l etes y hospedaje. e/las conceden becas en e/ marco de sus programas de movilidad". Correta.
e) a los trám ites invo l ucrados en e l retorno a i país d e origen, tales d) Há também informações sobre os trâmites necessários para as viagens
como l a com pro bación d e l a c o n c l u s i ón del c u rso. internacionais. Por exemplo, no trecho "Antes de salir rumbo a [rumo a]
Latinoamérica debemos habersolventado los problemas de! visado, a!oja­
3. Las expresiones aún, ya q u e y toca, d estacadas e n el texto, miento, seguro médico, billetes para viajar a/ país de destino, etc". Correta.
pod rían ser respectivamente sustituidas, s i n perj u icio de signi­ e) O texto não faz referência a o retorno ao país de origem. Incorreta.
ficado, por: Resposta: c e d
a) i nc l uso, a u n q u e, suena.
b) pues, aun q u e, sue na. 3-A questão exige vocabulário, em especial de sinônimos, especificamente
c) puesto, visto q u e, com pete. sobre o significado de "aún" e 'ya que". Aún (ainda) sempre se refere a tempo.
d) todavía, una vez q ue, com pete. O trecho "aún no nos hemos decidido por ninguno en concreto" poderia ser
e) todavía, pues, co m pete. traduzido por "ainda não nos decidimos por nenhum em particu!arJ'. Já a
expressão "ya que" ljá que), apesar de conter a palavra 'já'; não se refere a
4- En e l fragmento sacado del texto 2 "Es recomendable realizar tempo, mas introduz uma explicação. A expressão "toca"significa "compete".
todos los trámites con la máxima antelación posi ble, ya que la Não se trata exatamente de falso cognato. Em português, dentre os vários
obtención de I visado suele ! l evar u n par de meses como mínimo, significados do verbo tocar, há o de referir-se a ou ser da competência de.
y además cuanto antes se local i ce una vivienda y los bil letes de Resposta: d e e
avión, más barata resu ltará la estancia" las expresiones desta­
cadas pod rían ser respectivamente sustituidas, s i n perjuicio de 4- A questão exige apenas vocabulário. Neste caso, não é difícil, pois a
significado, por: maioria das palavras é m uito semelhante a sua versão em português.
a) aconsejable, anticipación, algu nos. Mas a questão pode ser resolvida apenas pela compreensão da expressão
b) i m presc i n d i ble, ante rioridad, m u chos. "recomendable" (recomendável). O termo é sinônimo de "aconsejable",
c) i nd ispensable, antici pación, d iversos. "conveniente" e "indicado", e não impõe uma obrigatoriedade - caso
d) conve n i ente, anterioridad, u nos. das palavras "imprescindible" (imprescindível) e "indispensable" (in­
e) i n d i cado, antici pac i ó n, u nos. dispensável). Com isso, eliminamos as alternativas b e c. "An telación" é
sinônimo de "anticipación" (antecipação). E ''un par de" é uma expressão
idiomática, sinônimo de "algunos" ou "unos" (alguns ou uns). (Veja mais
sobre e x p ressões i d i o m áticas na pág 101]
Resposta: a, d e e

GE IDIOMAS lOIS 1 125


DICAS CON EXÕES » COESÃO E C O E R Ê N C I A

Conheça dois importantes recursos


linguísticos que servem para ligar ideias

Os conce i tos presentes e m u m texto estabelecem e ntre Causal Suspendi e/ examen. No pude estudiar ayer.
s i vários tipos d e relação d e sentido. Para estabelecer essas
Consequência Ayer, estudié mucho. Tengo que aprobar e/ examen.
relações, é necessário e m p regar vários recu rsos l i ng u ísticos
que são responsávei s pela coesão e coerência do texto. A f i m Fui a/ teatro. Llegué temprano. Compré las entradas. Carlos
Temporal
d e realizar u m a l e i t u ra profi c i e n te do texto, ou seja, recon s­ 1/egó diez minutos antes de/ inicio de/ espectácu/o.
tru i r o sentido do q ue se leu, é necessário com p reender essas Comentário Suspendi e/ examen. iQué lástima!
relações e reco n hecer os recu rsos l i n g u ísticos e m pregad os
Cada dia hay más trabajadores en paro. Ayer demitieron a
p e l o autor para estabel ecê-las. Especificação
diez empleados de mi oficina.
A coe r ê n c i a textu al está re lacionad a ao m o d o como as
i de i as se arti c u l a m no texto. A coesão se art i c u l a com os Contraste I Estoy enfadada. Tú, ai contrario, estás muy tranquila.
m ecan i s m os formais q u e u t i l izamos para relacionar essas Comparação
i d e i as. Observe o exe m p l o a seg u i r: "Dura n te mucho tiempo
Correção Mi equipo ha perdido todos los partidos. O mejor, casi todos.
se h a pensado que la invención de la calculadora electrónica
condujo a un deterioro em nuestra capacidad para realizar
matemáticas menta/es. Ahora se cree que lo mismo podría E n cadeam ento p o r con exão: v i mos q u e é possível rel a­
estar ocurriendo con la tecnología . .. ". Entre as d uas orações se cionar e n u nciados tam bém com o e m prego de conecta res,
estabelece u ma relação d e coesão e coerência, marcada pelo q u e tornam as relações mais v i s íveis para o l e i to r. Há vários
e ncad eamento por j ustaposição, sem o uso d e con ectares. t i pos d e encadeamento pelo uso de conectares. Na frase: "En
E m outro exe m p l o, podemos observar tam bém o e ncadea­ afíos recientes varios científicos en e/ mundo han estado .. ". .

mento de enunciados, construído com o uso de conectares: "En A expressão s u b l i n hada é u m conectar e estabelece u m a
m i pueb/o estas cosas pasan (me he criado oyendo h istorias relação t e m p o ral. Veja a l g u n s dos conectares u sados para
de aparecidos), sin embargo yo no creo en las apariciones". estabelecer as relações temporais e ntre os e n u nciados de
Quando a relação entre e n u nciados se faz com o uso de u m um texto:
conecto r, temos o q u e se chama d e encadeame n to p o r cone­ » aún! todavía (ai n da): expressam a p e r s i st ê n c i a d e u m a

xão. No exe m p l o anterior, a expressão s u b l i n hada estabelece situação.


u ma relação lógica d e oposição entre os argu mentos dos dois » mientras/mientras tanto/entretanto (enq uanto/enq uanto

e n u nciados conectados. i sso): expressam a s i m u ltaneidade de ações.


» /uego/después/entonces (depois): expressam prox i m idade

i m ed iata no tem po.


Atenção! » pron to/rápidamente (cedo! rap idame nte)

O conectar sino pode estabelecer uma relação de correção entre » primero/en primer lugar/en segundo lugar/en tercer lugar/

dois enunciados. Para isso, é necessário que o primeiro deles seja para finalizar/para terminar: exp ressam a ordem na qual
negativo: No estoy cansado, sino enfermo. se d ão as ações.
» enseguida!ya (daq u i a pouco)

» expressões q u e d eterm inam q u a n d o ocorrem as ações:

E ncadeamento por j u staposição: Mesmo sem o auxíl i o d e datas (ano, mês e d i a)/hora/períodos etc.
conecta res, v i m os q u e é possível estabelecer d iversos t i pos
d e relação de sentido e n tre os e n u n ciados d e um texto:
Atenção!
O uso das palavras /o e/lo, esta, eso eaquello é um mecanismo de coesão
,

que nos permite retomar um enunciado do texto ou toda uma ide ia.
No tengo tiempo para estudiar. Pore/lo, noaprobaré matemáticas. / No
tenho tempo para estudar. Por isso, não serei aprovado em matemática.
� e/lo no tengo tiempo para estudiar
=

126 1 GE IDIOMAS 2015


DICAS PRETÉ RITO I M P E R F ECTO D E I N DI CATIVO » HÁBITOS ANTIGOS

Aprenda a conjugar os tempos verbais que tratam


de ações costumeiras e da descrição no passado

O pretérito i m perfecto é u sado para d escrever ações, pes­ Yo había trabajado.


soas, l u gares, c i rcu nstân cias e para falar d e hábitos/ações Tú habías trabajado.
cost u m e i ras ocorridos no passad o. Exe m plo: Cuando era Usted/ ÉI/ E/la había trabajado.
adolescente, estudiaba matemáticas todos los días. Nosotros/as habíamos trabajado.
O p retérito i m perfecto p o d e s e r u sado tam b é m c o m o Vosotros/as habíais trabajado.
fórm u l a d e cortesia, para expri m i r u m desejo ou u m p e d i d o Ustedes/ E/los/ Elias habían trabajado.
d e m a n e i ra g e n t i l : Quería hablar contigo, p o r favor.
Aco m panhe a conju gação desse tempo verbal:
Atenção redobrada!
Duas dicas im portantes a respeito da conj ugação verbal
Pronomes Verbos terminados em -ar Verbos terminadosem -e r / -ir

yo estudiaba comia I vivia


» A forma había, do verbo haber no pretérito i mperfeito,
exerce a mesma fu nção que a forma hay no presente do
indicativo. Ambas são impessoais (nunca vão ao pl ural) e têm
tú estudiabas comiasI vivias
o significado de existir: Habían muchas personas paseando
usted!
estudiaba comia I vivia por las calles. (errado) � Había muchas personas paseando
eljel/a
por las calles. (certo)
nosotros/as estudiábamos com/amos I vivíamos >> Com os verbos conjugados em todos os tempos do modo

indicativo, os pronomes átonos devem preceder as formas


vosostros/as estudiabais comíais 1 vivíais verbais. Veja exemplos: me ha provocado mucho dolor,
Cuando se inventá la caja tonta e pera ahora la usan como.
ustedes;
e/los/e/las
estudiaban comían 1 vivían No caso das locuções verbais, os pronomes podem vir
imediatamente antes ou depois dos verbos que formam
a locução e, nesse caso, formando uma só palavra com
o segundo verbo: es cómo nos estamos pareciendo, que
Pronomes Verbo Ir Verbo Ser Verbo Ver
poderia ser também: es cómo estamos pareciéndonos.
yo iba era veia Em espanhol, é proibido colocar pronomes entre dois verbos,
sejam de uma locução verbal ou de um tempo verbal composto
(pretérito perfecto compuesto, pretérito pluscuamperfecto
tú ibas eras veias
etc.): estoy me vistiendo I he me vestido (incorreto) � me estoy
usted/
iba era veia vistiendo! estoy vistiéndome/ me he vestido (correto).
eliel/a

nosotros/as íbamos éramos veiamos

vosostros/as ibais erais veiais Atenção redobrada!


Verbos com encontro vocálico sofrem u ma adaptação no
ustedes/
e/los/e/las
iban eran veían gerúndio: leer leiendo � leyendo. Verbos com a alteração
=

e > i no presente do indicativo mantêm essa alteração no


gerúndio: pedir � pidiendo. Os verbos morir!dormir/poder
Co m o p r eté r ito pl uscuam perfecto d e i n d i cativo, d escre­ trocam o por u: muriendo/durmiendo/pudiendo.
vemos ou explicamos c i rc u n stâncias e ações no passado,
anteriores a o u t ras, tam bém no passado. Exe m plo: Cuando
entré en casa, tú ya habías abierto las ventanas.
A c o nj u gação f u n c i o n a da s eg u i nte fo r m a: e s s e t e m ­
po com posto é formad o p e l o verbo haber, conj ugado n o
p reté r i to i m pe rfecto d o i n d i cativo, m a i s o part i c í p i o d o
v e r b o co rrespon d e n te . Le m b re-se d e q u e h á verbos q u e
t ê m parti c í p i o i rregu lar.

GE IDIOMAS 2015 I l27


DICAS P RET É RITO P E RF ECTO S I M P L E » AÇ ÕES/S ITUAÇÕ ES COMP LETAS E ACABADAS

Este tempo verbal descreve ações


que terminaram no passado

Vamos agora con hecer o p retérito perfecto s i m p l e do i ndi­ Poner (suponer.


cativo. Ele é usado para contar ações ou fatos de uma u n idade
•1 ' Hacer Querer Poder
componer1
d e tempo passada e encerrada, sem n e n h u ma relação com o
puse hice quise pude
presente ou com o m om ento em q u e se produz o e n u nciado.
Exe m p lo: Ayer bebí demasiado en la fiesta. pusiste hiciste quisiste pudiste
Ele costu ma vir acom pan hado das segu i n tes expressões
t e m p o rais: ayer ( o n te m ), anteayer (anteonte m), anoche/ puso hizo quiso pudo
ayer por la noche (ontem à n o i te), ayer por la mafiana/ tarde
pusimos hicimos quisimos pudimos
(ontem de man hã/à tarde), e/ mes pasado!afio pasado (o mês
passado/ano passado), e/ otro día (o o utro d i a), en e/ .2003 pusisteis hicisteis quisisteis pudisteis
(em 2003), hace dos dfas/una semana (há d o i s d ias/ há u m a
semana), una vez (certa vez, u m a vez), en esos momentos pusieron hicieron quisieron pudieron
( nesses/naqueles m o m e n tos), aquel día (aq u e l e d i a) etc.
O p retérito perfecto s i m p l e é bastante e m p regado n o s
textos p redo m i nantemen te n arrativos, j u ntamente com os Pedir (conseguir/sentir/vestir/preferir!
Saber (caber)
os pretéritos i m perfecto e o p luscuam perfecto d o i nd i cativo. pedir! medir! repetir/servir!divertir/elegir)
Com e l e, contamos fatos e, com os d o i s o utros pretéritos, pedí
supe (cupe)
narramos as ci rcu nstâncias em que eles ocorreram ou descre­
vemos os l u gares e pessoas envolvidas no relato. Essa relação supiste (cupiste) pediste
e n tre os pretéritos caracte riza a l g u n s g ê n e ro s l iterários,
como o conto, o romance e a biografia, mas tam bém pode supo (cupo) pidió
ser encontrada nas notícias e nas resen has d e fil m es e l ivros.
supimos (cupimos) pedimos
Q uanto à forma, tanto o pretérito perfecto s i m p l e como o
perfecto com puesto podem ser trad uzidos para o português supisteis (cupisteis) pedisteis
pelo pretérito perfeito: Hoy he cantado en e/ bafio (Hoje cantei
no banho) I Ayer canté en e/ bafio (Onte m cantei no ban ho). supieron (cupieron) pidieron
Porém, em espan h o l, eles têm u sos d iferentes: o pe rfecto
c o m p uesto é e m p regado q uando q ueremos mostrar q u e há
relação e ntre o q u e estamos contando e o presente (mesmo lnf!uir (oír/constituir/construirl
Morir Leer
q u e seja uma relação psicológica). hwr/conclwr)
O bserve o texto da página g8. Em vez do pretérito perfecto influi leí
morí
s i m p l e, pred o m i n a o uso do pretérito perfecto c o m puesto.
Essa esco l h a tem u m objetivo com u n icativo m u ito claro: ele moriste influíste leíste
.
quer tra n s m i t i r ao le itor a i d e ia d e q u e a ITiaioria das i d e ias
d iscutidas, apesar de pertencerem a um passado cronológico, murió influyó leyó
a i n d a afetam ao e m i ssor e ao l e itor ou estão relacionadas
morimos influímos leímos
com o presente. Se ele q u i sesse relatar os mesmos fatos de
maneira o bjetiva e se,m vínculo com o presente, teria em pre­ moristeis influisteis leísteis
gado os verbos no pretérito p e rfecto s i m ple.
murieron influyeron leyeron

128 1 GE IDIOMAS 2015


DICAS FU NÇÃO ADVÉRBIO » UM BOM CON ECTIVO RESOLVE QUASE TUDO

Orações subordinadas estabelecem


um sentido para as orações principais

As orações s u bordi nadas adverbiais funcionam como u m » Trabajaré/trabajo mucho a fin de que puedas vivi r con
advérbio e m relação à oração principal: En cuanto /legue a casa, dignidad. (final)
l/a maré a Enrique. (Quando chegar a m i nha casa, l i garei para
o Henrique.) Essas orações estabelecem com a o ração pri nci­ N o caso das subordi nadas f i n ais, se o sujeito dos verbos
pal u m a relação de sentido que pode ser causal, condicional, das d u as o rações, s u b o r d i nada e p r i n c i pal, é o m e s m o, o
temporal, de lugar, de modo, de com paração, final, consecutiva verbo da s u b o rd i nada d eve estar no i n f i n i t ivo.
ou concessiva. Os conectares que as unem são: Exemplo: i Tú crees que tendremos tiempo para adaptamos
a los efectos de/ cambio climático? (suje ito nosotros)=

porque, pues, puesto que, ya que, como, a causa de que, visto


que, en vista de que
Cuidado�
cuando, como, siempre que, ya que, con talque, con que, si O conectar si não obedece à regra anterior. Não é usado com
verbos no futuro simple nem no presente do subjuntivo. Para
en cuanto, como, antes (de que), después (de que), mientras,
expressar condição no futuro, deve ser usado com o presente do
apenas, tan pronto como, ni bien
indicativo: Si tengo sueíio, me acostaré un poco.
donde

como, según N o caso d as o rações t e m p o ra i s e c o n ce s s i vas, p ara


e x p ressar uma rel ação d e a n t e r i o r i d a d e e ntre as ações/
así/como, tal/cua/ fatos da oração p r i nc i pal e da subordi nada, usamos os verbos
nos seg u i n tes tem pos:
» verbo da oração princi pal n o futuro s i m p l e d o i nd icativo +
para, para que, a fin de que, a que
verbo da subord i n ada no presente del s u bj u n tivo:
luego, pues, por consiguiente, por (lo) tanto, por esto, así que, En wanto /legues a casa, ya habré preparado la cena.
así pues (tem poral)
aunque, aun, si bien, aun cuando, como, ya que, a pesar de » verbo da oração principal no futuro s i m pl e de i nd icativo
que, bien/ mal que + verbo da subord i n ada n o pretérito perfecto del
subj u n tivo:
Nas orações subordi nadas adverbiais tem porais, concessi­ Aunque hayas dicho la verdad, no te perdonarán.
vas, condicio nais, modais e f i n ais, o verbo da o ração subor­ (concessiva)
d i nada d eve estar no presente do s u bj u ntivo. E o verbo da
oração principal precisa estar no futuro s i m ple/ p resente do
i nd i cativo, quando a perspectiva tem poral das d uas o rações Conexão distante
é de futuro e expressamos h i póteses. Observe: As expressões de concessão também ligam parágrafos

» Cuando viajes a/ extranjero, comprarás muchos dólares. Os conectares concessivos não só estabelecem a oposição ou
(relação tem poral) o contraste de argumentos entre enu nciados do texto, mas
» Aunque ganes/ganas mucho dinero, no dejarás de trabajar. também entre sequências mais distantes umas das outras - por
(concessiva) exemplo, entre parágrafos. Há outros conectares que também
» Como vengas a encontrarme, te invitaré a un café. estabelecem oposições no texto, os adversativos, como pera e
(cond icional) sin embargo: Trabajó duro toda la semana, pera no /e pagaron
» Según se presente la situación, diremos la verdad o no. las horas extras.
(de modo)

GE IDIOMAS 2015 1 129


I NT E R P R ETAÇÃO D E TEXTO

Como
nosotros
Una canción de
Rubén Blades
sobre el pasado

.ollillll "CUANDO ERA NIAO Ml


BARRIO ERA UN CONTINENTE Y
CADA CALLE ERA UN CAMINO A
LA AVENTURA", dijo Blades sobre
Panamá de los afios 6o

TIEMPO una mudanza deja ai viejo barrio atrás,


Llévame de vuelta a los días cuando corria con mi perro a la tienda crece e! bigote y la responsabilidad ...
dei C hino, a comprar diez centavos de pan y cinco de queso blanco. Trabajaba y pensaba si otros, como yo,
Devuélveme las tardes cuando la mano de mi abuela me llevaba siendo tan jóvenes sentían mi soledad,
hasta e! final de la Calle 13 Oeste, en Santa Ana, a ver e! Mar dei Sur. si aún compartían nuestras almas la ilusión
Reg:résame a las noches cuando e! aroma a jazmín sembrado de que e! muchacho siempre triunfara ai final.
en latas, se derramaba desde los balcones por todas las calles de Me preguntaba si aún habría otros,
San Felipe. como nosotros, como yo,
aún resistiendo, aún sin rendirse,
Cuando era nino mi barrio era un continente aún recordando, como yo, como yo.
y cada calle era un camino a la aventura.
En cada esquina una memoria inolvidable, Y en la curva de los anos me enconLré
en cada cuarto una esperanza, ya madura. con los muchachos con los que solía jugar,
En nuestros viajes de ida y vuelta a los luceros fuimos piratas, con los que senderos de estrellas caminé,
saltimbanquis y vaqueros. cuando e! horizonte era un atajo sobre e! mar.
Nuestra pobreza nunca conquistá e! dinero, pero en las casas Y recobramos las memorias con café,
nunca se rindió e! "yo puedo". y nos tratamos aún de tú, y no de usted,
Me iba a la cama con la fe dei que ganó, me despertaba con la paz y reafirmamos la lección que e! tiempo da:
dei que aprendió que lo importante en esta vida es e! tratar, que lo que cuando h ay vida siempre hay posibilidad.
que cuesta es lo que no voy a olvidar. La lucha sigue y sobrevive como nosotros,
Crecí luchando, como los otros, los que crecieron como yo, de como yo,
humilde cuna, con su fortuna hecha de suenos, como yo, como yo. y en otros barrios hay otros ninos como nosotros,
como yo,
Mi adolescencia no fue fácil de llevar: como yo.
como tratar de atar un zapato ai caminar.
Nuestra inocencia retrocede ai comprender que, en la vida real, Como nosotros.
la injusticia puede golear a la verdad.
Muere família, se nos va e! primer amor, (Extraido de vestibular da UFPB 2010 - PSS2.
se confunde lo que una vez se afirmó; http://letras. mus.br/ru ben·blades/415871/)

130 I GE IDIOMAS 2015


EXERCfCIOS - UFPB 2010 (adaptado)

L En la primera estrofa de la canción, los verbos Llévame, RESPOSTAS


Devuélveme y Regrésame se usan para dirigirse a (ai) L Tiempo. Na primeira estrofe, o autor do texto se dirige a o tempo,
a) Chino. usa verbos no imperativo para pedir ao tempo as coisas que fizeram
b) La abuela. parte do seu passado. Resposta: c
c) Tiempo.
d) Los muchachos. 2. Proximidad. Como o pronome "tú" indica o discurso informal, o
e) Perro. trecho indica que se revela a proximidade/informalidade entre o
autor e seu interlocutor. Resposta: e
2. E n el contexto del siguiente verso: "y nos tratamos aún
de tú, y n o de usted,", el uso de la forma de tratami ento tú J. VFVFV. De acordo com as informações do texto, temos: VERDA­
revela: DEIRA. "Se expresa nostalgia de tiempos pasados." Esta alternativa
a) Distancia. é comprovada pela primeira parte da letra, uma vez que os verbos
b) Formalidad. no imperativo evocam ao tempo e as lembranças do a utor. FALSA.
c) Respeto. "Se manifiesta rabia tras toda una vida de lucha infructuosa." O
d) Rudeza. au tor não mostra raiva em consequência da vida que teve; ao con­
e) Proxi m idad. trário, está agradecido e orgulhoso. VERDADEIRA. "Se reconoce que
las viejas amistades son diffciles de olvidar." Essa afirmação é jus­
3- Teniendo en cuenta la l etra de la canción, identifiqu e las tificada por "Y en la curva de los anos me encon tré con los mucha­
sentencias correctas (V o F): chos con los que solía jugar". FALSA. "Se demuestra complacencia
a) ( ) Se ex presa nostalgia de tiem pos pasados. ante e/ triunfo de la injusticia." Sobre o personagem, é informado
b) ( ) Se manifiesta rabia tras toda una vida de lucha infruc­ que "mi adolescencia no fue fácil de /levar: "como tratar de atar un
tuosa. zapato a/ caminar". Isso indica que não há complacência, mas
c) ( ) Se reconoce que las viejas amistades son d iflciles de vontade de vencer. VERDADEIRA. "Se muestra una actitud positiva
olvidar. ante las dificultades de la vida." Afirma·se que "La lucha sigue y
d) ( ) Se demuestra complacencia ante el triunfo de la injusti­ sobrevive como nosotros, como yo". Isto é, as dificuldades da vida
cia. são consideradas algo corriqueiro.
e) ( ) Se m uestra u na actitud positiva ante las d ificultades
de la vida. �t.FVFVV. Temos, segundo o texto: FALSA. "Se anora a los seres queridos."
o texto não trata sobre o assunto da pena, ausência ou privação no
4- Teniendo en cuenta lo que la canción dice sobre la infancia, que se refere à lembrança dos entes queridos. VERDADEIRA. "Se vive
identifique las sentencias q u e caracterizan esta fase de la en un mundo mágico a través de/ juego." Tal alternativa pode ser
vida (V o F): justificada pela seguinte passagem: "en nuestros viajes de ida y
a) ( ) Se af'lora a los seres queridos. vuelta a los luceros / fuimos pira tas, saltimbanquis y vaqueros".
b) ( ) Se vive en un m u nd o mágico a través del j uego. FALSA. "Se pierde la inocencia." Na canção, o autor mostra que a
c) ( ) Se pierde la i nocencia. inocência continua, com a repetição: "los que crecieron como yo, I
d) ( ) Se percibe desproporcionadamente el espacio. de humilde cu na, con su fortuna hecha de suenos, 1 como yo, como
e) ( ) Se viven experiencias d iffc i les de olvidar. yo". VERDADEIRA. "Se percibe desproporcionadamente e/ espacio."
o texto informa: "Cuando era nino mi barrio era un continente". Isso
s. No e n unciado "Nuestra pobre riqu eza n u nca conguistó el significa a desproporcionalidade entre o pequeno garoto e a imensi·
d i ne ro, pero en las casas n unca se rindió el "yo puedo", os dão que o bairro significava para ele. VERDADEIRA. "Se viven expe­
verbos sublinhados est:l.o no perfecto simple do indicativo. riencias difíciles de olvidar." O texto diz "En cada esquina una memo­
Em q u e alternativa há um verbo conj u gado de forma i n cor­ ria inolvidable". Essa passagem justifica a assertiva acima.
reta nesse tem po verbal?
a) com pró, advirtió, d u rm ió. s. dice, respondió, m urió. Na terceira pessoa do singular do per­
b) respondió, mid ió, habló. fecto simpie, a forma correta do verbo "decir" é "dijo ". "Dice" é a
c) and uvo, estableció, q u iso. terceira pessoa do singular desse verbo no presente do indicativo.
d) dice, respondió, m urió. Resposta: d
e) pidió, estuvo, comió.

GE IDIOMAS201S 1 131
DICAS

Saiba como conjugar os verbos nesta condição

Em espanhol, o pretérito im perfecto e o pl uscuamperfecto Verbos i rregu lares: são formados a partir da 3" pessoa
del s u bj u n tivo conferem aos enunciados a noção tem poral do plural do perfecto simple de indicativo. Apresentam as
de passado. São empregados em orações su bordi nadas, con­ mesmas desinências dos verbos regulares. Por exem plo:
d icionais e concessivas, e também em orações desiderativas
(que expressam u m desejo): Ir/Ser perfecto simple (ellostellas)
= Tener perfecto simple (ellos/ellas)
=

fueron tuvieron
» Si dijera la verdad, se librarfa de la culpa. (Se d issesse a
fuese/fuera tuviese/tuviera
verdade, se livraria da cul pa.) (condicional)
» Si hubiera venido a la fies ta, no lo dejarfan entrar. (Se tivesse fueseS/fueras tuvieseS/tuvieras
vindo à festa, não o deixariam entrar.) (condicional)
» Aunque dijera la verdad, no se librarfa de la culpa. (Mes­ fuese/fuera tuvieseltuviera
mo q u e d i ssesse a verdade, não se livraria da cul pa.)
(concessiva) fuésemoS/fuéramos tuviésemosltuviéramos

» Aunque hubiera dicho la verdad, no se habrfa librado de


fueseis/fuerais tuvieseisltuvierais
la culpa. (Mesmo q u e tivesse d ito a verdade, não teria
se l ivrado da c u l pa.) (concessiva) fuesenlfueran tuviesen/tuvieran
» Ojalá hubiera venido a mi casa ayer. (Que bom seria se
tivesse vindo a minha casa ontem.) (desiderativa)
Já o pretérito pluscuam perfecto é formado pelo verbo ha­
Vamos c o n h e ce r agora a c o n j u gação do p retérito i m­ ber no pretérito imperfecto de I subjuntivo, mais o particfpio
perfecto. d o verbo principal. Por exemplo:
Verbos regulares: ad m item duas desinências d iferentes,
q u e servem para conjugar verbos das três termi nações.

hubiera/ hubiese visto


Estudiar comer/vivir

estudiaselestudiara hubieras I hubieses visto


comieselcomiera · vivieselviviera

estudiaseS/estudiaras comieseslcomieras - vivieseslvivieras hubiera 1 hubiese visto

estudiaselestudiara comieselcomiera · viviese/Viviera hubiéramoS/ hubiésemos visto

hubierais 1 hubieseis visto


estudiásemoS/ estudiáramos comiésemOS/comiéramos - viviésemoS/Viviéramos

hubieran 1 hubiesen visto


estudiaseislestudiarais comieseislcomierais - vivieseis/Vivierais

estudiasen/estudiaran comiesenlcomieran - viviesenlvivieran

1321 GE IDIOMAS 2015


DICAS R EG E N CIA » É PAREC I DO, MAS N ÃO É IG UAL .

Fique atento: algumas preposições são


usadas de maneira diferente do português

E m espan h o l, as preposiçõ e s são: a, an te, bajo, con, É importante também conhecer algumas locuções prepositivas
contra, de, desde, durante, en, entre, excepto, hacia, hasta, e o seu sign ificado:
incluso, mediante, para, por, pro, salvo, según, sin, sobre, » A cambio de � e m troca de.

trás. Vejamos: » A las oril/as de -7 às margens de.

» Alrededor de -7 ao redor de, por volta de.

» A preposição "a' é obrigatória antes d o objeto d i reto d e » En cuanto a -? com respeito a.

pessoa: Vi a }aime e n la cal/e. » Lejos de --t longe de.

» A. preposição ''con" pode equ ivaler a conju nção aunque » Cerca de perto de.
(apesar de): Con ser tan testarudo, te ha l/amado. » Pese a -? apesar de.

» As preposições "a" e "de" formam as d uas únicas contrações

de preposição e artigo do espan hol; a + e/ = ai e de + e/ = dei.


As demais preposições n u nca se contraem com os artigos "Zapatero a tus zapatosll
nem com os demonstrativos. Mais uma lista de expressões idiomáticas importantes
» A preposição "en" í n d ica meio de transporte: Voy en coche

a/ trabajo. 11 Saber a ... � Ter o sabor de ...


» "Hasta" e "hacia'' têm sign ificados d iferentes: hacia = tempo 11 Saber a ciencia cierta � Estar certo de algo.
ou d i reção aproximada e h�sta = tém i n o ou l i m ite. 11 Sacar a la luz 7 Tornar algo público.
.
» "Por" pode significar e m b usca de: Vengo por e/ dinero. 11 Sacar de quicio � Exasperar, irritar, atormentar, incomodar.
11 Sacar las unas � Mostrar as garras, demonstrar agressividade.
Algu n s verbos em espanhol regem preposições d i ferentes 11 Sacar los trapos sucios � jogar na cara, dizer a alguém algo de
do português: que se envergonha.
» Acercarse a � aproximar-se de. 11 Serde/gran mundo � Ser dojet set, da aristocracia.
» Acostumbrarse a � acostumar-se com. 11 Ser duro de oído 7 Ser (meio) surdo.
» Admirarse de � ad m i rar-se com. 11 Ser e/ cuento de nunca acabar � Ser uma história sem fim.
» Alarmarse de/por -? alarmar-se com. 11 Ser e/ mismo demonio -7 Ser um demônio, ser muito mau.
» Alegrarse de � ficar alegre com. 11 ;Tanto da! � Tanto faz, dá no mesmo.
» Aterrorizarse por -? aterrorizar-se com. 11 Tanto va e/ cántaro a/ agua hasta que un día se rompe � Agua
» Caerse a -? cair em. mole em pedra dura, tanto bate até que fura.
» Enamorarse de � apaixonar-se por. 11 Tener aga//as-7 Ser corajoso.
» Escandalizarse de � escandalizar-se com. 11 Tener envidia � Ter inveja.
» Estar en contra de alguien/de algo � ser contra, ser 11 Tener las manos largas -7 Ter tendência para agredir.
contrário a algo o u a alguém. 11 Tener mala feche -7 Ter caráter difícil, ser mal·humorado.
» Parecerse a -? parecer-se com. 11 Tener/poner cara de circunstancias � Assumir a expressão
» Participar en � pa r t i c i par d e. adequada a uma situação.
» Preocuparse por -? preocu par-se com. 11 Tener un buenjmal perder � Ser um bom/mau perdedor.
» Ser adicto a -? ser viciado em. 11 Tener una mala cabeza � Ser distraído,esquecer as coisas com
» Ser responsable de ser responsável por. facilidade.
» Tardar en � demorar para. 11 Tomar e/ pelo � Tirar um sarro, zombar.
» Tener miedo a -+ ter medo de. 11 Venir a/ pelo � Chegar em boa hora, ser muito propício.
11 Zapatero a tus zapatos -7 Cada macaco no seu galho.

GE IDIOMAS 2015 1 133


I NT E R P R ETAÇÃO D E TEXTO

El actor activo del cambio cultural


No cerrar los ojos ante el prójimo e s tan s ólo el inicio d e u n a larga jornada por la solidaridad

Ali SE REQUIERE, urgente,


un cambio cultural que acelere la
destrucción y el aniquilamiento
de todas aquellas barreras,
físicas o mentales, que impiden
la integración

esde chicos no aceptamos ai diferente, en todos los sentidos, existen físicamente porque mentalmente no estamos integrando a las

D mucho menos a aquel que visiblemente lo es. Esa crueldad,


llena de inocencia, se transforma con e! tiempo en indiferencia.
Olvidamos que ellos existen, tratamos de segregarias, no les damos
personas discapacitadas. De lo contrario, e! constructor haría las puertas
más anchas o diseíi.aría un bano para el ingreso en silla de ruedas.
Afortunadamente, esto ya existe en muchos lugares públicos
oportunidad, construímos pensando en nosotros y nos llevamos el y en los últimos tiempos hemos visto la proliferación de rampas,
mundo por delante. semáforos con sorridos y otros elementos que nos permiten
De tanto en tanto, imágenes terriblemente exploradas por los medios, visualizar con optimismo e] futuro. Falta, sin embargo, apurar e]
nos obligan a ser solidarios y salimos corriendo a depositar dinero en proceso que corre por dentro nuestro. Este debe empezar desde
una cuenta bancaria o Ie damos una moneda en e! semáforo a un hombre niíi.os y los colegíos deben integrar a niíi.os con discapacidad en sus
en silla de ruedas. Pareciera que la misión está cumplida y hasta nos aulas para que, naturalmente y dentro dei proceso de socialización,
sentimos satisfechos. iEso es todo lo que podemos hacer? aprendan a convivir con e] otro, aceptándolo con sus semejanzas
La respuesta es simple: no. Pero iqué se puede hacer? Salvo meter y diferencias, viviendo las dificultades diarias y derribando
la mano ai bolsillo, pareciera que todo lo que tiene que ver con la conjuntamente las barreras que !e impiden avanzar en la sociedad.
disca.pacidad no nos concierne. Y no es así. Se requiere, urgente, un Miremos a nuestro alrededor permanentemente y detengámonos a
cambio cultural que acelere la destrucción y e! aniquilamiento de pensar cómo haríamos para ir de nuestra casa ai trabajo, o ai fútbol, o
todas aquellas barreras, físicas o mentales, que impiden la integración. a una obra de teatro, si no tu viéramos la fortuna de no usar una silla
Un periodista discapacitado, como ya ocurrió una vez, no podría de ruedas o muletas, ver, oír u otra dificultad motora.
trabajar en mi oficina. Los cinco bafios que existen en un hermoso piso Hagamos el ejercicio y comentémoslo con otro, con aquel que no
de uno de los edificios más modernos de Santiago, no !e permitirían tiene tiempo para pensar en los demás, ayudemos a abrir las mentes
entrar a hacer sus necesidades. EI edificio, sin embargo, cuenta con para que esta barrera cultural, la más difícil de eliminar, se quede
una rampa y ascensores anchos. Pero e! obstáculo es el bano. definitivamente en el pasado.
En otros lugares las barreras son las escaleras, el medio de
transporte o la mentalidad de los empleadores. Estos obstáculos (Extraído de Uni rio, 2009. Francisco Martorell, Editor General Dei diário MTG, Chile)

134 1 GE IDIOMAS 2015


EXERCfCIOS - U N I RIO 2009 (adaptado)

L De acuerdo con el texto de la página anterior, podemos c) Prod u ce una crítica ai estilo d e las ropas h is pán icas en el
afirmar que: i nvierno septentrional.
a) c u m pli mos n uestra misión d e ayudar a los d iscapacitados d) Resalta los cam bios d e actitud de l a gente ai e m borrachar­
cuando depositamos di nero en una cuenta bancaria o damos se con los Reyes Magos.
l i m osna a algu ien en si lia de ruedas en e l semáforo.
b) d e be m o s ayudar a rom p e r las barrreras c u l t u rales q u e s. En el enunciado "Salvo meter la mano ai bolsillo" el verbo
i m piden a l o s d i scapacitados d e avanzar e n l a sociedad. meter pide el uso de la preposición a. La alternativa en la
c) los obstác u los que i m p i d e n a los d iscapacitados d e avan­ que la regencia preposicional está correcta es:
zar en la sociedad existen m ental m ente en l o s propios a) me alegré con tu l l egada.
d i scapacitados. b) tardaron m u c h o para resp o n d e rme.
d) debemos e n sefíar a los d i scapacitados a i n tegrarse a la c) f u i ai hospital e n taxi.
soci edad desde n i fíos. d) las c h i cas se enamoraron por los h o m b res mad u ros.
e) los n i fíos t i e n e n m i e d o d e d o r m i r solos.
z. "Desde chicos n o aceptamos ai d iferente, e n todos los
sentidos, mucho menos a aq uel que visiblemente lo es."
(primer párrafo). En ese fragmento el enu nciador empieza RESPOSTAS
su texto planteando el siguiente tema: 1. Debemos ayudar a romper las barrreras culturales que impiden
a) s e n s i b i l idad. a los discapacitados de avanzar en la sociedad. A resposta a essa
b) n i fí ez. q uestão está no parágra fo "Miremos a n u estro alrededor perma·
c) visi b i l i dad. nentemente y detengámonos a pensar cómo h a ríamos para ir de
d) e d u cación. n uestra casa ai trabajo, o ai fútbol o a una obra de teatro, si n o
e) alteridad. tuviéramos l a fortuna d e n o usar u n a si/la d e ruedas o muletas,
ver, oír u o tra dificultad motora". Resposta: b
3· En el trecho "Desde ch icos no aceptamos ai d iferente, en
todos los sentidos, mucho menos a aq uel que visi blemente 2. Alteridad. Para responder a essa pergun ta, é preciso e n tender
lo es. Esa crueldad, llena de i nocencia, se transforma con el o significado da pala vra "alteridad": capacidade d e enxerga r as
tiempo en ind iferencia. Olvidamos q u e ellos existen, trata­ coisas do ponto de vista do o utro, descobrir a existência do outro
mos de segregar� no les damos oportun idad( ... )", las pa­ a partir de si mesmo. t justamente disso q ue trata o trecho citado,
labras subrayadas su bstituyen respectivamente: da nossa incapacidade de pensar e ver o m undo a partir da pers·
a) c h i cos, c h i cos. pectiva alheia. Resposta: e
b) c h i cos, ai d iferente.
c) e I l os, ellos. 3. E/los, e/los. Nos dois casos, os p ro n omes retomam o mesmo
d) ai d i ferente, l a i n d iferencia. pronome pessoal, porém com funções gramaticais distintas. No
e) sentidos, la i n d ifere ncia. caso d o pronome á to n o "los': ta n t o "e!los" c o m o esse pronome
têm a função sintática d e comple m e n to direto e m relação ao
verbo segregar. No caso de "les", o p ro n o m e á tono e o p ro n o m e
4. A través d e I c o m u n i cado t ransmitido por el lenguaje "e/los" t êm a fu nção sin tá tica de complemento indireto em re·
verbal y n o verbal, se observa que la vi neta: /ação ao verbo "dar". Resposta: c

4. /roniza la confusión sobre las informaciones de/ uso de cadenas


en los coches. Essa q u estão exige que se conheça uma info rmação
cultural sobre a Espa nha. A tira é de Barcelona, onde é n ecessário
colocar correntes nas rodas dos carros para q u e eles possam cir­
cular na neve. A resposta do personagem à pergunta da mulher
joga tam bém com essa informação. Ele usa a pala vra "nievo",
clara referência à palavra "nieve". O homem ironiza ao responder
à mulher. Resposta: b

ORTIFUS. Ma.gaz.ine. Barcelona, 31 dez. 2006, p. 84. s. Fui ai hospital en taxi. As demais orações apresentam uso in cor·
reto da preposição. Para q u e estivessem corretas a preposição
seria: a) "Me alegré de tu 1/egada': b) "Ta rdaron mucho en respon­
a) H ac e refe rencia ai g u sto e u ro peo p o r la fantasia y l o s derme", d) "Las chicas se enamoraron de los h o m b res maduros':
d isfraces sadomasoq u i stas. e) "Los nií'ios tienen miedo a dormir solos". Resposta: c
b) l ro n iza l a confusión sobre las i nformaciones d e i uso de
cadenas e n los coches.

GE IDIOMAS 2015 1 135


DICAS ACE NTUAÇÃO >' REGRAS DE U SO DO AC ENTO G RÁ FICO

É difícil (e importante) não se confundir com o


português ao acentuar palavras em espanhol

Todas as palavras c o m mais de u ma sílaba têm u m a sílaba exceções a essa regra, ou seja, a l g u n s m o n ossílabos rece bem
tôn i ca, aq uela sobre a qual recai o acento da voz. Essas palavras a ti/de como um si nal d i sti ntivo q u a n d o, a u ma mesma forma
p o d e m o u não ser acentuadas graficamente. gráfi ca, correspondem duas categorias gramaticais d i ferentes.
Em e s p an h o l, as palavras se classificam em q uatro gru­ Esse acento gráfico d istintivo é chamado de ti/de diacrítica.
pos seg u n d o a sua s ílaba tôni ca: agudas (oxíton as), 1/anas Os m o n ossílabos q u e obedecem a essa regra são:
(paroxíto n as), esdrúju/as (proparoxíto nas) e sobresdrújulas
(anteproparoxítonas, q u e não existem e m p o rtugu ês). Nas
Diacríticos
palavras agudas, o acento tôn ico cai na ú l t i m a sílaba: razón,
ordenador, facultad. Nas llanas, o acento tônico cai na pen últi­ é/= pronome pessoal dé = verbo
e/ = artigo determinado de preposição
m a sílaba: acento, fácil, grupo. Nas palavras esdrújulas, cai na
=

antepe n ú l t i m a sílaba: sílaba, área, esdrúju/a. E, finalmente, nas


tú pronome pessoal
=
sí = advérbio de afirmação/
palavras sobresdrúju/as, o acento tônico cai na sflaba anterior pronome reflexivo
tu = possessivo si = conjunção
à antepe n ú l ti m a: dígamelo, en tregándose/o .
Você deve ter notado nos exe mplos anteriores q u e, e n tre as
té = substantivo más = advérbio
palavras agudas e as lianas, há palavras q u e são acentuadas
te = pronome mas = conjunção
grafi came nte e outras q ue não. Por o u tro lado, n o caso das
esdrújulas e sobresdrúju/as, todas as palavras são acentuadas mí= pronome aún = conjunção
graficamente. Portanto, n o caso das agudas e llanas, é neces­ mi possessivo
= aun = advérbio
sário ver as regras específi cas para dete rm inar q u ando devem
levar acento gráfico (ti/de):
» As palavras agudas (oxíto n as) te r m i nadas em "n", "s" o u
e m vogal s ã o acentuadas graficame nte: corazón, francés, Continuando...
ca fé. Outros casos de acento diacrítico
» As pal avras llanas (paroxítonas) termi nadas em consoantes,
exceto "n" e "s", são acentuadas graficamente: carácter, >> Todos os pronomes interrogativos e exclamativos são
lápiz , difícil . acentuados graficamente: qué, quién, cuál, por qué, dónde etc.
Exemplo: Hay que saber cómo fue escrita la obra para entender
Em e s p a n h o l, as palavras com e n contros vocálicos ( h i ato, cuánta verdad implica la respuesta.
d i tongo e tritongo) obedecem às mesmas regras de acentuação » Qué, cuándo, dónde, cómo, porqué recebem acento gráfico
gráfica q u e acabamos de ver: h uésped (paroxíto na, termi nada quando têm a função de substantivo: Lo hice sin saber ni e/
e m consoante "d", é acentuada graficamente), gente (paroxíto­ porqué. (Fiz isso sem sequer saber o porquê.)
na termi nada em vogal, não é ace ntuada graficamente), área » A conjunção o (ou) entre dois numerais é acentuada para não
(pro paroxítona: todas são ace n t u adas graficamente). A ú n ica ser confundida com o numeral zero: Vinieron 5 ó 6 personas a
exceção a essa regra são as pala,vras nas q u ais as vogais "i" e la reunión.
"u" dos e n contros vocálicos são tôn i cas e, por isso, se separam » Os advérbios terminados em "mente" formados a partir de
das vogais a/e/o q u e as aco m panham. Nesse caso, "i" e "u" têm adjetivos acentuados graficamente mantêm o acento do
de ser acentuadas graficamente: había, ataúd etc. adjetivo: rápido � rápidamente, fácil -+ fácilmente.
E m espanhol, os m o nossílabos geral m e nte não são acen­
tuados graficamente: me, ves, pan, es etc. H á, porém, algumas

,,

136 1 GE IDIOMAS 2015


DICAS H ETEROTÕ N ICOS » G RAFIA IG UAL, MAS . . .

Algumas palavras escritas como em


português têm sílaba tônica diferente

Em espan hol, há palavras com a grafia igual ou semel hante


à do portugu ês, m as q u e têm sílaba tôn ica d i fe re n te. São Verbos modais
chamadas heterotôn i cas. Você se I em bra do português? Pois eles
Vejamos alguns exem plos desse tipo de palavra com a sílaba também existem em espanhol
tônica em negrito:
» alcohol/ alergia/ alguien/ academia/ acrobacia; aristócrata/ Da mesma forma que em português, temos em espanhol uma série de
asfixia verbos que se unem a outro verbo no infinitivo para indicar o modo de
» b igam ia/ b u rocracia/ cerebro/ centígramo ação desse verbo. Chamamos a esses verbos de modais: deber, querer,
» d e m ocracia/ d e m agogia/ d i p l o mac i a/ d í s par/ e l o g i o/ saber, poder e soler (costumar).
e p i de m ia/ e u foria
» fútbol! filántropo/ gaucho/ hemo rragi a/ héroe/ i m béci l/ » Debes estudiar las matemáticas. (dever)
i m par » Quieres estudiar las matemáticas. (desejo)
» l im ite/ magia/ m edíocre/ m i crófono/ míope » Sabes estudiar las matemáticas. (conhecimento)

» n ivel/ n ostalgia/ neu rastenia/ oxigen o/ océano » Puedes estudiar las matemáticas. (possibilidade)

» periferia/ policia/ prototipo/ q u i romancia/ régimen/ reptil ,, Sueles estudiar las matemáticas. (costume)
» sintoma/ siderurgia/ taq u icard ia/ terapia/ tráq uea; vértigo

.olllillll A PALAVRA "FÜTBOL"


tem sílaba tônica distinta de
seu equivalente em português

GE IDIOMAS 2015 1 137


DICAS VE RBOS D E CÂM B I O » ESTRUTU RA I M PORTANTE

Saiba como expressar mudanças


de estado de uma pessoa

Os chamados verbos d e câmbio são m u ito usados e m espa­ >> M udanças c o m ê n fase no resultado: para expressá-las,
nhol, tanto na escrita como oral mente, para falar de mudanças usamos os verbos: quedarse, salirse, estar hecho/a, acabar
de estado de um sujeito. Há m uitos verbos que pertencem a + gerúndio.
essa categoria: aburrirse (ficar chateadot enojarse/enfadarse a. Quedarse: expressa o resu ltado ou conseq uência d e u ma
(ficar bravot disgustarse (ficar desgostoso), a/egrarse (ficar ale­ situação ou acontecimento, algumas vezes i nesperado: "Me
gre) etc. Porém, podemos também expressar mudanças usando quedé boquiabierto cuando supe de la noticia".
u ma estrutura com posta d e verbos como volverse, quedarse, b. Salirse: expressa o resultado final d e u ma m u d ança: "Salí
ponerse etc., acom panhados d e u m s u b stantivo, adjetivo/ curado de/ tratamiento".
partidpio o u advérbio: "Me pregunto por qué una mujer no c. Estar hechoja: expressa o resultado da m udança como u ma
puede /legar a presidente de la nación, por ejemplo". i m p ressão pessoal, s u bj etiva. � m u i to u sado para fazer
Podemos expressar vários ti pos de m u d ança de estado dos criticas, d e m o n strar reprovação ou expressar i ro n ia: "Mi
sujeitos e para cada tipo de m udança em pregaremos u m verbo amiga está hecha una ballena".
d iferente. Vejamos q uais são e o seu sign ificado: d . Acabar + gerú n d i o: expressa u m a m udança n egativa d o
» M u d anças essenciais: para expressá-las, usamos os verbos ponto d e v ista social: M i pobre padre acabó vendiendo
hacerse, convertirse, volverse, /legar a (ser). caramelos en la puerta de un cine.
a. Hacerse: expressa u ma m udança vo l u ntária d e u m sujeito
a n i m a d o (geral m ente pessoa) q u e tam bé m é revers lvel: » M udanças acid entais: para expressá-las usamos o verbo
"Se ha hecho vegetariano". Serve tam bém para expressar ponerse. Com esse verbo, falamos de m udanças q u e se
m u d an ças f i s i o l óg i cas, n at u rais o u para dar a i d e i a d e p rod uzem rap i d a m e n te, d e m a n e i ra i nstantânea e q u e
s u pe ração. parecem s e r pouco d u radouras, m u d anças d e estado, d e
b. Volverse: expressa uma m u dança involu ntária, súbita, rápi· saúde, d e cor, d e aspecto ffsico ou d e com portamento: "Me
da e m u i tas vezes i rreverslvel, c o m u m ente relacionad a ao pongo mejor cuando te encuentro". Com ponerse, podemos
caráter: "Me h e vue/to más lento con la edad". A expressão construi r tam b é m o utras expressões:
volverse loco significa ficar louco, perder o j u izo. � Ponerse co/orado!rojo como un tomate: Ficar verme l h o/
c. Llegar a (ser): expressa uma mudança social positiva, vol u n­ verme l h o como u m pimentão.
tária, frequentemente associada à profissão: "Mi padre ha � Ponerse verde de envldia: Ficar verde de i nveja.
/legado a ser un gran médico". Ao usarmos essa expressão, � Ponerse morado: Comer m uito/ficar em pantu rrado.
q ueremos tam bé m enfatizar o esforço d o sujeito o u a len· � Ponerse negro: Ficar verde de raiva.
tidão do processo de m udança. � Ponerse chu/o: Agir c o m e m páfia, com arrogância, com

d. Convertirse: expressa uma m udança i nvoluntária, rápida ou presunção.


inesperada. A m u d ança é s e mpre radi cal e m u ito evidente: � Ponerse como una moto: Ficar exultante.
"Después de la muerte de su hijo, se ha convertido en otra � Ponerse como una sopa: Ficar todo molhado, e nsopado.
persona". Serve tam bé m para i n d i car m u d anças d e estado � Ponerse de acuerdo: Com b i nar, pôr-se de acordo.
flsico: "E/ agua se convierte en hie/o a menos 1 grado". � Ponerse las botas: Comer o u beber m u ito.

138 1 GE IDIOMAS 2015


1.{UFG 2010) EL P U EBLO SOVIÉTICO los habitantes de la región, sus promotores optaron por
En e/ XX Congreso de/ PCUS (Partido Comunista de la capacitar a los empleados en tareas hoteleras. (...)
Unión Soviética), Nikita Kruchov presentó un informe (Revista Avianca, edlción 236, Septiembre 1998)
denunciando las via/aciones de Stalin a la legalidad socia·
lista. A decir de un conocido doctor en psiquiatria, e/ texto 3-(Ufla 2009) lCuál es el significado correcto de "finca" (ref. 1)
resultaba ser la descripción de los sintomas de un enfermo en e l texto?
paranoico con mania persecutoria, que se acentuaban a a) p ropiedad ru ral.
medida que pasaba e/ tiempo. Algo que corroboran los b) hotel ru rai.
hechos hoy conocidos. c) caso nas rurales.
(AZCARATE DIZ, Luis de. Mi héroe. La Aventura de la Historia. Madri, jun. 2008. p. 130. d) restaurantes e n el campo.
[Adaptado])
4. (Ufla 2009) Según el texto, lCuál es la diferencia entre los
En la exposición hecha por Kruchov se: modelos de turismo rural de Espaflay Colombia con respecto
a) presentaba a los m i l itantes l i bertarias como sujetos ater· a las tareas?
rados a la l ey. a) en Colombia, las personas no trabajan en el cam po.
b) s u b rayaba la violación q u e el co m u n is m o generaba del b) e n Espana, pred o m ina e l trabajo d e los cafeteros.
orden popu lar. c) los espanoles viven e n e l ocio agrícola.
c) d e n u nciaban las m e l las e n la salud m e n tal d e l p u e b l o d ) en Espana, las p e rsonas n o actúan d i rectamente e n el
soviético. proceso agrícola.
d) evidenciaba e l f u n cionam iento paranoico q u e i ba a ca­
racterizar a i PCUS. TEXTO PARA AS PROXIMAS DUAS Q U ESTOES:
e) c ri t i caba l o q ue h a b ía hecho Sta l i n d es d e s u d e l i ctiva
ad m i n istració n . LOS DIAS DE LA SOMBRA
E/ tiempo no tiene una sino sus muchas ruedas.
2. (UFG 2010) TI ERRAS SILVESTRES, DON DE "LA H U ELLA H U MA· Una rueda para las criaturas de corazón lento,
NA" NO PERMANECE y otra para las de corazón apresurado.
E/ término ti erras silvestres se refiere, en esencia, a los espa· Ruedas para las criaturas que envejecen lentamente,
cios na turales más intactos e indómitos, nevados o tórridos, Ruedas para las que se hacen viejas con e/ dia.
esos últimos lugares que verdaderamente conservan su estado (Lillana Bodoc. http://www.educared.org.ar/imaginaria/OB/4/sombra.htm}
natural y que e/ ser humano ni controla ni ha desarrollado
mediante carreteras u otra infraestructura industrial. s. (UFF 2009) EI poema de la argentina Liliana Bodoc:
(ROJO, Jaime. Magazine Reportaje. Ronda. Madri, out. 2009. p. 42. [Adaptado]) a) aclara la necesidad de sacar provecho del tiempo que pasa;
b) evi d e n c i a q u e el tiem po no pasa velozmente;
" La h uella h u m ana" d esaparece en las tierras s ilvestres c) s u giere q u e e l tiem po n o es igual para todos;
porq u e la: d) d e m uestra q u e el tiem po perdido ya no vuelve;
a) repercusión de la acci ó n h u mana se l i m ita. e) i nvita a vivi r el m om ento p resente.
b) red d e senderos se h a desmantelado.
c) n i eve ai cuajar la suei e I legar a c u brir. 6. (UFF 2009) La estructura adversativa de I primer verso equivale
d) i n d u stria logró garantizar la protección d e el las. a u n a de las siguientes versiones:
e) gente q u e las habita es naturalista. a) el tiempo no tiene sólo una rueda, en realidad tiene muchas.
b) e l tiem po tiene a veces u n a rueda, otras veces m uc h as.
TEXTO PARA AS PROXIMAS DUAS Q UESTOES: c) el tiem po no tiene ni una ni m u c has ruedas.
d) e l tiem po, a u n q u e n o lo parezca, só lo tiene una rueda.
( ..) E/ modelo de agroturismo, como se denomina esta
. e) e l tiempo n o tiene ruedas.
modalidad turistica que conecta a/ viajero con e/ turismo
rural, no es nuevo en e/ mundo. Los espalioles, pioneros en TEXTO PARA AS PROXIMAS TRtS Q U EST0ES:
ma teria de/ ocio, manejan un sistema muy parecido pero allf
las r'' fincas no son grandes casonas y las personas que las 25 PAISES EJ ERCEN CENSURA EN INTERNET
atienden no están comprometidas con e/ proceso y desarrollo Un estudio de Harvard, Oxford, Cambridge y Toronto concluye
agricola, como si sucede con los cafeteros colombianos. que e/ control gubernamental de la Red está aumentando.
También e/ modelo de atención a/ huésped y e/ manejo de las Más de mil millones de personas usan cada dia Internet
haciendas funciona de modo diferente. En Colombia, aunque para comunicarse y buscar información. Decenas de Gobier·
la vocación de buenos anfitriones les viene en la sangre a nos buscan, mientras, la manera de controlarles. Hoy se hace

GE IDIOMAS 2015 1 139


ESPANHOL SIMULADO

público un estudio de la prestigiosa organización OpenNet, d) a d i fusão de ide ias de terroristas e defensores dos d i re i tos
integrada por las universidades de Oxford, Cambrigde, h u manos na i nternet é controlada em Omã.
Harvard y Toronto. En é/ se conc/uye que 25 países - entre e) Myan mar, China e I rã o c u pam as três p r i m e i ras posições
e/los lrán, Chinay Corea de/ 5ur - ejercen la censura de webs q u anto à c e n s u ra d e conteúdos sociais.
con contenidos políticos o soeiales peligrosos, e impiden e/
acceso a aplicaciones como You Tube o Google Maps. Además, 9. (UEL 2009) Com base n o texto, considere as afirmativas a
sus métodos de censura están sofisticándose en muchos seg u i r:
casos gracias a la colaboración de empresas occidentales. I. Myanmar, Coreia do Norte, I rã, Arábia Saud ita e C h i na são al­
Los 25 países que, según e/ estudio, ejercen censura en la g u n s dos 25 países q ue cen s u ram conteúdos da i nternet.
Red no lo hacen de la misma manera, ni en la mesma medi­ 11. Um vídeo ofensivo ao p r i m e i ro p residente d a Repú b l i ca
da. Corea de/ Sur, por ejemplo, só/o impide e/ acceso a las da Tu rq u ia, M ustafá Kemal Ataturk, c i rc u l o u no YouTube.
informaciones relacionadas con Corea de/ Norte. Hay quien 1 1 1 . Países como Corei a do Sul e Turqu ia, apontados no estudo,
prefiere ejercer una censura puntual, comoTurquía, que im­ exercem a censu ra na i nternet de formas d i ferentes.
pidió e/ acceso temporal de sus ciudadanos a You Tube por IV. E m presas do Ocidente têm co n t r i b u ído para sofisticação
un vídeo que "ofendía" la memoria de/ primer presidente de de técn icas de c e n s u ra de conteúdos da i nternet.
la República, Mustafá Kemal Ataturk. E/ informe no incluye
Corea de/ Norte ni Cuba, no porque no se ejerza la censura Assinale a alternativa CORRETA.
sino porque los investigadores aseguran que no podrían a) somente as afi rmativas I e 1 1 são corretas.
garantizar la seguridad de sus fuentes en e/ país. b) somente as afi rmativas I e IV são co rretas.
Los autores de/ informe han detectado tres tipos de censu­ c) somente as afi rmativas 1 1 1 e IV são corretas.
ra. La más común es la política, ejercida para evitar la difusión d) somente as afi rmativas I, 11 e 1 1 1 são corretas.
de ideas por parte de partidos de la oposición, defensores e) somente as afi rmativas 1 1, 1 1 1 e IV são corretas.
de los derechos humanos o disidentes. E/ peor país para la
censura política es Myanmar (antiga Birmania), seguido de TEXTO PARA AS PRÓXIMAS CI NCO Q U ESTÕES:
China e lrán. E/ segundo tipo de censura es la social, que
impide acceder a contenidos vinculados con derechos de Un disfraz inédito tomá /as calles de Río de janeiro e/ pa­
grupos como m ujeres y homosexua/es. lrán, Omán y Arabia sado carnaval. Miles de nií'ios sonrientes, vestidos de negro,
Saudí encabezan la lista negra. Un tercer tipo de censura lucían en sus camisetas un siniestro escudo: una calavera
se relaciona con la "seguridad nacional'� es decir se impide atravesada por un machete y dos pistolas. Los populares
e/ acceso a webs y sítios de notícias de grupos insurgentes trajes de Superman o de princesa cogían polvo en las tiendas.
o terroristas. Myanm{lr, China e lrán repiten en los tres Este ano la moda fue sambar vestido como e/ capitán Nasci­
primeros puestos. mento, y repetir sus sobrecogedoras frases. Nascimento e/
(Adaptado. FERNANDEZ DE LIS, P. El País: Sociedad. lB/5/2007, n. 10.933. p. 32) héroe torturador que protagoniza e/ filme brasileno "Tropa
de élite", estrenado hace meses en su país de origen y hace
7.(UEL 2009) Com base no texto, é CORRETO afirmar: só/o diez días en Espana. Un éxito fulgurante que ganó e/ Oso
a) a o rganização O p e n N et real izou um estudo so b re países de Oro de/ Festival de Berlín y cuyos espectadores espafio/es
q u e p raticam a c e n s u ra d e conteúdos p o l íticos ou sociais ya se cuentan por decenas de miles.
da i ntern et. c:Cómo es posib/e que estas via/aciones sistemáticas de
b) cerca de 2 5 m i l h ões d e pessoas u t i l izam d i ariam e n te a los derechos humanos hayan sido mimetizados de manera
i nternet para controlar e censu rar conteúdos da rede. festiva, si no con admiración, por buena parte de la pob/a­
c) a orga n i zação O p e n N et é i n tegrad a por u n iversi dades de ción brasilena? La película trascendió e/ ámbito meramente
2 5 países e tem sedes e m Oxford, Cam b r i d ge, H arvard e cinematográfico desde antes de su estreno. La fuerte po­
Toronto. lémica que rodeaba ai filme se intensificá con su estreno.
d) a c e n s u ra d e 'webs' c o m c o n te ú d os sociais e p o l íticos Mientras parte de/ público se estremecía en la butaca por
é p rática c o m u m d e d ezen as d e gover n o s associados à la crudeza de las escenas, otros espectadores ap/audían. E/
Open N et. director de Tropa de é lite cree que la película canalizá e/
e) a o rgan ização Open Net e países como I rã, C h i n a e Core i a sentimiento de impotencia frente a/ crimen. "Los brasi/enos
do Su I real izaram u m estudo sobre controle governamental están hasta la coronilla de sufrir la violencia", explica por
dos conteúdos d a i n te r n et. teléfono a e/pais.com.
(www.elpais.com/articulotculturat28.7.20o8)
8. (UEL 2009) De acordo com o texto, é CORRETO afirmar:
a) h á três m o d a l i dades d e c e n s u ra n a i nternet: a política, a 10. (UFPR 2009) Según el texto, sobre el último carnaval de Rfo
social e a relacio nada à s e g u rança n acional. de janeiro, es CORRECTO afirmar:
b) e m Myan mar, a c e n s u ra mais com u m e mais con trolada a) los estableci m i entos comerc iales tuviero n q u e s u bastar
na i nternet é a d e conteúdos p o l íticos e sociais. sus popu lares trajes d e S u perman y de p r i n cesa.
c) I rã, Omã e Arábi a Saud ita o c u pam as p r i m e i ras posi ções b) las. p rendas d e S u perman y d e p r i ncesa se m ezclaron por
q uanto à c e n s u ra pol ítica n a i n ternet. igual con e l traje dei capitán Nasc i m e n to.

140 I G E IDIOMAS 2015


c) los afi c ionad os a los carnavales suelen com p rar a cada ano TEXTO PARA AS PRÓXIMAS DUAS Q UESTOES:
más d i sfraces de S u perman y d e p r i n cesa e n las tiendas.
d) los al macenes de p rendas carnavalescas mantuvieron su SI DA
b u e n a racha con los d isfraces d e Superman y d e p r i n cesa E/ SIDA (síndrome de inmunodeficiencia adquirida) es la
e n 2008. quinta causa importante de muerte en las personas entre 25
e) e n el carnaval d e este ano, l as ti endas no vend ieron casi y 44 afios de edad en los Estados Unidos, pero en l995 ocupa­
n i ng ú n traje d e S u perman ni d e p r i n cesa. ba e/ número uno. Alrededor de 25 millones de personas en
todo e/ mundo han m uerto a causa de esta infección desde
n (UFPR 2009) Según el texto, es CORRECTO afirmar q ue: e/ comienzo de la epidemia _V- en 2006, había alrededor de
a) las calles de Rio de janeiro se l le naron de c h i q u i l los con d is­ 40 millones de personas a/rededor de/ mundo viviendo con
fraces d e i h é roe tort u rador de la pel ícu l a Tropa de élite. VIH/SIDA. E/ virus de la inmunodeficiencia humana {VIH)
b) i rrecon o c i b l es trajes de l uces ocu paron por c o m pl eto l as causa e/ SI DA. Este vírus ataca a/ sistema inmunitario y deja
cal les de Río d e j a n e i ro d u rante los d ias de carnaval. a/ organismo vulnerable a una gran variedad de infecciones
c) U na i n d u m e ntaria n ueva q u e se puso e l h é roe d e Tropa y cánceres potencialmente mortales.
de élite d u rante el ú l t i m o carnaval l lamó la ate n c i ó n d e Se ha encontrado e/ VIH en saliva, lágrimas, tejido de/
l a g e n t e por l a s c a l l e s d e R í o d e janei ro. sistema nervioso, líquido cefalorraquídeo, sangre, semen,
d) jóvenes de Rio d e j a n e i ro sembraron desconfianza e n la f/ujo vaginal y feche materna. Sin embargo, só/o a través
población cuando e lia se d i o cuenta de los escudos s i n i es­ de la sangre, e/ semen, las secreciones vaginales y la feche
tros q u e l ucían e n sus cami setas. materna se transmite la infección a otras personas.
e) Los críos de Rio de janei ro presu m ían de tener en sus cam i­ La infección por VIH no se propaga por contacto casual
setas las asu stadoras frases dei capi tán Nasc i m ento. como un abrazo, por tocar cosas que han sido tocadas con
an terioridad por una persona infectada con e/ virus, ni
durante la participación en deportes ni por mosquitos. E/
u. (UFPR 2009) EI autor dei texto: SIDA comienza con una infección por VIH. Es posib/e que
a) ai aba a los b ras i l enos q u e han i m i tad o ai p rotagon ista d e las personas infectadas con e/ VIH no presenten sintomas
la p e l íc u l a Tropa d e élite con s u s cam isetas, pues e s e f i l m durante lO afios o más, aunque si pueden transmitir la
h a ten i d o u n gran éxito. infección a otros durante este período asintomático. Entre
b) p o l e m iza sobre e l h e c h o d e q u e l a p o b l a c i ó n b ras i le n a tanto, si la infección no se detecta y no se inicia e/ trata­
reproduce la vesti m enta y l a s frases d e u n pe rso naje q u e miento, e/ sistema inmunitario se debilita gradualmente y
desem pena e l papel d e tortu rado r e n Tropa d e élite. se desarrol/a e/ SI DA. La infección aguda por VIH progresa
c) ha i n te n tado i m pe d i r q u e la gente en Río de j a n e i ro se con e/ tiempo (generalmente unas pocas semanas a meses)
volcase en el p ropósito de p romocionar la película por las a una infección por VIH asintomática (sin sintomas) y lu ego
cal les de Río d e janeiro d u rante e l carnaval. a infección sintomática temprana por VI H. Posteriormente,
e) h a c e n s u rado ai d i rector d e l a pel íc u l a y a s u p rotago­ progresa a SI DA (infección por VIH avanzada con conteo de
n i sta p o r p re s e n tar e s c e n as extr em a m e n te c ru d as y células T por debajo de 200 células/mm3}.
p o l é m i cas. (Adaptado de: http://www.nlm.nih.gov/medlineplus/spanish/ency/article/ooos94-htm, el
e) se u fana d e q u e l a p e l íc u l a Tropa de élite esté e n tre las 24/9/2009)
más exitosas dei m o m en to ..
15- (PUC·PR 2010) EI nú mero de enfermos con SI DA:
13. (UFPR 2009) E n la frase "Los b rasi lenos están HASTA LA a) fue de vei nt i c i nco m i l l ones en todo e l m u n do, e n 1995.
CORON ILLA de sufri r la violencia", la expresión destacada b) fue de cuarenta m i l lones en todo el m u n d o, en 1995.
se p uede sustituir por: c) era d e c uarenta m i l lon es, e n 2006.
a) cansados y hartos. d) se q u i nt u p l i có en los Estados U n i d os.
b) d esechos. e) con edad e n tre vei nt i c i n co y c uarenta y cuatro anos, se
c) v i s i b le m e n te i n conformad os. q u i nt u p l icó d esde 1995.
d) atu rd idos.
e) i m potentes. 16. (PUC-PR 2010) Según el texto, es CORRECTO afirmar
que el vírus VI H :
J4. (UFPR 2009) La frase "Mientras parte del públ ico se estre­ 1. N o s e p ro paga p o r tocar o bj etos d e u s o co m ú n c o n persa­
mecia en la butaca ..." significa: nas i n fectadas.
a) a pesar de q u e a l g u n as p e rs o n as se e n treten la n en l a 11. Se t ra n s m i te a través d e l a sal i va d u rante el beso.
butaca ... 111. P u e d e estar presente s i n m a n i festar n i n g ú n s i n t o m a
b) a u n q u e algu nas person as se abu rrían e n l a butaca ... d u rante m uchos anos.
c) a la vez q u e a l g u n as p e rsonas t e n ía n escalofríos en la IV. Baja las defe n sas y hace vu l n erab l e ai o rga n i s m o .
b u taca ... V. Se tra n s m i te ai b e b é d u rante e l amamantam i e n to.
d) en relación a l a gente q u e sufría e n el s i l l ó n dei teatro ... a) todas las alte rnativas son correctas.
e) al iado d e l a gente q u e renía en e l s i l l ó n d e l a sala ... b) son correctas las alternativas l i, 1 1 1 y V.

GE IDIOMAS 1015 ) 141


SIMULADO

c) só l o es correcta l a a l ternativa I V. a) el riesgo de contraer la e n fermedad d i sm i n uye s i mante·


d) son correctas las alternativas I, 1 1 1, IV y V. n e m os activa la vida social.
e) son correctas las alte rnativas 1 1 1, IV y v. b) l a patoge n i a d e i v i r u s i n d ica m o rtal id ad, p o r lo que es
i n d i cado ce rrar colegi os.
TEXTO PARA AS PRÓXIMAS QUATRO QU ESTÕES: c) l as personas con u na e n fe r m edad l atente d e b e n evitar
l ugares con mucha gen te.
UN VIRUS Q U E PARECE LEVE d) los barbijos deben u t i l izarse s i n tem o r ai r i d íc u l o, pese a
Todos los aflos nos /lega "e/" vírus que proliferó en e/ no ser muy p e l i g rosa.
hemisferio Norte. Este afio, es la influenza tipo A (NlHl), y
es de/ tipo pandémico. 19. (UFMG 201o) A partir de las informaciones que trae el texto
La gripe A es una de las enfermedades respiratorias que acerca de la influenza tipo A, es I NCORRECTO afirmar que:
1/egan en otoflo, cuando e/ frio contribuye a que los ambientes a) e l contagio es m uy fác i l, razón por l a cual l a pan d e m i a ya
estén menos 2aireados y, en consecuencia, todos los vírus es u n hecho, y contra eso hay pocas acc i o n es pos i b l es.
que se transmiten por vía aérea pueden diseminarse más b) l a enfermedad l l ega e n l a estac i ó n fría, c u ando se mantie·
fácilmente. nen los a m b ie n tes s i n c i rcu lación de ai re.
Para contagiar_ e/ vírus influenza A necesita la proximidad c) las personas deben lavar las manos c u a n d o han estad o
de otra persona, a la que coloniza, a través de las secreciones en l u gares con m u c h a g e n te y no p e r m a n e c e r j u n to a
respiratorias, directamente, o de aquel/as que se depositaron enfermos.
en las manos con la tos o e/ estornudo. d) las personas que t i e n e n otra e nferm edad y se contagian
Si e/ contagio es sencillo y la condición es estar cerca de una con este vírus deben hace rse ver y tomar med icación.
persona infectada, la precaución suena simp/e: no acercarse
a una persona enferma y lavarse las manos si se estuvo en lu· zo. (UFMG 2010) Sefiale l a opción e n q u e, según e l texto, l a
gares muy poblados. La dificultad es que somos seres sociales correspondencia de sentido entre la palabra y su s i n ó n i m o
y la convivencia es obligatoria. Por lo tanto, cabe analizar e/ está I NCORRECTA:
riesgo de mantener activa nuestra vida comunitaria. a) ai reados (ref. l) = ventilad os.
Hasta e/ momento, parece un vírus muy poco peligroso b) asisti r (ref. 2) = ver.
y una barrera de precaución como los barbijos debería c) cepa (ref. 3) = forma.
quedar limitada a los enfermos y a los indivíduos con una d) reportados (ref. 4) = declarados.
enfermedad de base con mayor susceptibilidad. Lo que
este vírus nos enseíia es que esta nueva 3cepa se presenta TEXTO PARA AS PRÓXIMAS DUAS Q U ESTÕES:
como m uy poco patogénica, lo que significa que los muer·
tos •reportados han sido m uy pocos y la mortalidad, por CONTRA EL JET LAG
e/ momento, es menor a la de la gripe estaciona/ a la que UN TRASTORNO TIPICO DE LOS VU ELOS I NTERNACIONALES
estamos acostumbrados en in viemo, y que, dicho sea de Si viajamos de/ este a/ oeste o vice versa pasamos por di·
paso, nunca nos h a /levado a cerrar co/egios ni a evitar la ferentes franjas h orarias. E/lo provocará a/teraciones de los
concurrencia a centros multitudinarios. ritmos sueflo-vigilia, ocasionando trastornos intestina/es,
La conducta que surge es que las personas con una en· falta de concentración y sensación de fatiga.
fermedad subyacente deben evitar 2asistir a lugares muy Para superar este desfase horario es recomendable que
concurridos y, en e/ caso de adquiriria, deben procurar la ajustes, e/ dia antes, tu ritmo ai de/ país a/ que vas a viajar_
asistencia y medicación específica. procurando coger e/ vuelo que te permita ir durmiendo para
(La Nación. Disponible en: http://www.lanacion.eom.ar/nota.asp?nota_id=ll32641>. /legar allí de día. Toma una dieta hipoca/óricay equilibrada,
Acceso en: 27 mayo 2009, adaptado) no fumes ni bebas a/cohol_ café o excitantes, ya que estos
hábitos retrasan la recuperación.
17. (UFMG 2010) Segú n el texto, es I NCORRECTO afi rmar q u e (VARGAS, Rita. Viaja sin Riesgos. Semana. Madri, 26 ago. 2009. p. 78. [Adaptado])
u na de l a s estrategias para evitar e l v í r u s i nfluenza tipo A
consiste en: :u. (UFG 201o) EI trastorno li amado jet lag tratado en el artículo
a) alejarse d e u n a persona q u e p resenta los síntomas d e l a es causado por:
e n fe rm edad. a) l o i m pactante q u e a h o ra res u l ta el c o n traste e n tre l a
b) evi tar espacios e n l o s q u e n o hay c i rcu lación d e a i re y que c u l tu ra d e O ri ente y la de Occidente.
están l le n o s d e perso nas. b) las fajas h orarias de los círc u l o s paralelos ai Ecuador q u e
c) lavarse las manos d e s p u és d e haber estado en si tios m uy c i rcu ndao e l g l o b o terráq u eo.
concu rridos. c) los efectos sentidos ai atravesar los h usos q u e marcan el
d) no h a b l a r de n i n g u n a m a n era con p e rsonas q u e están cam b i o d e h o ra.
i n fectadas por e l v i rus. d) e l paso a desho ra desde un pu nto merid ional a su co rres·
p o n d ie n te septentrional.
18. (UFMG 2010) Según e l texto, a respecto de la pandemia por e) la i: o la de horas que se hace en l os p u estos fronterizos q ue
vírus i nfluenza tipo A (N1H1), se puede decir que: d e l i m itao las n aciones.

142 1 GE IDIOMAS 2015


n. (UFG 2010) Para sobreponerse ai desfase comentado en el TEXTO PARA AS PRÓXIMAS TRES Q UESTOES:
texto se aconseja:
a) viajar de d ía en la nave. Ú LTIMO MOM ENTO SOCI EDAD
b) tomar p í l d o ras para d o r m i r. A un ano d e su desaparición, el papá d e Sofia cree que suhija
c) aislarse en el país receptor. está viva
d) i ngeri r u n a i nf u s i ó n d e té. A un ano de la desaparición de Sofia Herrera en un camping
e) evitar las com idas pesadas. de Tierra de/ Fuego, su papá, Fabián Herrera, dijo hoy que
está convencido que su hija "está con vida".
TEXTO PARA AS PRÓXIMAS TRES Q UESTOES: "Recorri todas las estancias, me fui a Chile, a/ Calafate, no hay
indícios de nada. No fue encontrada ninguna pista para pensar
EL ALFE N I Q U E que mi hija no está con vida. Estoy seguro que alguien la tiene
L a palabra alfenique e s d e origen árabe y se usaba para escondida. Hasta varios videntes me dijeron que a mi hija la
l/amar a la "pasta de azúcar cocida y estirada en barras muy sacaron cuatro personas en una avioneta': comentó en diálogo
delgadas y retorcidas". En México, l/amamos, en la actuali­ con e/ programa Inocentes y Cu/pables, de Radio Mitre.
dad, alfenique a las figuras de azúcar harinosa o de azúcar Para Herrera, e/ cuidador de/ camping, que está sospechado
glass que se hacen especialmente para e/ día de muertos, de la desaparición de la nena, no tiene nada que ver con e/
con varios disenos de acuerdo a la celebración. caso. "Es un hombre grande, no creo que tenga relación. A mi
Los aztecas, durante su imperio, ya preparaban figuras para tampoco me convence que cualquiera esté detenido': sostuvo.
sus ofrendas, hechas de amaranto y otras pastas comestibles; Además, negó que é/ oculte información sobre la causa.
se dice que estos son los primeros alfeniques que conocieron "Cómo voy a ocultar información sobre mi hija. Hicimos
los habitantes de nuestro continente. cinco anos de tratamiento con mi mujer para tenerla", se
Lu ego, las monjas de los conventos relacionaron e/ alfeni­ defendió e/ papá, quien fue imputado por la ]usticia pero
que, como figura tradicional, con los meses de noviembre y recuperó su libertad a las pocas horas.
diciembre, y se consumieron en las celebraciones de/ Día de (Texto adaptado. Clarín, 26 /9/2009)
Muertos, en las posadas y en la cena de Navidad.
Los dulces de/ día de muertos son fascinantes, sobre todo z6.(UFPR 2010) Sobre el papá d e Sofia, es correcto afirmar:
para los ninos, ya que después de este día, ai quitar la ofren­ a) está conve n c i d o d e q u e su h ija está e n C h i l e o Cal afate.
da, e/los se dan un gran festín; además, son una muestra de b) s o l i c i tó ai cu i dador dei cam p i n g q u e I e ayu d e a encon trar
la cultura y tradición que tenemos en nuestro país, por lo a Sofía.
tanto, consérvala y no la cambies. c) acusa ai c u i dador dei cam p i n g d e ocultar i n formaciones
(ELALFEfliQUE. Disponível em: <http:// www.peques.com.mxtel_alfenique.htm>. sobre el paradero d e Sofía.
Acesso em: 28/l0/2009. [Adaptado]) d) estuvo preso acusado d e ser responsable d e la desapari·
ción d e Sofía.
23. (UFG 2010) En el texto se expone q u e el térm ino "alfenique" e) sigue creyen d o q u e pronto se e n c o n trará el cadáver de
se remonta a la: su h ij a.
a) América p re h istór i ca.
b) clau s u ra d e las m o njas. 27. (UFPR 2010) En relación a la desaparición de Sofía, es cor­
c) l e n gu a d e los árabes. recto afi rm ar:
d) i n m i graci ó n m u s u l mana. a) au n q u e el papá de Sofía h aya d e n u n c iad o ai c u i d a d o r
e) s i m b i os i s con el i n d i o. d e i cam pi n g como s i e n d o el c u l pable, la p o l i da no p u e d e
pre n d e r i a por falta d e pru e bas.
24- (UFG 2010) Según el texto, el alfeíiique mexicano contem­ b) la polida sigue b u scando el paradero de Sofra basánd ose
porâneo destaca por su: e n las i nfo rmaciones d e las videntes.
a) u t i l izac i ó n dei amaranto como ofrenda. c) la pol ida sigue la pista d e un i n fo rm ante que habría vi sto
b) d e manda e n las cartas d e las posadas. a Sofra s i e n d o ! levada e n u n a avi o neta.
c) venta a los aztecas e n los co nventos. d) m i e ntras no se pruebe l o contrari o, ai papá d e Sofía I e pa­
d) forma ajustada ai tipo de celeb ración. rece b i e n q u e la p o l icía prenda a cualq u i e r sospechoso.
e) consu m o e n barras ai efectuar un entie rro. e) e l papá d e Sofía d ec l a ra q ue no p u e d e ser c u l pa b l e de
l a d esaparic i ó n d e Sofra p o rq u e para t e n e r l a tuvo q u e
25. (UFG 2010) Concluyendo el texto, se e n u n cia que e s n e­ someterse a u n largo trata m i e n to.
cesario:
a) q u itar los alfe fi i q ues a los m uchachos e n las fiestas. zl. (UFPR 2010) .:Quién h a dicho qué?
b) am parar los rasgos de la i d entidad nacio nal. a) el papá d e Sofía h a d ic h o que consu ltó a los v i d entes para
c) fasci nar a los c h icos con j ug u etes f u n estos. conocer el paradero de Sofía.
d) mostrar a los márti res que e n salzaron e l país. b) el repo rtero d e la Rad i o M i tre d ij o q u e está d e acuerdo
e) cam biar los d u lces d e m uertos por hábitos patrios. con Fabián Herrera e n c uanto a la i n ocencia dei c u i dador
dei cam p i ng.

GE IDIOMAS 2015 1 143


SIMULADO

c) el reportero d e la Rad i o M itre declará q u e había recorrido b) las a n o réxicas son m e n os tolerantes a la fru strac i á n q u e
varias estancias hasta encontrar ai papá d e Sofía. l as b u l í m i cas.
d) e l cu idador d e i cam p i n g acusá a i papá d e Sofía d e ocu l tar c) l o s ú n i cos res p onsables d e estas e n fe r m edades son los
i nformac i o n es. m e nsajes refe r i d os a l a i magen c o r p o ral que apareceri
e) e l reportero dei programa Inocentes y Cul pables afi rma que e n l o s med ios de com u n icac i á n .
c u atro personas sacaron a Sofra e n u n a avio neta. d) l as b u l ím i cas se caracte rizan p o r s e r s i l e n c i osas p ue s
b u scan l a perfecci á n .
TEXTO PARA AS PRÓXI MAS DUAS Q U ESTÕES: e ) l os m e n sajes q u e hacen refer e n c i a a l a i m agen corporal
n o s o n u b i cuos.
ANOREXIA V B U LI MIA
Han bastado diez anos para que las cifras sobre inci· JO. (PUC·PR 201o) De acuerdo ai texto, la anorexia y la bulimia:
dencia de anorexia y bulimia en Espana se equiparen a las a) s á l o se las e n c u en tra en m uj e res.
de/ resto de los países europeos, convirtiéndose así en la b) s o n e n fe rm edades que se p rod u c e n como c o n s e c u e n c i a
tercera enfermedad (la primera es e/ asma, y la segunda d e trastornos a l i m e n tíci os.
la obesidad) más frecuente entre los adolescentes. A hora, c) afectan a los ad o l e s c e n te s de todas las e d ad e s en e l
aproximadamente, uno de cada 100 adolescentes de entre m is m o porce n taje.
14 y 18 anos cae en las garras de la anorexia, mientras que d) son pato logfas q u e no d e p e n d e n de l a personali dad d e i
un 2-4% desarrolla bulimia. Y n o solo e/ sexo femenino ado l escente .
(la anorexia nerviosa afecta 15 veces más a mujeres que e) los varones no padecen estas enfermedades pues e l sexo
a hombres) se está enganchando a los trastornos de la m ascu l i n o está genéticame n te p rotegido.
alimentación, también los varones han empezado a verse
reflejados significativamente en las estadfsticas. "E/ sexo
es un factor de protección frente a los trastornos de la ali·
R E S P OSTAS C O M E NTADAS
mentación. Hasta ahora ser varón era un arma eficaz contra
la anorexia y la bulimia, pero esto está cambiando. Cada
vez hay más casos de chicos que sufren estas patologias'� 1.Criticaba lo que había hecho Stalin desde su delictiva
destaca e/ M. Velilla, de/ Hospital Clínico de Zaragoza. administración. A resposta se justifica com a última passa·
La anorexia y la bulimia, conocidas popularmente a través gem do texto: "e/ texto resultaba ser la descripción de los
de/ rastro de a/gunos de sus afectados como lo fue Lady Diana, síntomas de un enfermo paranoico con manía persecutoria,
)ane Fonda o la bailarina de/ Boston Bal/et, Heide Guenther, que se acentuaban a medida que pasaba e/ tiempo. Algo que
que murió e/30 de junio como consecuencia de una anorexia, corroboran los hechos hoy conocidos". Esse trecho critica o
son patologias modernas que han sabido aprovecharse de que Stalin havia feito em sua administração. Resposta: E
las tendencias sociales de los países industrializados. En las
últimas dos décadas, la delgadez se ha convertido en la tarjeta 1. Repercusión de la acción humana se limita. A resposta
de visita de todos aquel/os hombres y mujeres que desean se justifica com "q u e e/ ser h u m a n o ni c o n t rola ni h a
triunfar social y profesionalmente. "Los mensajes que hacen desarrollado media n te carreteras u otra infraestructura
referenda a la imagen corporal son omnipresentesy con e/los industrial", indica n d o a repercussão da ação h u m a n a .
se transmite la idea de que estar delgado es e/ medio para Resposta: A
obtener la felicidad y e/ éxito. La presión es excesiva'� destaca
e/ doctor Gonza/o Morandé, de/ Hospital Niflo )esús de Madrid. J. Propiedad rural. O significado da palavra finca pode ser
Las vfctimas de la anorexia y de la buli mia poseen un carác· deduzido por eliminação das demais alternativas. A dúvida
ter obsesivo que acaba traduciéndose en una preocupación poderia estar entre "propiedad rural" e "hotel rural". Porém, o
constante por e/ peso y la dieta. Sin embargo, la personalidad agroturismo consiste na inserção do turista na vida cotidiana
de estos pacientes es diferente. As(, la anoréxica sue/e estar das propriedades rurais, e não o contrário. Resposta: A
considerada como "niíía modelo": perfeccionista, buena estu·
diante, con un nive/ intelectual alto y con tendencia a evitar 4. En Espafía, las personas no actúan directatpente .en e/
conflictos. En cambio, su preocupación por la opinión que los proceso agrícola. A resposta se justifica com "Los espano·
demás tienen de e/la es excesiva, así como su autocontrol. /es, pioneros en ma teria de/ ocio, manejan un sistema muy
Las bulímicas, por e/ contrario, suelen ser más impulsivas, parecido pera al/í las fincas no son grandes casonas y las
intolerantes y se frustran más. personas que las atienden n o es tán comprometidas con e/
(Adaptado de: MATEY, Patricia. La Nueva Epidemia de losAdolescentes. p roceso y desarro/lo agrfcola, como sí sucede con los cafe·
http://www. elmundo.estsalud/Snumeros/97/Sl6l/Sl6lalimentacion.html, el l4/9/l009) teros colombianos". Ou seja, os espanhóis não se envolvem
com o desenvolvimento agrfco/a. Resposta: D
19. (PUC·PR 2010) D e acuerdo con e l Dr. Gonzalo Morandé, d e i
Hospital N ii\o jesús d e Madrid: s. Sugiere que el tiempo no es igual para todos. A resposta está
a) estas patol ogías son res u l tado dei p re su p u esto q u e la no primeiro verso: "E/ tiempo no tiene una sino sus muchas
d e l gadez asegu ra e l éxito y l a fel i c i dad. ruedas". A poetisa afirma que o tempo tem m uitas rodas,

144 1 G E IDIOMAS 1015


ou seja, não é percebido da mesma maneira por todos. Para das de manera festiva, si no con admiración, por buena parte
alguns é mais len to, para o u tros, mais rápido. Resposta: C de la pob/ación brasilena?". A pergunta tem como objetivo
polemizar, ou seja, discutir a atitude dos brasileiros que trans­
&. E/ tiempo no tiene só/o una rueda, en reafidad tiene mu­ formaram a figura do torturador em herói. Resposta: B
chas. A estrutura a dversa tiva do primeiro verso, marcada
pelo emprego da conjunção "sino" ("mas sim'� em português), 13. Cansados y hartos. Tra ta-se de uma expressão idiomá­
implica u m a refo rm u la ção/explicação do q u e foi dito n o tica do espan h o l, equivalente a o u tra: "hasta las narices".
primeiro segme n to do verso. Resposta: A Resposta: A

7. Aorganização OpenNet realizou um estudo sobre países 14-A la vez que algunas personas tenían escalofríos en la bu·
que praticam a censura de conteúdos políticos ou sociais taca . . A expressão a "la vez" equivale a "m ien tras'� e expres­
.

da internet. A resposta correta está no primeiro parágra fo, sam simultaneidade de ações. E "se estremecia" corresponde
e m b o ra s eja n ecessário c onsiderar vários seg m e n tos de a "tenía escalofríos". O substan tivo "butaca" corresponde a
texto para compô-/a: "Hoy s e hace público un estudio de la assen to, cadeira de cinema ou teatro. Resposta: C
p res tigiosa orga nización OpenNet, in tegrada por las un iver­
sidades de Oxford, Cambrigde, Harvard y To ronto. En é/ se 15. Era de cuarenta millones, en 2006. De acordo com o texto,
concluye que 25 países - e n tre e/los l rán, China y Corea de/ "en 2006, ha bía a/rededor de 40 millones de personas a/rede­
Sur - ejercen la censura de webs con con tenidos po líticos o dor dei m u ndo víviendo con VIH/5/DA". Resposta: C
socia/es peligrosos, e impiden e/ acceso a aplicaciones como
You Tu be o Goog/e Maps". Resposta: A 16. Son correctas las alternativas I, 111, /Vy V. De acordo com
o texto, o vírus VIH foi encon trado também na saliva, mas
8. Há três modalidades de censura na internet: a política, não h á con tágio por meio de c o n ta t o com essa secreção
a social e a relacionada á segurança nacional. A resposta corporal: "Se h a encon trado e/ VIH en saliva, lágrimas, tejido
se encon tra no último parágrafo: "Los autores dei informe de/ sistema nervíoso, líquido cefalorraqufdeo, sangre, semen,
han detectado tres tipos de censura. La más común es la flujo vaginal y feche materna. Sin embargo, só/à a través de la
política, (... ) El segundo tipo de censura es Ia social, (. .. ). Un sangre, e/ semen, las secreciones vaginales y la feche materna
tercer tipo de censura se relaciona con la 'seguridad nacio­ se transmite la infección a o tras personas". Resposta: D
nal"'. Resposta: A
17.No hab/ar de ninguna manera con personas que están
g. Somente as afirmativas 11, I/I e IV são corretas. Segundo o infectadas por e/ virus. A resposta pode ser jus tificada no
texto, não é possível afirmar se há censura a con teúdos da texto pelo trecho n o q ua l se afirma que "La dificultad es q u e
in ternet na Core ia do Norte. Resposta: E s o m o s seres s o e ia les y t a con vivencia es obligatoria. P o r l o
tan to, cabe analizar e/ riesgo d e mantener activa nuestra vida
10. en
e/ carnaval de este afío, las tiendas no vendieron cas i comun ita ria". Isso reforça a ideia de que não se pode evitar
ningún traje de Superman ni de princesa. A resposta está o vírus "não falando com pessoas que esteja m infectadas
n o primeiro parágrafo: "Los pop ulares trajes de Superman o com o vfrus". Resposta: O
de princesa cogían polvo en las tiendas". A expressão "coger
polvo" significa "tomar pó", ou seja, as fa n tasias citadas não 11. Las personas con una enfermedad latente deben evitar
saíram das lojas, não foram vendidas. Resposta: E lugares con mucha gen te. A res p o s ta C se jus tifica c o m
a leitura do último parágrafo do texto: "La conducta q u e
11. Las ca/les de Río de janeiro se /lenaron de chiquil/os surge e s q u e las personas c o n u n a e n fermedad s u byacente
con disfraces de/ héroe torturador de la película Tropa d e deben evitar asistir a lugares muy concurridos y, en e/ caso
Élite. A resposta se encon tra n o prime iro parágra fo, m a s é de a d q uiriria, deben procurar la asistencia y m edicación
necessário jun tar informação que está distribuída por todo específica". Resposta: C
o parágrafo para compô-/a: "Un disfraz inédito t o m á las
cal/es de Río de janeiro e/ pasado carnaval. Miles de nifíos 19. p contagio es muy fácil, razón por la eua/ la pandemia
sonrien tes, vestidos de n egro, /ucían en sus camisetas un ya es un hecho, y contra eso hay pocas acciones posibles.
sin iestro escudo: una ca/a vera atravesada por un machete y Difere n te m e n te do q u e diz a a lternativa (A), apesar de o
dos pistolas. (...)Este ano la moda fue sambar vestido como e/ con tágio da nova gripe ser fácil, há varias ações possíveis
capitán Nascimen to, (. . .) e/ héroe torturador que protagoniza para preveni-lo. Ta mbém, em n e n h u m momento se afirma,
e/ filme brasileno Tro p a d e t l i te ". Resposta: A embora o vírus seja pandêmico, que já existe uma pandemia
causada por ele. Resposta: A
12.Polemiza sobre e/ hecho de que la población brasi/efía
reproduce la vestimenta y las frases de un personaje que 20. Asistir (ref. 2) = ver. No texto, asistir não q uer dizer ver,
desempena e/ papel de torturador en Tropa d e Élite. A res­ e sim "concurrir a una casa o reu n ión, tertulia, curs o, acto
posta está no trecho: "c!Cómo es posible que estas via/aciones público e tc"., segundo definição do dicionário da Real Aca­
sistemá ticas de los derechos humanos hayan sido mimetiza- demia Espa no/a. Resposta: B

GE IDIOMAS 2015 1 145


ESPANHOL S I M U LADO

u. Los efectos sentidos ai a travesar los husos que marcan :ai. Estuvo preso acusado de ser responsable de la desapa­
el cambio de hora. A justificativa para a resposta (C) recai rición de Sofia. O fim do texto informa que a alterna tiva (D)
sobre os efeitos sen tidos pelas pessoas ao a travessarem os está correta: "e/ papá, quien fue imputado por la}usticia pera
fusos que marcam as mudanças de horas. Esses efeitos são recuperó su libertad a las pocas horas". Resposta: D
descritos no trecho "pasamos por diferen tes franjas hora·
rias. E/lo provocará a/te raciones de los ritmos suefío·vigilia, 27. E/ papá de Sofía declara que no puede ser culpable de ta
ocasionando trastornos intestinales, fa lta de concen tración desaparición de Sofía porque para tenerla tuvo que some­
y sensación de fatiga". Resposta: C terse a un largo tratamiento. O último pa rágrafo do texto
informa q u e a alterna tiva (E) é a correta: "cinco a fí o s de
22. Evitar las comidas pesadas. O texto sugere "una dieta tra tamiento con mi mujer para tenerla .. . ". Resposta: E
hipocalórica y equilibrada, no fumes ni bebas alcohol, café
o excita n tes". A resposta está na expressão "dieta hipo­ 21. E/ papá de Sofia ha dicho que consultá a los videntes
calórica", o u s eja, dieta p o b re e m calorias, comida leve. para conocer e/ paradero de Sofia. O pai da garota consul­
Resposta: E tou videntes. Isso é dito no trecho: "Hasta varias videntes
m e dijeron que a mi hija la sacaron cua tro personas en una
23. Lengua de tos árabes. O seguinte trecho justifica a res­ avioneta", comentó en diálogo con e/ programa Inocen tes
posta: "La paiabra alfeflique es de origen árabe(. .)". O verbo y Culpables, de Radio Mitre". Resposta: A
"remon tarse", que aparece no enunciado da questão, significa
"ter origem em". Resposta: C 29. Estas patologias son resultado de/ presupuesto que ta
defgadez asegura e/ éxito y la felicidad. No texto, informa-se
24- Forma ajustada a/ tipo de celebración. Segundo o texto, que "Los mensajes que hacen referencia a la imagen corporal
"En M éxico, [/a m a m os, e n la a c tualidad, alfefíique a las son omnipresentes y con e/los se transmite la idea de que
figuras de aiúcar harinosa o de azúcar g/ass que se hacen estar delgado es e/ medio para obtener la felicidad y e / éxi­
especialm ente para e/ día de muertos, con varias diseflos de to (..)". Isto é, as doenças são resultado da crença de q u e a
acuerdo a la celebración". Ou s eja, o "a lfeflique" s e adapta magreza assegura o êxito e a felicidade. Resposta: A
às diferentes formas de comem oração do Dia dos Mortos
no México. Resposta: D JO. Son enfermedades que se producen como consecuencia
de trastornos alimentícios. O texto aponta os transtornos ali­
:as. Amparar los rasgos de la identidad nacional. A resposta mentares como um sinônimo ou resultado de doenças como
(B) se jus tifica com a passagem final do texto: "Los dulces de/ anorexia e bulimia. O trecho em que s e encontra a resposta:
día de muertos son fascinan tes, sobre todo para los niflos, ya "Y no solo e/ sexo femenino (la anorexia nerviosa afecta 15
que después de este día, a/ quitar la ofrenda, e/los se dan un veces más a m ujeres que a hom bres) s e está enganchando
gran festfn; además, son una muestra de la cultura y tradición a los trastornos de la alimen tación (...)". Resposta: 8
que tenemos en n u estro país, por lo tan to, consérvala y no
la cambies". Resposta: 8

146 1 GE IDIOMAS 2015

Você também pode gostar